You are on page 1of 724

Fundamentals of Mathematics

By:
Denny Burzynski
Wade Ellis
Fundamentals of Mathematics

By:
Denny Burzynski
Wade Ellis

Online:
< http://cnx.org/content/col10615/1.4/ >

CONNEXIONS

Rice University, Houston, Texas


This selection and arrangement of content as a collection is copyrighted by Denny Burzynski, Wade Ellis. It is licensed
under the Creative Commons Attribution 2.0 license (http://creativecommons.org/licenses/by/2.0/).
Collection structure revised: August 18, 2010
PDF generated: August 23, 2010
For copyright and attribution information for the modules contained in this collection, see p. 697.
Table of Contents
Preface ............................................................................ ................... 1
Acknowledgements ............................................................... ................... 7
1 Addition and Subtraction of Whole Numbers
1.1 Objectives . . . . . . . . . . . . . . . . . . . . . . . . . . . . . . . . . . . . . . . . . . . . . . . . . . . . . . . . . . . . . . . . . . . . . . . . . . . . . . . . . . . 9
1.2 Whole Numbers . . . . . . . . . . . . . . . . . . . . . . . . . . . . . . . . . . . . . . . . . . . . . . . . . . . . . . . . . . . . . . . . . . . . . . . . . . . . 10
1.3 Reading and Writing Whole Numbers . . . . . . . . . . . . . . . . . . . . . . . . . . . . . . . . . . . . . . . . . . . . . . . . . . . . . . 15
1.4 Rounding Whole Numbers . . . . . . . . . . . . . . . . . . . . . . . . . . . . . . . . . . . . . . . . . . . . . . . . . . . . . . . . . . . . . . . . . . 21
1.5 Addition of Whole Numbers . . . . . . . . . . . . . . . . . . . . . . . . . . . . . . . . . . . . . . . . . . . . . . . . . . . . . . . . . . . . . . . . 29
1.6 Subtraction of Whole Numbers . . . . . . . . . . . . . . . . . . . . . . . . . . . . . . . . . . . . . . . . . . . . . . . . . . . . . . . . . . . . . 44
1.7 Properties of Addition . . . . . . . . . . . . . . . . . . . . . . . . . . . . . . . . . . . . . . . . . . . . . . . . . . . . . . . . . . . . . . . . . . . . . . 62
1.8 Summary of Key Concepts . . . . . . . . . . . . . . . . . . . . . . . . . . . . . . . . . . . . . . . . . . . . . . . . . . . . . . . . . . . . . . . . . 67
1.9 Exercise Supplement . . . . . . . . . . . . . . . . . . . . . . . . . . . . . . . . . . . . . . . . . . . . . . . . . . . . . . . . . . . . . . . . . . . . . . . 69
1.10 Prociency Exam . . . . . . . . . . . . . . . . . . . . . . . . . . . . . . . . . . . . . . . . . . . . . . . . . . . . . . . . . . . . . . . . . . . . . . . . . 74
Solutions . . . . . . . . . . . . . . . . . . . . . . . . . . . . . . . . . . . . . . . . . . . . . . . . . . . . . . . . . . . . . . . . . . . . . . . . . . . . . . . . . . . . . . . . 76
2 Multiplication and Division of Whole Numbers
2.1 Objectives . . . . . . . . . . . . . . . . . . . . . . . . . . . . . . . . . . . . . . . . . . . . . . . . . . . . . . . . . . . . . . . . . . . . . . . . . . . . . . . . . . 91
2.2 Multiplication of Whole Numbers . . . . . . . . . . . . . . . . . . . . . . . . . . . . . . . . . . . . . . . . . . . . . .. . . . . . . . . . . . . 92
2.3 Concepts of Division of Whole Numbers . . . . . . . . . . . . . . . . . . . . . . . . . . . . . . . . . . . . . . . . . . . . . . . . . . . 105
2.4 Division of Whole Numbers . . . . . . . . . . . . . . . . . . . . . . . . . . . . . . . . . . . . . . . . . . . . . . . . . . . .. . . . . . . . . . . . 113
2.5 Some Interesting Facts about Division . . . . . . . . . . . . . . . . . . . . . . . . . . . . . . . . . . . . . . . . . . . . . . . . . . . . . 126
2.6 Properties of Multiplication . . . . . . . . . . . . . . . . . . . . . . . . . . . . . . . . . . . . . . . . . . . . . . . . . . . . . . . . . . . . . . . 130
2.7 Summary of Key Concepts . . . . . . . . . . . . . . . . . . . . . . . . . . . . . . . . . . . . . . . . . . . . . . . . . . . . . . . . . . . . . . . . 135
2.8 Exercise Supplement . . . . . . . . . . . . . . . . . . . . . . . . . . . . . . . . . . . . . . . . . . . . . . . . . . . . . . . . . . . . . . . . . . . . . . 136
2.9 Prociency Exam . . . . . . . . . . . . . . . . . . . . . . . . . . . . . . . . . . . . . . . . . . . . . . . . . . . . . . . . . . . . . .. . . . . . . . . . . . 139
Solutions . . . . . . . . . . . . . . . . . . . . . . . . . . . . . . . . . . . . . . . . . . . . . . . . . . . . . . . . . . . . . . . . . . . . . . . . . . . . . . . . . . . . . . . 141
3 Exponents, Roots, and Factorization of Whole Numbers
3.1 Objectives . . . . . . . . . . . . . . . . . . . . . . . . . . . . . . . . . . . . . . . . . . . . . . . . . . . . . . . . . . . . . . . . . . . . .. . . . . . . . . . . . 151
3.2 Exponents and Roots . . . . . . . . . . . . . . . . . . . . . . . . . . . . . . . . . . . . . . . . . . . . . . . . . . . . . . . . . .. . . . . . . . . . . . 152
3.3 Grouping Symbols and the Order of Operations . . . . . . . . . . . . . . . . . . . . . . . . . . . . . . . . . . . . . . . . . . . 160
3.4 Prime Factorization of Natural Numbers . . . . . . . . . . . . . . . . . . . . . . . . . . . . . . . . . . . . . . .. . . . . . . . . . . . 170
3.5 The Greatest Common Factor . . . . . . . . . . . . . . . . . . . . . . . . . . . . . . . . . . . . . . . . . . . . . . . . . . . . . . . . . . . . . 178
3.6 The Least Common Multiple . . . . . . . . . . . . . . . . . . . . . . . . . . . . . . . . . . . . . . . . . . . . . . . . . . . . . . . . . . . . . . 182
3.7 Summary of Key Concepts . . . . . . . . . . . . . . . . . . . . . . . . . . . . . . . . . . . . . . . . . . . . . . . . . . . . . . . . . . . . . . . . 188
3.8 Exercise Supplement . . . . . . . . . . . . . . . . . . . . . . . . . . . . . . . . . . . . . . . . . . . . . . . . . . . . . . . . . . . . . . . . . . . . . . 191
3.9 Prociency Exam . . . . . . . . . . . . . . . . . . . . . . . . . . . . . . . . . . . . . . . . . . . . . . . . . . . . . . . . . . . . . .. . . . . . . . . . . . 196
Solutions . . . . . . . . . . . . . . . . . . . . . . . . . . . . . . . . . . . . . . . . . . . . . . . . . . . . . . . . . . . . . . . . . . . . . . . . . . . . . . . . . . . . . . . 198
4 Introduction to Fractions and Multiplication and Division of Fractions
4.1 Objectives . . . . . . . . . . . . . . . . . . . . . . . . . . . . . . . . . . . . . . . . . . . . . . . . . . . . . . . . . . . . . . . . . . . . .. . . . . . . . . . . . 211
4.2 Fractions of Whole Numbers . . . . . . . . . . . . . . . . . . . . . . . . . . . . . . . . . . . . . . . . . . . . . . . . . . .. . . . . . . . . . . . 212
4.3 Proper Fractions, Improper Fractions, and Mixed Numbers . . . . . . . . . . . . . . . . . . . . . . . . . . . . . . . . 221
4.4 Equivalent Fractions, Reducing Fractions to Lowest Terms, and Raising Fractions
to Higher Terms . . . . . . . . . . . . . . . . . . . . . . . . . . . . . . . . . . . . . . . . . . . . . . . . . . . . . . . . . . . . . . . . . . . . . . . . . . . 231
4.5 Multiplication of Fractions . . . . . . . . . . . . . . . . . . . . . . . . . . . . . . . . . . . . . . . . . . . . . . . . . . . . . . . . . . . . . . . . 242
4.6 Division of Fractions . . . . . . . . . . . . . . . . . . . . . . . . . . . . . . . . . . . . . . . . . . . . . . . . . . . . . . . . . . . . . . . . . . . . . . 253
4.7 Applications Involving Fractions . . . . . . . . . . . . . . . . . . . . . . . . . . . . . . . . . . . . . . . . . . . . . . . . . . . . . . . . . . . 259
4.8 Summary of Key Concepts . . . . . . . . . . . . . . . . . . . . . . . . . . . . . . . . . . . . . . . . . . . . . . . . . . . . . . . . . . . . . . . . 265
4.9 Exercise Supplement . . . . . . . . . . . . . . . . . . . . . . . . . . . . . . . . . . . . . . . . . . . . . . . . . . . . . . . . . . . . . . . . . . . . . . 268
iv

4.10 Prociency Exam . . . . . . . . . . . . . . . . . . . . . . . . . . . . . . . . . . . . . . . . . . . . . . . . . . . . . . . . . . . . . . . . . . . . . . . . 274


Solutions . . . . . . . . . . . . . . . . . . . . . . . . . . . . . . . . . . . . . . . . . . . . . . . . . . . . . . . . . . . . . . . . . . . . . . . . . . . . . . . . . . . . . . . 276
5 Addition and Subtraction of Fractions, Comparing Fractions, and Complex Fractions
5.1 Objectives . . . . . . . . . . . . . . . . . . . . . . . . . . . . . . . . . . . . . . . . . . . . . . . . . . . . . . . . . . . . . . . . . . . . .. . . . . . . . . . . . 293
5.2 Addition and Subtraction of Fractions with Like Denominators . . . . . . . . . . . . . . . . . . . . . . . . . . . . 294
5.3 Addition and Subtraction of Fractions with Unlike Denominators . . . . . . . . . . . . . . . . . . . . . . . . . . 298
5.4 Addition and Subtraction of Mixed Numbers . . . . . . . . . . . . . . . . . . . . . . . . . . . . . . . . . . . . . . . . . . . . . . 304
5.5 Comparing Fractions . . . . . . . . . . . . . . . . . . . . . . . . . . . . . . . . . . . . . . . . . . . . . . . . . . . . . . . . . . . . . . . . . . . . . . 308
5.6 Complex Fractions . . . . . . . . . . . . . . . . . . . . . . . . . . . . . . . . . . . . . . . . . . . . . . . . . . . . . . . . . . . . . . . . . . . . . . . . 312
5.7 Combinations of Operations with Fractions . . . . . . . . . . . . . . . . . . . . . . . . . . . . . . . . . . . . . . . . . . . . . . . . 316
5.8 Summary of Key Concepts . . . . . . . . . . . . . . . . . . . . . . . . . . . . . . . . . . . . . . . . . . . . . . . . . . . . . . . . . . . . . . . . 320
5.9 Exercise Supplement . . . . . . . . . . . . . . . . . . . . . . . . . . . . . . . . . . . . . . . . . . . . . . . . . . . . . . . . . . . . . . . . . . . . . . 321
5.10 Prociency Exam . . . . . . . . . . . . . . . . . . . . . . . . . . . . . . . . . . . . . . . . . . . . . . . . . . . . . . . . . . . . . . . . . . . . . . . . 325
Solutions . . . . . . . . . . . . . . . . . . . . . . . . . . . . . . . . . . . . . . . . . . . . . . . . . . . . . . . . . . . . . . . . . . . . . . . . . . . . . . . . . . . . . . . 327
6 Decimals
6.1 Objectives . . . . . . . . . . . . . . . . . . . . . . . . . . . . . . . . . . . . . . . . . . . . . . . . . . . . . . . . . . . . . . . . . . . . .. . . . . . . . . . . . 337
6.2 Reading and Writing Decimals . . . . . . . . . . . . . . . . . . . . . . . . . . . . . . . . . . . . . . . . . . . . . . . . . . . . . . . . . . . . 338
6.3 Converting a Decimal to a Fraction . . . . . . . . . . . . . . . . . . . . . . . . . . . . . . . . . . . . . . . . . . . . . . . . . . . . . . . . 344
6.4 Rounding Decimals . . . . . . . . . . . . . . . . . . . . . . . . . . . . . . . . . . . . . . . . . . . . . . . . . . . . . . . . . . . .. . . . . . . . . . . . 348
6.5 Addition and Subtraction of Decimals . . . . . . . . . . . . . . . . . . . . . . . . . . . . . . . . . . . . . . . . . . . . . . . . . . . . . 352
6.6 Multiplication of Decimals . . . . . . . . . . . . . . . . . . . . . . . . . . . . . . . . . . . . . . . . . . . . . . . . . . . . .. . . . . . . . . . . . 358
6.7 Division of Decimals . . . . . . . . . . . . . . . . . . . . . . . . . . . . . . . . . . . . . . . . . . . . . . . . . . . . . . . . . . .. . . . . . . . . . . . 369
6.8 Nonterminating Divisions . . . . . . . . . . . . . . . . . . . . . . . . . . . . . . . . . . . . . . . . . . . . . . . . . . . . . .. . . . . . . . . . . . 381
6.9 Converting a Fraction to a Decimal . . . . . . . . . . . . . . . . . . . . . . . . . . . . . . . . . . . . . . . . . . . . . . . . . . . . . . . . 386
6.10 Combinations of Operations with Decimals and Fractions . . . . . . . . . . . . . . . . . . . . .. . . . . . . . . . . . 392
6.11 Summary of Key Concepts . . . . . . . . . . . . . . . . . . . . . . . . . . . . . . . . . . . . . . . . . . . . . . . . . . . . . . . . . . . . . . . 395
6.12 Exercise Supplement . . . . . . . . . . . . . . . . . . . . . . . . . . . . . . . . . . . . . . . . . . . . . . . . . . . . . . . . . . . . . . . . . . . . . 396
6.13 Prociency Exam . . . . . . . . . . . . . . . . . . . . . . . . . . . . . . . . . . . . . . . . . . . . . . . . . . . . . . . . . . . . . . . . . . . . . . . . 400
Solutions . . . . . . . . . . . . . . . . . . . . . . . . . . . . . . . . . . . . . . . . . . . . . . . . . . . . . . . . . . . . . . . . . . . . . . . . . . . . . . . . . . . . . . . 402
7 Ratios and Rates
7.1 Objectives . . . . . . . . . . . . . . . . . . . . . . . . . . . . . . . . . . . . . . . . . . . . . . . . . . . . . . . . . . . . . . . . . . . . .. . . . . . . . . . . . 417
7.2 Ratios and Rates . . . . . . . . . . . . . . . . . . . . . . . . . . . . . . . . . . . . . . . . . . . . . . . . . . . . . . . . . . . . . . . . . . . . . . . . . . 417
7.3 Proportions . . . . . . . . . . . . . . . . . . . . . . . . . . . . . . . . . . . . . . . . . . . . . . . . . . . . . . . . . . . . . . . . . . . . . . . . . . . . . . . 423
7.4 Applications of Proportions . . . . . . . . . . . . . . . . . . . . . . . . . . . . . . . . . . . . . . . . . . . . . . . . . . . .. . . . . . . . . . . . 429
7.5 Percent . . . . . . . . . . . . . . . . . . . . . . . . . . . . . . . . . . . . . . . . . . . . . . . . . . . . . . . . . . . . . . . . . . . . . . . .. . . . . . . . . . . . 435
7.6 Fractions of One Percent . . . . . . . . . . . . . . . . . . . . . . . . . . . . . . . . . . . . . . . . . . . . . . . . . . . . . . . . . . . . . . . . . . 442
7.7 Applications of Percents . . . . . . . . . . . . . . . . . . . . . . . . . . . . . . . . . . . . . . . . . . . . . . . . . . . . . . . . . . . . . . . . . . . 446
7.8 Summary of Key Concepts . . . . . . . . . . . . . . . . . . . . . . . . . . . . . . . . . . . . . . . . . . . . . . . . . . . . . . . . . . . . . . . . 456
7.9 Exercise Supplement . . . . . . . . . . . . . . . . . . . . . . . . . . . . . . . . . . . . . . . . . . . . . . . . . . . . . . . . . . . . . . . . . . . . . . 458
7.10 Prociency Exam . . . . . . . . . . . . . . . . . . . . . . . . . . . . . . . . . . . . . . . . . . . . . . . . . . . . . . . . . . . . . . . . . . . . . . . . 462
Solutions . . . . . . . . . . . . . . . . . . . . . . . . . . . . . . . . . . . . . . . . . . . . . . . . . . . . . . . . . . . . . . . . . . . . . . . . . . . . . . . . . . . . . . . 464
8 Techniques of Estimation
8.1 Objectives . . . . . . . . . . . . . . . . . . . . . . . . . . . . . . . . . . . . . . . . . . . . . . . . . . . . . . . . . . . . . . . . . . . . .. . . . . . . . . . . . 475
8.2 Estimation by Rounding . . . . . . . . . . . . . . . . . . . . . . . . . . . . . . . . . . . . . . . . . . . . . . . . . . . . . . .. . . . . . . . . . . . 475
8.3 Estimation by Clustering . . . . . . . . . . . . . . . . . . . . . . . . . . . . . . . . . . . . . . . . . . . . . . . . . . . . . . . . . . . . . . . . . . 482
8.4 Mental Arithmetic-Using the Distributive Property . . . . . . . . . . . . . . . . . . . . . . . . . . . . . . . . . . . . . . . . 485
8.5 Estimation by Rounding Fractions . . . . . . . . . . . . . . . . . . . . . . . . . . . . . . . . . . . . . . . . . . . . .. . . . . . . . . . . . 491
8.6 Summary of Key Concepts . . . . . . . . . . . . . . . . . . . . . . . . . . . . . . . . . . . . . . . . . . . . . . . . . . . . . . . . . . . . . . . . 493
8.7 Exercise Supplement . . . . . . . . . . . . . . . . . . . . . . . . . . . . . . . . . . . . . . . . . . . . . . . . . . . . . . . . . . . . . . . . . . . . . . 494
8.8 Prociency Exam . . . . . . . . . . . . . . . . . . . . . . . . . . . . . . . . . . . . . . . . . . . . . . . . . . . . . . . . . . . . . .. . . . . . . . . . . . 500
v

Solutions . . . . . . . . . . . . . . . . . . . . . . . . . . . . . . . . . . . . . . . . . . . . . . . . . . . . . . . . . . . . . . . . . . . . . . . . . . . . . . . . . . . . . . . 502
9 Measurement and Geometry
9.1 Objectives . . . . . . . . . . . . . . . . . . . . . . . . . . . . . . . . . . . . . . . . . . . . . . . . . . . . . . . . . . . . . . . . . . . . .. . . . . . . . . . . . 511
9.2 Measurement and the United States System . . . . . . . . . . . . . . . . . . . . . . . . . . . . . . . . . . . . . . . . . . . . . . . 512
9.3 The Metric System of Measurement . . . . . . . . . . . . . . . . . . . . . . . . . . . . . . . . . . . . . . . . . . . . . . . . . . . . . . . 517
9.4 Simplication of Denominate Numbers . . . . . . . . . . . . . . . . . . . . . . . . . . . . . . . . . . . . . . . . . . . . . . . . . . . . 522
9.5 Perimeter and Circumference of Geometric Figures . . . . . . . . . . . . . . . . . . . . . . . . . . . . . . . . . . . . . . . . 530
9.6 Area and Volume of Geometric Figures and Objects . . . . . . . . . . . . . . . . . . . . . . . . . . . . . . . . . . . . . . . 540
9.7 Summary of Key Concepts . . . . . . . . . . . . . . . . . . . . . . . . . . . . . . . . . . . . . . . . . . . . . . . . . . . . . . . . . . . . . . . . 555
9.8 Exercise Supplement . . . . . . . . . . . . . . . . . . . . . . . . . . . . . . . . . . . . . . . . . . . . . . . . . . . . . . . . . . . . . . . . . . . . . . 557
9.9 Prociency Exam . . . . . . . . . . . . . . . . . . . . . . . . . . . . . . . . . . . . . . . . . . . . . . . . . . . . . . . . . . . . . .. . . . . . . . . . . . 562
Solutions . . . . . . . . . . . . . . . . . . . . . . . . . . . . . . . . . . . . . . . . . . . . . . . . . . . . . . . . . . . . . . . . . . . . . . . . . . . . . . . . . . . . . . . 567
10 Signed Numbers
10.1 Objectives . . . . . . . . . . . . . . . . . . . . . . . . . . . . . . . . . . . . . . . . . . . . . . . . . . . . . . . . . . . . . . . . . . . . . . . . . . . . . . . 575
10.2 Variables, Constants, and Real Numbers . . . . . . . . . . . . . . . . . . . . . . . . . . . . . . . . . . . . . . . . . . . . . . . . . 576
10.3 Signed Numbers . . . . . . . . . . . . . . . . . . . . . . . . . . . . . . . . . . . . . . . . . . . . . . . . . . . . . . . . . . . . . . . . . . . . . . . . . 582
10.4 Absolute Value . . . . . . . . . . . . . . . . . . . . . . . . . . . . . . . . . . . . . . . . . . . . . . . . . . . . . . . . . . . . . . .. . . . . . . . . . . . 586
10.5 Addition of Signed Numbers . . . . . . . . . . . . . . . . . . . . . . . . . . . . . . . . . . . . . . . . . . . . . . . . . . . . . . . . . . . . . 590
10.6 Subtraction of Signed Numbers . . . . . . . . . . . . . . . . . . . . . . . . . . . . . . . . . . . . . . . . . . . . . . .. . . . . . . . . . . . 597
10.7 Multiplication and Division of Signed Numbers . . . . . . . . . . . . . . . . . . . . . . . . . . . . . . . . . . . . . . . . . . . 602
10.8 Summary of Key Concepts . . . . . . . . . . . . . . . . . . . . . . . . . . . . . . . . . . . . . . . . . . . . . . . . . . . . . . . . . . . . . . . 611
10.9 Exercise Supplement . . . . . . . . . . . . . . . . . . . . . . . . . . . . . . . . . . . . . . . . . . . . . . . . . . . . . . . . . . . . . . . . . . . . . 612
10.10 Prociency Exam . . . . . . . . . . . . . . . . . . . . . . . . . . . . . . . . . . . . . . . . . . . . . . . . . . . . . . . . . . . . . . . . . . . . . . . 616
Solutions . . . . . . . . . . . . . . . . . . . . . . . . . . . . . . . . . . . . . . . . . . . . . . . . . . . . . . . . . . . . . . . . . . . . . . . . . . . . . . . . . . . . . . . 618
11 Algebraic Expressions and Equations
11.1 Objectives . . . . . . . . . . . . . . . . . . . . . . . . . . . . . . . . . . . . . . . . . . . . . . . . . . . . . . . . . . . . . . . . . . . . . . . . . . . . . . . 629
11.2 Algebraic Expressions . . . . . . . . . . . . . . . . . . . . . . . . . . . . . . . . . . . . . . . . . . . . . . . . . . . . . . . . . . . . . . . . . . . . 630
11.3 Combining Like Terms Using Addition and Subtraction . . . . . . . . . . . . . . . . . . . . . . . . . . . . . . . . . . 636
11.4 Solving Equations of the Form x+a=b and x-a=b . . . . . . . . . . . . . . . . . . . . . . . . . . . . . . . . . . . . . . . . 639
11.5 Solving Equations of the Form ax=b and x/a=b . . . . . . . . . . . . . . . . . . . . . . . . . . . . . . . . . . . . . . . . . 647
11.6 Applications I: Translating Words to Mathematical Symbols . . . . . . . . . . . . . . . . . .. . . . . . . . . . . . 655
11.7 Applications II: Solving Problems . . . . . . . . . . . . . . . . . . . . . . . . . . . . . . . . . . . . . . . . . . . . . . . . . . . . . . . . 660
11.8 Summary of Key Concepts . . . . . . . . . . . . . . . . . . . . . . . . . . . . . . . . . . . . . . . . . . . . . . . . . . . . . . . . . . . . . . . 669
11.9 Exercise Supplement . . . . . . . . . . . . . . . . . . . . . . . . . . . . . . . . . . . . . . . . . . . . . . . . . . . . . . . . . . . . . . . . . . . . . 671
11.10 Prociency Exam . . . . . . . . . . . . . . . . . . . . . . . . . . . . . . . . . . . . . . . . . . . . . . . . . . . . . . . . . . . . . . . . . . . . . . . 678
Solutions . . . . . . . . . . . . . . . . . . . . . . . . . . . . . . . . . . . . . . . . . . . . . . . . . . . . . . . . . . . . . . . . . . . . . . . . . . . . . . . . . . . . . . . 680
Index . . . . . . . . . . . . . . . . . . . . . . . . . . . . . . . . . . . . . . . . . . . . . . . . . . . . . . . . . . . . . . . . . . . . . . . . . . . . . . . . . . . . . . . . . . . . . . . 692
Attributions . . . . . . . . . . . . . . . . . . . . . . . . . . . . . . . . . . . . . . . . . . . . . . . . . . . . . . . . . . . . . . . . . . . . . . . . . . . . . . . . . . . . . . . .697
vi
1
Preface

To the next generation of explorers: Kristi, BreAnne, Lindsey, Randi, Piper, Meghan, Wyatt, Lara, Mason,
and Sheanna.

Fundamentals of Mathematics is a work text that covers the traditional topics studied in a modern prealgebra
course, as well as the topics of estimation, elementary analytic geometry, and introductory algebra. It is
intended for students who

1. have had a previous course in prealgebra,


2. wish to meet the prerequisite of a higher level course such as elementary algebra, and
3. need to review fundamental mathematical concepts and techniques.

This text will help the student develop the insight and intuition necessary to master arithmetic techniques
and manipulative skills. It was written with the following main objectives:

1. to provide the student with an understandable and usable source of information,


2. to provide the student with the maximum opportunity to see that arithmetic concepts and techniques
are logically based,
3. to instill in the student the understanding and intuitive skills necessary to know how and when to use
particular arithmetic concepts in subsequent material, courses, and nonclassroom situations, and
4. to give the student the ability to correctly interpret arithmetically obtained results.

We have tried to meet these objectives by presenting material dynamically, much the way an instructor
might present the material visually in a classroom. (See the development of the concept of addition and
subtraction of fractions in Section 5.3, for example.) Intuition and understanding are some of the keys
to creative thinking; we believe that the material presented in this text will help the student realize that
mathematics is a creative subject.

This text can be used in standard lecture or self-paced classes. To help meet our objectives and to make
the study of prealgebra a pleasant and rewarding experience, Fundamentals of Mathematics is organized as
follows.

Pedagogical Features
The work text format gives the student space to practice mathematical skills with ready reference to sample
problems. The chapters are divided into sections, and each section is a complete treatment of a particular
topic, which includes the following features:

• Section Overview
• Sample Sets
• Practice Sets
1 This content is available online at <http://cnx.org/content/m18884/1.4/>.

1
2

• Section Exercises
• Exercises for Review
• Answers to Practice Sets

The chapters begin with Objectives and end with a Summary of Key Concepts, an Exercise Supple-
ment, and a Prociency Exam.

Objectives
Each chapter begins with a set of objectives identifying the material to be covered. Each section begins with
an overview that repeats the objectives for that particular section. Sections are divided into subsections that
correspond to the section objectives, which makes for easier reading.

Sample Sets
Fundamentals of Mathematics contains examples that are set o in boxes for easy reference. The examples
are referred to as Sample Sets for two reasons:

1. They serve as a representation to be imitated, which we believe will foster understanding of mathe-
matical concepts and provide experience with mathematical techniques.
2. Sample Sets also serve as a preliminary representation of problem-solving techniques that may be used
to solve more general and more complicated problems.

The examples have been carefully chosen to illustrate and develop concepts and techniques in the most
instructive, easily remembered way. Concepts and techniques preceding the examples are introduced at
a level below that normally used in similar texts and are thoroughly explained, assuming little previous
knowledge.

Practice Sets
A parallel Practice Set follows each Sample Set, which reinforces the concepts just learned. There is adequate
space for the student to work each problem directly on the page.

Answers to Practice Sets


The Answers to Practice Sets are given at the end of each section and can be easily located by referring to
the page number, which appears after the last Practice Set in each section.

Section Exercises
The exercises at the end of each section are graded in terms of diculty, although they are not grouped into
categories. There is an ample number of problems, and after working through the exercises, the student will
be capable of solving a variety of challenging problems.

The problems are paired so that the odd-numbered problems are equivalent in kind and diculty to the
even-numbered problems. Answers to the odd-numbered problems are provided at the back of the book.

Exercises for Review


This section consists of ve problems that form a cumulative review of the material covered in the preceding
sections of the text and is not limited to material in that chapter. The exercises are keyed by section for
easy reference. Since these exercises are intended for review only, no work space is provided.

Summary of Key Concepts


A summary of the important ideas and formulas used throughout the chapter is included at the end of
each chapter. More than just a list of terms, the summary is a valuable tool that reinforces concepts in
preparation for the Prociency Exam at the end of the chapter, as well as future exams. The summary keys
each item to the section of the text where it is discussed.
3

Exercise Supplement
In addition to numerous section exercises, each chapter includes approximately 100 supplemental problems,
which are referenced by section. Answers to the odd-numbered problems are included in the back of the
book.

Prociency Exam
Each chapter ends with a Prociency Exam that can serve as a chapter review or evaluation. The Prociency
Exam is keyed to sections, which enables the student to refer back to the text for assistance. Answers to all
the problems are included in the Answer Section at the end of the book.

Content
The writing style used in Fundamentals of Mathematics is informal and friendly, oering a straightforward
approach to prealgebra mathematics. We have made a deliberate eort not to write another text that mini-
mizes the use of words because we believe that students can best study arithmetic concepts and understand
arithmetic techniques by using words and symbols rather than symbols alone. It has been our experience
that students at the prealgebra level are not nearly experienced enough with mathematics to understand
symbolic explanations alone; they need literal explanations to guide them through the symbols.

We have taken great care to present concepts and techniques so they are understandable and easily remem-
bered. After concepts have been developed, students are warned about common pitfalls. We have tried to
make the text an information source accessible to prealgebra students.

Addition and Subtraction of Whole Numbers


This chapter includes the study of whole numbers, including a discussion of the Hindu-Arabic numeration
and the base ten number systems. Rounding whole numbers is also presented, as are the commutative and
associative properties of addition.

Multiplication and Division of Whole Numbers


The operations of multiplication and division of whole numbers are explained in this chapter. Multiplication is
described as repeated addition. Viewing multiplication in this way may provide students with a visualization
of the meaning of algebraic terms such as 8x when they start learning algebra. The chapter also includes
the commutative and associative properties of multiplication.

Exponents, Roots, and Factorizations of Whole Numbers


The concept and meaning of the word root is introduced in this chapter. A method of reading root notation
and a method of determining some common roots, both mentally and by calculator, is then presented. We
also present grouping symbols and the order of operations, prime factorization of whole numbers, and the
greatest common factor and least common multiple of a collection of whole numbers.

Introduction to Fractions and Multiplication and Division of Fractions


We recognize that fractions constitute one of the foundations of problem solving. We have, therefore, given
a detailed treatment of the operations of multiplication and division of fractions and the logic behind these
operations. We believe that the logical treatment and many practice exercises will help students retain the
information presented in this chapter and enable them to use it as a foundation for the study of rational
expressions in an algebra course.
4

Addition and Subtraction of Fractions, Comparing Fractions, and Complex


Fractions
A detailed treatment of the operations of addition and subtraction of fractions and the logic behind these
operations is given in this chapter. Again, we believe that the logical treatment and many practice exercises
will help students retain the information, thus enabling them to use it in the study of rational expressions
in an algebra course. We have tried to make explanations dynamic. A method for comparing fractions
is introduced, which gives the student another way of understanding the relationship between the words
denominator and denomination. This method serves to show the student that it is sometimes possible to
compare two dierent types of quantities. We also study a method of simplifying complex fractions and of
combining operations with fractions.

Decimals
The student is introduced to decimals in terms of the base ten number system, fractions, and digits occurring
to the right of the units position. A method of converting a fraction to a decimal is discussed. The logic
behind the standard methods of operating on decimals is presented and many examples of how to apply the
methods are given. The word of as related to the operation of multiplication is discussed. Nonterminating
divisions are examined, as are combinations of operations with decimals and fractions.

Ratios and Rates


We begin by dening and distinguishing the terms ratio and rate. The meaning of proportion and some
applications of proportion problems are described. Proportion problems are solved using the "Five-Step
Method." We hope that by using this method the student will discover the value of introducing a variable
as a rst step in problem solving and the power of organization. The chapter concludes with discussions of
percent, fractions of one percent, and some applications of percent.

Techniques of Estimation
One of the most powerful problem-solving tools is a knowledge of estimation techniques. We feel that
estimation is so important that we devote an entire chapter to its study. We examine three estimation
techniques: estimation by rounding, estimation by clustering, and estimation by rounding fractions. We also
include a section on the distributive property, an important algebraic property.

Measurement and Geometry


This chapter presents some of the techniques of measurement in both the United States system and the
metric system. Conversion from one unit to another (in a system) is examined in terms of unit fractions. A
discussion of the simplication of denominate numbers is also included. This discussion helps the student
understand more clearly the association between pure numbers and dimensions. The chapter concludes with
a study of perimeter and circumference of geometric gures and area and volume of geometric gures and
objects.

Signed Numbers
A look at algebraic concepts and techniques is begun in this chapter. Basic to the study of algebra is a working
knowledge of signed numbers. Denitions of variables, constants, and real numbers are introduced. We then
distinguish between positive and negative numbers, learn how to read signed numbers, and examine the
origin and use of the double-negative property of real numbers. The concept of absolute value is presented
both geometrically (using the number line) and algebraically. The algebraic denition is followed by an
interpretation of its meaning and several detailed examples of its use. Addition, subtraction, multiplication,
and division of signed numbers are presented rst using the number line, then with absolute value.
5

Algebraic Expressions and Equations


The student is introduced to some elementary algebraic concepts and techniques in this nal chapter. Alge-
braic expressions and the process of combining like terms are discussed in Section 11.2 and Section 11.3.
The method of combining like terms in an algebraic expression is explained by using the interpretation of
multiplication as a description of repeated addition (as in Section 2.1).
6
2
Acknowledgements

Many extraordinarily talented people are responsible for helping to create this text. We wish to acknowledge
the eorts and skill of the following mathematicians. Their contributions have been invaluable.

Barbara Conway, Berkshire Community College


Bill Hajdukiewicz, Miami-Dade Community College
Virginia Hamilton, Shawnee State University
David Hares, El Centro College
Norman Lee, Ball State University
Ginger Y. Manchester, Hinds Junior College
John R. Martin, Tarrant County Junior College
Shelba Mormon, Northlake College
Lou Ann Pate, Pima Community College
Gus Pekara, Oklahoma City Community College
David Price, Tarrant County Junior College
David Schultz, Virginia Western Community College
Sue S. Watkins, Lorain County Community College
Elizabeth M. Wayt, Tennessee State University
Prentice E. Whitlock, Jersey City State College
Thomas E. Williamson, Montclair State College

Special thanks to the following individuals for their careful accuracy reviews of manuscript, galleys, and
page proofs: Steve Blasberg, West Valley College; Wade Ellis, Sr., University of Michigan; John R. Martin,
Tarrant County Junior College; and Jane Ellis. We would also like to thank Amy Miller and Guy Sanders,
Branham High School.

Our sincere thanks to Debbie Wiedemann for her encouragement, suggestions concerning psychobiological
examples, proofreading much of the manuscript, and typing many of the section exercises; Sandi Wiedemann
for collating the annotated reviews, counting the examples and exercises, and untiring use of "white-out";
and Jane Ellis for solving and typing all of the exercise solutions.

We thank the following people for their excellent work on the various ancillary items that accompany Fun-
damentals of Mathematics : Steve Blasberg, West Valley College; Wade Ellis, Sr., University of Michigan;
and Jane Ellis ( Instructor's Manual); John R. Martin, Tarrant County Junior College (Student Solutions
Manual and Study Guide); Virginia Hamilton, Shawnee State University (Computerized Test Bank); Pa-
tricia Morgan, San Diego State University (Prepared Tests); and George W. Bergeman, Northern Virginia
Community College (Maxis Interactive Software).

We also thank the talented people at Saunders College Publishing whose eorts made this text run smoothly
and less painfully than we had imagined. Our particular thanks to Bob Stern, Mathematics Editor, Ellen
Newman, Developmental Editor, and Janet Nuciforo, Project Editor. Their guidance, suggestions, open
2 This content is available online at <http://cnx.org/content/m34775/1.2/>.

7
8

minds to our suggestions and concerns, and encouragement have been extraordinarily helpful. Although
there were times we thought we might be permanently damaged from rereading and rewriting, their eorts
have improved this text immensely. It is a pleasure to work with such high-quality professionals.

Denny Burzynski
Wade Ellis, Jr.
San Jose, California
December 1988

I would like to thank Doug Campbell, Ed Lodi, and Guy Sanders for listening to my frustrations and
encouraging me on. Thanks also go to my cousin, David Raety, who long ago in Sequoia National Forest
told me what a dierential equation is.

Particular thanks go to each of my colleagues at West Valley College. Our everyday conversations regarding
mathematics instruction have been of the utmost importance to the development of this text and to my
teaching career.

D.B.
Chapter 1

Addition and Subtraction of Whole

Numbers

1.1 Objectives1
After completing this chapter, you should
Whole Numbers (Section 1.2)

• know the dierence between numbers and numerals


• know why our number system is called the Hindu-Arabic numeration system
• understand the base ten positional number system
• be able to identify and graph whole numbers

Reading and Writing Whole Numbers (Section 1.3)

• be able to read and write a whole number

Rounding Whole Numbers (Section 1.4)

• understand that rounding is a method of approximation


• be able to round a whole number to a specied position

Addition of Whole Numbers (Section 1.5)

• understand the addition process


• be able to add whole numbers
• be able to use the calculator to add one whole number to another

Subtraction of Whole Numbers (Section 1.6)

• understand the subtraction process


• be able to subtract whole numbers
• be able to use a calculator to subtract one whole number from another whole number

Properties of Addition (Section 1.7)

• understand the commutative and associative properties of addition


• understand why 0 is the additive identity

1 This content is available online at <http://cnx.org/content/m18885/1.4/>.

9
10 CHAPTER 1. ADDITION AND SUBTRACTION OF WHOLE NUMBERS

1.2 Whole Numbers2


1.2.1 Section Overview
• Numbers and Numerals
• The Hindu-Arabic Numeration System
• The Base Ten Positional Number System
• Whole Numbers
• Graphing Whole Numbers

1.2.2 Numbers and Numerals


We begin our study of introductory mathematics by examining its most basic building block, the number.

Number
A number is a concept. It exists only in the mind.

The earliest concept of a number was a thought that allowed people to mentally picture the size of some
collection of objects. To write down the number being conceptualized, a numeral is used.

Numeral
A numeral is a symbol that represents a number.

In common usage today we do not distinguish between a number and a numeral. In our study of introductory
mathematics, we will follow this common usage.

1.2.2.1 Sample Set A

The following are numerals. In each case, the rst represents the number four, the second represents the num-
ber one hundred twenty-three, and the third, the number one thousand ve. These numbers are represented
in dierent ways.

• Hindu-Arabic numerals
4, 123, 1005
• Roman numerals
IV, CXXIII, MV
• Egyptian numerals

1.2.2.2 Practice Set A

Exercise 1.1 (Solution on p. 76.)


Do the phrases "four," "one hundred twenty-three," and "one thousand ve" qualify as numerals?
Yes or no?

2 This content is available online at <http://cnx.org/content/m34795/1.4/>.


11

1.2.3 The Hindu-Arabic Numeration System


Hindu-Arabic Numeration System
Our society uses the Hindu-Arabic numeration system. This system of numeration began shortly before
the third century when the Hindus invented the numerals

0123456789

Leonardo Fibonacci
About a thousand years later, in the thirteenth century, a mathematician named Leonardo Fibonacci of Pisa
introduced the system into Europe. It was then popularized by the Arabs. Thus, the name, Hindu-Arabic
numeration system.

1.2.4 The Base Ten Positional Number System


Digits
The Hindu-Arabic numerals 0 1 2 3 4 5 6 7 8 9 are called digits. We can form any number in the number
system by selecting one or more digits and placing them in certain positions. Each position has a particular
value. The Hindu mathematician who devised the system about A.D. 500 stated that "from place to place
each is ten times the preceding."

Base Ten Positional Systems


It is for this reason that our number system is called a positional number system with base ten.

Commas
When numbers are composed of more than three digits, commas are sometimes used to separate the digits
into groups of three.

Periods
These groups of three are called periods and they greatly simplify reading numbers.

In the Hindu-Arabic numeration system, a period has a value assigned to each or its three positions, and
the values are the same for each period. The position values are

Thus, each period contains a position for the values of one, ten, and hundred. Notice that, in looking from
right to left, the value of each position is ten times the preceding. Each period has a particular name.

As we continue from right to left, there are more periods. The ve periods listed above are the most common,
and in our study of introductory mathematics, they are sucient.

The following diagram illustrates our positional number system to trillions. (There are, to be sure, other
periods.)
12 CHAPTER 1. ADDITION AND SUBTRACTION OF WHOLE NUMBERS

In our positional number system, the value of a digit is determined by its position in the number.

1.2.4.1 Sample Set B

Example 1.1
Find the value of 6 in the number 7,261.

Since 6 is in the tens position of the units period, its value is 6 tens.

6 tens = 60
Example 1.2
Find the value of 9 in the number 86,932,106,005.

Since 9 is in the hundreds position of the millions period, its value is 9 hundred millions.

9 hundred millions = 9 hundred million


Example 1.3
Find the value of 2 in the number 102,001.

Since 2 is in the ones position of the thousands period, its value is 2 one thousands.

2 one thousands = 2 thousand

1.2.4.2 Practice Set B

Exercise 1.2 (Solution on p. 76.)


Find the value of 5 in the number 65,000.
Exercise 1.3 (Solution on p. 76.)
Find the value of 4 in the number 439,997,007,010.
Exercise 1.4 (Solution on p. 76.)
Find the value of 0 in the number 108.

1.2.5 Whole Numbers


Whole Numbers
Numbers that are formed using only the digits
0123456789
are called whole numbers. They are
0, 1, 2, 3, 4, 5, 6, 7, 8, 9, 10, 11, 12, 13, 14, 15, . . .

The three dots at the end mean "and so on in this same pattern."
13

1.2.6 Graphing Whole Numbers


Number Line
Whole numbers may be visualized by constructing a number line. To construct a number line, we simply
draw a straight line and choose any point on the line and label it 0.

Origin
This point is called the origin. We then choose some convenient length, and moving to the right, mark o
consecutive intervals (parts) along the line starting at 0. We label each new interval endpoint with the next
whole number.

Graphing
We can visually display a whole number by drawing a closed circle at the point labeled with that whole
number. Another phrase for visually displaying a whole number is graphing the whole number. The word
graph means to "visually display."

1.2.6.1 Sample Set C

Example 1.4
Graph the following whole numbers: 3, 5, 9.

Example 1.5
Specify the whole numbers that are graphed on the following number line. The break in the number
line indicates that we are aware of the whole numbers between 0 and 106, and 107 and 872, but we
are not listing them due to space limitations.

The numbers that have been graphed are

0, 106, 873, 874

1.2.6.2 Practice Set C

Exercise 1.5 (Solution on p. 76.)


Graph the following whole numbers: 46, 47, 48, 325, 327.

Exercise 1.6 (Solution on p. 76.)


Specify the whole numbers that are graphed on the following number line.

A line is composed of an endless number of points. Notice that we have labeled only some of them. As we
proceed, we will discover new types of numbers and determine their location on the number line.
14 CHAPTER 1. ADDITION AND SUBTRACTION OF WHOLE NUMBERS

1.2.7 Exercises
Exercise 1.7 (Solution on p. 76.)
What is a number?
Exercise 1.8
What is a numeral?
Exercise 1.9 (Solution on p. 76.)
Does the word "eleven" qualify as a numeral?
Exercise 1.10
How many dierent digits are there?
Exercise 1.11 (Solution on p. 76.)
Our number system, the Hindu-Arabic number system, is a number system with
base .
Exercise 1.12
Numbers composed of more than three digits are sometimes separated into groups of three by
commas. These groups of three are called .
Exercise 1.13 (Solution on p. 76.)
In our number system, each period has three values assigned to it. These values are the same for
each period. From right to left, what are they?
Exercise 1.14
Each period has its own particular name. From right to left, what are the names of the rst four?
Exercise 1.15 (Solution on p. 76.)
In the number 841, how many tens are there?
Exercise 1.16
In the number 3,392, how many ones are there?
Exercise 1.17 (Solution on p. 76.)
In the number 10,046, how many thousands are there?
Exercise 1.18
In the number 779,844,205, how many ten millions are there?
Exercise 1.19 (Solution on p. 76.)
In the number 65,021, how many hundred thousands are there?
For following problems, give the value of the indicated digit in the given number.
Exercise 1.20
5 in 599
Exercise 1.21 (Solution on p. 76.)
1 in 310,406
Exercise 1.22
9 in 29,827
Exercise 1.23 (Solution on p. 76.)
6 in 52,561,001,100
Exercise 1.24
Write a two-digit number that has an eight in the tens position.
Exercise 1.25 (Solution on p. 76.)
Write a four-digit number that has a one in the thousands position and a zero in the ones position.
Exercise 1.26
How many two-digit whole numbers are there?
15

Exercise 1.27 (Solution on p. 76.)


How many three-digit whole numbers are there?
Exercise 1.28
How many four-digit whole numbers are there?
Exercise 1.29 (Solution on p. 76.)
Is there a smallest whole number? If so, what is it?
Exercise 1.30
Is there a largest whole number? If so, what is it?
Exercise 1.31 (Solution on p. 76.)
Another term for "visually displaying" is .
Exercise 1.32
The whole numbers can be visually displayed on a .
Exercise 1.33 (Solution on p. 76.)
Graph (visually display) the following whole numbers on the number line below: 0, 1, 31, 34.

Exercise 1.34
Construct a number line in the space provided below and graph (visually display) the following
whole numbers: 84, 85, 901, 1006, 1007.
Exercise 1.35 (Solution on p. 76.)
Specify, if any, the whole numbers that are graphed on the following number line.

Exercise 1.36
Specify, if any, the whole numbers that are graphed on the following number line.

1.3 Reading and Writing Whole Numbers3


1.3.1 Section Overview
• Reading Whole Numbers
• Writing Whole Numbers

Because our number system is a positional number system, reading and writing whole numbers is quite
simple.
3 This content is available online at <http://cnx.org/content/m34778/1.5/>.
16 CHAPTER 1. ADDITION AND SUBTRACTION OF WHOLE NUMBERS

1.3.2 Reading Whole Numbers


To convert a number that is formed by digits into a verbal phrase, use the following method:

1. Beginning at the right and working right to left, separate the number into distinct periods by inserting
commas every three digits.
2. Beginning at the left, read each period individually, saying the period name.

1.3.2.1 Sample Set A

Write the following numbers as words.


Example 1.6
Read 42958.

1. Beginning at the right, we can separate this number into distinct periods by inserting a comma
between the 2 and 9.
42,958
2. Beginning at the left, we read each period individually:

Forty-two thousand, nine hundred fty-eight.

Example 1.7
Read 307991343.

1. Beginning at the right, we can separate this number into distinct periods by placing commas
between the 1 and 3 and the 7 and 9.
307,991,343
2. Beginning at the left, we read each period individually.

Three hundred seven million, nine hundred ninety-one thousand, three hundred forty-three.

Example 1.8
Read 36000000000001.

1. Beginning at the right, we can separate this number into distinct periods by placing commas.
36,000,000,001
17

2. Beginning at the left, we read each period individually.

Thirty-six trillion, one.

1.3.2.2 Practice Set A

Write each number in words.


Exercise 1.37 (Solution on p. 76.)
12,542
Exercise 1.38 (Solution on p. 76.)
101,074,003
Exercise 1.39 (Solution on p. 76.)
1,000,008

1.3.3 Writing Whole Numbers


To express a number in digits that is expressed in words, use the following method:

1. Notice rst that a number expressed as a verbal phrase will have its periods set o by commas.
2. Starting at the beginning of the phrase, write each period of numbers individually.
3. Using commas to separate periods, combine the periods to form one number.

1.3.3.1 Sample Set B

Write each number using digits.


Example 1.9
Seven thousand, ninety-two.

Using the comma as a period separator, we have


18 CHAPTER 1. ADDITION AND SUBTRACTION OF WHOLE NUMBERS

7,092
Example 1.10
Fifty billion, one million, two hundred thousand, fourteen.

Using the commas as period separators, we have

50,001,200,014
Example 1.11
Ten million, ve hundred twelve.

The comma sets o the periods. We notice that there is no thousands period. We'll have to insert
this ourselves.

10,000,512

1.3.3.2 Practice Set B

Express each number using digits.


Exercise 1.40 (Solution on p. 77.)
One hundred three thousand, twenty-ve.
Exercise 1.41 (Solution on p. 77.)
Six million, forty thousand, seven.
Exercise 1.42 (Solution on p. 77.)
Twenty trillion, three billion, eighty million, one hundred nine thousand, four hundred two.
Exercise 1.43 (Solution on p. 77.)
Eighty billion, thirty-ve.

1.3.4 Exercises
For the following problems, write all numbers in words.
Exercise 1.44 (Solution on p. 77.)
912
Exercise 1.45
84
19

Exercise 1.46 (Solution on p. 77.)


1491
Exercise 1.47
8601
Exercise 1.48 (Solution on p. 77.)
35,223
Exercise 1.49
71,006
Exercise 1.50 (Solution on p. 77.)
437,105
Exercise 1.51
201,040
Exercise 1.52 (Solution on p. 77.)
8,001,001
Exercise 1.53
16,000,053
Exercise 1.54 (Solution on p. 77.)
770,311,101
Exercise 1.55
83,000,000,007
Exercise 1.56 (Solution on p. 77.)
106,100,001,010
Exercise 1.57
3,333,444,777
Exercise 1.58 (Solution on p. 77.)
800,000,800,000
Exercise 1.59
A particular community college has 12,471 students enrolled.
Exercise 1.60 (Solution on p. 77.)
A person who watches 4 hours of television a day spends 1460 hours a year watching T.V.
Exercise 1.61
Astronomers believe that the age of the earth is about 4,500,000,000 years.
Exercise 1.62 (Solution on p. 77.)
Astronomers believe that the age of the universe is about 20,000,000,000 years.
Exercise 1.63
There are 9690 ways to choose four objects from a collection of 20.
Exercise 1.64 (Solution on p. 77.)
If a 412 page book has about 52 sentences per page, it will contain about 21,424 sentences.
Exercise 1.65
In 1980, in the United States, there was $1,761,000,000,000 invested in life insurance.
Exercise 1.66 (Solution on p. 77.)
In 1979, there were 85,000 telephones in Alaska and 2,905,000 telephones in Indiana.
Exercise 1.67
In 1975, in the United States, it is estimated that 52,294,000 people drove to work alone.
Exercise 1.68 (Solution on p. 77.)
In 1980, there were 217 prisoners under death sentence that were divorced.
20 CHAPTER 1. ADDITION AND SUBTRACTION OF WHOLE NUMBERS

Exercise 1.69
In 1979, the amount of money spent in the United States for regular-session college education was
$50,721,000,000,000.
Exercise 1.70 (Solution on p. 77.)
In 1981, there were 1,956,000 students majoring in business in U.S. colleges.
Exercise 1.71
In 1980, the average fee for initial and follow up visits to a medical doctors oce was about $34.
Exercise 1.72 (Solution on p. 77.)
In 1980, there were approximately 13,100 smugglers of aliens apprehended by the Immigration
border patrol.
Exercise 1.73
In 1980, the state of West Virginia pumped 2,000,000 barrels of crude oil, whereas Texas pumped
975,000,000 barrels.
Exercise 1.74 (Solution on p. 77.)
The 1981 population of Uganda was 12,630,000 people.
Exercise 1.75
In 1981, the average monthly salary oered to a person with a Master's degree in mathematics
was $1,685.
For the following problems, write each number using digits.
Exercise 1.76 (Solution on p. 77.)
Six hundred eighty-one
Exercise 1.77
Four hundred ninety
Exercise 1.78 (Solution on p. 77.)
Seven thousand, two hundred one
Exercise 1.79
Nineteen thousand, sixty-ve
Exercise 1.80 (Solution on p. 77.)
Five hundred twelve thousand, three
Exercise 1.81
Two million, one hundred thirty-three thousand, eight hundred fty-nine
Exercise 1.82 (Solution on p. 77.)
Thirty-ve million, seven thousand, one hundred one
Exercise 1.83
One hundred million, one thousand
Exercise 1.84 (Solution on p. 77.)
Sixteen billion, fty-nine thousand, four
Exercise 1.85
Nine hundred twenty billion, four hundred seventeen million, twenty-one thousand
Exercise 1.86 (Solution on p. 78.)
Twenty-three billion
Exercise 1.87
Fifteen trillion, four billion, nineteen thousand, three hundred ve
Exercise 1.88 (Solution on p. 78.)
One hundred trillion, one
21

1.3.4.1 Exercises for Review

Exercise 1.89
(Section 1.2) How many digits are there?
Exercise 1.90 (Solution on p. 78.)
(Section 1.2) In the number 6,641, how many tens are there?
Exercise 1.91
(Section 1.2) What is the value of 7 in 44,763?
Exercise 1.92 (Solution on p. 78.)
(Section 1.2) Is there a smallest whole number? If so, what is it?
Exercise 1.93
(Section 1.2) Write a four-digit number with a 9 in the tens position.

1.4 Rounding Whole Numbers4


1.4.1 Section Overview
• Rounding as an Approximation
• The Method of Rounding Numbers

1.4.2 Rounding as an Approximation


A primary use of whole numbers is to keep count of how many objects there are in a collection. Sometimes
we're only interested in the approximate number of objects in the collection rather than the precise number.
For example, there are approximately 20 symbols in the collection below.

The precise number of symbols in the above collection is 18.

Rounding
We often approximate the number of objects in a collection by mentally seeing the collection as occurring
in groups of tens, hundreds, thousands, etc. This process of approximation is called rounding. Rounding
is very useful in estimation. We will study estimation in Chapter 8.

When we think of a collection as occurring in groups of tens, we say we're rounding to the nearest ten. When
we think of a collection as occurring in groups of hundreds, we say we're rounding to the nearest hundred.
This idea of rounding continues through thousands, ten thousands, hundred thousands, millions, etc.

The process of rounding whole numbers is illustrated in the following examples.


Example 1.12
Round 67 to the nearest ten.

4 This content is available online at <http://cnx.org/content/m34780/1.3/>.


22 CHAPTER 1. ADDITION AND SUBTRACTION OF WHOLE NUMBERS

On the number line, 67 is more than halfway from 60 to 70. The digit immediately to the right of
the tens digit, the round-o digit, is the indicator for this.

Thus, 67, rounded to the near-


est ten, is 70.
Example 1.13
Round 4,329 to the nearest hundred.

On the number line, 4,329 is less than halfway from 4,300 to 4,400. The digit to the immediate
right of the hundreds digit, the round-o digit, is the indicator.

Thus, 4,329, rounded to the


nearest hundred is 4,300.
Example 1.14
Round 16,500 to the nearest thousand.

On the number line, 16,500 is exactly halfway from 16,000 to 17,000.

By convention, when the number to be rounded is exactly halfway between two numbers, it is
rounded to the higher number.

Thus, 16,500, rounded to the nearest thousand, is 17,000.


Example 1.15
A person whose salary is $41,450 per year might tell a friend that she makes $41,000 per year. She
has rounded 41,450 to the nearest thousand. The number 41,450 is closer to 41,000 than it is to
42,000.

1.4.3 The Method of Rounding Whole Numbers


From the observations made in the preceding examples, we can use the following method to round a whole
number to a particular position.

1. Mark the position of the round-o digit.


2. Note the digit to the immediate right of the round-o digit.
23

a. If it is less than 5, replace it and all the digits to its right with zeros. Leave the round-o digit
unchanged.
b. If it is 5 or larger, replace it and all the digits to its right with zeros. Increase the round-o digit
by 1.

1.4.3.1 Sample Set A

Use the method of rounding whole numbers to solve the following problems.
Example 1.16
Round 3,426 to the nearest ten.

1. We are rounding to the tens position. Mark the digit in the tens position

2. Observe the digit immediately to the right of the tens position. It is 6. Since 6 is greater
than 5, we round up by replacing 6 with 0 and adding 1 to the digit in the tens position (the
round-o position): 2 + 1 = 3 .
3,430

Thus, 3,426 rounded to the nearest ten is 3,430.


Example 1.17
Round 9,614,018,007 to the nearest ten million.

1. We are rounding to the nearest ten million.

2. Observe the digit immediately to the right of the ten millions position. It is 4. Since 4 is less
than 5, we round down by replacing 4 and all the digits to its right with zeros.
9,610,000,000

Thus, 9,614,018,007 rounded to the nearest ten million is 9,610,000,000.


Example 1.18
Round 148,422 to the nearest million.

1. Since we are rounding to the nearest million, we'll have to imagine a digit in the millions
position. We'll write 148,422 as 0,148,422.

2. The digit immediately to the right is 1. Since 1 is less than 5, we'll round down by replacing
it and all the digits to its right with zeros.
0,000,000
This number is 0.
24 CHAPTER 1. ADDITION AND SUBTRACTION OF WHOLE NUMBERS

Thus, 148,422 rounded to the nearest million is 0.


Example 1.19
Round 397,000 to the nearest ten thousand.

1. We are rounding to the nearest ten thousand.

2. The digit immediately to the right of the ten thousand position is 7. Since 7 is greater than
5, we round up by replacing 7 and all the digits to its right with zeros and adding 1 to the
digit in the ten thousands position. But 9 + 1 = 10 and we must carry the 1 to the next (the
hundred thousands) position.
400,000

Thus, 397,000 rounded to the nearest ten thousand is 400,000.

1.4.3.2 Practice Set A

Use the method of rounding whole numbers to solve each problem.


Exercise 1.94 (Solution on p. 78.)
Round 3387 to the nearest hundred.
Exercise 1.95 (Solution on p. 78.)
Round 26,515 to the nearest thousand.
Exercise 1.96 (Solution on p. 78.)
Round 30,852,900 to the nearest million.
Exercise 1.97 (Solution on p. 78.)
Round 39 to the nearest hundred.
Exercise 1.98 (Solution on p. 78.)
Round 59,600 to the nearest thousand.

1.4.4 Exercises
For the following problems, complete the table by rounding each number to the indicated positions.
Exercise 1.99 (Solution on p. 78.)
1,642

hundred thousand ten thousand million

Table 1.1

Exercise 1.100
5,221
25

hundred thousand ten thousand million

Table 1.2

Exercise 1.101 (Solution on p. 78.)


91,803

Hundred thousand ten thousand million

Table 1.3

Exercise 1.102
106,007

hundred thousand ten thousand million

Table 1.4

Exercise 1.103 (Solution on p. 78.)


208

hundred thousand ten thousand million

Table 1.5

Exercise 1.104
199

hundred thousand ten thousand million

Table 1.6

Exercise 1.105 (Solution on p. 78.)


863

hundred thousand ten thousand million

Table 1.7
26 CHAPTER 1. ADDITION AND SUBTRACTION OF WHOLE NUMBERS

Exercise 1.106
794

hundred thousand ten thousand million

Table 1.8

Exercise 1.107 (Solution on p. 79.)


925

hundred thousand ten thousand million

Table 1.9

Exercise 1.108
909

hundred thousand ten thousand million

Table 1.10

Exercise 1.109 (Solution on p. 79.)


981

hundred thousand ten thousand million

Table 1.11

Exercise 1.110
965

hundred thousand ten thousand million

Table 1.12

Exercise 1.111 (Solution on p. 79.)


551,061,285

hundred thousand ten thousand million


27

Table 1.13

Exercise 1.112
23,047,991,521

hundred thousand ten thousand million

Table 1.14

Exercise 1.113 (Solution on p. 79.)


106,999,413,206

Hundred thousand ten thousand million

Table 1.15

Exercise 1.114
5,000,000

hundred thousand ten thousand million

Table 1.16

Exercise 1.115 (Solution on p. 79.)


8,006,001

hundred thousand ten thousand million

Table 1.17

Exercise 1.116
94,312

hundred thousand ten thousand million

Table 1.18

Exercise 1.117 (Solution on p. 79.)


33,486
28 CHAPTER 1. ADDITION AND SUBTRACTION OF WHOLE NUMBERS

hundred thousand ten thousand million

Table 1.19

Exercise 1.118
560,669

hundred thousand ten thousand million

Table 1.20

Exercise 1.119 (Solution on p. 80.)


388,551

hundred thousand ten thousand million

Table 1.21

Exercise 1.120
4,752

hundred thousand ten thousand million

Table 1.22

Exercise 1.121 (Solution on p. 80.)


8,209

hundred thousand ten thousand million

Table 1.23

Exercise 1.122
In 1950, there were 5,796 cases of diphtheria reported in the United States. Round to the nearest
hundred.
Exercise 1.123 (Solution on p. 80.)
In 1979, 19,309,000 people in the United States received federal food stamps. Round to the nearest
ten thousand.
Exercise 1.124
In 1980, there were 1,105,000 people between 30 and 34 years old enrolled in school. Round to the
nearest million.
29

Exercise 1.125 (Solution on p. 80.)


In 1980, there were 29,100,000 reports of aggravated assaults in the United States. Round to the
nearest million.
For the following problems, round the numbers to the position you think is most reasonable for the situation.
Exercise 1.126
In 1980, for a city of one million or more, the average annual salary of police and reghters was
$16,096.
Exercise 1.127 (Solution on p. 80.)
The average percentage of possible sunshine in San Francisco, California, in June is 73%.
Exercise 1.128
In 1980, in the state of Connecticut, $3,777,000,000 in defense contract payroll was awarded.
Exercise 1.129 (Solution on p. 80.)
In 1980, the federal government paid $5,463,000,000 to Viet Nam veterans and dependants.
Exercise 1.130
In 1980, there were 3,377,000 salespeople employed in the United States.
Exercise 1.131 (Solution on p. 80.)
In 1948, in New Hampshire, 231,000 popular votes were cast for the president.
Exercise 1.132
In 1970, the world production of cigarettes was 2,688,000,000,000.
Exercise 1.133 (Solution on p. 80.)
In 1979, the total number of motor vehicle registrations in Florida was 5,395,000.
Exercise 1.134
In 1980, there were 1,302,000 registered nurses the United States.

1.4.4.1 Exercises for Review

Exercise 1.135 (Solution on p. 80.)


(Section 1.2) There is a term that describes the visual displaying of a number. What is the term?
Exercise 1.136
(Section 1.2) What is the value of 5 in 26,518,206?
Exercise 1.137 (Solution on p. 80.)
(Section 1.3) Write 42,109 as you would read it.
Exercise 1.138
(Section 1.3) Write "six hundred twelve" using digits.
Exercise 1.139 (Solution on p. 80.)
(Section 1.3) Write "four billion eight" using digits.

1.5 Addition of Whole Numbers5


1.5.1 Section Overview
• Addition
• Addition Visualized on the Number Line
• The Addition Process
5 This content is available online at <http://cnx.org/content/m34786/1.4/>.
30 CHAPTER 1. ADDITION AND SUBTRACTION OF WHOLE NUMBERS

• Addition Involving Carrying


• Calculators

1.5.2 Addition
Suppose we have two collections of objects that we combine together to form a third collection. For example,

We are combining a collection of four objects with a collection of three objects to obtain a collection of seven
objects.

Addition
The process of combining two or more objects (real or intuitive) to form a third, the total, is called addition.

In addition, the numbers being added are called addends or terms, and the total is called the sum. The
plus symbol (+) is used to indicate addition, and the equal symbol (=) is used to represent the word
"equal." For example, 4 + 3 = 7 means "four added to three equals seven."

1.5.3 Addition Visualized on the Number Line


Addition is easily visualized on the number line. Let's visualize the addition of 4 and 3 using the number
line.

To nd 4 + 3,

1. Start at 0.
2. Move to the right 4 units. We are now located at 4.
3. From 4, move to the right 3 units. We are now located at 7.

Thus, 4 + 3 = 7.

1.5.4 The Addition Process


We'll study the process of addition by considering the sum of 25 and 43.

25
means
+43

We write this as 68.

We can suggest the following procedure for adding whole numbers using this example.
Example 1.20: The Process of Adding Whole Numbers
To add whole numbers,

The process:
31

1. Write the numbers vertically, placing corresponding positions in the same column.
25
+43
2. Add the digits in each column. Start at the right (in the ones position) and move to the left,
placing the sum at the bottom.
25
+43
68

Caution: Confusion and incorrect sums can occur when the numbers are not aligned in columns
properly. Avoid writing such additions as

25
+43
25
+43

1.5.4.1 Sample Set A

Example 1.21
Add 276 and 103.

276 6+3=9.
+103 7+0=7.
379 2+1=3.
Example 1.22
Add 1459 and 130

9+0=9.
1459
5+3=8.
+130
4+1=5.
1589
1+0=1.
In each of these examples, each individual sum does not exceed 9. We will examine individual sums that
exceed 9 in the next section.

1.5.4.2 Practice Set A

Perform each addition. Show the expanded form in problems 1 and 2.


Exercise 1.140 (Solution on p. 80.)
Add 63 and 25.
Exercise 1.141 (Solution on p. 80.)
Add 4,026 and 1,501.
Exercise 1.142 (Solution on p. 80.)
Add 231,045 and 36,121.
32 CHAPTER 1. ADDITION AND SUBTRACTION OF WHOLE NUMBERS

1.5.5 Addition Involving Carrying


It often happens in addition that the sum of the digits in a column will exceed 9. This happens when we
add 18 and 34. We show this in expanded form as follows.

Notice that when we add the 8 ones to the 4 ones we get 12 ones. We then convert the 12 ones to 1 ten and
2 ones. In vertical addition, we show this conversion by carrying the ten to the tens column. We write a 1
at the top of the tens column to indicate the carry. This same example is shown in a shorter form as follows:

8 + 4 = 12 Write 2, carry 1 ten to the top of the next column to the left.

1.5.5.1 Sample Set B

Perform the following additions. Use the process of carrying when needed.
Example 1.23
Add 1875 and 358.

5 + 8 = 13 Write 3, carry 1 ten.


1 + 7 + 5 = 13 Write 3, carry 1 hundred.
1 + 8 + 3 = 12 Write 2, carry 1 thousand.
1+1=2

The sum is 2233.


Example 1.24
Add 89,208 and 4,946.
33

8 + 6 = 14 Write 4, carry 1 ten.


1+0+4=5 Write the 5 (nothing to carry).
2 + 9 = 11 Write 1, carry one thousand.
1 + 9 + 4 = 14 Write 4, carry one ten thousand.
1+8=9

The sum is 94,154.


Example 1.25
Add 38 and 95.

8 + 5 = 13 Write 3, carry 1 ten.


1 + 3 + 9 = 13 Write 3, carry 1 hundred.
1+0=1

As you proceed with the addition, it is a good idea to keep in mind what is actually happening.

The sum is 133.


Example 1.26
Find the sum 2648, 1359, and 861.

8 + 9 + 1 = 18 Write 8, carry 1 ten.


1 + 4 + 5 + 6 = 16 Write 6, carry 1 hundred.
1 + 6 + 3 + 8 = 18 Write 8, carry 1 thousand.
1+2+1=4

The sum is 4,868.


Numbers other than 1 can be carried as illustrated in Example 1.27.
Example 1.27
Find the sum of the following numbers.
34 CHAPTER 1. ADDITION AND SUBTRACTION OF WHOLE NUMBERS

6 + 5 + 1 + 7 = 19 Write 9, carry the 1.


1+1+0+5+1=8 Write 8.
0 + 9 + 9 + 8 = 26 Write 6, carry the 2.
2 + 8 + 9 + 8 + 6 = 33 Write 3, carry the 3.
3 + 7 + 3 + 5 = 18 Write 8, carry the 1.
1+8=9 Write 9.

The sum is 983,689.


Example 1.28
The number of students enrolled at Riemann College in the years 1984, 1985, 1986, and 1987 was
10,406, 9,289, 10,108, and 11,412, respectively. What was the total number of students enrolled at
Riemann College in the years 1985, 1986, and 1987?

We can determine the total number of students enrolled by adding 9,289, 10,108, and 11,412, the
number of students enrolled in the years 1985, 1986, and 1987.

The total number of students enrolled at Riemann College in the years 1985, 1986, and 1987 was
30,809.

1.5.5.2 Practice Set B

Perform each addition. For the next three problems, show the expanded form.
Exercise 1.143 (Solution on p. 80.)
Add 58 and 29.
Exercise 1.144 (Solution on p. 81.)
Add 476 and 85.
Exercise 1.145 (Solution on p. 81.)
Add 27 and 88.
Exercise 1.146 (Solution on p. 81.)
Add 67,898 and 85,627.
For the next three problems, nd the sums.
Exercise 1.147 (Solution on p. 81.)
57
26
84
35

Exercise 1.148 (Solution on p. 81.)


847
825
796
Exercise 1.149 (Solution on p. 81.)
16, 945
8, 472
387, 721
21, 059
629

1.5.6 Calculators
Calculators provide a very simple and quick way to nd sums of whole numbers. For the two problems in
Sample Set C, assume the use of a calculator that does not require the use of an ENTER key (such as many
Hewlett-Packard calculators).

1.5.6.1 Sample Set C

Use a calculator to nd each sum.


Example 1.29

34 + 21 Display Reads
Type 34 34
Press + 34
Type 21 21
Press = 55

Table 1.24

The sum is 55.


Example 1.30
36 CHAPTER 1. ADDITION AND SUBTRACTION OF WHOLE NUMBERS

106 + 85 + 322 + 406 Display Reads


Type 106 106 The calculator keeps a running subtotal
Press + 106
Type 85 85
Press = 191 ← 106 + 85
Type 322 322
Press + 513 ← 191 + 322
Type 406 406
Press = 919 ← 513 + 406

Table 1.25

The sum is 919.

1.5.6.2 Practice Set C

Use a calculator to nd the following sums.


Exercise 1.150 (Solution on p. 81.)
62 + 81 + 12
Exercise 1.151 (Solution on p. 81.)
9, 261 + 8, 543 + 884 + 1, 062
Exercise 1.152 (Solution on p. 81.)
10, 221 + 9, 016 + 11, 445

1.5.7 Exercises
For the following problems, perform the additions. If you can, check each sum with a calculator.
Exercise 1.153 (Solution on p. 81.)
14 + 5
Exercise 1.154
12 + 7
Exercise 1.155 (Solution on p. 81.)
46 + 2
Exercise 1.156
83 + 16
Exercise 1.157 (Solution on p. 81.)
77 + 21
Exercise 1.158
321
+ 42
37

Exercise 1.159 (Solution on p. 82.)


916
+ 62
Exercise 1.160
104
+561
Exercise 1.161 (Solution on p. 82.)
265
+103
Exercise 1.162
552 + 237
Exercise 1.163 (Solution on p. 82.)
8, 521 + 4, 256
Exercise 1.164
16, 408
+ 3, 101
Exercise 1.165 (Solution on p. 82.)
16, 515
+42, 223
Exercise 1.166
616, 702 + 101, 161
Exercise 1.167 (Solution on p. 82.)
43, 156, 219 + 2, 013, 520
Exercise 1.168
17 + 6
Exercise 1.169 (Solution on p. 82.)
25 + 8
Exercise 1.170
84
+ 7
Exercise 1.171 (Solution on p. 82.)
75
+ 6
Exercise 1.172
36 + 48
Exercise 1.173 (Solution on p. 82.)
74 + 17
Exercise 1.174
486 + 58
Exercise 1.175 (Solution on p. 82.)
743 + 66
Exercise 1.176
381 + 88
38 CHAPTER 1. ADDITION AND SUBTRACTION OF WHOLE NUMBERS

Exercise 1.177 (Solution on p. 82.)


687
+175
Exercise 1.178
931
+853
Exercise 1.179 (Solution on p. 82.)
1, 428 + 893
Exercise 1.180
12, 898 + 11, 925
Exercise 1.181 (Solution on p. 82.)
631, 464
+509, 740
Exercise 1.182
805, 996
+ 98, 516
Exercise 1.183 (Solution on p. 82.)
38, 428, 106
+522, 936, 005
Exercise 1.184
5, 288, 423, 100 + 16, 934, 785, 995
Exercise 1.185 (Solution on p. 82.)
98, 876, 678, 521, 402 + 843, 425, 685, 685, 658
Exercise 1.186
41 + 61 + 85 + 62
Exercise 1.187 (Solution on p. 82.)
21 + 85 + 104 + 9 + 15
Exercise 1.188
116
27
110
110
+ 8
Exercise 1.189 (Solution on p. 82.)
75, 206
4, 152
+16, 007
39

Exercise 1.190
8, 226
143
92, 015
8
487, 553
5, 218
Exercise 1.191 (Solution on p. 82.)
50, 006
1, 005
100, 300
20, 008
1, 000, 009
800, 800
Exercise 1.192
616
42, 018
1, 687
225
8, 623, 418
12, 506, 508
19
2, 121
195, 643
For the following problems, perform the additions and round to the nearest hundred.
Exercise 1.193 (Solution on p. 82.)
1, 468
2, 183
Exercise 1.194
928, 725
15, 685
Exercise 1.195 (Solution on p. 82.)
82, 006
3, 019, 528
Exercise 1.196
18, 621
5, 059
40 CHAPTER 1. ADDITION AND SUBTRACTION OF WHOLE NUMBERS

Exercise 1.197 (Solution on p. 82.)


92
48
Exercise 1.198
16
37
Exercise 1.199 (Solution on p. 82.)
21
16
Exercise 1.200
11, 172
22, 749
12, 248
Exercise 1.201 (Solution on p. 82.)
240
280
210
310
Exercise 1.202
9, 573
101, 279
122, 581
For the next ve problems, replace the letter m with the whole number that will make the addition true.
Exercise 1.203 (Solution on p. 82.)
62
+ m
67
Exercise 1.204
106
+ m
113
Exercise 1.205 (Solution on p. 82.)
432
+ m
451
Exercise 1.206
803
+ m
830
41

Exercise 1.207 (Solution on p. 82.)


1, 893
+ m
1, 981
Exercise 1.208
The number of nursing and related care facilities in the United States in 1971 was 22,004. In 1978,
the number was 18,722. What was the total number of facilities for both 1971 and 1978?
Exercise 1.209 (Solution on p. 83.)
The number of persons on food stamps in 1975, 1979, and 1980 was 19,179,000, 19,309,000, and
22,023,000, respectively. What was the total number of people on food stamps for the years 1975,
1979, and 1980?
Exercise 1.210
The enrollment in public and nonpublic schools in the years 1965, 1970, 1975, and 1984 was
54,394,000, 59,899,000, 61,063,000, and 55,122,000, respectively. What was the total enrollment for
those years?
Exercise 1.211 (Solution on p. 83.)
The area of New England is 3,618,770 square miles. The area of the Mountain states is 863,563
square miles. The area of the South Atlantic is 278,926 square miles. The area of the Pacic states
is 921,392 square miles. What is the total area of these regions?
Exercise 1.212
In 1960, the IRS received 1,188,000 corporate income tax returns. In 1965, 1,490,000 returns were
received. In 1970, 1,747,000 returns were received. In 1972 1977, 1,890,000; 1,981,000; 2,043,000;
2,100,000; 2,159,000; and 2,329,000 returns were received, respectively. What was the total number
of corporate tax returns received by the IRS during the years 1960, 1965, 1970, 1972 1977?
Exercise 1.213 (Solution on p. 83.)
Find the total number of scientists employed in 1974.

Exercise 1.214
Find the total number of sales for space vehicle systems for the years 1965-1980.
42 CHAPTER 1. ADDITION AND SUBTRACTION OF WHOLE NUMBERS

Exercise 1.215 (Solution on p. 83.)


Find the total baseball attendance for the years 1960-1980.

Exercise 1.216
Find the number of prosecutions of federal ocials for 1970-1980.

For the following problems, try to add the numbers mentally.


43

Exercise 1.217 (Solution on p. 83.)


5
5
3
7
Exercise 1.218
8
2
6
4
Exercise 1.219 (Solution on p. 83.)
9
1
8
5
2
Exercise 1.220
5
2
5
8
3
7
Exercise 1.221 (Solution on p. 83.)
6
4
3
1
6
7
9
4
Exercise 1.222
20
30
Exercise 1.223 (Solution on p. 83.)
15
35
44 CHAPTER 1. ADDITION AND SUBTRACTION OF WHOLE NUMBERS

Exercise 1.224
16
14
Exercise 1.225 (Solution on p. 83.)
23
27
Exercise 1.226
82
18
Exercise 1.227 (Solution on p. 83.)
36
14

1.5.7.1 Exercises for Review

Exercise 1.228
(Section 1.2) Each period of numbers has its own name. From right to left, what is the name of
the fourth period?
Exercise 1.229 (Solution on p. 83.)
(Section 1.2) In the number 610,467, how many thousands are there?
Exercise 1.230
(Section 1.3) Write 8,840 as you would read it.
Exercise 1.231 (Solution on p. 83.)
(Section 1.4) Round 6,842 to the nearest hundred.
Exercise 1.232
(Section 1.4) Round 431,046 to the nearest million.

1.6 Subtraction of Whole Numbers6


1.6.1 Section Overview
• Subtraction
• Subtraction as the Opposite of Addition
• The Subtraction Process
• Subtraction Involving Borrowing
• Borrowing From Zero
• Calculators

6 This content is available online at <http://cnx.org/content/m34784/1.5/>.


45

1.6.2 Subtraction
Subtraction
Subtraction is the process of determining the remainder when part of the total is removed.

Suppose the sum of two whole numbers is 11, and from 11 we remove 4. Using the number line to help our
visualization, we see that if we are located at 11 and move 4 units to the left, and thus remove 4 units, we
will be located at 7. Thus, 7 units remain when we remove 4 units from 11 units.

The Minus Symbol


The minus symbol (-) is used to indicate subtraction. For example, 11 − 4 indicates that 4 is to be
subtracted from 11.

Minuend
The number immediately in front of or the minus symbol is called the minuend, and it represents the
original number of units.
Subtrahend
The number immediately following or below the minus symbol is called the subtrahend, and it represents
the number of units to be removed.

Dierence
The result of the subtraction is called the dierence of the two numbers. For example, in 11 − 4 = 7, 11 is
the minuend, 4 is the subtrahend, and 7 is the dierence.

1.6.3 Subtraction as the Opposite of Addition


Subtraction can be thought of as the opposite of addition. We show this in the problems in Sample Set A.

1.6.3.1 Sample Set A

Example 1.31
8 − 5 = 3 since 3 + 5 = 8.
Example 1.32
9 − 3 = 6 since 6 + 3 = 9.

1.6.3.2 Practice Set A

Complete the following statements.


Exercise 1.233 (Solution on p. 83.)
7−5= since +5 = 7.
Exercise 1.234 (Solution on p. 83.)
9−1= since +1 = 9.
Exercise 1.235 (Solution on p. 83.)
17 − 8 = since +8 = 17.
46 CHAPTER 1. ADDITION AND SUBTRACTION OF WHOLE NUMBERS

1.6.4 The Subtraction Process


We'll study the process of the subtraction of two whole numbers by considering the dierence between 48
and 35.

which we write as 13.


Example 1.33: The Process of Subtracting Whole Numbers
To subtract two whole numbers,

The process
1. Write the numbers vertically, placing corresponding positions in the same column.
48
−35
2. Subtract the digits in each column. Start at the right, in the ones position, and move to the
left, placing the dierence at the bottom.
48
−35
13

1.6.4.1 Sample Set B

Perform the following subtractions.


Example 1.34
275
−142
133

5-2=3.
7-4=3.
2-1=1.
Example 1.35
46, 042
− 1, 031
45, 011

2-1=1.
4-3=1.
0-0=0.
6-1=5.
4-0=4.
47

Example 1.36
Find the dierence between 977 and 235.

Write the numbers vertically, placing the larger number on top. Line up the columns properly.

977
−235
742

The dierence between 977 and 235 is 742.


Example 1.37
In Keys County in 1987, there were 809 cable television installations. In Flags County in 1987,
there were 1,159 cable television installations. How many more cable television installations were
there in Flags County than in Keys County in 1987?

We need to determine the dierence between 1,159 and 809.

There were 350 more cable television installations in Flags County than in Keys County in 1987.

1.6.4.2 Practice Set B

Perform the following subtractions.


Exercise 1.236 (Solution on p. 83.)
534
−203
Exercise 1.237 (Solution on p. 83.)
857
− 43
Exercise 1.238 (Solution on p. 83.)
95, 628
−34, 510
Exercise 1.239 (Solution on p. 83.)
11, 005
− 1, 005
Exercise 1.240 (Solution on p. 83.)
Find the dierence between 88,526 and 26,412.
In each of these problems, each bottom digit is less than the corresponding top digit. This may not always
be the case. We will examine the case where the bottom digit is greater than the corresponding top digit in
the next section.
48 CHAPTER 1. ADDITION AND SUBTRACTION OF WHOLE NUMBERS

1.6.5 Subtraction Involving Borrowing


Minuend and Subtrahend
It often happens in the subtraction of two whole numbers that a digit in the minuend (top number) will
be less than the digit in the same position in the subtrahend (bottom number). This happens when we
subtract 27 from 84.

84
−27

We do not have a name for 4 − 7. We need to rename 84 in order to continue. We'll do so as follows:

Our new name for 84 is 7 tens + 14 ones.

= 57

Notice that we converted 8 tens to 7 tens + 1 ten, and then we converted the 1 ten to 10 ones. We then had
14 ones and were able to perform the subtraction.

Borrowing
The process of borrowing (converting) is illustrated in the problems of Sample Set C.

1.6.5.1 Sample Set C

Example 1.38

1. Borrow 1 ten from the 8 tens. This leaves 7 tens.


2. Convert the 1 ten to 10 ones.
3. Add 10 ones to 4 ones to get 14 ones.

Example 1.39

1. Borrow 1 hundred from the 6 hundreds. This leaves 5 hundreds.


49

2. Convert the 1 hundred to 10 tens.


3. Add 10 tens to 7 tens to get 17 tens.

1.6.5.2 Practice Set C

Perform the following subtractions. Show the expanded form for the rst three problems.
Exercise 1.241 (Solution on p. 83.)
53
−35
Exercise 1.242 (Solution on p. 84.)
76
−28
Exercise 1.243 (Solution on p. 84.)
872
−565
Exercise 1.244 (Solution on p. 84.)
441
−356
Exercise 1.245 (Solution on p. 84.)
775
− 66
Exercise 1.246 (Solution on p. 84.)
5, 663
−2, 559
Borrowing More Than Once

Sometimes it is necessary to borrow more than once. This is shown in the problems in Section 1.6.5.3
(Sample Set D).

1.6.5.3 Sample Set D

Perform the Subtractions. Borrowing more than once if necessary

Example 1.40

1. Borrow 1 ten from the 4 tens. This leaves 3 tens.


2. Convert the 1 ten to 10 ones.
3. Add 10 ones to 1 one to get 11 ones. We can now perform 11 − 8.
4. Borrow 1 hundred from the 6 hundreds. This leaves 5 hundreds.
50 CHAPTER 1. ADDITION AND SUBTRACTION OF WHOLE NUMBERS

5. Convert the 1 hundred to 10 tens.


6. Add 10 tens to 3 tens to get 13 tens.
7. Now 13 − 5 = 8.
8. 5 − 3 = 2.

Example 1.41

1. Borrow 1 ten from the 3 tens. This leaves 2 tens.


2. Convert the 1 ten to 10 ones.
3. Add 10 ones to 4 ones to get 14 ones. We can now perform 14 − 5.
4. Borrow 1 hundred from the 5 hundreds. This leaves 4 hundreds.
5. Convert the 1 hundred to 10 tens.
6. Add 10 tens to 2 tens to get 12 tens. We can now perform 12 − 8 = 4.
7. Finally, 4 − 0 = 4.

Example 1.42
71529
- 6952

After borrowing, we have

1.6.5.4 Practice Set D

Perform the following subtractions.


Exercise 1.247 (Solution on p. 84.)
526
−358
Exercise 1.248 (Solution on p. 84.)
63, 419
− 7, 779
Exercise 1.249 (Solution on p. 84.)
4, 312
−3, 123
51

1.6.6 Borrowing from Zero


It often happens in a subtraction problem that we have to borrow from one or more zeros. This occurs in
problems such as

503
1.
− 37
and
5000
2.
− 37

We'll examine each case.


Example 1.43: Borrowing from a single zero.
503
Consider the problem
− 37

Since we do not have a name for 3 − 7, we must borrow from 0.

Since there are no tens to borrow, we must borrow 1 hundred. One hundred = 10 tens.

We can now borrow 1 ten from 10 tens (leaving 9 tens). One ten = 10 ones and 10 ones + 3 ones
= 13 ones.

Now we can suggest the following method for borrowing from a single zero.

Borrowing from a Single Zero


To borrow from a single zero,

1. Decrease the digit to the immediate left of zero by one.


2. Draw a line through the zero and make it a 10.
3. Proceed to subtract as usual.

1.6.6.1 Sample Set E

Example 1.44
Perform this subtraction.

503
− 37

The number 503 contains a single zero


52 CHAPTER 1. ADDITION AND SUBTRACTION OF WHOLE NUMBERS

1. The number to the immediate left of 0 is 5. Decrease 5 by 1.

5−1=4

2. Draw a line through the zero and make it a 10.


3. Borrow from the 10 and proceed.

1 ten + 10 ones

10 ones + 3 ones = 13 ones

1.6.6.2 Practice Set E

Perform each subtraction.


Exercise 1.250 (Solution on p. 84.)
906
− 18
Exercise 1.251 (Solution on p. 84.)
5102
− 559
Exercise 1.252 (Solution on p. 85.)
9055
− 386
Example 1.45: Borrowing from a group of zeros
5000
Consider the problem
− 37

In this case, we have a group of zeros.

Since we cannot borrow any tens or hundreds, we must borrow 1 thousand. One thousand = 10
hundreds.

We can now borrow 1 hundred from 10 hundreds. One hundred = 10 tens.


53

We can now borrow 1 ten from 10 tens. One ten = 10 ones.

From observations made in this procedure we can suggest the following method for borrowing from
a group of zeros.
Borrowing from a Group of zeros
To borrow from a group of zeros,

1. Decrease the digit to the immediate left of the group of zeros by one.
2. Draw a line through each zero in the group and make it a 9, except the rightmost zero, make it 10.
3. Proceed to subtract as usual.

1.6.6.3 Sample Set F

Perform each subtraction.


Example 1.46
40, 000
− 125

The number 40,000 contains a group of zeros.

1. The number to the immediate left of the group is 4. Decrease 4 by 1.

4−1=3
2. Make each 0, except the rightmost one, 9. Make the rightmost 0 a 10.

3. Subtract as usual.

Example 1.47
8, 000, 006
− 41, 107

The number 8,000,006 contains a group of zeros.

1. The number to the immediate left of the group is 8. Decrease 8 by 1. 8 − 1 = 7


2. Make each zero, except the rightmost one, 9. Make the rightmost 0 a 10.
54 CHAPTER 1. ADDITION AND SUBTRACTION OF WHOLE NUMBERS

3. To perform the subtraction, we'll need to borrow from the ten.

1 ten = 10 ones
10 ones + 6 ones = 16 ones

1.6.6.4 Practice Set F

Perform each subtraction.


Exercise 1.253 (Solution on p. 85.)
21, 007
− 4, 873
Exercise 1.254 (Solution on p. 85.)
10, 004
− 5, 165
Exercise 1.255 (Solution on p. 85.)
16, 000, 000
− 201, 060

1.6.7 Calculators
In practice, calculators are used to nd the dierence between two whole numbers.

1.6.7.1 Sample Set G

Find the dierence between 1006 and 284.

Display Reads
Type 1006 1006
Press − 1006
Type 284 284
Press = 722

Table 1.26
55

The dierence between 1006 and 284 is 722.

(What happens if you type 284 rst and then 1006? We'll study such numbers in Chapter 10.)

1.6.7.2 Practice Set G

Exercise 1.256 (Solution on p. 85.)


Use a calculator to nd the dierence between 7338 and 2809.
Exercise 1.257 (Solution on p. 85.)
Use a calculator to nd the dierence between 31,060,001 and 8,591,774.

1.6.8 Exercises
For the following problems, perform the subtractions. You may check each dierence with a calculator.
Exercise 1.258 (Solution on p. 85.)
15
− 8
Exercise 1.259
19
− 8
Exercise 1.260 (Solution on p. 85.)
11
− 5
Exercise 1.261
14
− 6
Exercise 1.262 (Solution on p. 85.)
12
− 9
Exercise 1.263
56
−12
Exercise 1.264 (Solution on p. 85.)
74
−33
Exercise 1.265
80
−61
Exercise 1.266 (Solution on p. 85.)
350
−141
56 CHAPTER 1. ADDITION AND SUBTRACTION OF WHOLE NUMBERS

Exercise 1.267
800
−650
Exercise 1.268 (Solution on p. 85.)
35, 002
−14, 001
Exercise 1.269
5, 000, 566
−2, 441, 326
Exercise 1.270 (Solution on p. 85.)
400, 605
−121, 352
Exercise 1.271
46, 400
− 2, 012
Exercise 1.272 (Solution on p. 85.)
77, 893
− 421
Exercise 1.273
42
−18
Exercise 1.274 (Solution on p. 85.)
51
−27
Exercise 1.275
622
− 88
Exercise 1.276 (Solution on p. 85.)
261
− 73
Exercise 1.277
242
−158
Exercise 1.278 (Solution on p. 85.)
3, 422
−1, 045
Exercise 1.279
5, 565
−3, 985
57

Exercise 1.280 (Solution on p. 85.)


42, 041
−15, 355
Exercise 1.281
304, 056
− 20, 008
Exercise 1.282 (Solution on p. 85.)
64, 000, 002
− 856, 743
Exercise 1.283
4, 109
− 856
Exercise 1.284 (Solution on p. 85.)
10, 113
− 2, 079
Exercise 1.285
605
− 77
Exercise 1.286 (Solution on p. 85.)
59
−26
Exercise 1.287
36, 107
− 8, 314
Exercise 1.288 (Solution on p. 85.)
92, 526, 441, 820
−59, 914, 805, 253
Exercise 1.289
1, 605
− 881
Exercise 1.290 (Solution on p. 85.)
30, 000
−26, 062
Exercise 1.291
600
−216
Exercise 1.292 (Solution on p. 85.)
9, 000, 003
− 726, 048
For the following problems, perform each subtraction.
58 CHAPTER 1. ADDITION AND SUBTRACTION OF WHOLE NUMBERS

Exercise 1.293
Subtract 63 from 92.

Hint: The word "from" means "beginning at." Thus, 63 from 92 means beginning at 92, or
92 − 63.

Exercise 1.294 (Solution on p. 85.)


Subtract 35 from 86.
Exercise 1.295
Subtract 382 from 541.
Exercise 1.296 (Solution on p. 86.)
Subtract 1,841 from 5,246.
Exercise 1.297
Subtract 26,082 from 35,040.
Exercise 1.298 (Solution on p. 86.)
Find the dierence between 47 and 21.
Exercise 1.299
Find the dierence between 1,005 and 314.
Exercise 1.300 (Solution on p. 86.)
Find the dierence between 72,085 and 16.
Exercise 1.301
Find the dierence between 7,214 and 2,049.
Exercise 1.302 (Solution on p. 86.)
Find the dierence between 56,108 and 52,911.
Exercise 1.303
How much bigger is 92 than 47?
Exercise 1.304 (Solution on p. 86.)
How much bigger is 114 than 85?
Exercise 1.305
How much bigger is 3,006 than 1,918?
Exercise 1.306 (Solution on p. 86.)
How much bigger is 11,201 than 816?
Exercise 1.307
How much bigger is 3,080,020 than 1,814,161?
Exercise 1.308 (Solution on p. 86.)
In Wichita, Kansas, the sun shines about 74% of the time in July and about 59% of the time in
November. How much more of the time (in percent) does the sun shine in July than in November?
Exercise 1.309
The lowest temperature on record in Concord, New Hampshire in May is 21 ◦ F, and in July it is
35 ◦ F. What is the dierence in these lowest temperatures?
Exercise 1.310 (Solution on p. 86.)
In 1980, there were 83,000 people arrested for prostitution and commercialized vice and 11,330,000
people arrested for driving while intoxicated. How many more people were arrested for drunk
driving than for prostitution?
Exercise 1.311
In 1980, a person with a bachelor's degree in accounting received a monthly salary oer of $1,293,
and a person with a marketing degree a monthly salary oer of $1,145. How much more was oered
to the person with an accounting degree than the person with a marketing degree?
59

Exercise 1.312 (Solution on p. 86.)


In 1970, there were about 793 people per square mile living in Puerto Rico, and 357 people per
square mile living in Guam. How many more people per square mile were there in Puerto Rico
than Guam?
Exercise 1.313
The 1980 population of Singapore was 2,414,000 and the 1980 population of Sri Lanka was
14,850,000. How many more people lived in Sri Lanka than in Singapore in 1980?
Exercise 1.314 (Solution on p. 86.)
In 1977, there were 7,234,000 hospitals in the United States and 64,421,000 in Mainland China.
How many more hospitals were there in Mainland China than in the United States in 1977?
Exercise 1.315
In 1978, there were 3,095,000 telephones in use in Poland and 4,292,000 in Switzerland. How many
more telephones were in use in Switzerland than in Poland in 1978?
For the following problems, use the corresponding graphs to solve the problems.
Exercise 1.316 (Solution on p. 86.)
How many more life scientists were there in 1974 than mathematicians? (this image)
Exercise 1.317
How many more social, psychological, mathematical, and environmental scientists were there than
life, physical, and computer scientists? (this image)

Exercise 1.318 (Solution on p. 86.)


How many more prosecutions were there in 1978 than in 1974? (this image)
Exercise 1.319
How many more prosecutions were there in 1976-1980 than in 1970-1975? (this image)
60 CHAPTER 1. ADDITION AND SUBTRACTION OF WHOLE NUMBERS

Exercise 1.320 (Solution on p. 86.)


How many more dry holes were drilled in 1960 than in 1975? (this image)
Exercise 1.321
How many more dry holes were drilled in 1960, 1965, and 1970 than in 1975, 1978 and 1979? (this
image)

For the following problems, replace the [U+2610] with the whole number that will make the subtraction
true.
Exercise 1.322 (Solution on p. 86.)
14
−
3
61

Exercise 1.323
21
−
14
Exercise 1.324 (Solution on p. 86.)
35
−
25
Exercise 1.325
16
−
9
Exercise 1.326 (Solution on p. 86.)
28
−
16
For the following problems, nd the solutions.
Exercise 1.327
Subtract 42 from the sum of 16 and 56.
Exercise 1.328 (Solution on p. 86.)
Subtract 105 from the sum of 92 and 89.
Exercise 1.329
Subtract 1,127 from the sum of 2,161 and 387.
Exercise 1.330 (Solution on p. 86.)
Subtract 37 from the dierence between 263 and 175.
Exercise 1.331
Subtract 1,109 from the dierence between 3,046 and 920.
Exercise 1.332 (Solution on p. 86.)
Add the dierence between 63 and 47 to the difference between 55 and 11.
Exercise 1.333
Add the dierence between 815 and 298 to the dierence between 2,204 and 1,016.
Exercise 1.334 (Solution on p. 86.)
Subtract the dierence between 78 and 43 from the sum of 111 and 89.
Exercise 1.335
Subtract the dierence between 18 and 7 from the sum of the dierences between 42 and 13, and
81 and 16.
Exercise 1.336 (Solution on p. 86.)
Find the dierence between the dierences of 343 and 96, and 521 and 488.
62 CHAPTER 1. ADDITION AND SUBTRACTION OF WHOLE NUMBERS

1.6.8.1 Exercises for Review

Exercise 1.337
(Section 1.2) In the number 21,206, how many hundreds are there?
Exercise 1.338 (Solution on p. 86.)
(Section 1.2) Write a three-digit number that has a zero in the ones position.
Exercise 1.339
(Section 1.2) How many three-digit whole numbers are there?
Exercise 1.340 (Solution on p. 86.)
(Section 1.4) Round 26,524,016 to the nearest million.
Exercise 1.341
(Section 1.5) Find the sum of 846 + 221 + 116.

1.7 Properties of Addition7


1.7.1 Section Overview
• The Commutative Property of Addition
• The Associative Property of Addition
• The Additive Identity

We now consider three simple but very important properties of addition.

1.7.2 The Commutative Property of Addition


Commutative Property of Addition
If two whole numbers are added in any order, the sum will not change.

1.7.2.1 Sample Set A

Example 1.48
Add the whole numbers

8 + 5 = 13
5 + 8 = 13

The numbers 8 and 5 can be added in any order. Regardless of the order they are added, the sum
is 13.

7 This content is available online at <http://cnx.org/content/m34802/1.5/>.


63

1.7.2.2 Practice Set A

Exercise 1.342 (Solution on p. 86.)


Use the commutative property of addition to nd the sum of 12 and 41 in two dierent ways.

Exercise 1.343 (Solution on p. 86.)


Add the whole numbers

1.7.3 The Associative Property of Addition


Associative Property of Addition
If three whole numbers are to be added, the sum will be the same if the rst two are added rst, then that
sum is added to the third, or, the second two are added rst, and that sum is added to the rst.

Using Parentheses
It is a common mathematical practice to use parentheses to show which pair of numbers we wish to
combine rst.

1.7.3.1 Sample Set B

Example 1.49
Add the whole numbers.

1.7.3.2 Practice Set B

Exercise 1.344 (Solution on p. 87.)


Use the associative property of addition to add the following whole numbers two dierent ways.

Exercise 1.345 (Solution on p. 87.)


64 CHAPTER 1. ADDITION AND SUBTRACTION OF WHOLE NUMBERS

1.7.4 The Additive Identity


0 Is the Additive Identity
The whole number 0 is called the additive identity, since when it is added to any whole number, the sum
is identical to that whole number.

1.7.4.1 Sample Set C

Example 1.50
Add the whole numbers.

29 + 0 = 29
0 + 29 = 29

Zero added to 29 does not change the identity of 29.

1.7.4.2 Practice Set C

Add the following whole numbers.


Exercise 1.346 (Solution on p. 87.)

Exercise 1.347 (Solution on p. 87.)

Suppose we let the letter x represent a choice for some whole number. For the rst two problems, nd the
sums. For the third problem, nd the sum provided we now know that x represents the whole number 17.
Exercise 1.348 (Solution on p. 87.)

Exercise 1.349 (Solution on p. 87.)


65

Exercise 1.350 (Solution on p. 87.)

1.7.5 Exercises
For the following problems, add the numbers in two ways.
Exercise 1.351 (Solution on p. 87.)

Exercise 1.352

Exercise 1.353 (Solution on p. 87.)

Exercise 1.354

Exercise 1.355 (Solution on p. 87.)

Exercise 1.356

Exercise 1.357 (Solution on p. 87.)


66 CHAPTER 1. ADDITION AND SUBTRACTION OF WHOLE NUMBERS

Exercise 1.358

Exercise 1.359 (Solution on p. 87.)

Exercise 1.360

Exercise 1.361 (Solution on p. 87.)

Exercise 1.362

Exercise 1.363 (Solution on p. 87.)

Exercise 1.364

Exercise 1.365 (Solution on p. 87.)

For the following problems, show that the pairs of quantities yield the same sum.
Exercise 1.366
(11 + 27) + 9 and 11 + (27 + 9)
Exercise 1.367 (Solution on p. 87.)
(80 + 52) + 6 and 80 + (52 + 6)
67

Exercise 1.368
(114 + 226) + 108 and 114 + (226 + 108)
Exercise 1.369 (Solution on p. 87.)
(731 + 256) + 171 and 731 + (256 + 171)
Exercise 1.370
The fact that (a rst number + a second number) + third number = a rst number + (a second
number + a third number) is an example of the property of addition.
Exercise 1.371 (Solution on p. 87.)
The fact that 0 + any number = that particular number is an example of the prop-
erty of addition.
Exercise 1.372
The fact that a rst number + a second number = a second number + a rst number is an example
of the property of addition.
Exercise 1.373 (Solution on p. 87.)
Use the numbers 15 and 8 to illustrate the commutative property of addition.
Exercise 1.374
Use the numbers 6, 5, and 11 to illustrate the associative property of addition.
Exercise 1.375 (Solution on p. 87.)
The number zero is called the additive identity. Why is the term identity so appropriate?

1.7.5.1 Exercises for Review

Exercise 1.376
(Section 1.2) How many hundreds in 46,581?
Exercise 1.377 (Solution on p. 87.)
(Section 1.3) Write 2,218 as you would read it.
Exercise 1.378
(Section 1.4) Round 506,207 to the nearest thousand.
Exercise 1.379 (Solution on p. 87.)
482
(Section 1.5) Find the sum of
+ 68
Exercise 1.380
3, 318
(Section 1.6) Find the dierence:
− 429

1.8 Summary of Key Concepts8


1.8.1 Summary of Key Concepts
Number / Numeral (Section 1.2)
A number is a concept. It exists only in the mind. A numeral is a symbol that represents a number. It is
customary not to distinguish between the two (but we should remain aware of the dierence).

8 This content is available online at <http://cnx.org/content/m34798/1.3/>.


68 CHAPTER 1. ADDITION AND SUBTRACTION OF WHOLE NUMBERS

Hindu-Arabic Numeration System (Section 1.2)


In our society, we use the Hindu-Arabic numeration system. It was invented by the Hindus shortly before
the third century and popularized by the Arabs about a thousand years later.

Digits (Section 1.2)


The numbers 0, 1, 2, 3, 4, 5, 6, 7, 8, 9 are called digits.

Base Ten Positional System (Section 1.2)


The Hindu-Arabic numeration system is a positional number system with base ten. Each position has value
that is ten times the value of the position to its right.

Commas / Periods (Section 1.2)


Commas are used to separate digits into groups of three. Each group of three is called a period. Each
period has a name. From right to left, they are ones, thousands, millions, billions, etc.

Whole Numbers (Section 1.2)


A whole number is any number that is formed using only the digits (0, 1, 2, 3, 4, 5, 6, 7, 8, 9).

Number Line (Section 1.2)


The number line allows us to visually display the whole numbers.

Graphing (Section 1.2)


Graphing a whole number is a term used for visually displaying the whole number. The graph of 4 appears
below.

Reading Whole Numbers (Section 1.3)


To express a whole number as a verbal phrase:

1. Begin at the right and, working right to left, separate the number into distinct periods by inserting
commas every three digits.
2. Begin at the left, and read each period individually.

Writing Whole Numbers (Section 1.3)


To rename a number that is expressed in words to a number expressed in digits:

1. Notice that a number expressed as a verbal phrase will have its periods set o by commas.
2. Start at the beginning of the sentence, and write each period of numbers individually.
3. Use commas to separate periods, and combine the periods to form one number.

Rounding (Section 1.4)


Rounding is the process of approximating the number of a group of objects by mentally "seeing" the
collection as occurring in groups of tens, hundreds, thousands, etc.
69

Addition (Section 1.5)


Addition is the process of combining two or more objects (real or intuitive) to form a new, third object,
the total, or sum.

Addends / Sum (Section 1.5)


In addition, the numbers being added are called addends and the result, or total, the sum.

Subtraction (Section 1.6)


Subtraction is the process of determining the remainder when part of the total is removed.

Minuend / Subtrahend Dierence (Section 1.6)

Commutative Property of Addition (Section 1.7)


If two whole numbers are added in either of two orders, the sum will not change.
3+5=5+3

Associative Property of Addition (Section 1.7)


If three whole numbers are to be added, the sum will be the same if the rst two are added and that sum is
then added to the third, or if the second two are added and the rst is added to that sum.
(3 + 5) + 2 = 3 + (5 + 2)

Parentheses in Addition (Section 1.7)


Parentheses in addition indicate which numbers are to be added rst.

Additive Identity (Section 1.7)


The whole number 0 is called the additive identity since, when it is added to any particular whole number,
the sum is identical to that whole number.
0+7=7
7+0=7

1.9 Exercise Supplement9


1.9.1 Exercise Supplement
For problems 1-35, nd the sums and dierences.
Exercise 1.381 (Solution on p. 87.)
908
+ 29
Exercise 1.382
529
+161
Exercise 1.383 (Solution on p. 87.)
549
+ 16

9 This content is available online at <http://cnx.org/content/m34800/1.5/>.


70 CHAPTER 1. ADDITION AND SUBTRACTION OF WHOLE NUMBERS

Exercise 1.384
726
+892
Exercise 1.385 (Solution on p. 87.)
390
+169
Exercise 1.386
166
+660
Exercise 1.387 (Solution on p. 88.)
391
+951
Exercise 1.388
48
+36
Exercise 1.389 (Solution on p. 88.)
1, 103
+ 898
Exercise 1.390
1, 642
+ 899
Exercise 1.391 (Solution on p. 88.)
807
+1, 156
Exercise 1.392
80, 349
+ 2, 679
Exercise 1.393 (Solution on p. 88.)
70, 070
+ 9, 386
Exercise 1.394
90, 874
+ 2, 945
Exercise 1.395 (Solution on p. 88.)
45, 292
+51, 661
Exercise 1.396
1, 617
+54, 923
71

Exercise 1.397 (Solution on p. 88.)


702, 607
+ 89, 217
Exercise 1.398
6, 670, 006
+ 2, 495
Exercise 1.399 (Solution on p. 88.)
267
+8, 034
Exercise 1.400
7, 007
+11, 938
Exercise 1.401 (Solution on p. 88.)
131, 294
+ 9, 087
Exercise 1.402
5, 292
+ 161
Exercise 1.403 (Solution on p. 88.)
17, 260
+58, 964
Exercise 1.404
7, 006
−5, 382
Exercise 1.405 (Solution on p. 88.)
7, 973
−3, 018
Exercise 1.406
16, 608
− 1, 660
Exercise 1.407 (Solution on p. 88.)
209, 527
− 23, 916
Exercise 1.408
584
−226
Exercise 1.409 (Solution on p. 88.)
3, 313
−1, 075
72 CHAPTER 1. ADDITION AND SUBTRACTION OF WHOLE NUMBERS

Exercise 1.410
458
−122
Exercise 1.411 (Solution on p. 88.)
1, 007
+ 331
Exercise 1.412
16, 082
+ 2, 013
Exercise 1.413 (Solution on p. 88.)
926
− 48
Exercise 1.414
736
+5, 869
Exercise 1.415 (Solution on p. 88.)
676, 504
− 58, 277
For problems 36-39, add the numbers.
Exercise 1.416
769
795
298
746
Exercise 1.417 (Solution on p. 88.)
554
184
883
Exercise 1.418
30, 188
79, 731
16, 600
66, 085
39, 169
95, 170
73

Exercise 1.419 (Solution on p. 88.)


2, 129
6, 190
17, 044
30, 447
292
41
428, 458
For problems 40-50, combine the numbers as indicated.
Exercise 1.420
2, 957 + 9, 006
Exercise 1.421 (Solution on p. 88.)
19, 040 + 813
Exercise 1.422
350, 212 + 14, 533
Exercise 1.423 (Solution on p. 88.)
970 + 702 + 22 + 8
Exercise 1.424
3, 704 + 2, 344 + 429 + 10, 374 + 74
Exercise 1.425 (Solution on p. 88.)
874 + 845 + 295 − 900
Exercise 1.426
904 + 910 − 881
Exercise 1.427 (Solution on p. 88.)
521 + 453 − 334 + 600
Exercise 1.428
892 − 820 − 9
Exercise 1.429 (Solution on p. 88.)
159 + 4, 085 − 918 − 608
Exercise 1.430
2, 562 + 8, 754 − 393 − 385 − 910
For problems 51-63, add and subtract as indicated.
Exercise 1.431 (Solution on p. 88.)
Subtract 671 from 8,027.
Exercise 1.432
Subtract 387 from 6,342.
Exercise 1.433 (Solution on p. 88.)
Subtract 2,926 from 6,341.
Exercise 1.434
Subtract 4,355 from the sum of 74 and 7,319.
Exercise 1.435 (Solution on p. 88.)
Subtract 325 from the sum of 7,188 and 4,964.
Exercise 1.436
Subtract 496 from the dierence of 60,321 and 99.
74 CHAPTER 1. ADDITION AND SUBTRACTION OF WHOLE NUMBERS

Exercise 1.437 (Solution on p. 89.)


Subtract 20,663 from the dierence of 523,150 and 95,225.
Exercise 1.438
Add the dierence of 843 and 139 to the dierence of 4,450 and 839.
Exercise 1.439 (Solution on p. 89.)
Add the dierence of 997,468 and 292,513 to the dierence of 22,140 and 8,617.
Exercise 1.440
Subtract the dierence of 8,412 and 576 from the sum of 22,140 and 8,617.
Exercise 1.441 (Solution on p. 89.)
Add the sum of 2,273, 3,304, 847, and 16 to the dierence of 4,365 and 864.
Exercise 1.442
Add the sum of 19,161, 201, 166,127, and 44 to the dierence of the sums of 161, 2,455, and 85,
and 21, 26, 48, and 187.
Exercise 1.443 (Solution on p. 89.)
Is the sum of 626 and 1,242 the same as the sum of 1,242 and 626? Justify your claim.

1.10 Prociency Exam10


1.10.1 Prociency Exam
Exercise 1.444 (Solution on p. 89.)
(Section 1.2) What is the largest digit?
Exercise 1.445 (Solution on p. 89.)
(Section 1.2) In the Hindu-Arabic number system, each period has three values assigned to it.
These values are the same for each period. From right to left, what are they?
Exercise 1.446 (Solution on p. 89.)
(Section 1.2) In the number 42,826, how many hundreds are there?
Exercise 1.447 (Solution on p. 89.)
(Section 1.2) Is there a largest whole number? If so, what is it?
Exercise 1.448 (Solution on p. 89.)
(Section 1.2) Graph the following whole numbers on the number line: 2, 3, 5.

Exercise 1.449 (Solution on p. 89.)


(Section 1.3) Write the number 63,425 as you would read it aloud.
Exercise 1.450 (Solution on p. 89.)
(Section 1.3) Write the number eighteen million, three hundred fty-nine thousand, seventy-two.
Exercise 1.451 (Solution on p. 89.)
(Section 1.4) Round 427 to the nearest hundred.
Exercise 1.452 (Solution on p. 89.)
(Section 1.4) Round 18,995 to the nearest ten.
10 This content is available online at <http://cnx.org/content/m34805/1.4/>.
75

Exercise 1.453 (Solution on p. 89.)


(Section 1.4) Round to the most reasonable digit: During a semester, a mathematics instructor
uses 487 pieces of chalk.
For problems 11-17, nd the sums and dierences.
Exercise 1.454 (Solution on p. 89.)
627
(Section 1.5)
+ 48
Exercise 1.455 (Solution on p. 89.)
(Section 1.5) 3106 + 921
Exercise 1.456 (Solution on p. 89.)
152
(Section 1.5)
+ 36
Exercise 1.457 (Solution on p. 89.)
5, 189
6, 189
(Section 1.5)
4, 122
+8, 001
Exercise 1.458 (Solution on p. 89.)
(Section 1.5) 21 + 16 + 42 + 11
Exercise 1.459 (Solution on p. 89.)
(Section 1.6) 520 − 216
Exercise 1.460 (Solution on p. 89.)
80, 001
(Section 1.6)
− 9, 878
Exercise 1.461 (Solution on p. 89.)
(Section 1.6) Subtract 425 from 816.
Exercise 1.462 (Solution on p. 89.)
(Section 1.6) Subtract 712 from the sum of 507 and 387.
Exercise 1.463 (Solution on p. 89.)
(Section 1.7) Is the sum of 219 and 412 the same as the sum of 412 and 219? If so, what makes it
so?
76 CHAPTER 1. ADDITION AND SUBTRACTION OF WHOLE NUMBERS

Solutions to Exercises in Chapter 1


Solution to Exercise 1.1 (p. 10)
Yes. Letters are symbols. Taken as a collection (a written word), they represent a number.
Solution to Exercise 1.2 (p. 12)
ve thousand
Solution to Exercise 1.3 (p. 12)
four hundred billion
Solution to Exercise 1.4 (p. 12)
zero tens, or zero
Solution to Exercise 1.5 (p. 13)

Solution to Exercise 1.6 (p. 13)


4, 5, 6, 113, 978
Solution to Exercise 1.7 (p. 14)
concept
Solution to Exercise 1.9 (p. 14)
Yes, since it is a symbol that represents a number.
Solution to Exercise 1.11 (p. 14)
positional; 10
Solution to Exercise 1.13 (p. 14)
units, tens, hundreds
Solution to Exercise 1.15 (p. 14)
4
Solution to Exercise 1.17 (p. 14)
0
Solution to Exercise 1.19 (p. 14)
0
Solution to Exercise 1.21 (p. 14)
ten thousand
Solution to Exercise 1.23 (p. 14)
ten million
Solution to Exercise 1.25 (p. 14)
1,340 (answers may vary)
Solution to Exercise 1.27 (p. 15)
900
Solution to Exercise 1.29 (p. 15)
yes; zero
Solution to Exercise 1.31 (p. 15)
graphing
Solution to Exercise 1.33 (p. 15)

Solution to Exercise 1.35 (p. 15)


61, 99, 100, 102
Solution to Exercise 1.37 (p. 17)
Twelve thousand, ve hundred forty-two
Solution to Exercise 1.38 (p. 17)
One hundred one million, seventy-four thousand, three
77

Solution to Exercise 1.39 (p. 17)


One million, eight
Solution to Exercise 1.40 (p. 18)
103,025
Solution to Exercise 1.41 (p. 18)
6,040,007
Solution to Exercise 1.42 (p. 18)
20,003,080,109,402
Solution to Exercise 1.43 (p. 18)
80,000,000,035
Solution to Exercise 1.44 (p. 18)
nine hundred twelve
Solution to Exercise 1.46 (p. 19)
one thousand, four hundred ninety-one
Solution to Exercise 1.48 (p. 19)
thirty-ve thousand, two hundred twenty-three
Solution to Exercise 1.50 (p. 19)
four hundred thirty-seven thousand, one hundred ve
Solution to Exercise 1.52 (p. 19)
eight million, one thousand, one
Solution to Exercise 1.54 (p. 19)
seven hundred seventy million, three hundred eleven thousand, one hundred one
Solution to Exercise 1.56 (p. 19)
one hundred six billion, one hundred million, one thousand ten
Solution to Exercise 1.58 (p. 19)
eight hundred billion, eight hundred thousand
Solution to Exercise 1.60 (p. 19)
four; one thousand, four hundred sixty
Solution to Exercise 1.62 (p. 19)
twenty billion
Solution to Exercise 1.64 (p. 19)
four hundred twelve; fty-two; twenty-one thousand, four hundred twenty-four
Solution to Exercise 1.66 (p. 19)
one thousand, nine hundred seventy-nine; eighty-ve thousand; two million, nine hundred ve thousand
Solution to Exercise 1.68 (p. 19)
one thousand, nine hundred eighty; two hundred seventeen
Solution to Exercise 1.70 (p. 20)
one thousand, nine hundred eighty one; one million, nine hundred fty-six thousand
Solution to Exercise 1.72 (p. 20)
one thousand, nine hundred eighty; thirteen thousand, one hundred
Solution to Exercise 1.74 (p. 20)
twelve million, six hundred thirty thousand
Solution to Exercise 1.76 (p. 20)
681
Solution to Exercise 1.78 (p. 20)
7,201
Solution to Exercise 1.80 (p. 20)
512,003
Solution to Exercise 1.82 (p. 20)
35,007,101
78 CHAPTER 1. ADDITION AND SUBTRACTION OF WHOLE NUMBERS

Solution to Exercise 1.84 (p. 20)


16,000,059,004
Solution to Exercise 1.86 (p. 20)
23,000,000,000
Solution to Exercise 1.88 (p. 20)
100,000,000,000,001
Solution to Exercise 1.90 (p. 21)
4
Solution to Exercise 1.92 (p. 21)
yes, zero
Solution to Exercise 1.94 (p. 24)
3400
Solution to Exercise 1.95 (p. 24)
27,000
Solution to Exercise 1.96 (p. 24)
31,000,000
Solution to Exercise 1.97 (p. 24)
0
Solution to Exercise 1.98 (p. 24)
60,000
Solution to Exercise 1.99 (p. 24)

hundred thousand ten thousand million


1,600 2000 0 0

Table 1.27

Solution to Exercise 1.101 (p. 25)

Hundred thousand ten thousand million


91,800 92,000 90,000 0

Table 1.28

Solution to Exercise 1.103 (p. 25)

hundred thousand ten thousand million


200 0 0 0

Table 1.29

Solution to Exercise 1.105 (p. 25)

hundred thousand ten thousand million


900 1,000 0 0
79

Table 1.30

Solution to Exercise 1.107 (p. 26)

hundred thousand ten thousand million


900 1,000 0 0

Table 1.31

Solution to Exercise 1.109 (p. 26)

hundred thousand ten thousand million


1,000 1,000 0 0

Table 1.32

Solution to Exercise 1.111 (p. 26)

hundred thousand ten thousand million


551,061,300 551,061,000 551,060,000 551,000,000

Table 1.33

Solution to Exercise 1.113 (p. 27)

hundred thousand ten thousand million


106,999,413,200 106,999,413,000 106,999,410,000 106,999,000,000

Table 1.34

Solution to Exercise 1.115 (p. 27)

Hundred Thousand ten thousand Million


8,006,000 8,006,000 8,010,000 8,000,000

Table 1.35

Solution to Exercise 1.117 (p. 27)

hundred thousand ten thousand million


33,500 33,000 30,000 0
80 CHAPTER 1. ADDITION AND SUBTRACTION OF WHOLE NUMBERS

Table 1.36

Solution to Exercise 1.119 (p. 28)

hundred thousand ten thousand million


388,600 389,000 390,000 0

Table 1.37

Solution to Exercise 1.121 (p. 28)

hundred thousand ten thousand million


8,200 8,000 10,000 0

Table 1.38

Solution to Exercise 1.123 (p. 28)


19,310,000
Solution to Exercise 1.125 (p. 29)
29,000,000
Solution to Exercise 1.127 (p. 29)
70% or 75%
Solution to Exercise 1.129 (p. 29)
$5,500,000,000
Solution to Exercise 1.131 (p. 29)
230,000
Solution to Exercise 1.133 (p. 29)
5,400,000
Solution to Exercise 1.135 (p. 29)
graphing
Solution to Exercise 1.137 (p. 29)
Forty-two thousand, one hundred nine
Solution to Exercise 1.139 (p. 29)
4,000,000,008
Solution to Exercise 1.140 (p. 31)
88

Solution to Exercise 1.141 (p. 31)


5,527

Solution to Exercise 1.142 (p. 31)


267,166
81

Solution to Exercise 1.143 (p. 34)


87

= 7 tens + 1 ten + 7 ones


= 8 tens + 7 ones
= 87
Solution to Exercise 1.144 (p. 34)
561

= 4 hundreds + 15 tens + 1 ten + 1 one


= 4 hundreds + 16 tens + 1 one
= 4 hundreds + 1 hundred + 6 tens + 1 one
= 5 hundreds + 6 tens + 1 one
= 561
Solution to Exercise 1.145 (p. 34)
115

= 10 tens + 1 ten + 5 ones


= 11 tens + 5 ones
= 1 hundred + 1 ten + 5 ones
= 115
Solution to Exercise 1.146 (p. 34)
153,525
Solution to Exercise 1.147 (p. 34)
167
Solution to Exercise 1.148 (p. 35)
2,468
Solution to Exercise 1.149 (p. 35)
434,826
Solution to Exercise 1.150 (p. 36)
155
Solution to Exercise 1.151 (p. 36)
19,750
Solution to Exercise 1.152 (p. 36)
30,682
Solution to Exercise 1.153 (p. 36)
19
Solution to Exercise 1.155 (p. 36)
48
82 CHAPTER 1. ADDITION AND SUBTRACTION OF WHOLE NUMBERS

Solution to Exercise 1.157 (p. 36)


98
Solution to Exercise 1.159 (p. 36)
978
Solution to Exercise 1.161 (p. 37)
368
Solution to Exercise 1.163 (p. 37)
12,777
Solution to Exercise 1.165 (p. 37)
58,738
Solution to Exercise 1.167 (p. 37)
45,169,739
Solution to Exercise 1.169 (p. 37)
33
Solution to Exercise 1.171 (p. 37)
81
Solution to Exercise 1.173 (p. 37)
91
Solution to Exercise 1.175 (p. 37)
809
Solution to Exercise 1.177 (p. 38)
862
Solution to Exercise 1.179 (p. 38)
2,321
Solution to Exercise 1.181 (p. 38)
1,141,204
Solution to Exercise 1.183 (p. 38)
562,364,111
Solution to Exercise 1.185 (p. 38)
942,302,364,207,060
Solution to Exercise 1.187 (p. 38)
234
Solution to Exercise 1.189 (p. 38)
95,365
Solution to Exercise 1.191 (p. 39)
1,972,128
Solution to Exercise 1.193 (p. 39)
3,700
Solution to Exercise 1.195 (p. 39)
3,101,500
Solution to Exercise 1.197 (p. 39)
100
Solution to Exercise 1.199 (p. 40)
0
Solution to Exercise 1.201 (p. 40)
1,000
Solution to Exercise 1.203 (p. 40)
5
Solution to Exercise 1.205 (p. 40)
19
83

Solution to Exercise 1.207 (p. 40)


88
Solution to Exercise 1.209 (p. 41)
60,511,000
Solution to Exercise 1.211 (p. 41)
5,682,651 square miles
Solution to Exercise 1.213 (p. 41)
1,190,000
Solution to Exercise 1.215 (p. 42)
271,564,000
Solution to Exercise 1.217 (p. 43)
20
Solution to Exercise 1.219 (p. 43)
25
Solution to Exercise 1.221 (p. 43)
40
Solution to Exercise 1.223 (p. 43)
50
Solution to Exercise 1.225 (p. 44)
50
Solution to Exercise 1.227 (p. 44)
50
Solution to Exercise 1.229 (p. 44)
0
Solution to Exercise 1.231 (p. 44)
6,800
Solution to Exercise 1.233 (p. 45)
7 − 5 = 2 since 2 + 5 = 7
Solution to Exercise 1.234 (p. 45)
9 − 1 = 8 since 8 + 1 = 9
Solution to Exercise 1.235 (p. 45)
17 − 8 = 9 since 9 + 8 = 17
Solution to Exercise 1.236 (p. 47)
331
Solution to Exercise 1.237 (p. 47)
814
Solution to Exercise 1.238 (p. 47)
61,118
Solution to Exercise 1.239 (p. 47)
10,000
Solution to Exercise 1.240 (p. 47)
62,114
84 CHAPTER 1. ADDITION AND SUBTRACTION OF WHOLE NUMBERS

Solution to Exercise 1.241 (p. 49)

Solution to Exercise 1.242 (p. 49)

Solution to Exercise 1.243 (p. 49)

Solution to Exercise 1.244 (p. 49)


85
Solution to Exercise 1.245 (p. 49)
709
Solution to Exercise 1.246 (p. 49)
3,104
Solution to Exercise 1.247 (p. 50)
168
Solution to Exercise 1.248 (p. 50)
55,640
Solution to Exercise 1.249 (p. 50)
1,189
Solution to Exercise 1.250 (p. 52)
888
85

Solution to Exercise 1.251 (p. 52)


4,543
Solution to Exercise 1.252 (p. 52)
8,669
Solution to Exercise 1.253 (p. 54)
16,134
Solution to Exercise 1.254 (p. 54)
4,839
Solution to Exercise 1.255 (p. 54)
15,789,940
Solution to Exercise 1.256 (p. 55)
4,529
Solution to Exercise 1.257 (p. 55)
22,468,227
Solution to Exercise 1.258 (p. 55)
7
Solution to Exercise 1.260 (p. 55)
6
Solution to Exercise 1.262 (p. 55)
3
Solution to Exercise 1.264 (p. 55)
41
Solution to Exercise 1.266 (p. 55)
209
Solution to Exercise 1.268 (p. 56)
21,001
Solution to Exercise 1.270 (p. 56)
279,253
Solution to Exercise 1.272 (p. 56)
77,472
Solution to Exercise 1.274 (p. 56)
24
Solution to Exercise 1.276 (p. 56)
188
Solution to Exercise 1.278 (p. 56)
2,377
Solution to Exercise 1.280 (p. 56)
26,686
Solution to Exercise 1.282 (p. 57)
63,143,259
Solution to Exercise 1.284 (p. 57)
8,034
Solution to Exercise 1.286 (p. 57)
33
Solution to Exercise 1.288 (p. 57)
32,611,636,567
Solution to Exercise 1.290 (p. 57)
3,938
Solution to Exercise 1.292 (p. 57)
8,273,955
86 CHAPTER 1. ADDITION AND SUBTRACTION OF WHOLE NUMBERS

Solution to Exercise 1.294 (p. 58)


51
Solution to Exercise 1.296 (p. 58)
3,405
Solution to Exercise 1.298 (p. 58)
26
Solution to Exercise 1.300 (p. 58)
72,069
Solution to Exercise 1.302 (p. 58)
3,197
Solution to Exercise 1.304 (p. 58)
29
Solution to Exercise 1.306 (p. 58)
10,385
Solution to Exercise 1.308 (p. 58)
15%
Solution to Exercise 1.310 (p. 58)
11,247,000
Solution to Exercise 1.312 (p. 59)
436
Solution to Exercise 1.314 (p. 59)
57,187,000
Solution to Exercise 1.316 (p. 59)
165,000
Solution to Exercise 1.318 (p. 59)
74
Solution to Exercise 1.320 (p. 60)
4,547
Solution to Exercise 1.322 (p. 60)
11
Solution to Exercise 1.324 (p. 61)
10
Solution to Exercise 1.326 (p. 61)
12
Solution to Exercise 1.328 (p. 61)
76
Solution to Exercise 1.330 (p. 61)
51
Solution to Exercise 1.332 (p. 61)
60
Solution to Exercise 1.334 (p. 61)
165
Solution to Exercise 1.336 (p. 61)
214
Solution to Exercise 1.338 (p. 62)
330 (answers may vary)
Solution to Exercise 1.340 (p. 62)
27,000,000
Solution to Exercise 1.342 (p. 63)
12 + 41 = 53 and 41 + 12 = 53
87

Solution to Exercise 1.343 (p. 63)


837 + 1, 958 = 2, 795 and 1, 958 + 837 = 2, 795
Solution to Exercise 1.344 (p. 63)
(17 + 32) + 25 = 49 + 25 = 74 and 17 + (32 + 25) = 17 + 57 = 74
Solution to Exercise 1.345 (p. 63)
(1, 629 + 806) + 429 = 2, 435 + 429 = 2, 864

1, 629 + (806 + 429) = 1, 629 + 1, 235 = 2, 864


Solution to Exercise 1.346 (p. 64)
8
Solution to Exercise 1.347 (p. 64)
5
Solution to Exercise 1.348 (p. 64)
x
Solution to Exercise 1.349 (p. 64)
x
Solution to Exercise 1.350 (p. 65)
17
Solution to Exercise 1.351 (p. 65)
37
Solution to Exercise 1.353 (p. 65)
45
Solution to Exercise 1.355 (p. 65)
568
Solution to Exercise 1.357 (p. 65)
122,323
Solution to Exercise 1.359 (p. 66)
45
Solution to Exercise 1.361 (p. 66)
100
Solution to Exercise 1.363 (p. 66)
556
Solution to Exercise 1.365 (p. 66)
43,461
Solution to Exercise 1.367 (p. 66)
132 + 6 =80 + 58 = 138
Solution to Exercise 1.369 (p. 67)
987 + 171 =731 + 427 = 1, 158
Solution to Exercise 1.371 (p. 67)
Identity
Solution to Exercise 1.373 (p. 67)
15 + 8 = 8 + 15 = 23
Solution to Exercise 1.375 (p. 67)
. . .because its partner in addition remains identically the same after that addition
Solution to Exercise 1.377 (p. 67)
Two thousand, two hundred eighteen.
Solution to Exercise 1.379 (p. 67)
550
Solution to Exercise 1.381 (p. 69)
937
Solution to Exercise 1.383 (p. 69)
565
88 CHAPTER 1. ADDITION AND SUBTRACTION OF WHOLE NUMBERS

Solution to Exercise 1.385 (p. 70)


559
Solution to Exercise 1.387 (p. 70)
1,342
Solution to Exercise 1.389 (p. 70)
2,001
Solution to Exercise 1.391 (p. 70)
1,963
Solution to Exercise 1.393 (p. 70)
79,456
Solution to Exercise 1.395 (p. 70)
96,953
Solution to Exercise 1.397 (p. 70)
791,824
Solution to Exercise 1.399 (p. 71)
8,301
Solution to Exercise 1.401 (p. 71)
140,381
Solution to Exercise 1.403 (p. 71)
76,224
Solution to Exercise 1.405 (p. 71)
4,955
Solution to Exercise 1.407 (p. 71)
185,611
Solution to Exercise 1.409 (p. 71)
2,238
Solution to Exercise 1.411 (p. 72)
1,338
Solution to Exercise 1.413 (p. 72)
878
Solution to Exercise 1.415 (p. 72)
618,227
Solution to Exercise 1.417 (p. 72)
1,621
Solution to Exercise 1.419 (p. 72)
484,601
Solution to Exercise 1.421 (p. 73)
19,853
Solution to Exercise 1.423 (p. 73)
1,702
Solution to Exercise 1.425 (p. 73)
1,114
Solution to Exercise 1.427 (p. 73)
1,300
Solution to Exercise 1.429 (p. 73)
2,718
Solution to Exercise 1.431 (p. 73)
7,356
Solution to Exercise 1.433 (p. 73)
3,415
89

Solution to Exercise 1.435 (p. 73)


11,827
Solution to Exercise 1.437 (p. 74)
407,262
Solution to Exercise 1.439 (p. 74)
718,478
Solution to Exercise 1.441 (p. 74)
9,941
Solution to Exercise 1.443 (p. 74)
626 + 1, 242 = 1, 242 + 626 = 1, 868
Solution to Exercise 1.444 (p. 74)
9
Solution to Exercise 1.445 (p. 74)
ones, tens, hundreds
Solution to Exercise 1.446 (p. 74)
8
Solution to Exercise 1.447 (p. 74)
no
Solution to Exercise 1.448 (p. 74)

Solution to Exercise 1.449 (p. 74)


Sixty-three thousand, four hundred twenty-ve
Solution to Exercise 1.450 (p. 74)
18,359,072
Solution to Exercise 1.451 (p. 74)
400
Solution to Exercise 1.452 (p. 74)
19,000
Solution to Exercise 1.453 (p. 75)
500
Solution to Exercise 1.454 (p. 75)
675
Solution to Exercise 1.455 (p. 75)
4,027
Solution to Exercise 1.456 (p. 75)
188
Solution to Exercise 1.457 (p. 75)
23,501
Solution to Exercise 1.458 (p. 75)
90
Solution to Exercise 1.459 (p. 75)
304
Solution to Exercise 1.460 (p. 75)
70,123
Solution to Exercise 1.461 (p. 75)
391
Solution to Exercise 1.462 (p. 75)
182
Solution to Exercise 1.463 (p. 75)
Yes, commutative property of addition
90 CHAPTER 1. ADDITION AND SUBTRACTION OF WHOLE NUMBERS
Chapter 2

Multiplication and Division of Whole

Numbers

2.1 Objectives1
After completing this chapter, you should
Multiplication of Whole Numbers (Section 2.2)

• understand the process of multiplication


• be able to multiply whole numbers
• be able to simplify multiplications with numbers ending in zero
• be able to use a calculator to multiply one whole number by another

Concepts of Division of Whole Numbers (Section 2.3)

• understand the process of division


• understand division of a nonzero number into zero
• understand why division by zero is undened
• be able to use a calculator to divide one whole number by another

Division of Whole Numbers (Section 2.4)

• be able to divide a whole number by a single or multiple digit divisor


• be able to interpret a calculator statement that a division results in a remainder

Some Interesting Facts about Division (Section 2.5)

• be able to recognize a whole number that is divisible by 2, 3, 4, 5, 6, 8, 9, or 10

Properties of Multiplication (Section 2.6)

• understand and appreciate the commutative and associative properties of multiplication


• understand why 1 is the multiplicative identity

1 This content is available online at <http://cnx.org/content/m18888/1.4/>.

91
92 CHAPTER 2. MULTIPLICATION AND DIVISION OF WHOLE NUMBERS

2.2 Multiplication of Whole Numbers2


2.2.1 Section Overview
• Multiplication
• The Multiplication Process With a Single Digit Multiplier
• The Multiplication Process With a Multiple Digit Multiplier
• Multiplication With Numbers Ending in Zero
• Calculators

2.2.2 Multiplication
Multiplication is a description of repeated addition.

In the addition of

5+5+5

the number 5 is repeated 3 times. Therefore, we say we have three times ve and describe it by writing

3×5

Thus,

3×5=5+5+5

Multiplicand
In a multiplication, the repeated addend (number being added) is called the multiplicand. In 3 × 5, the 5
is the multiplicand.

Multiplier
Also, in a multiplication, the number that records the number of times the multiplicand is used is called the
multiplier. In 3 × 5, the 3 is the multiplier.

2.2.2.1 Sample Set A

Express each repeated addition as a multiplication. In each case, specify the multiplier and the multiplicand.
Example 2.1
7+7+7+7+7+7

6 × 7. Multiplier is 6. Multiplicand is 7.
Example 2.2
18 + 18 + 18

3 × 18. Multiplier is 3. Multiplicand is 18.

2 This content is available online at <http://cnx.org/content/m34863/1.2/>.


93

2.2.2.2 Practice Set A

Express each repeated addition as a multiplication. In each case, specify the multiplier and the multiplicand.
Exercise 2.1 (Solution on p. 141.)
12 + 12 + 12 + 12

. Multiplier is . Multiplicand is .
Exercise 2.2 (Solution on p. 141.)
36 + 36 + 36 + 36 + 36 + 36 + 36 + 36

. Multiplier is . Multiplicand is .
Exercise 2.3 (Solution on p. 141.)
0+0+0+0+0

. Multiplier is . Multiplicand is .
Exercise 2.4 (Solution on p. 141.)
1847 + 1847 + ... + 1847
| {z }
12, 000 times

. Multiplier is . Multiplicand is .
Factors

In a multiplication, the numbers being multiplied are also called factors.

Products
The result of a multiplication is called the product. In 3 × 5 = 15, the 3 and 5 are not only called the
multiplier and multiplicand, but they are also called factors. The product is 15.

Indicators of Multiplication ×,·,( )


The multiplication symbol (×) is not the only symbol used to indicate multiplication. Other symbols include
the dot ( · ) and pairs of parentheses ( ). The expressions

3 × 5, 3 · 5, 3 (5), (3) 5, (3) (5)

all represent the same product.

2.2.3 The Multiplication Process With a Single Digit Multiplier


Since multiplication is repeated addition, we should not be surprised to notice that carrying can occur.
Carrying occurs when we nd the product of 38 and 7:

First, we compute 7 × 8 = 56. Write the 6 in the ones column. Carry the 5. Then take 7 × 3 = 21. Add to
21 the 5 that was carried: 21 + 5 = 26. The product is 266.
94 CHAPTER 2. MULTIPLICATION AND DIVISION OF WHOLE NUMBERS

2.2.3.1 Sample Set B

Find the following products.


Example 2.3

3 × 4 = 12 Write the 2, carry the 1.


3 × 6 = 18 Add to 18 the 1 that was carried: 18 + 1 = 19.

The product is 192.


Example 2.4

5 × 6 = 30 Write the 0, carry the 3.


5 × 2 = 10 Add to 10 the 3 that was carried: 10 + 3 = 13. Write the 3, carry the 1.
5 × 5 = 25 Add to 25 the 1 that was carried: 25 + 1 = 6.

The product is 2,630.


Example 2.5

9 × 4 = 36 Write the 6, carry the 3.


9×0=0 Add to the 0 the 3 that was carried: 0 + 3 = 3. Write the 3.
9 × 8 = 72 Write the 2, carry the 7.
Add to the 9 the 7 that was carried: 9 + 7 = 16.
9×1=9
Since there are no more multiplications to perform,write both the 1 and 6.

The product is 16,236.

2.2.3.2 Practice Set B

Find the following products.


Exercise 2.5 (Solution on p. 141.)
37
× 5
95

Exercise 2.6 (Solution on p. 141.)


78
× 8
Exercise 2.7 (Solution on p. 141.)
536
× 7
Exercise 2.8 (Solution on p. 141.)
40, 019
× 8
Exercise 2.9 (Solution on p. 141.)
301, 599
× 3

2.2.4 The Multiplication Process With a Multiple Digit Multiplier


In a multiplication in which the multiplier is composed of two or more digits, the multiplication must take
place in parts. The process is as follows:
Part 1: First Partial Product Multiply the multiplicand by the ones digit of the multiplier. This product
is called the rst partial product.
Part 2: Second Partial Product Multiply the multiplicand by the tens digit of the multiplier. This
product is called the second partial product. Since the tens digit is used as a factor, the second
partial product is written below the rst partial product so that its rightmost digit appears in the tens
column.
Part 3: If necessary, continue this way nding partial products. Write each one below the previous one so
that the rightmost digit appears in the column directly below the digit that was used as a factor.
Part 4: Total Product Add the partial products to obtain the total product.

note: It may be necessary to carry when nding each partial product.

2.2.4.1 Sample Set C

Example 2.6
Multiply 326 by 48.

Part 1:

Part 2:
96 CHAPTER 2. MULTIPLICATION AND DIVISION OF WHOLE NUMBERS

Part 3: This step is unnecessary since all of the digits in the multiplier have been used.
Part 4: Add the partial products to obtain the total product.

The product is 15,648.

Example 2.7
Multiply 5,369 by 842.

Part 1:

Part 2:

Part 3:
The product is 4,520,698.

Example 2.8
Multiply 1,508 by 206.

Part 1:

Part 2:
97

Since 0 times 1508 is 0, the partial product will not change the identity of the total product
(which is obtained by addition). Go to the next partial product.

Part 3:
The product is 310,648

2.2.4.2 Practice Set C

Exercise 2.10 (Solution on p. 141.)


Multiply 73 by 14.
Exercise 2.11 (Solution on p. 141.)
Multiply 86 by 52.
Exercise 2.12 (Solution on p. 141.)
Multiply 419 by 85.
Exercise 2.13 (Solution on p. 141.)
Multiply 2,376 by 613.
Exercise 2.14 (Solution on p. 141.)
Multiply 8,107 by 304.
Exercise 2.15 (Solution on p. 141.)
Multiply 66,260 by 1,008.
Exercise 2.16 (Solution on p. 141.)
Multiply 209 by 501.
Exercise 2.17 (Solution on p. 141.)
Multiply 24 by 10.
Exercise 2.18 (Solution on p. 141.)
Multiply 3,809 by 1,000.
Exercise 2.19 (Solution on p. 141.)
Multiply 813 by 10,000.

2.2.5 Multiplications With Numbers Ending in Zero


Often, when performing a multiplication, one or both of the factors will end in zeros. Such multiplications
can be done quickly by aligning the numbers so that the rightmost nonzero digits are in the same column.
98 CHAPTER 2. MULTIPLICATION AND DIVISION OF WHOLE NUMBERS

2.2.5.1 Sample Set D

Perform the multiplication (49, 000) (1, 200).

(49,000)(1,200) = 49000
× 1200

Since 9 and 2 are the rightmost nonzero digits, put them in the same column.

Draw (perhaps mentally) a vertical line to separate the zeros from the nonzeros.

Multiply the numbers to the left of the vertical line as usual, then attach to the right end of this product
the total number of zeros.

The product is 58,800,000

2.2.5.2 Practice Set D

Exercise 2.20 (Solution on p. 141.)


Multiply 1,800 by 90.
Exercise 2.21 (Solution on p. 141.)
Multiply 420,000 by 300.
Exercise 2.22 (Solution on p. 141.)
Multiply 20,500,000 by 140,000.

2.2.6 Calculators
Most multiplications are performed using a calculator.

2.2.6.1 Sample Set E

Example 2.9
Multiply 75,891 by 263.

Display Reads
Type 75891 75891
Press × 75891
Type 263 263
Press = 19959333
99

Table 2.1

The product is 19,959,333.


Example 2.10
Multiply 4,510,000,000,000 by 1,700.

Display Reads
Type 451 451
Press × 451
Type 17 17
Press = 7667

Table 2.2

The display now reads 7667. We'll have to add the zeros ourselves. There are a total of 12 zeros.
Attaching 12 zeros to 7667, we get 7,667,000,000,000,000.

The product is 7,667,000,000,000,000.


Example 2.11
Multiply 57,847,298 by 38,976.

Display Reads
Type 57847298 57847298
Press × 57847298
Type 38976 38976
Press = 2.2546563 12

Table 2.3

The display now reads 2.2546563 12. What kind of number is this? This is an example of a whole
number written in scientic notation. We'll study this concept when we get to decimal numbers.

2.2.6.2 Practice Set E

Use a calculator to perform each multiplication.


Exercise 2.23 (Solution on p. 141.)
52 × 27
Exercise 2.24 (Solution on p. 141.)
1, 448 × 6, 155
Exercise 2.25 (Solution on p. 141.)
8, 940, 000 × 205, 000
100 CHAPTER 2. MULTIPLICATION AND DIVISION OF WHOLE NUMBERS

2.2.7 Exercises
For the following problems, perform the multiplications. You may check each product with a calculator.
Exercise 2.26 (Solution on p. 141.)
8
×3
Exercise 2.27
3
×5
Exercise 2.28 (Solution on p. 142.)
8
×6
Exercise 2.29
5
×7
Exercise 2.30 (Solution on p. 142.)
6×1
Exercise 2.31
4×5
Exercise 2.32 (Solution on p. 142.)
75 × 3
Exercise 2.33
35 × 5
Exercise 2.34 (Solution on p. 142.)
45
× 6
Exercise 2.35
31
× 7
Exercise 2.36 (Solution on p. 142.)
97
× 6
Exercise 2.37
75
×57
Exercise 2.38 (Solution on p. 142.)
64
×15
Exercise 2.39
73
×15
101

Exercise 2.40 (Solution on p. 142.)


81
×95
Exercise 2.41
31
×33
Exercise 2.42 (Solution on p. 142.)
57 × 64
Exercise 2.43
76 × 42
Exercise 2.44 (Solution on p. 142.)
894 × 52
Exercise 2.45
684 × 38
Exercise 2.46 (Solution on p. 142.)
115
× 22
Exercise 2.47
706
× 81
Exercise 2.48 (Solution on p. 142.)
328
× 21
Exercise 2.49
550
× 94
Exercise 2.50 (Solution on p. 142.)
930 × 26
Exercise 2.51
318 × 63
Exercise 2.52 (Solution on p. 142.)
582
×127
Exercise 2.53
247
×116
Exercise 2.54 (Solution on p. 142.)
305
×225
102 CHAPTER 2. MULTIPLICATION AND DIVISION OF WHOLE NUMBERS

Exercise 2.55
782
×547
Exercise 2.56 (Solution on p. 142.)
771
×663
Exercise 2.57
638
×516
Exercise 2.58 (Solution on p. 142.)
1,905 × 710
Exercise 2.59
5,757 × 5,010
Exercise 2.60 (Solution on p. 142.)
3, 106
×1, 752
Exercise 2.61
9, 300
×1, 130
Exercise 2.62 (Solution on p. 142.)
7, 057
×5, 229
Exercise 2.63
8, 051
×5, 580
Exercise 2.64 (Solution on p. 142.)
5, 804
×4, 300
Exercise 2.65
357
×16
Exercise 2.66 (Solution on p. 142.)
724
× 0
Exercise 2.67
2, 649
× 41
Exercise 2.68 (Solution on p. 142.)
5, 173
× 8
103

Exercise 2.69
1, 999
× 0
Exercise 2.70 (Solution on p. 142.)
1, 666
× 0
Exercise 2.71
51, 730
× 142
Exercise 2.72 (Solution on p. 142.)
387
×190
Exercise 2.73
3, 400
× 70
Exercise 2.74 (Solution on p. 142.)
460, 000
× 14, 000
Exercise 2.75
558, 000, 000
× 81, 000
Exercise 2.76 (Solution on p. 142.)
37, 000
× 120
Exercise 2.77
498, 000
× 0
Exercise 2.78 (Solution on p. 143.)
4, 585, 000
× 140
Exercise 2.79
30, 700, 000
× 180
Exercise 2.80 (Solution on p. 143.)
8, 000
× 10
Exercise 2.81
Suppose a theater holds 426 people. If the theater charges $4 per ticket and sells every seat, how
much money would they take in?
104 CHAPTER 2. MULTIPLICATION AND DIVISION OF WHOLE NUMBERS

Exercise 2.82 (Solution on p. 143.)


In an English class, a student is expected to read 12 novels during the semester and prepare a report
on each one of them. If there are 32 students in the class, how many reports will be prepared?
Exercise 2.83
In a mathematics class, a nal exam consists of 65 problems. If this exam is given to 28 people,
how many problems must the instructor grade?
Exercise 2.84 (Solution on p. 143.)
A business law instructor gives a 45 problem exam to two of her classes. If each class has 37 people
in it, how many problems will the instructor have to grade?
Exercise 2.85
An algebra instructor gives an exam that consists of 43 problems to four of his classes. If the classes
have 25, 28, 31, and 35 students in them, how many problems will the instructor have to grade?
Exercise 2.86 (Solution on p. 143.)
In statistics, the term "standard deviation" refers to a number that is calculated from certain data.
If the data indicate that one standard deviation is 38 units, how many units is three standard
deviations?
Exercise 2.87
Soft drinks come in cases of 24 cans. If a supermarket sells 857 cases during one week, how many
individual cans were sold?
Exercise 2.88 (Solution on p. 143.)
There are 60 seconds in 1 minute and 60 minutes in 1 hour. How many seconds are there in 1 hour?
Exercise 2.89
There are 60 seconds in 1 minute, 60 minutes in one hour, 24 hours in one day, and 365 days in one
year. How many seconds are there in 1 year?
Exercise 2.90 (Solution on p. 143.)
Light travels 186,000 miles in one second. How many miles does light travel in one year? (Hint:
Can you use the result of the previous problem?)
Exercise 2.91
An elementary school cafeteria sells 328 lunches every day. Each lunch costs $1. How much money
does the cafeteria bring in in 2 weeks?
Exercise 2.92 (Solution on p. 143.)
A computer company is selling stock for $23 a share. If 87 people each buy 55 shares, how much
money would be brought in?

2.2.7.1 Exercises for Review

Exercise 2.93
(Section 1.2) In the number 421,998, how may ten thousands are there?
Exercise 2.94 (Solution on p. 143.)
(Section 1.4) Round 448,062,187 to the nearest hundred thousand.
Exercise 2.95
(Section 1.5) Find the sum. 22,451 + 18,976.
Exercise 2.96 (Solution on p. 143.)
(Section 1.6) Subtract 2,289 from 3,001.
Exercise 2.97
(Section 1.7) Specify which property of addition justies the fact that (a rst whole number + a
second whole number) = (the second whole number + the rst whole number)
105

2.3 Concepts of Division of Whole Numbers3


2.3.1 Section Overview
• Division
• Division into Zero (Zero As a Dividend: a0 , a 6= 0)
• Division by Zero (Zero As a Divisor: a0 , a 6= 0)
• Division by and into Zero (Zero As a Dividend and Divisor: 0)
0

• Calculators

2.3.2 Division
Division is a description of repeated subtraction.

In the process of division, the concern is how many times one number is contained in another number. For
example, we might be interested in how many 5's are contained in 15. The word times is signicant because
it implies a relationship between division and multiplication.

There are several notations used to indicate division. Suppose Q records the number of times 5 is contained
in 15. We can indicate this by writing

Q 15
=Q
5)15 |5 {z }
| {z } 15 divided by 5
5 into 15

15/5 = Q 15 ÷ 5 = Q
| {z } | {z }
15 divided by 5 15 divided by 5

Each of these division notations describes the same number, represented here by the symbol Q. Each notation
also converts to the same multiplication form. It is 15 = 5 × Q

In division,

Dividend
the number being divided into is called the dividend.

Divisor
the number dividing into the dividend is the divisor.

Quotient
the result of the division is called the quotient.

quotient
divisor)dividend
dividend = quotient
divisor
dividend/divisor = quotient dividend ÷ divisor = quotient
3 This content is available online at <http://cnx.org/content/m34864/1.3/>.
106 CHAPTER 2. MULTIPLICATION AND DIVISION OF WHOLE NUMBERS

2.3.2.1 Sample Set A

Find the following quotients using multiplication facts.


Example 2.12
18 ÷ 6

Since 6 × 3 = 18,

18 ÷ 6 = 3

Notice also that

18
−6
12
−6 }Repeated subtraction
6
−6
0

Thus, 6 is contained in 18 three times.


Example 2.13
24
3

Since 3 × 8 = 24,
24 = 8
3

Notice also that 3 could be subtracted exactly 8 times from 24. This implies that 3 is contained in
24 eight times.
Example 2.14
36
6

Since 6 × 6 = 36,
36
6 =6

Thus, there are 6 sixes in 36.


Example 2.15
9)72

Since 9 × 8 = 72,

8
9)72

Thus, there are 8 nines in 72.


107

2.3.2.2 Practice Set A

Use multiplication facts to determine the following quotients.


Exercise 2.98 (Solution on p. 143.)
32 ÷ 8
Exercise 2.99 (Solution on p. 143.)
18 ÷ 9
Exercise 2.100 (Solution on p. 143.)
25
5
Exercise 2.101 (Solution on p. 143.)
48
8
Exercise 2.102 (Solution on p. 143.)
28
7
Exercise 2.103 (Solution on p. 143.)
4)36

0
2.3.3 Division into Zero (Zero as a Dividend: a , a 6= 0)
Let's look at what happens when the dividend (the number being divided into) is zero, and the divisor (the
number doing the dividing) is any whole number except zero. The question is

What number, if any, is any nonzero whole


0
number ?
Let's represent this unknown quotient by Q. Then,
0
any nonzero whole number = Q
Converting this division problem to its corresponding multiplication problem, we get

0 = Q × (any nonzero whole number)

From our knowledge of multiplication, we can understand that if the product of two whole numbers is zero,
then one or both of the whole numbers must be zero. Since any nonzero whole number is certainly not zero,
Q must represent zero. Then,
0
any nonzero whole number = 0
Zero Divided By Any Nonzero Whole Number Is Zero
Zero divided any nonzero whole number is zero.

a
2.3.4 Division by Zero (Zero as a Divisor: 0 , a 6= 0)
Now we ask,

What number, if any, is any nonzero whole


0
number ?

Letting Q represent a possible quotient, we get


any nonzero whole number = Q
0

Converting to the corresponding multiplication form, we have


108 CHAPTER 2. MULTIPLICATION AND DIVISION OF WHOLE NUMBERS

(any nonzero whole number) = Q × 0

Since Q × 0 = 0, (any nonzero whole number) = 0. But this is absurd. This would mean that 6 = 0, or
37 = 0. A nonzero whole number cannot equal 0! Thus,
any nonzero whole number does not name a number
0

Division by Zero is Undened


Division by zero does not name a number. It is, therefore, undened.

0
2.3.5 Division by and Into Zero (Zero as a Dividend and Divisor: 0 )

We are now curious about zero divided by zero 0


. If we let Q represent a potential quotient, we get

0
0
0 =Q

Converting to the multiplication form,

0=Q×0

This results in

0=0

This is a statement that is true regardless of the number used in place of Q. For example,
0
0 = 5, since 0 = 5 × 0.
0
0 = 31, since 0 = 31 × 0.
0
0 = 286, since 0 = 286 × 0.

A unique quotient cannot be determined.

Indeterminant
Since the result of the division is inconclusive, we say that 0
0 is indeterminant.
0
0 is Indeterminant
The division 00 is indeterminant.

2.3.5.1 Sample Set B

Perform, if possible, each division.


Example 2.16
19 . Since division by 0 does not name a whole number, no quotient exists, and we state 19 is
0 0
undened
Example 2.17
0)14 . Since division by 0 does not name a dened number, no quotient exists, and we state
0)14 is undened
Example 2.18
0
9)0 . Since division into 0 by any nonzero whole number results in 0, we have
9)0
109

Example 2.19
7. Since division into 0 by any nonzero whole number results in 0, we have
0 0
7 =0

2.3.5.2 Practice Set B

Perform, if possible, the following divisions.


Exercise 2.104 (Solution on p. 143.)
5
0
Exercise 2.105 (Solution on p. 143.)
0
4
Exercise 2.106 (Solution on p. 143.)
0)0
Exercise 2.107 (Solution on p. 143.)
0)8
Exercise 2.108 (Solution on p. 143.)
9
0
Exercise 2.109 (Solution on p. 143.)
0
1

2.3.6 Calculators
Divisions can also be performed using a calculator.

2.3.6.1 Sample Set C

Example 2.20
Divide 24 by 3.

Display Reads
Type 24 24
Press ÷ 24
Type 3 3
Press = 8

Table 2.4

The display now reads 8, and we conclude that 24 ÷ 3 = 8.


Example 2.21
Divide 0 by 7.
110 CHAPTER 2. MULTIPLICATION AND DIVISION OF WHOLE NUMBERS

Display Reads
Type 0 0
Press ÷ 0
Type 7 7
Press = 0

Table 2.5

The display now reads 0, and we conclude that 0 ÷ 7 = 0.


Example 2.22
Divide 7 by 0.

Since division by zero is undened, the calculator should register some kind of error message.

Display Reads
Type 7 7
Press ÷ 7
Type 0 0
Press = Error

Table 2.6

The error message indicates an undened operation was attempted, in this case, division by zero.

2.3.6.2 Practice Set C

Use a calculator to perform each division.


Exercise 2.110 (Solution on p. 143.)
35 ÷ 7
Exercise 2.111 (Solution on p. 143.)
56 ÷ 8
Exercise 2.112 (Solution on p. 143.)
0÷6
Exercise 2.113 (Solution on p. 144.)
3÷0
Exercise 2.114 (Solution on p. 144.)
0÷0
111

2.3.7 Exercises
For the following problems, determine the quotients (if possible). You may use a calculator to check the
result.
Exercise 2.115 (Solution on p. 144.)
4)32
Exercise 2.116
7)42
Exercise 2.117 (Solution on p. 144.)
6)18
Exercise 2.118
2)14
Exercise 2.119 (Solution on p. 144.)
3)27
Exercise 2.120
1)6
Exercise 2.121 (Solution on p. 144.)
4)28
Exercise 2.122
30
5
Exercise 2.123 (Solution on p. 144.)
16
4
Exercise 2.124
24 ÷ 8
Exercise 2.125 (Solution on p. 144.)
10 ÷ 2
Exercise 2.126
21 ÷ 7
Exercise 2.127 (Solution on p. 144.)
21 ÷ 3
Exercise 2.128
0÷6
Exercise 2.129 (Solution on p. 144.)
8÷0
Exercise 2.130
12 ÷ 4
Exercise 2.131 (Solution on p. 144.)
3)9
Exercise 2.132
0)0
Exercise 2.133 (Solution on p. 144.)
7)0
Exercise 2.134
6)48
112 CHAPTER 2. MULTIPLICATION AND DIVISION OF WHOLE NUMBERS

Exercise 2.135 (Solution on p. 144.)


15
3
Exercise 2.136
35
0
Exercise 2.137 (Solution on p. 144.)
56 ÷ 7
Exercise 2.138
0
9
Exercise 2.139 (Solution on p. 144.)
72 ÷ 8
Exercise 2.140
Write 16
2 = 8 using three dierent notations.
Exercise 2.141 (Solution on p. 144.)
Write 27
9 = 3 using three dierent notations.
Exercise 2.142
4
In the statement
6)24

6 is called the .

24 is called the .

4 is called the .
Exercise 2.143 (Solution on p. 144.)
In the statement 56 ÷ 8 = 7,

7 is called the .

8 is called the .

56 is called the .

2.3.7.1 Exercises for Review

Exercise 2.144
(Section 1.2) What is the largest digit?
Exercise 2.145 (Solution on p. 144.)
8, 006
(Section 1.5) Find the sum.
+4, 118
Exercise 2.146
631
(Section 1.6) Find the dierence.
−589
Exercise 2.147 (Solution on p. 144.)
(Section 1.7) Use the numbers 2, 3, and 7 to illustrate the associative property of addition.
Exercise 2.148
86
(Section 2.2) Find the product.
× 12
113

2.4 Division of Whole Numbers 4


2.4.1 Section Overview
• Division with a Single Digit Divisor
• Division with a Multiple Digit Divisor
• Division with a Remainder
• Calculators

2.4.2 Division with a Single Digit Divisor


Our experience with multiplication of whole numbers allows us to perform such divisions as 75÷5. We perform
the division by performing the corresponding multiplication, 5 × Q = 75. Each division we considered in
Section 2.3 had a one-digit quotient. Now we will consider divisions in which the quotient may consist of
two or more digits. For example, 75 ÷ 5.

Let's examine the division 75 ÷ 5. We are asked to determine how many 5's are contained in 75. We'll
approach the problem in the following way.

1. Make an educated guess based on experience with multiplication.


2. Find how close the estimate is by multiplying the estimate by 5.
3. If the product obtained in step 2 is less than 75, nd out how much less by subtracting it from 75.
4. If the product obtained in step 2 is greater than 75, decrease the estimate until the product is less than
75. Decreasing the estimate makes sense because we do not wish to exceed 75.

We can suggest from this discussion that the process of division consists of
The Four Steps in Division

1. an educated guess
2. a multiplication
3. a subtraction
4. bringing down the next digit (if necessary)

The educated guess can be made by determining how many times the divisor is contained in the dividend
by using only one or two digits of the dividend.

2.4.2.1 Sample Set A

Example 2.23
Find 75 ÷ 5.

5)75 Rewrite the problem using a division bracket.

10
5)75
Make an educated guess by noting that one 5 is contained in 75 at most 10 times.
Since 7 is the tens digit, we estimate that 5 goes into 75 at most 10 times.

4 This content is available online at <http://cnx.org/content/m34865/1.2/>.


114 CHAPTER 2. MULTIPLICATION AND DIVISION OF WHOLE NUMBERS

10
5)75
−50
25
Now determine how close the estimate is.
10 ves is 10 × 5 = 50. Subtract 50 from 75.
Estimate the number of 5's in 25.
There are exactly 5 ves in 25.

5 10 ves + 5 ves = 15 ves.


}
10 There are 15 ves contained in 75.
5)75
−50
25
−25
0

Check:

Thus, 75 ÷ 5 = 15.

The notation in this division can be shortened by writing.

15
5)75
−5 ↓
25
−25
0
Divide: 5 goes into 7 at most 1 time. Divide: 5 goes into 25 exactly 5 times.
{ Multiply: 1 × 5 = 5. Write 5 below 7. { Multiply: 5 × 5 = 25. Write 25 below 25.
Subtract: 7 - 5 = 2. Bring down the 5. Subtract: 25 - 25 = 0.
Example 2.24
Find 4, 944 ÷ 8.

8)4944
Rewrite the problem using a division bracket.

600
8)4944
−4800
144
115

8 goes into 49 at most 6 times, and 9 is in the hundreds column. We'll guess 600.
Then, 8 × 600 = 4800.

10
600
8)4944
−4800
144
− 80
64
8 goes into 14 at most 1 time, and 4 is in the tens column. We'll guess 10.

8
10
600
8)4944
−4800
144
− 80
64
−64
0

8 goes into 64 exactly 8 times.


600 eights + 10 eights + 8 eights = 618 eights.

Check:

Thus, 4, 944 ÷ 8 = 618.

As in the rst problem, the notation in this division can be shortened by eliminating the subtraction
signs and the zeros in each educated guess.
116 CHAPTER 2. MULTIPLICATION AND DIVISION OF WHOLE NUMBERS

Divide: 8 goes into 49 at most 6 times. Divide: 8 goes into 14 at most 1 time.
{ Multiply: 6 × 8 = 48. Write 48 below 49. { Multiply: 1 × 8 = 8. Write 8 below 14.
Subtract: 49 - 48 = 1. Bring down the 4. Subtract: 14 - 8 = 6. Bring down the 4.
Divide: 8 goes into 64 exactly 8 times.
{ Multiply: 8 × 8 = 64. Write 64 below 64.
Subtract: 64 - 64 = 0.

note: Not all divisions end in zero. We will examine such divisions in a subsequent subsection.

2.4.2.2 Practice Set A

Perform the following divisions.


Exercise 2.149 (Solution on p. 144.)
126 ÷ 7
Exercise 2.150 (Solution on p. 144.)
324 ÷ 4
Exercise 2.151 (Solution on p. 144.)
2, 559 ÷ 3
Exercise 2.152 (Solution on p. 144.)
5, 645 ÷ 5
Exercise 2.153 (Solution on p. 144.)
757, 125 ÷ 9

2.4.3 Division with a Multiple Digit Divisor


The process of division also works when the divisor consists of two or more digits. We now make educated
guesses using the rst digit of the divisor and one or two digits of the dividend.

2.4.3.1 Sample Set B

Example 2.25
Find 2, 232 ÷ 36.

36)2232

Use the rst digit of the divisor and the rst two digits of the dividend to make the educated guess.

3 goes into 22 at most 7 times.

Try 7: 7 × 36 = 252 which is greater than 223. Reduce the estimate.

Try 6: 6 × 36 = 216 which is less than 223.


117

Multiply: 6 × 36 = 216. Write 216 below 223.


Subtract: 223 - 216 = 7. Bring down the 2.

Divide 3 into 7 to estimate the number of times 36 goes into 72. The 3 goes into 7 at most 2 times.

Try 2: 2 × 36 = 72.

Check :
Thus, 2, 232 ÷ 36 = 62.
Example 2.26
Find 2, 417, 228 ÷ 802.

802)2417228

First, the educated guess: 24 ÷ 8 = 3. Then 3 × 802 = 2406, which is less than 2417. Use 3 as the
guess. Since 3 × 802 = 2406, and 2406 has four digits, place the 3 above the fourth digit of the
dividend.

Subtract: 2417 - 2406 = 11.


Bring down the 2.

The divisor 802 goes into 112 at most 0 times. Use 0.


118 CHAPTER 2. MULTIPLICATION AND DIVISION OF WHOLE NUMBERS

Multiply: 0 × 802 = 0.
Subtract: 112 - 0 = 112.
Bring down the 2.

The 8 goes into 11 at most 1 time, and 1 × 802 = 802, which is less than 1122. Try 1.

Subtract 1122 − 802 = 320


Bring down the 8.

8 goes into 32 at most 4 times.

4 × 802 = 3208.

Use 4.

Check:

Thus, 2, 417, 228 ÷ 802 = 3, 014.

2.4.3.2 Practice Set B

Perform the following divisions.


Exercise 2.154 (Solution on p. 145.)
1, 376 ÷ 32
Exercise 2.155 (Solution on p. 145.)
6, 160 ÷ 55
Exercise 2.156 (Solution on p. 145.)
18, 605 ÷ 61
Exercise 2.157 (Solution on p. 145.)
144, 768 ÷ 48
119

2.4.4 Division with a Remainder


We might wonder how many times 4 is contained in 10. Repeated subtraction yields

10
− 4
6
−4
2

Since the remainder is less than 4, we stop the subtraction. Thus, 4 goes into 10 two times with 2 remaining.
We can write this as a division as follows.

2
4)10
− 8
2
Divide: 4 goes into 10 at most 2 times.
Multiply: 2 × 4 = 8. Write 8 below 0.
Subtract: 10 - 8 = 2.

Since 4 does not divide into 2 (the remainder is less than the divisor) and there are no digits to bring down
to continue the process, we are done. We write

2 R2
4) 10
or 10 ÷ 4 = 2R2
−8
|{z}
2 with remainder 2
2

2.4.4.1 Sample Set C

Example 2.27
Find 85 ÷ 3.
120 CHAPTER 2. MULTIPLICATION AND DIVISION OF WHOLE NUMBERS

Divide: 3 goes into 8 at most 2 times. Divide: 3 goes into 25 at most 8 times.
{ Multiply: 2 × 3 = 6. Write 6 below 8. { Multiply: 3 × 8 = 24. Write 24 below 25.
Subtract: 8 - 6 = 2. Bring down the 5. Subtract: 25 - 24 = 1.

There are no more digits to bring down to continue the process. We are done. One is the remainder.

Check: Multiply 28 and 3, then add 1.

28
× 3
84
+ 1
85

Thus, 85 ÷ 3 = 28R1.
Example 2.28
Find 726 ÷ 23.

Check: Multiply 31 by 23, then add 13.

Thus, 726 ÷ 23 = 31R13.

2.4.4.2 Practice Set C

Perform the following divisions.


Exercise 2.158 (Solution on p. 145.)
75 ÷ 4
Exercise 2.159 (Solution on p. 145.)
346 ÷ 8
Exercise 2.160 (Solution on p. 145.)
489 ÷ 21
Exercise 2.161 (Solution on p. 145.)
5, 016 ÷ 82
121

Exercise 2.162 (Solution on p. 145.)


41, 196 ÷ 67

2.4.5 Calculators
The calculator can be useful for nding quotients with single and multiple digit divisors. If, however, the
division should result in a remainder, the calculator is unable to provide us with the particular value of the
remainder. Also, some calculators (most nonscientic) are unable to perform divisions in which one of the
numbers has more than eight digits.

2.4.5.1 Sample Set D

Use a calculator to perform each division.


Example 2.29
328 ÷ 8

Type 328
Press ÷
Type 8
Press =

Table 2.7

The display now reads 41.


Example 2.30
53, 136 ÷ 82

Type 53136
Press ÷
Type 82
Press =

Table 2.8

The display now reads 648.


Example 2.31
730, 019, 001 ÷ 326

We rst try to enter 730,019,001 but nd that we can only enter 73001900. If our calculator has only
an eight-digit display (as most nonscientic calculators do), we will be unable to use the calculator
to perform this division.
Example 2.32
3727 ÷ 49
122 CHAPTER 2. MULTIPLICATION AND DIVISION OF WHOLE NUMBERS

Type 3727
Press ÷
Type 49
Press =

Table 2.9

The display now reads 76.061224.

This number is an example of a decimal number (see Section 6.1). When a decimal number results
in a calculator division, we can conclude that the division produces a remainder.

2.4.5.2 Practice Set D

Use a calculator to perform each division.


Exercise 2.163 (Solution on p. 145.)
3, 330 ÷ 74
Exercise 2.164 (Solution on p. 145.)
63, 365 ÷ 115
Exercise 2.165 (Solution on p. 145.)
21, 996, 385, 287 ÷ 53
Exercise 2.166 (Solution on p. 145.)
4, 558 ÷ 67

2.4.6 Exercises
For the following problems, perform the divisions.

The rst 38 problems can be checked with a calculator by multiplying the divisor and quotient then adding
the remainder.
Exercise 2.167 (Solution on p. 145.)
52 ÷ 4
Exercise 2.168
776 ÷ 8
Exercise 2.169 (Solution on p. 145.)
603 ÷ 9
Exercise 2.170
240 ÷ 8
Exercise 2.171 (Solution on p. 145.)
208 ÷ 4
Exercise 2.172
576 ÷ 6
Exercise 2.173 (Solution on p. 145.)
21 ÷ 7
123

Exercise 2.174
0÷0
Exercise 2.175 (Solution on p. 145.)
140 ÷ 2
Exercise 2.176
528 ÷ 8
Exercise 2.177 (Solution on p. 145.)
244 ÷ 4
Exercise 2.178
0÷7
Exercise 2.179 (Solution on p. 145.)
177 ÷ 3
Exercise 2.180
96 ÷ 8
Exercise 2.181 (Solution on p. 145.)
67 ÷ 1
Exercise 2.182
896 ÷ 56
Exercise 2.183 (Solution on p. 145.)
1, 044 ÷ 12
Exercise 2.184
988 ÷ 19
Exercise 2.185 (Solution on p. 145.)
5, 238 ÷ 97
Exercise 2.186
2, 530 ÷ 55
Exercise 2.187 (Solution on p. 145.)
4, 264 ÷ 82
Exercise 2.188
637 ÷ 13
Exercise 2.189 (Solution on p. 146.)
3, 420 ÷ 90
Exercise 2.190
5, 655 ÷ 87
Exercise 2.191 (Solution on p. 146.)
2, 115 ÷ 47
Exercise 2.192
9, 328 ÷ 22
Exercise 2.193 (Solution on p. 146.)
55, 167 ÷ 71
Exercise 2.194
68, 356 ÷ 92
Exercise 2.195 (Solution on p. 146.)
27, 702 ÷ 81
Exercise 2.196
6, 510 ÷ 31
124 CHAPTER 2. MULTIPLICATION AND DIVISION OF WHOLE NUMBERS

Exercise 2.197 (Solution on p. 146.)


60, 536 ÷ 94
Exercise 2.198
31, 844 ÷ 38
Exercise 2.199 (Solution on p. 146.)
23, 985 ÷ 45
Exercise 2.200
60, 606 ÷ 74
Exercise 2.201 (Solution on p. 146.)
2, 975, 400 ÷ 285
Exercise 2.202
1, 389, 660 ÷ 795
Exercise 2.203 (Solution on p. 146.)
7, 162, 060 ÷ 879
Exercise 2.204
7, 561, 060 ÷ 909
Exercise 2.205 (Solution on p. 146.)
38 ÷ 9
Exercise 2.206
97 ÷ 4
Exercise 2.207 (Solution on p. 146.)
199 ÷ 3
Exercise 2.208
573 ÷ 6
Exercise 2.209 (Solution on p. 146.)
10, 701 ÷ 13
Exercise 2.210
13, 521 ÷ 53
Exercise 2.211 (Solution on p. 146.)
3, 628 ÷ 90
Exercise 2.212
10, 592 ÷ 43
Exercise 2.213 (Solution on p. 146.)
19, 965 ÷ 30
Exercise 2.214
8, 320 ÷ 21
Exercise 2.215 (Solution on p. 146.)
61, 282 ÷ 64
Exercise 2.216
1, 030 ÷ 28
Exercise 2.217 (Solution on p. 146.)
7, 319 ÷ 11
Exercise 2.218
3, 628 ÷ 90
Exercise 2.219 (Solution on p. 146.)
35, 279 ÷ 77
125

Exercise 2.220
52, 196 ÷ 55
Exercise 2.221 (Solution on p. 146.)
67, 751 ÷ 68
For the following 5 problems, use a calculator to nd the quotients.
Exercise 2.222
4, 346 ÷ 53
Exercise 2.223 (Solution on p. 146.)
3, 234 ÷ 77
Exercise 2.224
6, 771 ÷ 37
Exercise 2.225 (Solution on p. 146.)
4, 272, 320 ÷ 520
Exercise 2.226
7, 558, 110 ÷ 651
Exercise 2.227 (Solution on p. 146.)
A mathematics instructor at a high school is paid $17,775 for 9 months. How much money does
this instructor make each month?
Exercise 2.228
A couple pays $4,380 a year for a one-bedroom apartment. How much does this couple pay each
month for this apartment?
Exercise 2.229 (Solution on p. 146.)
Thirty-six people invest a total of $17,460 in a particular stock. If they each invested the same
amount, how much did each person invest?
Exercise 2.230
Each of the 28 students in a mathematics class buys a textbook. If the bookstore sells $644 worth
of books, what is the price of each book?
Exercise 2.231 (Solution on p. 146.)
A certain brand of refrigerator has an automatic ice cube maker that makes 336 ice cubes in one
day. If the ice machine makes ice cubes at a constant rate, how many ice cubes does it make each
hour?
Exercise 2.232
A beer manufacturer bottles 52,380 ounces of beer each hour. If each bottle contains the same
number of ounces of beer, and the manufacturer lls 4,365 bottles per hour, how many ounces of
beer does each bottle contain?
Exercise 2.233 (Solution on p. 146.)
A computer program consists of 68,112 bits. 68,112 bits equals 8,514 bytes. How many bits in one
byte?
Exercise 2.234
A 26-story building in San Francisco has a total of 416 oces. If each oor has the same number
of oces, how many oors does this building have?
Exercise 2.235 (Solution on p. 146.)
A college has 67 classrooms and a total of 2,546 desks. How many desks are in each classroom if
each classroom has the same number of desks?
126 CHAPTER 2. MULTIPLICATION AND DIVISION OF WHOLE NUMBERS

2.4.6.1 Exercises for Review

Exercise 2.236
(Section 1.2) What is the value of 4 in the number 124,621?
Exercise 2.237 (Solution on p. 146.)
(Section 1.4) Round 604,092 to the nearest hundred thousand.
Exercise 2.238
(Section 1.7) What whole number is the additive identity?
Exercise 2.239 (Solution on p. 147.)
(Section 2.2) Find the product. 6, 256 × 100.
Exercise 2.240
(Section 2.3) Find the quotient. 0 ÷ 11.

2.5 Some Interesting Facts about Division 5


2.5.1 Section Overview
• Division by 2, 3, 4, and 5
• Division by 6, 8, 9, and 10

Quite often, we are able to determine if a whole number is divisible by another whole number just by
observing some simple facts about the number. Some of these facts are listed in this section.

2.5.2 Division by 2, 3, 4, and 5


Division by 2
A whole number is divisible by 2 if its last digit is 0, 2, 4, 6, or 8.

The numbers 80, 112, 64, 326, and 1,008 are all divisible by 2 since the last digit of each is 0, 2, 4,
6, or 8, respectively.

The numbers 85 and 731 are not divisible by 2.

Division by 3
A whole number is divisible by 3 if the sum of its digits is divisible by 3.

The number 432 is divisible by 3 since 4 + 3 + 2 = 9 and 9 is divisible by 3.

432 ÷ 3 = 144

The number 25 is not divisible by 3 since 2 + 5 = 7, and 7 is not divisible by 3.

Division by 4
A whole number is divisible by 4 if its last two digits form a number that is divisible by 4.

The number 31,048 is divisible by 4 since the last two digits, 4 and 8, form a number, 48, that is
divisible by 4.

31048 ÷ 4 = 7262
5 This content is available online at <http://cnx.org/content/m34866/1.2/>.
127

The number 137 is not divisible by 4 since 37 is not divisible by 4.

Division by 5
A whole number is divisible by 5 if its last digit is 0 or 5.

2.5.2.1 Sample Set A

Example 2.33
The numbers 65, 110, 8,030, and 16,955 are each divisible by 5 since the last digit of each is 0 or 5.

2.5.2.2 Practice Set A

State which of the following whole numbers are divisible by 2, 3, 4, or 5. A number may be divisible by
more than one number.
Exercise 2.241 (Solution on p. 147.)
26
Exercise 2.242 (Solution on p. 147.)
81
Exercise 2.243 (Solution on p. 147.)
51
Exercise 2.244 (Solution on p. 147.)
385
Exercise 2.245 (Solution on p. 147.)
6,112
Exercise 2.246 (Solution on p. 147.)
470
Exercise 2.247 (Solution on p. 147.)
113,154

2.5.3 Division by 6, 8, 9, 10
Division by 6
A number is divisible by 6 if it is divisible by both 2 and 3.

The number 234 is divisible by 2 since its last digit is 4. It is also divisible by 3 since 2 + 3 + 4 = 9 and 9 is
divisible by 3. Therefore, 234 is divisible by 6.

The number 6,532 is not divisible by 6. Although its last digit is 2, making it divisible by 2, the
sum of its digits, 6 + 5 + 3 + 2 = 16, and 16 is not divisible by 3.

Division by 8
A whole number is divisible by 8 if its last three digits form a number that is divisible by 8.
The number 4,000 is divisible by 8 since 000 is divisible by 8.
The number 13,128 is divisible by 8 since 128 is divisible by 8.
The number 1,170 is not divisible by 8 since 170 is not divisible by 8.
128 CHAPTER 2. MULTIPLICATION AND DIVISION OF WHOLE NUMBERS

Division by 9
A whole number is divisible by 9 if the sum of its digits is divisible by 9.
The number 702 is divisible by 9 since 7 + 0 + 2 is divisible by 9.
The number 6588 is divisible by 9 since 6 + 5 + 8 + 8 = 27 is divisible by 9.
The number 14,123 is not divisible by 9 since 1 + 4 + 1 + 2 + 3 = 11 is not divisible by 9.

Division by 10
A Whole number is divisible by 10 if its last digit is 0.

2.5.3.1 Sample Set B

Example 2.34
The numbers 30, 170, 16,240, and 865,000 are all divisible by 10.

2.5.3.2 Practice Set B

State which of the following whole numbers are divisible 6, 8, 9, or 10. Some numbers may be divisible by
more than one number.
Exercise 2.248 (Solution on p. 147.)
900
Exercise 2.249 (Solution on p. 147.)
6,402
Exercise 2.250 (Solution on p. 147.)
6,660
Exercise 2.251 (Solution on p. 147.)
55,116

2.5.4 Exercises
For the following 30 problems, specify if the whole number is divisible by 2, 3, 4, 5, 6, 8, 9, or 10. Write
"none" if the number is not divisible by any digit other than 1. Some numbers may be divisible by more
than one number.
Exercise 2.252 (Solution on p. 147.)
48
Exercise 2.253
85
Exercise 2.254 (Solution on p. 147.)
30
Exercise 2.255
83
Exercise 2.256 (Solution on p. 147.)
98
Exercise 2.257
972
Exercise 2.258 (Solution on p. 147.)
892
129

Exercise 2.259
676
Exercise 2.260 (Solution on p. 147.)
903
Exercise 2.261
800
Exercise 2.262 (Solution on p. 147.)
223
Exercise 2.263
836
Exercise 2.264 (Solution on p. 147.)
665
Exercise 2.265
4,381
Exercise 2.266 (Solution on p. 147.)
2,195
Exercise 2.267
2,544
Exercise 2.268 (Solution on p. 147.)
5,172
Exercise 2.269
1,307
Exercise 2.270 (Solution on p. 147.)
1,050
Exercise 2.271
3,898
Exercise 2.272 (Solution on p. 147.)
1,621
Exercise 2.273
27,808
Exercise 2.274 (Solution on p. 147.)
45,764
Exercise 2.275
49,198
Exercise 2.276 (Solution on p. 147.)
296,122
Exercise 2.277
178,656
Exercise 2.278 (Solution on p. 148.)
5,102,417
Exercise 2.279
16,990,792
Exercise 2.280 (Solution on p. 148.)
620,157,659
Exercise 2.281
457,687,705
130 CHAPTER 2. MULTIPLICATION AND DIVISION OF WHOLE NUMBERS

2.5.4.1 Exercises for Review

Exercise 2.282 (Solution on p. 148.)


(Section 1.2) In the number 412, how many tens are there?
Exercise 2.283
(Section 1.6) Subtract 613 from 810.
Exercise 2.284 (Solution on p. 148.)
(Section 1.7) Add 35, 16, and 7 in two dierent ways.
Exercise 2.285
(Section 2.3) Find the quotient 35 ÷ 0, if it exists.
Exercise 2.286 (Solution on p. 148.)
(Section 2.4) Find the quotient. 3654 ÷ 42.

2.6 Properties of Multiplication 6

2.6.1 Section Overview


• The Commutative Property of Multiplication
• The Associative Property of Multiplication
• The Multiplicative Identity

We will now examine three simple but very important properties of multiplication.

2.6.2 The Commutative Property of Multiplication


Commutative Property of Multiplication
The product of two whole numbers is the same regardless of the order of the factors.

2.6.2.1 Sample Set A

Example 2.35
Multiply the two whole numbers.

6 · 7 = 42

7 · 6 = 42

The numbers 6 and 7 can be multiplied in any order. Regardless of the order they are multiplied,
the product is 42.

6 This content is available online at <http://cnx.org/content/m34867/1.2/>.


131

2.6.2.2 Practice Set A

Use the commutative property of multiplication to nd the products in two ways.
Exercise 2.287 (Solution on p. 148.)

Exercise 2.288 (Solution on p. 148.)

2.6.3 The Associative Property of Multiplication


Associative Property of Multiplication
If three whole numbers are multiplied, the product will be the same if the rst two are multiplied rst and
then that product is multiplied by the third, or if the second two are multiplied rst and that product is
multiplied by the rst. Note that the order of the factors is maintained.

It is a common mathematical practice to use parentheses to show which pair of numbers is to be combined
rst.

2.6.3.1 Sample Set B

Example 2.36
Multiply the whole numbers.

(8 · 3) · 14 = 24 · 14 = 336

8 · (3 · 14) = 8 · 42 = 336

2.6.3.2 Practice Set B

Use the associative property of multiplication to nd the products in two ways.
Exercise 2.289 (Solution on p. 148.)
132 CHAPTER 2. MULTIPLICATION AND DIVISION OF WHOLE NUMBERS

Exercise 2.290 (Solution on p. 148.)

2.6.4 The Multiplicative Identity


The Multiplicative Identity is 1
The whole number 1 is called the multiplicative identity, since any whole number multiplied by 1 is not
changed.

2.6.4.1 Sample Set C

Example 2.37
Multiply the whole numbers.

12 · 1 = 12

1 · 12 = 12

2.6.4.2 Practice Set C

Multiply the whole numbers.


Exercise 2.291 (Solution on p. 148.)

2.6.5 Exercises
For the following problems, multiply the numbers.
Exercise 2.292 (Solution on p. 148.)

Exercise 2.293
133

Exercise 2.294 (Solution on p. 148.)

Exercise 2.295

Exercise 2.296 (Solution on p. 148.)

Exercise 2.297

Exercise 2.298 (Solution on p. 148.)

Exercise 2.299

Exercise 2.300 (Solution on p. 148.)

Exercise 2.301
134 CHAPTER 2. MULTIPLICATION AND DIVISION OF WHOLE NUMBERS

Exercise 2.302 (Solution on p. 148.)

Exercise 2.303

For the following 4 problems, show that the quantities yield the same products by performing the multipli-
cations.
Exercise 2.304 (Solution on p. 148.)
(4 · 8) · 2 and 4 · (8 · 2)
Exercise 2.305
(100 · 62) · 4 and 100 · (62 · 4)
Exercise 2.306 (Solution on p. 148.)
23 · (11 · 106) and (23 · 11) · 106
Exercise 2.307
1 · (5 · 2) and (1 · 5) · 2
Exercise 2.308 (Solution on p. 148.)
The fact that (a rst number · a second number) · a third number = a rst number ·
(a second number · a third number)is an example of the property of multiplication.
Exercise 2.309
The fact that 1 · any number = that particular numberis an example of the property of
multiplication.
Exercise 2.310 (Solution on p. 148.)
Use the numbers 7 and 9 to illustrate the commutative property of multiplication.
Exercise 2.311
Use the numbers 6, 4, and 7 to illustrate the associative property of multiplication.

2.6.5.1 Exercises for Review

Exercise 2.312 (Solution on p. 148.)


(Section 1.2) In the number 84,526,098,441, how many millions are there?
Exercise 2.313
85
(Section 1.5) Replace the letter m with the whole number that makes the addition true. + m
97
Exercise 2.314 (Solution on p. 148.)
(Section 1.7) Use the numbers 4 and 15 to illustrate the commutative property of addition.
Exercise 2.315
(Section 2.3) Find the product. 8, 000, 000 × 1, 000.
135

Exercise 2.316 (Solution on p. 148.)


(Section 2.5) Specify which of the digits 2, 3, 4, 5, 6, 8,10 are divisors of the number 2,244.

2.7 Summary of Key Concepts 7


2.7.1 Summary of Key Concepts
Multiplication (Section 2.2)
Multiplication is a description of repeated addition.
7+7+7+7
| {z }
7 appears 4 times
This expression is described by writing 4 X 7.

Multiplicand/Multiplier/Product (Section 2.2)


In a multiplication of whole numbers, the repeated addend is called the multiplicand, and the number that
records the number of times the multiplicand is used is the multiplier. The result of the multiplication is
the product.

Factors (Section 2.2)


In a multiplication, the numbers being multiplied are also called factors. Thus, the multiplicand and the
multiplier can be called factors.

Division (Section 2.3)


Division is a description of repeated subtraction.

Dividend/Divisor/Quotient (Section 2.3)


In a division, the number divided into is called the dividend, and the number dividing into the dividend is
called the divisor. The result of the division is called the quotient.
quotient
divisor)dividend

Division into Zero (Section 2.3)


Zero divided by any nonzero whole number is zero.

Division by Zero (Section 2.3)


Division by zero does not name a whole number. It is, therefore, undened. The quotient 0
0 is indeterminant.

Division by 2, 3, 4, 5, 6, 8, 9, 10 (Section 2.5)


Division by the whole numbers 2, 3, 4, 5, 6, 8, 9, and 10 can be determined by noting some certain properties
of the particular whole number.

Commutative Property of Multiplication (Section 2.6)


The product of two whole numbers is the same regardless of the order of the factors. 3 × 5 = 5 × 3

Associative Property of Multiplication (Section 2.6)


If three whole numbers are to be multiplied, the product will be the same if the rst two are multiplied
rst and then that product is multiplied by the third, or if the second two are multiplied rst and then that
product is multiplied by the rst.
(3 × 5) × 2 = 3 × (5 × 2)
Note that the order of the factors is maintained.
7 This content is available online at <http://cnx.org/content/m34868/1.2/>.
136 CHAPTER 2. MULTIPLICATION AND DIVISION OF WHOLE NUMBERS

Multiplicative Identity (Section 2.6)


The whole number 1 is called the multiplicative identity since any whole number multiplied by 1 is not
changed.
4×1=4
1×4=4

2.8 Exercise Supplement 8


2.8.1 Exercise Supplement
2.8.1.1 Multiplication of Whole Numbers (Section 2.2)

Exercise 2.317 (Solution on p. 148.)


In the multiplication 5 × 9 = 45, 5 and 9 are called and 45 is called the .
Exercise 2.318
In the multiplication 4 × 8 = 32, 4 and 8 are called and 32 is called the .

2.8.1.2 Concepts of Division of Whole Numbers (Section 2.3)

Exercise 2.319 (Solution on p. 149.)


In the division 24 ÷ 6 = 4, 6 is called the , and 4 is called the .
Exercise 2.320
In the division 36 ÷ 2 = 18, 2 is called the , and 18 is called the .

2.8.1.3 Some Interesting Facts about Division (Section 2.5)

Exercise 2.321 (Solution on p. 149.)


A number is divisible by 2 only if its last digit is .
Exercise 2.322
A number is divisible by 3 only if of its digits is divisible by 3.
Exercise 2.323 (Solution on p. 149.)
A number is divisible by 4 only if the rightmost two digits form a number that is .

2.8.1.4 Multiplication and Division of Whole Numbers (Section 2.2,Section 2.4)

Find each product or quotient.


Exercise 2.324
24
×3
Exercise 2.325 (Solution on p. 149.)
14
×8
Exercise 2.326
21 ÷ 7
8 This content is available online at <http://cnx.org/content/m34869/1.2/>.
137

Exercise 2.327 (Solution on p. 149.)


35 ÷ 5
Exercise 2.328
36
×22
Exercise 2.329 (Solution on p. 149.)
87
×35
Exercise 2.330
117
×42
Exercise 2.331 (Solution on p. 149.)
208 ÷ 52
Exercise 2.332
521
× 87
Exercise 2.333 (Solution on p. 149.)
1005
× 15
Exercise 2.334
1338 ÷ 446
Exercise 2.335 (Solution on p. 149.)
2814 ÷ 201
Exercise 2.336
5521
×8
Exercise 2.337 (Solution on p. 149.)
6016
× 7
Exercise 2.338
576 ÷ 24
Exercise 2.339 (Solution on p. 149.)
3969 ÷ 63
Exercise 2.340
5482
× 322
Exercise 2.341 (Solution on p. 149.)
9104
× 115
Exercise 2.342
6102
×1000
138 CHAPTER 2. MULTIPLICATION AND DIVISION OF WHOLE NUMBERS

Exercise 2.343 (Solution on p. 149.)


10101
×10000
Exercise 2.344
162, 006 ÷ 31
Exercise 2.345 (Solution on p. 149.)
0 ÷ 25
Exercise 2.346
25 ÷ 0
Exercise 2.347 (Solution on p. 149.)
4280 ÷ 10
Exercise 2.348
2126000 ÷ 100
Exercise 2.349 (Solution on p. 149.)
84 ÷ 15
Exercise 2.350
126 ÷ 4
Exercise 2.351 (Solution on p. 149.)
424 ÷ 0
Exercise 2.352
1198 ÷ 46
Exercise 2.353 (Solution on p. 149.)
995 ÷ 31
Exercise 2.354
0 ÷ 18
Exercise 2.355 (Solution on p. 149.)
2162
×1421
Exercise 2.356
0×0
Exercise 2.357 (Solution on p. 149.)
5×0
Exercise 2.358
64 × 1
Exercise 2.359 (Solution on p. 149.)
1×0
Exercise 2.360
0÷3
Exercise 2.361 (Solution on p. 149.)
14 ÷ 0
Exercise 2.362
35 ÷ 1
Exercise 2.363 (Solution on p. 149.)
1÷1
139

2.8.1.5 Properties of Multiplication (Section 2.6)

Exercise 2.364
Use the commutative property of multiplication to rewrite 36 × 128.
Exercise 2.365 (Solution on p. 149.)
Use the commutative property of multiplication to rewrite 114 × 226.
Exercise 2.366
Use the associative property of multiplication to rewrite (5 · 4) · 8.
Exercise 2.367 (Solution on p. 149.)
Use the associative property of multiplication to rewrite 16 · (14 · 0).

2.8.1.6 Multiplication and Division of Whole Numbers (Section 2.2,Section 2.4)

Exercise 2.368
A computer store is selling diskettes for $4 each. At this price, how much would 15 diskettes cost?
Exercise 2.369 (Solution on p. 150.)
Light travels 186,000 miles in one second. How far does light travel in 23 seconds?
Exercise 2.370
A dinner bill for eight people comes to exactly $112. How much should each person pay if they all
agree to split the bill equally?
Exercise 2.371 (Solution on p. 150.)
Each of the 33 students in a math class buys a textbook. If the bookstore sells $1089 worth of
books, what is the price of each book?

2.9 Prociency Exam 9

2.9.1 Prociency Exam


Exercise 2.372 (Solution on p. 150.)
(Section 2.2) In the multiplication of 8 × 7 = 56, what are the names given to the 8 and 7 and the
56?
Exercise 2.373 (Solution on p. 150.)
(Section 2.2) Multiplication is a description of what repeated process?
Exercise 2.374 (Solution on p. 150.)
(Section 2.3) In the division 12 ÷ 3 = 4, what are the names given to the 3 and the 4?
Exercise 2.375 (Solution on p. 150.)
(Section 2.5) Name the digits that a number must end in to be divisible by 2.
Exercise 2.376 (Solution on p. 150.)
(Section 2.6) Name the property of multiplication that states that the order of the factors in a
multiplication can be changed without changing the product.
Exercise 2.377 (Solution on p. 150.)
(Section 2.6) Which number is called the multiplicative identity?
For problems 7-17, nd the product or quotient.
Exercise 2.378 (Solution on p. 150.)
(Section 2.2) 14 × 6
9 This content is available online at <http://cnx.org/content/m34870/1.2/>.
140 CHAPTER 2. MULTIPLICATION AND DIVISION OF WHOLE NUMBERS

Exercise 2.379 (Solution on p. 150.)


(Section 2.2) 37 × 0
Exercise 2.380 (Solution on p. 150.)
(Section 2.2) 352 × 1000
Exercise 2.381 (Solution on p. 150.)
(Section 2.2) 5986 × 70
Exercise 2.382 (Solution on p. 150.)
(Section 2.2) 12 × 12
Exercise 2.383 (Solution on p. 150.)
(Section 2.3) 856 ÷ 0
Exercise 2.384 (Solution on p. 150.)
(Section 2.3) 0 ÷ 8
Exercise 2.385 (Solution on p. 150.)
(Section 2.4) 136 ÷ 8
Exercise 2.386 (Solution on p. 150.)
(Section 2.4) 432 ÷ 24
Exercise 2.387 (Solution on p. 150.)
(Section 2.4) 5286 ÷ 37
Exercise 2.388 (Solution on p. 150.)
(Section 2.6) 211 × 1
For problems 18-20, use the numbers 216, 1,005, and 640.
Exercise 2.389 (Solution on p. 150.)
(Section 2.5) Which numbers are divisible by 3?
Exercise 2.390 (Solution on p. 150.)
(Section 2.5) Which number is divisible by 4?
Exercise 2.391 (Solution on p. 150.)
(Section 2.5) Which number(s) is divisible by 5?
141

Solutions to Exercises in Chapter 2


Solution to Exercise 2.1 (p. 93)
4 × 12. Multiplier is 4. Multiplicand is 12.
Solution to Exercise 2.2 (p. 93)
8 × 36. Multiplier is 8. Multiplicand is 36.
Solution to Exercise 2.3 (p. 93)
5 × 0. Multiplier is 5. Multiplicand is 0.
Solution to Exercise 2.4 (p. 93)
12, 000 × 1, 847. Multiplier is 12,000. Multiplicand is 1,847.
Solution to Exercise 2.5 (p. 94)
185
Solution to Exercise 2.6 (p. 94)
624
Solution to Exercise 2.7 (p. 95)
3,752
Solution to Exercise 2.8 (p. 95)
320,152
Solution to Exercise 2.9 (p. 95)
904,797
Solution to Exercise 2.10 (p. 97)
1,022
Solution to Exercise 2.11 (p. 97)
4,472
Solution to Exercise 2.12 (p. 97)
35,615
Solution to Exercise 2.13 (p. 97)
1,456,488
Solution to Exercise 2.14 (p. 97)
2,464,528
Solution to Exercise 2.15 (p. 97)
66,790,080
Solution to Exercise 2.16 (p. 97)
104,709
Solution to Exercise 2.17 (p. 97)
240
Solution to Exercise 2.18 (p. 97)
3,809,000
Solution to Exercise 2.19 (p. 97)
8,130,000
Solution to Exercise 2.20 (p. 98)
162,000
Solution to Exercise 2.21 (p. 98)
126,000,000
Solution to Exercise 2.22 (p. 98)
2,870,000,000,000
Solution to Exercise 2.23 (p. 99)
1,404
Solution to Exercise 2.24 (p. 99)
8,912,440
Solution to Exercise 2.25 (p. 99)
1,832,700,000,000
142 CHAPTER 2. MULTIPLICATION AND DIVISION OF WHOLE NUMBERS

Solution to Exercise 2.26 (p. 100)


24
Solution to Exercise 2.28 (p. 100)
48
Solution to Exercise 2.30 (p. 100)
6
Solution to Exercise 2.32 (p. 100)
225
Solution to Exercise 2.34 (p. 100)
270
Solution to Exercise 2.36 (p. 100)
582
Solution to Exercise 2.38 (p. 100)
960
Solution to Exercise 2.40 (p. 100)
7,695
Solution to Exercise 2.42 (p. 101)
3,648
Solution to Exercise 2.44 (p. 101)
46,488
Solution to Exercise 2.46 (p. 101)
2,530
Solution to Exercise 2.48 (p. 101)
6,888
Solution to Exercise 2.50 (p. 101)
24,180
Solution to Exercise 2.52 (p. 101)
73,914
Solution to Exercise 2.54 (p. 101)
68,625
Solution to Exercise 2.56 (p. 102)
511,173
Solution to Exercise 2.58 (p. 102)
1,352,550
Solution to Exercise 2.60 (p. 102)
5,441,712
Solution to Exercise 2.62 (p. 102)
36,901,053
Solution to Exercise 2.64 (p. 102)
24,957,200
Solution to Exercise 2.66 (p. 102)
0
Solution to Exercise 2.68 (p. 102)
41,384
Solution to Exercise 2.70 (p. 103)
0
Solution to Exercise 2.72 (p. 103)
73,530
Solution to Exercise 2.74 (p. 103)
6,440,000,000
143

Solution to Exercise 2.76 (p. 103)


4,440,000
Solution to Exercise 2.78 (p. 103)
641,900,000
Solution to Exercise 2.80 (p. 103)
80,000
Solution to Exercise 2.82 (p. 104)
384 reports
Solution to Exercise 2.84 (p. 104)
3,330 problems
Solution to Exercise 2.86 (p. 104)
114 units
Solution to Exercise 2.88 (p. 104)
3,600 seconds
Solution to Exercise 2.90 (p. 104)
5,865,696,000,000 miles per year
Solution to Exercise 2.92 (p. 104)
$110,055
Solution to Exercise 2.94 (p. 104)
448,100,000
Solution to Exercise 2.96 (p. 104)
712
Solution to Exercise 2.98 (p. 107)
4
Solution to Exercise 2.99 (p. 107)
2
Solution to Exercise 2.100 (p. 107)
5
Solution to Exercise 2.101 (p. 107)
6
Solution to Exercise 2.102 (p. 107)
4
Solution to Exercise 2.103 (p. 107)
9
Solution to Exercise 2.104 (p. 109)
undened
Solution to Exercise 2.105 (p. 109)
0
Solution to Exercise 2.106 (p. 109)
indeterminant
Solution to Exercise 2.107 (p. 109)
undened
Solution to Exercise 2.108 (p. 109)
undened
Solution to Exercise 2.109 (p. 109)
0
Solution to Exercise 2.110 (p. 110)
5
Solution to Exercise 2.111 (p. 110)
7
144 CHAPTER 2. MULTIPLICATION AND DIVISION OF WHOLE NUMBERS

Solution to Exercise 2.112 (p. 110)


0
Solution to Exercise 2.113 (p. 110)
An error message tells us that this operation is undened. The particular message depends on the calculator.
Solution to Exercise 2.114 (p. 110)
An error message tells us that this operation cannot be performed. Some calculators actually set 0 ÷ 0 equal
to 1. We know better! 0 ÷ 0 is indeterminant.
Solution to Exercise 2.115 (p. 111)
8
Solution to Exercise 2.117 (p. 111)
3
Solution to Exercise 2.119 (p. 111)
9
Solution to Exercise 2.121 (p. 111)
7
Solution to Exercise 2.123 (p. 111)
4
Solution to Exercise 2.125 (p. 111)
5
Solution to Exercise 2.127 (p. 111)
7
Solution to Exercise 2.129 (p. 111)
not dened
Solution to Exercise 2.131 (p. 111)
3
Solution to Exercise 2.133 (p. 111)
0
Solution to Exercise 2.135 (p. 111)
5
Solution to Exercise 2.137 (p. 112)
8
Solution to Exercise 2.139 (p. 112)
9
Solution to Exercise 2.141 (p. 112)
27 ÷ 9 = 3; 9)27 = 3 ; 27
9 =3
Solution to Exercise 2.143 (p. 112)
7 is quotient; 8 is divisor; 56 is dividend
Solution to Exercise 2.145 (p. 112)
12,124
Solution to Exercise 2.147 (p. 112)
(2 + 3) + 7 = 2 + (3 + 7) = 12
5+7=2+10=12
Solution to Exercise 2.149 (p. 116)
18
Solution to Exercise 2.150 (p. 116)
81
Solution to Exercise 2.151 (p. 116)
853
Solution to Exercise 2.152 (p. 116)
1,129
145

Solution to Exercise 2.153 (p. 116)


84,125
Solution to Exercise 2.154 (p. 118)
43
Solution to Exercise 2.155 (p. 118)
112
Solution to Exercise 2.156 (p. 118)
305
Solution to Exercise 2.157 (p. 118)
3,016
Solution to Exercise 2.158 (p. 120)
18 R3
Solution to Exercise 2.159 (p. 120)
43 R2
Solution to Exercise 2.160 (p. 120)
23 R6
Solution to Exercise 2.161 (p. 120)
61 R14
Solution to Exercise 2.162 (p. 121)
614 R58
Solution to Exercise 2.163 (p. 122)
45
Solution to Exercise 2.164 (p. 122)
551
Solution to Exercise 2.165 (p. 122)
Since the dividend has more than eight digits, this division cannot be performed on most nonscientic
calculators. On others, the answer is 415,026,137.4
Solution to Exercise 2.166 (p. 122)
This division results in 68.02985075, a decimal number, and therefore, we cannot, at this time, nd the value
of the remainder. Later, we will discuss decimal numbers.
Solution to Exercise 2.167 (p. 122)
13
Solution to Exercise 2.169 (p. 122)
67
Solution to Exercise 2.171 (p. 122)
52
Solution to Exercise 2.173 (p. 122)
3
Solution to Exercise 2.175 (p. 123)
70
Solution to Exercise 2.177 (p. 123)
61
Solution to Exercise 2.179 (p. 123)
59
Solution to Exercise 2.181 (p. 123)
67
Solution to Exercise 2.183 (p. 123)
87
Solution to Exercise 2.185 (p. 123)
54
146 CHAPTER 2. MULTIPLICATION AND DIVISION OF WHOLE NUMBERS

Solution to Exercise 2.187 (p. 123)


52
Solution to Exercise 2.189 (p. 123)
38
Solution to Exercise 2.191 (p. 123)
45
Solution to Exercise 2.193 (p. 123)
777
Solution to Exercise 2.195 (p. 123)
342
Solution to Exercise 2.197 (p. 124)
644
Solution to Exercise 2.199 (p. 124)
533
Solution to Exercise 2.201 (p. 124)
10,440
Solution to Exercise 2.203 (p. 124)
8,147 remainder 847
Solution to Exercise 2.205 (p. 124)
4 remainder 2
Solution to Exercise 2.207 (p. 124)
66 remainder 1
Solution to Exercise 2.209 (p. 124)
823 remainder 2
Solution to Exercise 2.211 (p. 124)
40 remainder 28
Solution to Exercise 2.213 (p. 124)
665 remainder 15
Solution to Exercise 2.215 (p. 124)
957 remainder 34
Solution to Exercise 2.217 (p. 124)
665 remainder 4
Solution to Exercise 2.219 (p. 124)
458 remainder 13
Solution to Exercise 2.221 (p. 125)
996 remainder 23
Solution to Exercise 2.223 (p. 125)
42
Solution to Exercise 2.225 (p. 125)
8,216
Solution to Exercise 2.227 (p. 125)
$1,975 per month
Solution to Exercise 2.229 (p. 125)
$485 each person invested
Solution to Exercise 2.231 (p. 125)
14 cubes per hour
Solution to Exercise 2.233 (p. 125)
8 bits in each byte
Solution to Exercise 2.235 (p. 125)
38
147

Solution to Exercise 2.237 (p. 126)


600,000
Solution to Exercise 2.239 (p. 126)
625,600
Solution to Exercise 2.241 (p. 127)
2
Solution to Exercise 2.242 (p. 127)
3
Solution to Exercise 2.243 (p. 127)
3
Solution to Exercise 2.244 (p. 127)
5
Solution to Exercise 2.245 (p. 127)
2, 4
Solution to Exercise 2.246 (p. 127)
2, 5
Solution to Exercise 2.247 (p. 127)
2, 3
Solution to Exercise 2.248 (p. 128)
6, 9, 10
Solution to Exercise 2.249 (p. 128)
6
Solution to Exercise 2.250 (p. 128)
6, 9, 10
Solution to Exercise 2.251 (p. 128)
6, 9
Solution to Exercise 2.252 (p. 128)
2, 3, 4, 6, 8
Solution to Exercise 2.254 (p. 128)
2, 3, 5, 6, 10
Solution to Exercise 2.256 (p. 128)
2
Solution to Exercise 2.258 (p. 128)
2, 4
Solution to Exercise 2.260 (p. 129)
3
Solution to Exercise 2.262 (p. 129)
none
Solution to Exercise 2.264 (p. 129)
5
Solution to Exercise 2.266 (p. 129)
5
Solution to Exercise 2.268 (p. 129)
2, 3, 4, 6
Solution to Exercise 2.270 (p. 129)
2, 3, 5, 6, 10
Solution to Exercise 2.272 (p. 129)
none
Solution to Exercise 2.274 (p. 129)
2, 4
148 CHAPTER 2. MULTIPLICATION AND DIVISION OF WHOLE NUMBERS

Solution to Exercise 2.276 (p. 129)


2
Solution to Exercise 2.278 (p. 129)
none
Solution to Exercise 2.280 (p. 129)
none
Solution to Exercise 2.282 (p. 130)
1
Solution to Exercise 2.284 (p. 130)
(35 + 16) + 7 = 51 + 7 = 58
35 + (16 + 7) = 35 + 23 = 58
Solution to Exercise 2.286 (p. 130)
87
Solution to Exercise 2.287 (p. 131)
15 · 6 = 90 and 6 · 15 = 90
Solution to Exercise 2.288 (p. 131)
432 · 428 = 184, 896 and 428 · 432 = 184, 896
Solution to Exercise 2.289 (p. 131)
168
Solution to Exercise 2.290 (p. 132)
165,564
Solution to Exercise 2.291 (p. 132)
843
Solution to Exercise 2.292 (p. 132)
234
Solution to Exercise 2.294 (p. 133)
4,032
Solution to Exercise 2.296 (p. 133)
326,000
Solution to Exercise 2.298 (p. 133)
252
Solution to Exercise 2.300 (p. 133)
21,340
Solution to Exercise 2.302 (p. 134)
8,316
Solution to Exercise 2.304 (p. 134)
32 · 2 = 64 = 4 · 16
Solution to Exercise 2.306 (p. 134)
23 · 1, 166 = 26, 818 = 253 · 106
Solution to Exercise 2.308 (p. 134)
associative
Solution to Exercise 2.310 (p. 134)
7 · 9 = 63 = 9 · 7
Solution to Exercise 2.312 (p. 134)
6
Solution to Exercise 2.314 (p. 134)
4 + 15 = 19

15 + 4 = 19
Solution to Exercise 2.316 (p. 135)
2, 3, 4, 6
149

Solution to Exercise 2.317 (p. 136)


factors; product
Solution to Exercise 2.319 (p. 136)
divisor; quotient
Solution to Exercise 2.321 (p. 136)
an even digit (0, 2, 4, 6, or 8)
Solution to Exercise 2.323 (p. 136)
divisible by 4
Solution to Exercise 2.325 (p. 136)
112
Solution to Exercise 2.327 (p. 136)
7
Solution to Exercise 2.329 (p. 137)
3,045
Solution to Exercise 2.331 (p. 137)
4
Solution to Exercise 2.333 (p. 137)
15,075
Solution to Exercise 2.335 (p. 137)
14
Solution to Exercise 2.337 (p. 137)
42,112
Solution to Exercise 2.339 (p. 137)
63
Solution to Exercise 2.341 (p. 137)
1,046,960
Solution to Exercise 2.343 (p. 138)
101,010,000
Solution to Exercise 2.345 (p. 138)
0
Solution to Exercise 2.347 (p. 138)
428
Solution to Exercise 2.349 (p. 138)
5 remainder 9
Solution to Exercise 2.351 (p. 138)
not dened
Solution to Exercise 2.353 (p. 138)
32 remainder 3
Solution to Exercise 2.355 (p. 138)
3,072,202
Solution to Exercise 2.357 (p. 138)
0
Solution to Exercise 2.359 (p. 138)
0
Solution to Exercise 2.361 (p. 138)
not dened
Solution to Exercise 2.363 (p. 138)
1
Solution to Exercise 2.365 (p. 139)
226 · 114
150 CHAPTER 2. MULTIPLICATION AND DIVISION OF WHOLE NUMBERS

Solution to Exercise 2.367 (p. 139)


(16 · 14) · 0
Solution to Exercise 2.369 (p. 139)
4,278,000
Solution to Exercise 2.371 (p. 139)
$33
Solution to Exercise 2.372 (p. 139)
8 and 7 are factors; 56 is the product
Solution to Exercise 2.373 (p. 139)
Addition
Solution to Exercise 2.374 (p. 139)
3 is the divisor; 4 is the quotient
Solution to Exercise 2.375 (p. 139)
0, 2, 4, 6, or 8
Solution to Exercise 2.376 (p. 139)
commutative
Solution to Exercise 2.377 (p. 139)
1
Solution to Exercise 2.378 (p. 139)
84
Solution to Exercise 2.379 (p. 140)
0
Solution to Exercise 2.380 (p. 140)
352,000
Solution to Exercise 2.381 (p. 140)
419,020
Solution to Exercise 2.382 (p. 140)
252
Solution to Exercise 2.383 (p. 140)
not dened
Solution to Exercise 2.384 (p. 140)
0
Solution to Exercise 2.385 (p. 140)
17
Solution to Exercise 2.386 (p. 140)
18
Solution to Exercise 2.387 (p. 140)
142 remainder 32
Solution to Exercise 2.388 (p. 140)
211
Solution to Exercise 2.389 (p. 140)
216; 1,005
Solution to Exercise 2.390 (p. 140)
216; 640
Solution to Exercise 2.391 (p. 140)
1,005; 640
Chapter 3

Exponents, Roots, and Factorization of

Whole Numbers

3.1 Objectives1
After completing this chapter, you should
Exponents and Roots (Section 3.2)

• understand and be able to read exponential notation


• understand the concept of root and be able to read root notation
• be able to use a calculator having the y x key to determine a root

Grouping Symbols and the Order of Operations (Section 3.3)

• understand the use of grouping symbols


• understand and be able to use the order of operations
• use the calculator to determine the value of a numerical expression

Prime Factorization of Natural Numbers (Section 3.4)

• be able to determine the factors of a whole number


• be able to distinguish between prime and composite numbers
• be familiar with the fundamental principle of arithmetic
• be able to nd the prime factorization of a whole number

The Greatest Common Factor (Section 3.5)

• be able to nd the greatest common factor of two or more whole numbers

The Least Common Multiple (Section 3.6)

• be able to nd the least common multiple of two or more whole numbers

1 This content is available online at <http://cnx.org/content/m18890/1.4/>.

151
152
CHAPTER 3. EXPONENTS, ROOTS, AND FACTORIZATION OF WHOLE
NUMBERS
3.2 Exponents and Roots 2
3.2.1 Section Overview
• Exponential Notation
• Reading Exponential Notation
• Roots
• Reading Root Notation
• Calculators

3.2.2 Exponential Notation


Exponential Notation
We have noted that multiplication is a description of repeated addition. Exponential notation is a
description of repeated multiplication.

Suppose we have the repeated multiplication

8·8·8·8·8

Exponent
The factor 8 is repeated 5 times. Exponential notation uses a superscript for the number of times the factor
is repeated. The superscript is placed on the repeated factor, 85 , in this case. The superscript is called an
exponent.

The Function of an Exponent


An exponent records the number of identical factors that are repeated in a multiplication.

3.2.2.1 Sample Set A

Write the following multiplication using exponents.


Example 3.1
3 · 3. Since the factor 3 appears 2 times, we record this as

32
Example 3.2
62 · 62 · 62 · 62 · 62 · 62 · 62 · 62 · 62. Since the factor 62 appears 9 times, we record this as

629
Expand (write without exponents) each number.
Example 3.3
124 . The exponent 4 is recording 4 factors of 12 in a multiplication. Thus,

124 = 12 · 12 · 12 · 12
Example 3.4
7063 . The exponent 3 is recording 3 factors of 706 in a multiplication. Thus,

7063 = 706 · 706 · 706

2 This content is available online at <http://cnx.org/content/m34871/1.2/>.


153

3.2.2.2 Practice Set A

Write the following using exponents.


Exercise 3.1 (Solution on p. 198.)
37 · 37
Exercise 3.2 (Solution on p. 198.)
16 · 16 · 16 · 16 · 16
Exercise 3.3 (Solution on p. 198.)
9·9·9·9·9·9·9·9·9·9
Write each number without exponents.
Exercise 3.4 (Solution on p. 198.)
853
Exercise 3.5 (Solution on p. 198.)
47
Exercise 3.6 (Solution on p. 198.)
1, 7392

3.2.3 Reading Exponential Notation


In a number such as 85 ,

Base
8 is called the base.

Exponent, Power
5 is called the exponent, or power. 85 is read as "eight to the fth power," or more simply as "eight to
the fth," or "the fth power of eight."

Squared
When a whole number is raised to the second power, it is said to be squared. The number 52 can be read
as

5 to the second power, or


5 to the second, or
5 squared.

Cubed
When a whole number is raised to the third power, it is said to be cubed. The number 53 can be read as

5 to the third power, or


5 to the third, or
5 cubed.

When a whole number is raised to the power of 4 or higher, we simply say that that number is raised to that
particular power. The number 58 can be read as

5 to the eighth power, or just


5 to the eighth.
154
CHAPTER 3. EXPONENTS, ROOTS, AND FACTORIZATION OF WHOLE
NUMBERS
3.2.4 Roots
In the English language, the word "root" can mean a source of something. In mathematical terms, the word
"root" is used to indicate that one number is the source of another number through repeated multiplication.

Square Root
We know that 49 = 72 , that is, 49 = 7 · 7. Through repeated multiplication, 7 is the source of 49. Thus, 7 is
a root of 49. Since two 7's must be multiplied together to produce 49, the 7 is called the second or square
root of 49.

Cube Root
We know that 8 = 23 , that is, 8 = 2 · 2 · 2. Through repeated multiplication, 2 is the source of 8. Thus, 2 is a
root of 8. Since three 2's must be multiplied together to produce 8, 2 is called the third or cube root of 8.

We can continue this way to see such roots as fourth roots, fth roots, sixth roots, and so on.

3.2.5 Reading Root Notation



There is a symbol used to indicate roots of a number. It is called the radical sign n


The Radical Sign n

The symbol n is called a radical sign and indicates the nth root of a number.

We discuss particular roots using the radical sign as follows:

Square
√ Root
2
number indicates the square root of the number under the radical sign. It is customary to drop the 2 in

the radical sign when discussing square roots. The symbol is understood to be the square root radical
sign.

49 = 7 since 7 · 7 = 72 = 49

Cube
√ Root
3
number indicates the cube root of the number under the radical sign.

3
8 = 2 since 2 · 2 · 2 = 23 = 8

Fourth
√ Root
4
number indicates the fourth root of the number under the radical sign.

4
81 = 3 since 3 · 3 · 3 · 3 = 34 = 81

In an expression such as 5 32

Radical Sign

is called the radical sign.

Index
5 is called the index. (The index describes the indicated root.)

Radicand
32 is called the radicand.

Radical

5
32 is called a radical (or radical expression).
155

3.2.5.1 Sample Set B

Find each root.


Example
√ 3.5
25 To determine the square root of 25, we ask, "What whole number squared equals 25?" From
our experience with multiplication, we know this number to be 5. Thus,

25 = 5

Check: 5 · 5 = 52 = 25
Example
√ 3.6
5
32 To determine the fth root of 32, we ask, "What whole number raised to the fth power
equals 32?" This number is 2.
√5
32 = 2

Check: 2 · 2 · 2 · 2 · 2 = 25 = 32

3.2.5.2 Practice Set B

Find the following roots using only a knowledge of multiplication.


Exercise
√ 3.7 (Solution on p. 198.)
64
Exercise
√ 3.8 (Solution on p. 198.)
100
Exercise
√ 3.9 (Solution on p. 198.)
3
64
Exercise
√ 3.10 (Solution on p. 198.)
6
64

3.2.6 Calculators

Calculators with the x, y x , and 1/x keys can be used to nd or approximate roots.

3.2.6.1 Sample Set C

Example 3.7 √
Use the calculator to nd 121

Display Reads
Type 121 121

Press x 11

Table 3.1

Example
√ 3.8
Find 7
2187.
156
CHAPTER 3. EXPONENTS, ROOTS, AND FACTORIZATION OF WHOLE
NUMBERS

Display Reads
Type 2187 2187
Press y x
2187
Type 7 7
Press 1/x .14285714
Press = 3

Table 3.2


7
2187 = 3 (Which means that 37 = 2187 .)

3.2.6.2 Practice Set C

Use a calculator to nd the following roots.


Exercise
√ 3.11 (Solution on p. 198.)
3
729
Exercise
√ 3.12 (Solution on p. 198.)
4
8503056
Exercise
√ 3.13 (Solution on p. 198.)
53361
Exercise
√ 3.14 (Solution on p. 198.)
12
16777216

3.2.7 Exercises
For the following problems, write the expressions using exponential notation.
Exercise 3.15 (Solution on p. 198.)
4·4
Exercise 3.16
12 · 12
Exercise 3.17 (Solution on p. 198.)
9·9·9·9
Exercise 3.18
10 · 10 · 10 · 10 · 10 · 10
Exercise 3.19 (Solution on p. 198.)
826 · 826 · 826
Exercise 3.20
3, 021 · 3, 021 · 3, 021 · 3, 021 · 3, 021
Exercise 3.21 (Solution on p. 198.)
|6 · 6 ·{z· · · · 6}
85 factors of 6
Exercise 3.22
|2 · 2 ·{z· · · · 2}
112 factors of 2
157

Exercise 3.23 (Solution on p. 198.)


|1 · 1 ·{z· · · · 1}
3,008 factors of 1
For the following problems, expand the terms. (Do not nd the actual value.)
Exercise 3.24
53
Exercise 3.25 (Solution on p. 198.)
74
Exercise 3.26
152
Exercise 3.27 (Solution on p. 198.)
1175
Exercise 3.28
616
Exercise 3.29 (Solution on p. 198.)
302
For the following problems, determine the value of each of the powers. Use a calculator to check each result.
Exercise 3.30
32
Exercise 3.31 (Solution on p. 198.)
42
Exercise 3.32
12
Exercise 3.33 (Solution on p. 198.)
102
Exercise 3.34
112
Exercise 3.35 (Solution on p. 198.)
122
Exercise 3.36
132
Exercise 3.37 (Solution on p. 198.)
152
Exercise 3.38
14
Exercise 3.39 (Solution on p. 199.)
34
Exercise 3.40
73
Exercise 3.41 (Solution on p. 199.)
103
Exercise 3.42
1002
Exercise 3.43 (Solution on p. 199.)
83
158
CHAPTER 3. EXPONENTS, ROOTS, AND FACTORIZATION OF WHOLE
NUMBERS
Exercise 3.44
55
Exercise 3.45 (Solution on p. 199.)
93
Exercise 3.46
62
Exercise 3.47 (Solution on p. 199.)
71
Exercise 3.48
128
Exercise 3.49 (Solution on p. 199.)
27
Exercise 3.50
05
Exercise 3.51 (Solution on p. 199.)
84
Exercise 3.52
58
Exercise 3.53 (Solution on p. 199.)
69
Exercise 3.54
253
Exercise 3.55 (Solution on p. 199.)
422
Exercise 3.56
313
Exercise 3.57 (Solution on p. 199.)
155
Exercise 3.58
220
Exercise 3.59 (Solution on p. 199.)
8162
For the following problems, nd the roots (using your knowledge of multiplication). Use a calculator to check
each result.
Exercise
√ 3.60
9
Exercise
√ 3.61 (Solution on p. 199.)
16
Exercise
√ 3.62
36
Exercise
√ 3.63 (Solution on p. 199.)
64
Exercise
√ 3.64
121
Exercise
√ 3.65 (Solution on p. 199.)
144
159

Exercise
√ 3.66
169
Exercise
√ 3.67 (Solution on p. 199.)
225
Exercise
√ 3.68
3
27
Exercise
√ 3.69 (Solution on p. 199.)
5
32
Exercise
√ 3.70
4
256
Exercise
√ 3.71 (Solution on p. 199.)
3
216
Exercise
√ 3.72
7
1
Exercise
√ 3.73 (Solution on p. 199.)
400
Exercise
√ 3.74
900
Exercise 3.75 (Solution on p. 199.)

10, 000
Exercise
√ 3.76
324
Exercise 3.77 (Solution on p. 199.)

3, 600

For the following problems, use a calculator with the keys x, y x , and 1/x to nd each of the values.
Exercise
√ 3.78
676
Exercise 3.79 (Solution on p. 199.)

1, 156
Exercise 3.80

46, 225
Exercise 3.81 (Solution on p. 199.)

17, 288, 964
Exercise 3.82

3
3, 375
Exercise 3.83 (Solution on p. 199.)

4
331, 776
Exercise 3.84

8
5, 764, 801
Exercise 3.85 (Solution on p. 199.)

12
16, 777, 216
Exercise 3.86

8
16, 777, 216
Exercise 3.87 (Solution on p. 199.)

10
9, 765, 625
160
CHAPTER 3. EXPONENTS, ROOTS, AND FACTORIZATION OF WHOLE
NUMBERS
Exercise 3.88

4
160, 000
Exercise 3.89 (Solution on p. 200.)

3
531, 441

3.2.7.1 Exercises for Review

Exercise 3.90
(Section 1.7) Use the numbers 3, 8, and 9 to illustrate the associative property of addition.
Exercise 3.91 (Solution on p. 200.)
(Section 2.2) In the multiplication 8 · 4 = 32, specify the name given to the numbers 8 and 4.
Exercise 3.92
(Section 2.3) Does the quotient 15 ÷ 0 exist? If so, what is it?
Exercise 3.93 (Solution on p. 200.)
(Section 2.3) Does the quotient 0 ÷ 15exist? If so, what is it?
Exercise 3.94
(Section 2.6) Use the numbers 4 and 7 to illustrate the commutative property of multiplication.

3.3 Grouping Symbols and the Order of Operations 3

3.3.1 Section Overview


• Grouping Symbols
• Multiple Grouping Symbols
• The Order of Operations
• Calculators

3.3.2 Grouping Symbols


Grouping symbols are used to indicate that a particular collection of numbers and meaningful operations are
to be grouped together and considered as one number. The grouping symbols commonly used in mathematics
are the following:

( ), [ ], { },
Parentheses: ()
Brackets: [ ]
Braces: { }
Bar:

In a computation in which more than one operation is involved, grouping symbols indicate which operation
to perform rst. If possible, we perform operations inside grouping symbols rst.
3 This content is available online at <http://cnx.org/content/m34872/1.2/>.
161

3.3.2.1 Sample Set A

If possible, determine the value of each of the following.


Example 3.9
9 + (3 · 8)

Since 3 and 8 are within parentheses, they are to be combined rst.

9 + (3 · 8) = 9 + 24
= 33

Thus,

9 + (3 · 8) = 33
Example 3.10
(10 ÷ 0) · 6

Since 10 ÷ 0 is undened, this operation is meaningless, and we attach no value to it. We write,
"undened."

3.3.2.2 Practice Set A

If possible, determine the value of each of the following.


Exercise 3.95 (Solution on p. 200.)
16 − (3 · 2)
Exercise 3.96 (Solution on p. 200.)
5 + (7 · 9)
Exercise 3.97 (Solution on p. 200.)
(4 + 8) · 2
Exercise 3.98 (Solution on p. 200.)
28 ÷ (18 − 11)
Exercise 3.99 (Solution on p. 200.)
(33 ÷ 3) − 11
Exercise 3.100 (Solution on p. 200.)
4 + (0 ÷ 0)

3.3.3 Multiple Grouping Symbols


When a set of grouping symbols occurs inside another set of grouping symbols, we perform the operations
within the innermost set rst.
162
CHAPTER 3. EXPONENTS, ROOTS, AND FACTORIZATION OF WHOLE
NUMBERS
3.3.3.1 Sample Set B

Determine the value of each of the following.


Example 3.11
2 + (8 · 3) − (5 + 6)

Combine 8 and 3 rst, then combine 5 and 6.

2 + 24 − 11 Now combine left to right.


26 − 11
15
Example 3.12
10 + [30 − (2 · 9)]

Combine 2 and 9 since they occur in the innermost set of parentheses.

10 + [30 − 18] Now combine 30 and 18.


10 + 12
22

3.3.3.2 Practice Set B

Determine the value of each of the following.


Exercise 3.101 (Solution on p. 200.)
(17 + 8) + (9 + 20)
Exercise 3.102 (Solution on p. 200.)
(55 − 6) − (13 · 2)
Exercise 3.103 (Solution on p. 200.)
23 + (12 ÷ 4) − (11 · 2)
Exercise 3.104 (Solution on p. 200.)
86 + [14 ÷ (10 − 8)]
Exercise 3.105 (Solution on p. 200.)
31 + {9 + [1 + (35 − 2)]}
Exercise 3.106 (Solution on p. 200.)
3
{6 − [24 ÷ (4 · 2)]}

3.3.4 The Order of Operations


Sometimes there are no grouping symbols indicating which operations to perform rst. For example, suppose
we wish to nd the value of 3 + 5 · 2. We could do either of two things:

Add 3 and 5, then multiply this sum by 2.

3+5·2 = 8 · 2
= 16

Multiply 5 and 2, then add 3 to this product.


163

3+5·2 = 3 + 10
= 13

We now have two values for one number. To determine the correct value, we must use the accepted order of
operations.
Order of Operations

1. Perform all operations inside grouping symbols, beginning with the innermost set, in the order 2, 3, 4
described below,
2. Perform all exponential and root operations.
3. Perform all multiplications and divisions, moving left to right.
4. Perform all additions and subtractions, moving left to right.

3.3.4.1 Sample Set C

Determine the value of each of the following.


Example 3.13
21 + 3 · 12 Multiply rst.
21 + 36 Add.
57
Example 3.14
(15 − 8) + 5 · (6 + 4) . Simplify inside parentheses rst.
7 + 5 · 10 Multiply.
7 + 50 Add.
57
Example 3.15
63 − (4 + 6 · 3) + 76 − 4 Simplify rst within the parenthesis by multiplying, then adding.
63 − (4 + 18) + 76 − 4
63 − 22 + 76 − 4 Now perform the additions and subtractions, moving left to right.
41 + 76 − 4 Add 41 and 76: 41 + 76 = 117.
117 − 4 Subtract 4 from 117: 117 − 4 = 113.
113
Example 3.16
7 · 6 − 42 + 15 Evaluate the exponential forms, moving left to right.
7 · 6 − 16 + 1 Multiply 7 and 6: 7 · 6 = 42
42 − 16 + 1 Subtract 16 from 42: 42 − 16 = 26
26 + 1 Add 26 and 1: 26 + 1 = 27
27
164
CHAPTER 3. EXPONENTS, ROOTS, AND FACTORIZATION OF WHOLE
NUMBERS
Example 3.17
Evaluate the exponential forms in the parentheses: 32 = 9 and 22 = 4

6 · 32 + 2 2 + 4 2
6 · (9 + 4) + 42 Add the 9 and 4 in the parentheses: 9 + 4 = 13
6 · (13) + 42 Evaluate the exponential form: 42 = 16
6 · (13) + 16 Multiply 6 and 13: 6 · 13 = 78
78 + 16 Add 78 and 16: 78 + 16 = 94
94
Example 3.18

62 +22 3 2 Recall that the bar is a grouping symbol.


42 +6·22 + 1012 −+8
19·5 2
+22
The fraction 462 +6·22 is equivalent to 6 + 2
2 2
 
÷ 42 + 6 · 22
36+4 1+64
16+6·4 + 100−19·5
36+4 1+64
16+24 + 100−95
40 65
40 + 5
1 + 13
14

3.3.4.2 Practice Set C

Determine the value of each of the following.


Exercise 3.107 (Solution on p. 200.)
8 + (32 − 7)
Exercise 3.108 (Solution on p. 200.)
(34 + 18 − 2 · 3) + 11
Exercise 3.109 (Solution on p. 200.)
8 (10) + 4 (2 + 3) − (20 + 3 · 15 + 40 − 5)
Exercise 3.110 (Solution on p. 200.)
5 · 8 + 42 − 22
Exercise 3.111
  (Solution on p. 200.)
4 62 − 33 ÷ 42 − 4
Exercise 3.112 (Solution on p. 200.)
(8 + 9 · 3) ÷ 7 + 5 · (8 ÷ 4 + 7 + 3 · 5)
Exercise 3.113
  (Solution on p. 200.)
33 +23 82 +24 8·3+18
62 −29 +5 72 −32 ÷ 23 −3

3.3.5 Calculators
Using a calculator is helpful for simplifying computations that involve large numbers.
165

3.3.5.1 Sample Set D

Use a calculator to determine each value.


Example 3.19
9, 842 + 56 · 85

Key Display Reads


Perform the multiplication rst. Type 56 56
Press × 56
Type 85 85
Now perform the addition. Press + 4760
Type 9842 9842
Press = 14602

Table 3.3

The display now reads 14,602.


Example 3.20
42 (27 + 18) + 105 (810 ÷ 18)

Key Display Reads


Operate inside the parentheses Type 27 27
Press + 27
Type 18 18
Press = 45
Multiply by 42. Press × 45
Type 42 42
Press = 1890

Table 3.4

Place this result into memory by pressing the memory key.


166
CHAPTER 3. EXPONENTS, ROOTS, AND FACTORIZATION OF WHOLE
NUMBERS

Key Display Reads


Now operate in the other parentheses. Type 810 810
Press ÷ 810
Type 18 18
Press = 45
Now multiply by 105. Press × 45
Type 105 105
Press = 4725
We are now ready to add these two quantities together. Press + 4725
Press the memory recall key. 1890
Press = 6615

Table 3.5

Thus, 42 (27 + 18) + 105 (810 ÷ 18) = 6, 615


Example 3.21
164 + 373

Nonscientic Calculators
Key Display Reads
Type 16 16
Press × 16
Type 16 16
Press × 256
Type 16 16
Press × 4096
Type 16 16
Press = 65536
Press the memory key
Type 37 37
Press × 37
Type 37 37
Press × 1396
Type 37 37
Press × 50653
Press + 50653
Press memory recall key 65536
Press = 116189
167

Table 3.6

Calculators with y x Key


Key Display Reads
Type 16 16
Press yx 16
Type 4 4
Press = 4096
Press + 4096
Type 37 37
Press y x
37
Type 3 3
Press = 116189

Table 3.7

Thus, 164 + 373 = 116, 189

We can certainly see that the more powerful calculator simplies computations.
Example 3.22
Nonscientic calculators are unable to handle calculations involving very large numbers.

85612 · 21065

Key Display Reads


Type 85612 85612
Press × 85612
Type 21065 21065
Press =

Table 3.8

This number is too big for the display of some calculators and we'll probably get some kind of error
message. On some scientic calculators such large numbers are coped with by placing them in a
form called "scientic notation." Others can do the multiplication directly. (1803416780)

3.3.5.2 Practice Set D

Use a calculator to nd each value.


Exercise 3.114 (Solution on p. 200.)
9, 285 + 86 (49)
Exercise 3.115 (Solution on p. 200.)
55 (84 − 26) + 120 (512 − 488)
168
CHAPTER 3. EXPONENTS, ROOTS, AND FACTORIZATION OF WHOLE
NUMBERS
Exercise 3.116 (Solution on p. 200.)
1063 − 174
Exercise 3.117 (Solution on p. 201.)
6, 0533

3.3.6 Exercises
For the following problems, nd each value. Check each result with a calculator.
Exercise 3.118 (Solution on p. 201.)
2 + 3 · (8)
Exercise 3.119
18 + 7 · (4 − 1)
Exercise 3.120 (Solution on p. 201.)
3 + 8 · (6 − 2) + 11
Exercise 3.121
1 − 5 · (8 − 8)
Exercise 3.122 (Solution on p. 201.)
37 − 1 · 62
Exercise 3.123
98 ÷ 2 ÷ 72
Exercise 3.124
 (Solution on p. 201.)
42 − 2 · 4 − 23
Exercise
√ 3.125
9 + 14
Exercise
√ √3.126 (Solution on p. 201.)
100 + 81 − 42
Exercise
√ 3.127
3
8+8−2·5
Exercise
√ 3.128 (Solution on p. 201.)
4
16 − 1 + 52
Exercise 3.129
61 − 22 + 4 [3 · (10) + 11]
Exercise 3.130 (Solution on p. 201.)
121 − 4 · [(4) · (5) − 12] + 16
2
Exercise 3.131
(1+16)−3
7 + 5 · (12)
Exercise 3.132 (Solution on p. 201.)
8·(6+20) 3·(6+16)
8 + 22
Exercise 3.133
10 · [8 + 2 · (6 + 7)]
Exercise 3.134 (Solution on p. 201.)
21 ÷ 7 ÷ 3
Exercise 3.135
102 · 3 ÷ 52 · 3 − 2 · 3
169

Exercise 3.136 (Solution on p. 201.)


85 ÷ 5 · 5 − 85
Exercise 3.137
51 + 7 − 2 · 5 · 12 
17 3
Exercise 3.138 (Solution on p. 201.)
22 · 3 + 23 · (6 − 2) − (3 + 17) + 11 (6)
Exercise 3.139
26 − 2 · { 6+ 20
13 }
Exercise 3.140 (Solution on p. 201.)
2 · {(7 + 7) + 6 · [4 · (8 + 2)]}
Exercise 3.141
0 + 10 (0) + 15 · {4 · 3 + 1}
Exercise 3.142 (Solution on p. 201.)
18 + 7+2
9
Exercise 3.143
(4 + 7) · (8 − 3)
Exercise 3.144 (Solution on p. 201.)
(6 + 8) · (5 + 2 − 4)
Exercise 3.145
(21 − 3) · (6 − 1) · (7) + 4 (6 + 3)
Exercise 3.146 (Solution on p. 201.)
(10 + 5) · (10 + 5) − 4 · (60 − 4)
Exercise 3.147
6 · {2 · 8 + 3} − (5) · (2) + 8
4 + (1 + 8) · (1 + 11)
Exercise 3.148 (Solution on p. 201.)
25 + 3 · (8 + 1)
Exercise 3.149
34 + 24 · (1 + 5)
Exercise 3.150 (Solution on p. 201.)
3
16 + 08 + 52 · (2 + 8)
Exercise 3.151
(7) · (16) − 34 + 22 · 17 + 32


Exercise 3.152 (Solution on p. 201.)


23 −7
52
Exercise 3.153
(1+6)2 +2
3·6+1
Exercise 3.154 (Solution on p. 201.)
62 −1 43 +2·3
23 −3 + 2·5
Exercise 3.155
5(82 −9·6) 72 −42
25 −7 + 24 −5
Exercise 3.156 (Solution on p. 201.)
(2+1)3 +23 +110 152 −[2·5]2
62 − 5·52
Exercise 3.157
63 −2·102 18(23 +72 )
22 + 2(19)−33
170
CHAPTER 3. EXPONENTS, ROOTS, AND FACTORIZATION OF WHOLE
NUMBERS
Exercise 3.158 √  (Solution on p. 201.)
2 · {6 + 102 − 6 25 }
Exercise 3.159
√ √ 
181 − 3 · 2 36 + 3 3 64
Exercise 3.160 (Solution on p. 201.)
√ √
2·( 81− 3 125)
4 −10+22
2

3.3.6.1 Exercises for Review

Exercise 3.161
(Section 1.7) The fact that 0 + any whole number = that particular whole number is an example
of which property of addition?
Exercise 3.162 (Solution on p. 201.)
(Section 2.2) Find the product. 4, 271 × 630.
Exercise 3.163
(Section 2.3) In the statement 27 ÷ 3 = 9, what name is given to the result 9?
Exercise 3.164 (Solution on p. 201.)
(Section 2.7) What number is the multiplicative identity?
Exercise 3.165
(Section 2.7) Find the value of 24 .

3.4 Prime Factorization of Natural Numbers4


3.4.1 Section Overview
• Factors
• Determining the Factors of a Whole Number
• Prime and Composite Numbers
• The Fundamental Principle of Arithmetic
• The Prime Factorization of a Natural Number

3.4.2 Factors
From observations made in the process of multiplication, we have seen that

(factor) · (factor) = product

Factors, Product
The two numbers being multiplied are the factors and the result of the multiplication is the product. Now,
using our knowledge of division, we can see that a rst number is a factor of a second number if the rst
number divides into the second number a whole number of times (without a remainder).

One Number as a Factor of Another


A rst number is a factor of a second number if the rst number divides into the second number a whole
number of times (without a remainder).

4 This content is available online at <http://cnx.org/content/m34873/1.2/>.


171

We show this in the following examples:


Example 3.23
3 is a factor of 27, since 27 ÷ 3 = 9, or 3 · 9 = 27.
Example 3.24
7 is a factor of 56, since 56 ÷ 7 = 8, or 7 · 8 = 56.
Example 3.25
4 is not a factor of 10, since 10 ÷ 4 = 2R2. (There is a remainder.)

3.4.3 Determining the Factors of a Whole Number


We can use the tests for divisibility from Section 2.5 to determine all the factors of a whole number.

3.4.3.1 Sample Set A

Example 3.26
Find all the factors of 24.

Try 1: 24 ÷ 1 = 24 1 and 24 are factors


Try 2: 24 is even, so 24 is divisible by 2.
24 ÷ 2 = 12 2 and 12 are factors
Try 3: 2 + 4 = 6 and 6 is divisible by 3, so 24 is divisible by 3.
24 ÷ 3 = 8 3 and 8 are factors
Try 4: 24 ÷ 4 = 6 4 and 6 are factors
Try 5: 24 ÷ 5 = 4R4 5 is not a factor.

The next number to try is 6, but we already have that 6 is a factor. Once we come upon a factor
that we already have discovered, we can stop.

All the whole number factors of 24 are 1, 2, 3, 4, 6, 8, 12, and 24.

3.4.3.2 Practice Set A

Find all the factors of each of the following numbers.


Exercise 3.166 (Solution on p. 202.)
6
Exercise 3.167 (Solution on p. 202.)
12
Exercise 3.168 (Solution on p. 202.)
18
Exercise 3.169 (Solution on p. 202.)
5
Exercise 3.170 (Solution on p. 202.)
10
Exercise 3.171 (Solution on p. 202.)
33
172
CHAPTER 3. EXPONENTS, ROOTS, AND FACTORIZATION OF WHOLE
NUMBERS
Exercise 3.172 (Solution on p. 202.)
19

3.4.4 Prime and Composite Numbers


Notice that the only factors of 7 are 1 and 7 itself, and that the only factors of 3 are 1 and 3 itself. However,
the number 8 has the factors 1, 2, 4, and 8, and the number 10 has the factors 1, 2, 5, and 10. Thus, we
can see that a whole number can have only two factors (itself and 1) and another whole number can have
several factors.
We can use this observation to make a useful classication for whole numbers: prime numbers and composite
numbers.

Prime Number
A whole number (greater than one) whose only factors are itself and 1 is called a prime number.

The Number 1 is Not a Prime Number


The rst seven prime numbers are 2, 3, 5, 7, 11, 13, and 17. Notice that the whole number 1 is not considered
to be a prime number, and the whole number 2 is the rst prime and the only even prime number.

Composite Number
A whole number composed of factors other than itself and 1 is called a composite number. Composite
numbers are not prime numbers.

Some composite numbers are 4, 6, 8, 9, 10, 12, and 15.

3.4.4.1 Sample Set B

Determine which whole numbers are prime and which are composite.
Example 3.27
39. Since 3 divides into 39, the number 39 is composite: 39 ÷ 3 = 13
Example 3.28
47. A few division trials will assure us that 47 is only divisible by 1 and 47. Therefore, 47 is prime.

3.4.4.2 Practice Set B

Determine which of the following whole numbers are prime and which are composite.
Exercise 3.173 (Solution on p. 202.)
3
Exercise 3.174 (Solution on p. 202.)
16
Exercise 3.175 (Solution on p. 202.)
21
Exercise 3.176 (Solution on p. 202.)
35
Exercise 3.177 (Solution on p. 202.)
47
Exercise 3.178 (Solution on p. 202.)
29
173

Exercise 3.179 (Solution on p. 202.)


101
Exercise 3.180 (Solution on p. 202.)
51

3.4.5 The Fundamental Principle of Arithmetic


Prime numbers are very useful in the study of mathematics. We will see how they are used in subsequent
sections. We now state the Fundamental Principle of Arithmetic.

Fundamental Principle of Arithmetic


Except for the order of the factors, every natural number other than 1 can be factored in one and only one
way as a product of prime numbers.

Prime Factorization
When a number is factored so that all its factors are prime numbers. the factorization is called the prime
factorization of the number.

The technique of prime factorization is illustrated in the following three examples.

1. 10 = 5 · 2. Both 2 and 5 are primes. Therefore, 2 · 5 is the prime factorization of 10.


2. 11. The number 11 is a prime number. Prime factorization applies only to composite numbers. Thus,
11 has no prime factorization.
3. 60 = 2 · 30. The number 30 is not prime: 30 = 2 · 15.

60 = 2 · 2 · 15

The number 15 is not prime: 15 = 3 · 5

60 = 2 · 2 · 3 · 5

We'll use exponents.

60 = 22 · 3 · 5

The numbers 2, 3, and 5 are each prime. Therefore, 22 · 3 · 5 is the prime factorization of 60.

3.4.6 The Prime Factorization of a Natural Number


The following method provides a way of nding the prime factorization of a natural number.
The Method of Finding the Prime Factorization of a Natural Number

1. Divide the number repeatedly by the smallest prime number that will divide into it a whole number
of times (without a remainder).
2. When the prime number used in step 1 no longer divides into the given number without a remainder,
repeat the division process with the next largest prime that divides the given number.
3. Continue this process until the quotient is smaller than the divisor.
4. The prime factorization of the given number is the product of all these prime divisors. If the number
has no prime divisors, it is a prime number.

We may be able to use some of the tests for divisibility we studied in Section 2.5 to help nd the primes
that divide the given number.
174
CHAPTER 3. EXPONENTS, ROOTS, AND FACTORIZATION OF WHOLE
NUMBERS
3.4.6.1 Sample Set C

Example 3.29
Find the prime factorization of 60.

Since the last digit of 60 is 0, which is even, 60 is divisible by 2. We will repeatedly divide by 2
until we no longer can. We shall divide as follows:

30 is divisible by 2 again.
15 is not divisible by 2, but it is divisible by 3, the next prime.
5 is not divisble by 3, but it is divisible by 5, the next prime.

The quotient 1 is nally smaller than the divisor 5, and the prime factorization of 60 is the product
of these prime divisors.

60 = 2 · 2 · 3 · 5

We use exponents when possible.

60 = 22 · 3 · 5
Example 3.30
Find the prime factorization of 441.

441 is not divisible by 2 since its last digit is not divisible by 2.

441 is divisible by 3 since 4 + 4 + 1 = 9 and 9 is divisible by 3.

147 is divisible by 3 (1 + 4 + 7 = 12) .


49 is not divisible by 3, nor is it divisible by 5. It is divisible by 7.

The quotient 1 is nally smaller than the divisor 7, and the prime factorization of 441 is the product
of these prime divisors.

441 = 3 · 3 · 7 · 7

Use exponents.

441 = 32 · 72
Example 3.31
Find the prime factorization of 31.
175

31 is not divisible by 2 Its last digit is not even


31 ÷ 2 = 15R1
The quotient, 15, is larger than the divisor, 3. Continue.
31 is not divisible by 3 The digits 3 + 1 = 4, and 4 is not divisible by 3.
31 ÷ 3 = 10R1
The quotient, 10, is larger than the divisor, 3. Continue.
31 is not divisible by 5 The last digit of 31 is not 0 or 5.
31 ÷ 5 = 6R1
The quotient, 6, is larger than the divisor, 5. Continue.
31 is not divisible by 7. Divide by 7.
31 ÷ 7 = 4R1
The quotient, 4, is smaller than the divisor, 7.
We can stop the process and conclude that 31 is a prime number.

The number 31 is a prime number

3.4.6.2 Practice Set C

Find the prime factorization of each whole number.


Exercise 3.181 (Solution on p. 202.)
22
Exercise 3.182 (Solution on p. 202.)
40
Exercise 3.183 (Solution on p. 202.)
48
Exercise 3.184 (Solution on p. 202.)
63
Exercise 3.185 (Solution on p. 202.)
945
Exercise 3.186 (Solution on p. 202.)
1,617
Exercise 3.187 (Solution on p. 202.)
17
Exercise 3.188 (Solution on p. 202.)
61

3.4.7 Exercises
For the following problems, determine the missing factor(s).
Exercise 3.189 (Solution on p. 202.)
14=7·
176
CHAPTER 3. EXPONENTS, ROOTS, AND FACTORIZATION OF WHOLE
NUMBERS
Exercise 3.190
20=4·
Exercise 3.191 (Solution on p. 202.)
36=9·
Exercise 3.192
42 = 21·
Exercise 3.193 (Solution on p. 203.)
44=4·
Exercise 3.194
38=2·
Exercise 3.195 (Solution on p. 203.)
18=3· ·
Exercise 3.196
28=2· ·
Exercise 3.197 (Solution on p. 203.)
300=2·5· ·
Exercise 3.198
840=2· · ·
For the following problems, nd all the factors of each of the numbers.
Exercise 3.199 (Solution on p. 203.)
16
Exercise 3.200
22
Exercise 3.201 (Solution on p. 203.)
56
Exercise 3.202
105
Exercise 3.203 (Solution on p. 203.)
220
Exercise 3.204
15
Exercise 3.205 (Solution on p. 203.)
32
Exercise 3.206
80
Exercise 3.207 (Solution on p. 203.)
142
Exercise 3.208
218
For the following problems, determine which of the whole numbers are prime and which are composite.
Exercise 3.209 (Solution on p. 203.)
23
Exercise 3.210
25
177

Exercise 3.211 (Solution on p. 203.)


27
Exercise 3.212
2
Exercise 3.213 (Solution on p. 203.)
3
Exercise 3.214
5
Exercise 3.215 (Solution on p. 203.)
7
Exercise 3.216
9
Exercise 3.217 (Solution on p. 203.)
11
Exercise 3.218
34
Exercise 3.219 (Solution on p. 203.)
55
Exercise 3.220
63
Exercise 3.221 (Solution on p. 203.)
1,044
Exercise 3.222
924
Exercise 3.223 (Solution on p. 203.)
339
Exercise 3.224
103
Exercise 3.225 (Solution on p. 203.)
209
Exercise 3.226
667
Exercise 3.227 (Solution on p. 203.)
4,575
Exercise 3.228
119
For the following problems, nd the prime factorization of each of the whole numbers.
Exercise 3.229 (Solution on p. 203.)
26
Exercise 3.230
38
Exercise 3.231 (Solution on p. 203.)
54
Exercise 3.232
62
178
CHAPTER 3. EXPONENTS, ROOTS, AND FACTORIZATION OF WHOLE
NUMBERS
Exercise 3.233 (Solution on p. 203.)
56
Exercise 3.234
176
Exercise 3.235 (Solution on p. 203.)
480
Exercise 3.236
819
Exercise 3.237 (Solution on p. 203.)
2,025
Exercise 3.238
148,225

3.4.7.1 Exercises For Review

Exercise 3.239 (Solution on p. 203.)


(Section 1.4) Round 26,584 to the nearest ten.
Exercise 3.240
(Section 1.6) How much bigger is 106 than 79?
Exercise 3.241 (Solution on p. 203.)
(Section 2.3) True or false? Zero divided by any nonzero whole number is zero.
Exercise 3.242
(Section 2.4) Find the quotient. 10, 584 ÷ 126.
Exercise 3.243 √ √ (Solution on p. 204.)
(Section 3.3) Find the value of 121 − 81 + 62 ÷ 3.

3.5 The Greatest Common Factor 5


3.5.1 Section Overview
• The Greatest Common Factor (GCF)
• A Method for Determining the Greatest Common Factor

3.5.2 The Greatest Common Factor (GCF)


Using the method we studied in Section 3.4, we could obtain the prime factorizations of 30 and 42.

30 = 2 · 3 · 5

42 = 2 · 3 · 7

Common Factor
We notice that 2 appears as a factor in both numbers, that is, 2 is a common factor of 30 and 42. We also
notice that 3 appears as a factor in both numbers. Three is also a common factor of 30 and 42.

5 This content is available online at <http://cnx.org/content/m34874/1.2/>.


179

Greatest Common Factor (GCF)


When considering two or more numbers, it is often useful to know if there is a largest common factor of the
numbers, and if so, what that number is. The largest common factor of two or more whole numbers is called
the greatest common factor, and is abbreviated by GCF. The greatest common factor of a collection of
whole numbers is useful in working with fractions (which we will do in Section 4.1).

3.5.3 A Method for Determining the Greatest Common Factor


A straightforward method for determining the GCF of two or more whole numbers makes use of both the
prime factorization of the numbers and exponents.

Finding the GCF


To nd the greatest common factor (GCF) of two or more whole numbers:
1. Write the prime factorization of each number, using exponents on repeated factors.
2. Write each base that is common to each of the numbers.
3. To each base listed in step 2, attach the smallest exponent that appears on it in either of the prime
factorizations.
4. The GCF is the product of the numbers found in step 3.

3.5.3.1 Sample Set A

Find the GCF of the following numbers.


Example 3.32
12 and 18

12 = 2 · 6 = 2 · 2 · 3 = 22 · 3
1.
18 = 2 · 9 = 2 · 3 · 3 = 2 · 32
2. The common bases are 2 and 3.
3. The smallest exponents appearing on 2 and 3 in the prime factorizations are, respectively, 1
and 1 ( 21 and 31 ), or 2 and 3.
4. The GCF is the product of these numbers. 2 · 3 = 6

The GCF of 30 and 42 is 6 because 6 is the largest number that divides both 30 and 42 without a
remainder.
Example 3.33
18, 60, and 72

18 = 2 · 9 = 2 · 3 · 3 = 2 · 32
1. 60 = 2 · 30 = 2 · 2 · 15 = 2 · 2 · 3 · 5 = 22 · 3 · 5
72 = 2 · 36 = 2 · 2 · 18 = 2 · 2 · 2 · 9 = 2 · 2 · 2 · 3 · 3 = 23 · 32
2. The common bases are 2 and 3.
3. The smallest exponents appearing on 2 and 3 in the prime factorizations are, respectively, 1
and 1:

21 from 18

31 from 60
4. The GCF is the product of these numbers.

GCF is 2 · 3 = 6
180
CHAPTER 3. EXPONENTS, ROOTS, AND FACTORIZATION OF WHOLE
NUMBERS
Thus, 6 is the largest number that divides 18, 60, and 72 without a remainder.
Example 3.34
700, 1,880, and 6,160

700 = 2 · 350 = 2 · 2 · 175 = 2 · 2 · 5 · 35


= 2·2·5·5·7
= 22 · 52 · 7
1, 880 = 2 · 940 = 2 · 2 · 470 = 2 · 2 · 2 · 235
= 2 · 2 · 2 · 5 · 47
1. = 23 · 5 · 47
6, 160 = 2 · 3, 080 = 2 · 2 · 1, 540 = 2 · 2 · 2 · 770
= 2 · 2 · 2 · 2 · 385
= 2 · 2 · 2 · 2 · 5 · 77
= 2 · 2 · 2 · 2 · 5 · 7 · 11
= 24 · 5 · 7 · 11
2. The common bases are 2 and 5
3. The smallest exponents appearing on 2 and 5 in the prime factorizations are, respectively, 2
and 1.

22 from 700.

51 from either 1,880 or 6,160.


4. The GCF is the product of these numbers.

GCF is 22 · 5 = 4 · 5 = 20

Thus, 20 is the largest number that divides 700, 1,880, and 6,160 without a remainder.

3.5.3.2 Practice Set A

Find the GCF of the following numbers.


Exercise 3.244 (Solution on p. 204.)
24 and 36
Exercise 3.245 (Solution on p. 204.)
48 and 72
Exercise 3.246 (Solution on p. 204.)
50 and 140
Exercise 3.247 (Solution on p. 204.)
21 and 225
Exercise 3.248 (Solution on p. 204.)
450, 600, and 540
181

3.5.4 Exercises
For the following problems, nd the greatest common factor (GCF) of the numbers.
Exercise 3.249 (Solution on p. 204.)
6 and 8
Exercise 3.250
5 and 10
Exercise 3.251 (Solution on p. 204.)
8 and 12
Exercise 3.252
9 and 12
Exercise 3.253 (Solution on p. 204.)
20 and 24
Exercise 3.254
35 and 175
Exercise 3.255 (Solution on p. 204.)
25 and 45
Exercise 3.256
45 and 189
Exercise 3.257 (Solution on p. 204.)
66 and 165
Exercise 3.258
264 and 132
Exercise 3.259 (Solution on p. 204.)
99 and 135
Exercise 3.260
65 and 15
Exercise 3.261 (Solution on p. 204.)
33 and 77
Exercise 3.262
245 and 80
Exercise 3.263 (Solution on p. 204.)
351 and 165
Exercise 3.264
60, 140, and 100
Exercise 3.265 (Solution on p. 204.)
147, 343, and 231
Exercise 3.266
24, 30, and 45
Exercise 3.267 (Solution on p. 204.)
175, 225, and 400
Exercise 3.268
210, 630, and 182
Exercise 3.269 (Solution on p. 204.)
14, 44, and 616
182
CHAPTER 3. EXPONENTS, ROOTS, AND FACTORIZATION OF WHOLE
NUMBERS
Exercise 3.270
1,617, 735, and 429
Exercise 3.271 (Solution on p. 204.)
1,573, 4,862, and 3,553
Exercise 3.272
3,672, 68, and 920
Exercise 3.273 (Solution on p. 204.)
7, 2,401, 343, 16, and 807
Exercise 3.274
500, 77, and 39
Exercise 3.275 (Solution on p. 204.)
441, 275, and 221

3.5.4.1 Exercises for Review

Exercise 3.276
(Section 2.2) Find the product. 2, 753 × 4, 006.
Exercise 3.277 (Solution on p. 204.)
(Section 2.4) Find the quotient. 954 ÷ 18.
Exercise 3.278
(Section 2.5) Specify which of the digits 2, 3, or 4 divide into 9,462.
Exercise 3.279 (Solution on p. 204.)
(Section 3.2) Write 8 × 8 × 8 × 8 × 8 × 8 using exponents.
Exercise 3.280
(Section 3.4) Find the prime factorization of 378.

3.6 The Least Common Multiple6


3.6.1 Section Overview
• Multiples
• Common Multiples
• The Least Common Multiple (LCM)
• Finding the Least Common Multiple

3.6.2 Multiples
When a whole number is multiplied by other whole numbers, with the exception of zero, the resulting
products are called multiples of the given whole number. Note that any whole number is a multiple of
itself.
6 This content is available online at <http://cnx.org/content/m34876/1.2/>.
183

3.6.2.1 Sample Set A

Multiples of 2 Multiples of 3 Multiples of 8 Multiples of 10


2×1=2 3×1=3 8×1=8 10 × 1 = 10
2×2=4 3×2=6 8 × 2 = 16 10 × 2 = 20
2×3=6 3×3=9 8 × 3 = 24 10 × 3 = 30
2×4=8 3 × 4 = 12 8 × 4 = 32 10 × 4 = 40
2 × 5 = 10 3 × 5 = 15 8 × 5 = 40 10 × 5 = 50
.. .. .. ..
. . . .

Table 3.9

3.6.2.2 Practice Set A

Find the rst ve multiples of the following numbers.


Exercise 3.281 (Solution on p. 204.)
4
Exercise 3.282 (Solution on p. 204.)
5
Exercise 3.283 (Solution on p. 204.)
6
Exercise 3.284 (Solution on p. 205.)
7
Exercise 3.285 (Solution on p. 205.)
9

3.6.3 Common Multiples


There will be times when we are given two or more whole numbers and we will need to know if there are any
multiples that are common to each of them. If there are, we will need to know what they are. For example,
some of the multiples that are common to 2 and 3 are 6, 12, and 18.

3.6.3.1 Sample Set B

Example 3.35
We can visualize common multiples using the number line.

Notice that the common multiples can be divided by both whole numbers.
184
CHAPTER 3. EXPONENTS, ROOTS, AND FACTORIZATION OF WHOLE
NUMBERS
3.6.3.2 Practice Set B

Find the rst ve common multiples of the following numbers.


Exercise 3.286 (Solution on p. 205.)
2 and 4
Exercise 3.287 (Solution on p. 205.)
3 and 4
Exercise 3.288 (Solution on p. 205.)
2 and 5
Exercise 3.289 (Solution on p. 205.)
3 and 6
Exercise 3.290 (Solution on p. 205.)
4 and 5

3.6.4 The Least Common Multiple (LCM)


Notice that in our number line visualization of common multiples (above), the rst common multiple is also
the smallest, or least common multiple, abbreviated by LCM.

Least Common Multiple


The least common multiple, LCM, of two or more whole numbers is the smallest whole number that
each of the given numbers will divide into without a remainder.

The least common multiple will be extremely useful in working with fractions (Section 4.1).

3.6.5 Finding the Least Common Multiple


Finding the LCM
To nd the LCM of two or more numbers:

1. Write the prime factorization of each number, using exponents on repeated factors.
2. Write each base that appears in each of the prime factorizations.
3. To each base, attach the largest exponent that appears on it in the prime factorizations.
4. The LCM is the product of the numbers found in step 3.

There are some major dierences between using the processes for obtaining the GCF and the LCM that we
must note carefully:
The Dierence Between the Processes for Obtaining the GCF and the LCM

1. Notice the dierence between step 2 for the LCM and step 2 for the GCF. For the GCF, we use only
the bases that are common in the prime factorizations, whereas for the LCM, we use each base that
appears in the prime factorizations.
2. Notice the dierence between step 3 for the LCM and step 3 for the GCF. For the GCF, we attach
the smallest exponents to the common bases, whereas for the LCM, we attach the largest exponents
to the bases.
185

3.6.5.1 Sample Set C

Find the LCM of the following numbers.


Example 3.36
9 and 12
9 = 3 · 3 = 32
1.
12 = 2 · 6 = 2 · 2 · 3 = 22 · 3
2. The bases that appear in the prime factorizations are 2 and 3.
3. The largest exponents appearing on 2 and 3 in the prime factorizations are, respectively, 2
and 2: 22 from 12.

32 from 9.
4. The LCM is the product of these numbers. LCM = 22 · 32 = 4 · 9 = 36
Thus, 36 is the smallest number that both 9 and 12 divide into without remainders.
Example 3.37
90 and 630
90 = 2 · 45 = 2 · 3 · 15 = 2 · 3 · 3 · 5 = 2 · 32 · 5
1. 630 = 2 · 315 = 2 · 3 · 105 = 2 · 3 · 3 · 35 =2·3·3·5·7
= 2 · 32 · 5 · 7
2. The bases that appear in the prime factorizations are 2, 3, 5, and 7.
3. The largest exponents that appear on 2, 3, 5, and 7 are, respectively, 1, 2, 1, and 1:
21 from either 90 or 630.
32 from either 90 or 630.
51 from either 90 or 630.
71 from 630.
4. The LCM is the product of these numbers. LCM = 2 · 32 · 5 · 7 = 2 · 9 · 5 · 7 = 630
Thus, 630 is the smallest number that both 90 and 630 divide into with no remainders.
Example 3.38
33, 110, and 484

33 = 3 · 11
1. 110 = 2 · 55 = 2 · 5 · 11
484 = 2 · 242 = 2 · 2 · 121 = 2 · 2 · 11 · 11 = 22 · 112 .
2. The bases that appear in the prime factorizations are 2, 3, 5, and 11.
3. The largest exponents that appear on 2, 3, 5, and 11 are, respectively, 2, 1, 1, and 2:
22 from 484.
31 from 33.
51 from 110
112 from 484.
LCM = 22 · 3 · 5 · 112
4. The LCM is the product of these numbers. = 4 · 3 · 5 · 121
= 7260

Thus, 7260 is the smallest number that 33, 110, and 484 divide into without remainders.
186
CHAPTER 3. EXPONENTS, ROOTS, AND FACTORIZATION OF WHOLE
NUMBERS
3.6.5.2 Practice Set C

Find the LCM of the following numbers.


Exercise 3.291 (Solution on p. 205.)
20 and 54
Exercise 3.292 (Solution on p. 205.)
14 and 28
Exercise 3.293 (Solution on p. 205.)
6 and 63
Exercise 3.294 (Solution on p. 205.)
28, 40, and 98
Exercise 3.295 (Solution on p. 205.)
16, 27, 125, and 363

3.6.6 Exercises
For the following problems, nd the least common multiple of the numbers.
Exercise 3.296 (Solution on p. 205.)
8 and 12
Exercise 3.297
6 and 15
Exercise 3.298 (Solution on p. 205.)
8 and 10
Exercise 3.299
10 and 14
Exercise 3.300 (Solution on p. 205.)
4 and 6
Exercise 3.301
6 and 12
Exercise 3.302 (Solution on p. 205.)
9 and 18
Exercise 3.303
6 and 8
Exercise 3.304 (Solution on p. 205.)
5 and 6
Exercise 3.305
7 and 8
Exercise 3.306 (Solution on p. 205.)
3 and 4
Exercise 3.307
2 and 9
Exercise 3.308 (Solution on p. 205.)
7 and 9
Exercise 3.309
28 and 36
187

Exercise 3.310 (Solution on p. 205.)


24 and 36
Exercise 3.311
28 and 42
Exercise 3.312 (Solution on p. 205.)
240 and 360
Exercise 3.313
162 and 270
Exercise 3.314 (Solution on p. 205.)
20 and 24
Exercise 3.315
25 and 30
Exercise 3.316 (Solution on p. 205.)
24 and 54
Exercise 3.317
16 and 24
Exercise 3.318 (Solution on p. 205.)
36 and 48
Exercise 3.319
24 and 40
Exercise 3.320 (Solution on p. 205.)
15 and 21
Exercise 3.321
50 and 140
Exercise 3.322 (Solution on p. 206.)
7, 11, and 33
Exercise 3.323
8, 10, and 15
Exercise 3.324 (Solution on p. 206.)
18, 21, and 42
Exercise 3.325
4, 5, and 21
Exercise 3.326 (Solution on p. 206.)
45, 63, and 98
Exercise 3.327
15, 25, and 40
Exercise 3.328 (Solution on p. 206.)
12, 16, and 20
Exercise 3.329
84 and 96
Exercise 3.330 (Solution on p. 206.)
48 and 54
Exercise 3.331
12, 16, and 24
Exercise 3.332 (Solution on p. 206.)
12, 16, 24, and 36
188
CHAPTER 3. EXPONENTS, ROOTS, AND FACTORIZATION OF WHOLE
NUMBERS
Exercise 3.333
6, 9, 12, and 18
Exercise 3.334 (Solution on p. 206.)
8, 14, 28, and 32
Exercise 3.335
18, 80, 108, and 490
Exercise 3.336 (Solution on p. 206.)
22, 27, 130, and 225
Exercise 3.337
38, 92, 115, and 189
Exercise 3.338 (Solution on p. 206.)
8 and 8
Exercise 3.339
12, 12, and 12
Exercise 3.340 (Solution on p. 206.)
3, 9, 12, and 3

3.6.6.1 Exercises for Review

Exercise 3.341
(Section 1.4) Round 434,892 to the nearest ten thousand.
Exercise 3.342 (Solution on p. 206.)
(Section 1.6) How much bigger is 14,061 than 7,509?
Exercise 3.343
(Section 2.4) Find the quotient. 22, 428 ÷ 14.
Exercise 3.344 (Solution on p. 206.)
(Section 3.2) Expand 843 . Do not nd the value.
Exercise 3.345
(Section 3.5) Find the greatest common factor of 48 and 72.

3.7 Summary of Key Concepts7


3.7.1 Summary of Key Concepts
Exponential Notation (Section 3.2)
Exponential notation is a description of repeated multiplication.

Exponent (Section 3.2)


An exponent records the number of identical factors repeated in a multiplication.

In a number such as 73 ,

Base (Section 3.2)


7 is called the base.

7 This content is available online at <http://cnx.org/content/m34877/1.2/>.


189

Exponent (Section 3.2)


3 is called the exponent, or power.

Power (Section 3.2)


73 is read "seven to the third power," or "seven cubed."

Squared, Cubed (Section 3.2)


A number raised to the second power is often called squared. A number raised to the third power is often
called cubed.

Root (Section 3.2)


In mathematics, the word root is used to indicate that, through repeated multiplication, one number is the
source of another number.

The Radical Sign (Section 3.2)
√ √
The symbol is called a radical sign and indicates the square root of a number. The symbol n

represents the nth root.

Radical, Index, Radicand


√ (Section 3.2)
An expression such as 4 16 is called a radical and 4 is called the index. The number 16 is called the
radicand.

Grouping Symbols (Section 3.3)


Grouping symbols are used to indicate that a particular collection of numbers and meaningful operations are
to be grouped together and considered as one number. The grouping symbols commonly used in mathematics
are

Parentheses: ( )
Brackets: [ ]
Braces: { }
Bar:

Order of Operations (Section 3.3)

1. Perform all operations inside grouping symbols, beginning with the innermost set, in the order of 2, 3,
and 4 below.
2. Perform all exponential and root operations, moving left to right.
3. Perform all multiplications and division, moving left to right.
4. Perform all additions and subtractions, moving left to right.

One Number as the Factor of Another (Section 3.4)


A rst number is a factor of a second number if the rst number divides into the second number a whole
number of times.

Prime Number (Section 3.4)


A whole number greater than one whose only factors are itself and 1 is called a prime number. The whole
number 1 is not a prime number. The whole number 2 is the rst prime number and the only even prime
number.

Composite Number (Section 3.4)


A whole number greater than one that is composed of factors other than itself and 1 is called a composite
number.
190
CHAPTER 3. EXPONENTS, ROOTS, AND FACTORIZATION OF WHOLE
NUMBERS
Fundamental Principle of Arithmetic (Section 3.4)
Except for the order of factors, every whole number other than 1 can be written in one and only one way as
a product of prime numbers.

Prime Factorization (Section 3.4)


The prime factorization of 45 is 3 · 3 · 5. The numbers that occur in this factorization of 45 are each prime.

Determining the Prime Factorization of a Whole Number (Section 3.4)


There is a simple method, based on division by prime numbers, that produces the prime factorization of a
whole number. For example, we determine the prime factorization of 132 as follows.

The prime factorization of 132 is 2 · 2 · 3 · 11 = 22 · 3 · 11.

Common Factor (Section 3.5)


A factor that occurs in each number of a group of numbers is called a common factor. 3 is a common
factor to the group 18, 6, and 45

Greatest Common Factor (GCF) (Section 3.5)


The largest common factor of a group of whole numbers is called the greatest common factor. For
example, to nd the greatest common factor of 12 and 20,

12 = 2 · 2 · 3 = 22 · 3
1. Write the prime factorization of each number.
60 = 2 · 2 · 3 · 5 = 22 · 3 · 5
2. Write each base that is common to each of the numbers: 2 and 3
3. The smallest exponent appearing on 2 is 2.
The smallest exponent appearing on 3 is 1.
4. The GCF of 12 and 60 is the product of the numbers 22 and 3. 22 · 3 = 4 · 3 = 12

Thus, 12 is the largest number that divides both 12 and 60 without a remainder.

Finding the GCF (Section 3.5)


There is a simple method, based on prime factorization, that determines the GCF of a group of whole
numbers.

Multiple (Section 3.6)


When a whole number is multiplied by all other whole numbers, with the exception of zero, the resulting
individual products are called multiples of that whole number. Some multiples of 7 are 7, 14, 21, and 28.

Common Multiples (Section 3.6)


Multiples that are common to a group of whole numbers are called common multiples. Some common
multiples of 6 and 9 are 18, 36, and 54.

The LCM (Section 3.6)


The least common multiple (LCM) of a group of whole numbers is the smallest whole number that each
of the given whole numbers divides into without a remainder. The least common multiple of 9 and 6 is 18.

Finding the LCM (Section 3.6)


There is a simple method, based on prime factorization, that determines the LCM of a group of whole
numbers. For example, the least common multiple of 28 and 72 is found in the following way.
191

28 = 2 · 2 · 7 = 22 · 7
1. Write the prime factorization of each number
72 = 2 · 2 · 2 · 3 · 3 = 23 · 32
2. Write each base that appears in each of the prime factorizations, 2, 3, and 7.
3. To each of the bases listed in step 2, attach the largest exponent that appears on it in the prime
factorization. 23 , 32 , and 7
4. The LCM is the product of the numbers found in step 3. 23 · 32 · 7 = 8 · 9 · 7 = 504

Thus, 504 is the smallest number that both 28 and 72 will divide into without a remainder.

The Dierence Between the GCF and the LCM (Section 3.6)
The GCF of two or more whole numbers is the largest number that divides into each of the given whole
numbers. The LCM of two or more whole numbers is the smallest whole number that each of the given
numbers divides into without a remainder.

3.8 Exercise Supplement 8


3.8.1 Exercise Supplement
3.8.1.1 Exponents and Roots (Section 3.2)

For problems 1 -25, determine the value of each power and root.
Exercise 3.346 (Solution on p. 206.)
33
Exercise 3.347
43
Exercise 3.348 (Solution on p. 206.)
05
Exercise 3.349
14
Exercise 3.350 (Solution on p. 206.)
122
Exercise 3.351
72
Exercise 3.352 (Solution on p. 206.)
82
Exercise 3.353
112
Exercise 3.354 (Solution on p. 206.)
25
Exercise 3.355
34
Exercise 3.356 (Solution on p. 206.)
152
Exercise 3.357
202
Exercise 3.358 (Solution on p. 206.)
252
8 This content is available online at <http://cnx.org/content/m34878/1.2/>.
192
CHAPTER 3. EXPONENTS, ROOTS, AND FACTORIZATION OF WHOLE
NUMBERS
Exercise
√ 3.359
36
Exercise
√ 3.360 (Solution on p. 206.)
225
Exercise
√ 3.361
3
64
Exercise
√ 3.362 (Solution on p. 206.)
4
16
Exercise
√ 3.363
0
Exercise
√ 3.364 (Solution on p. 206.)
3
1
Exercise
√ 3.365
3
216
Exercise
√ 3.366 (Solution on p. 206.)
144
Exercise
√ 3.367
196
Exercise
√ 3.368 (Solution on p. 206.)
1
Exercise
√ 3.369
4
0
Exercise
√ 3.370 (Solution on p. 206.)
6
64

3.8.1.2 Section 3.2

For problems 26-45, use the order of operations to determine each value.
Exercise 3.371
23 − 2 · 4
Exercise 3.372 (Solution on p. 207.)
52 − 10 · 2 − 5
Exercise
√ 3.373
81 − 32 + 6 · 2
Exercise 3.374 (Solution on p. 207.)
152 + 52 · 22
Exercise 3.375

3 · 22 + 32
Exercise 3.376 (Solution on p. 207.)
64 · 32 − 23


Exercise 3.377
52 +1 3 +13

13 + 14
Exercise 3.378 (Solution on p. 207.)
62 −1
5·7 − 492·7
+7
193

Exercise 3.379
2·[3+5(22 +1)]
5·23 −32
Exercise 3.380 (Solution on p. 207.)
32 ·[25 −14 (23 +25)]
2·52 +5+2
Exercise 3.381
(52 −23 )−2·7
h i
32 −3
22 −1 +5· 2 +1
Exercise 3.382 (Solution on p. 207.)
2 2 2

(8 − 3) + 2 + 3
Exercise 3.383
√  √
25 + 23 · 81 − 32

32 · 42 +
Exercise 3.384 (Solution on p. 207.)

16 + 9
Exercise
√ √3.385
16 + 9
Exercise 3.386 (Solution on p. 207.)
Compare the results of problems 39 and 40. What might we conclude?
Exercise
√ 3.387
18 · 2
Exercise
√ 3.388 (Solution on p. 207.)
6·6
Exercise
√ 3.389
7·7
Exercise
√ 3.390 (Solution on p. 207.)
8·8
Exercise 3.391
An records the number of identical factors that are repeated in a multiplication.

3.8.1.3 Prime Factorization of Natural Numbers (Section 3.4)

For problems 47- 53, nd all the factors of each number.
Exercise 3.392 (Solution on p. 207.)
18
Exercise 3.393
24
Exercise 3.394 (Solution on p. 207.)
11
Exercise 3.395
12
Exercise 3.396 (Solution on p. 207.)
51
Exercise 3.397
25
Exercise 3.398 (Solution on p. 207.)
2
194
CHAPTER 3. EXPONENTS, ROOTS, AND FACTORIZATION OF WHOLE
NUMBERS
Exercise 3.399
What number is the smallest prime number?

3.8.1.4 Grouping Symbol and the Order of Operations (Section 3.3)

For problems 55 -64, write each number as a product of prime factors.


Exercise 3.400 (Solution on p. 207.)
55
Exercise 3.401
20
Exercise 3.402 (Solution on p. 207.)
80
Exercise 3.403
284
Exercise 3.404 (Solution on p. 207.)
700
Exercise 3.405
845
Exercise 3.406 (Solution on p. 207.)
1,614
Exercise 3.407
921
Exercise 3.408 (Solution on p. 207.)
29
Exercise 3.409
37

3.8.1.5 The Greatest Common Factor (Section 3.5)

For problems 65 - 75, nd the greatest common factor of each collection of numbers.
Exercise 3.410 (Solution on p. 207.)
5 and 15
Exercise 3.411
6 and 14
Exercise 3.412 (Solution on p. 207.)
10 and 15
Exercise 3.413
6, 8, and 12
Exercise 3.414 (Solution on p. 207.)
18 and 24
Exercise 3.415
42 and 54
Exercise 3.416 (Solution on p. 207.)
40 and 60
195

Exercise 3.417
18, 48, and 72
Exercise 3.418 (Solution on p. 207.)
147, 189, and 315
Exercise 3.419
64, 72, and 108
Exercise 3.420 (Solution on p. 207.)
275, 297, and 539

3.8.1.6 The Least Common Multiple (Section 3.6)

For problems 76-86, nd the least common multiple of each collection of numbers.
Exercise 3.421
5 and 15
Exercise 3.422 (Solution on p. 208.)
6 and 14
Exercise 3.423
10 and 15
Exercise 3.424 (Solution on p. 208.)
36 and 90
Exercise 3.425
42 and 54
Exercise 3.426 (Solution on p. 208.)
8, 12, and 20
Exercise 3.427
40, 50, and 180
Exercise 3.428 (Solution on p. 208.)
135, 147, and 324
Exercise 3.429
108, 144, and 324
Exercise 3.430 (Solution on p. 208.)
5, 18, 25, and 30
Exercise 3.431
12, 15, 18, and 20
Exercise 3.432 (Solution on p. 208.)
Find all divisors of 24.
Exercise 3.433
Find all factors of 24.
Exercise 3.434 (Solution on p. 208.)
Write all divisors of 23 · 52 · 7.
Exercise 3.435
Write all divisors of 6 · 82 · 103 .
Exercise 3.436 (Solution on p. 208.)
Does 7 divide 53 · 64 · 72 · 85 ?
196
CHAPTER 3. EXPONENTS, ROOTS, AND FACTORIZATION OF WHOLE
NUMBERS
Exercise 3.437
Does 13 divide 83 · 102 · 114 · 132 · 15?

3.9 Prociency Exam 9

3.9.1 Prociency Exam


Exercise 3.438 (Solution on p. 208.)
(Section 3.2) In the number 85 , write the names used for the number 8 and the number 5.
Exercise 3.439 (Solution on p. 208.)
(Section 3.2) Write using exponents. 12 × 12 × 12 × 12 × 12 × 12 × 12
Exercise 3.440 (Solution on p. 208.)
(Section 3.2) Expand 94 .
For problems 4-15, determine the value of each expression.
Exercise 3.441 (Solution on p. 208.)
(Section 3.3) 43
Exercise 3.442 (Solution on p. 208.)
(Section 3.3) 15
Exercise 3.443 (Solution on p. 208.)
(Section 3.3) 03
Exercise 3.444 (Solution on p. 208.)
(Section 3.3) 26
Exercise 3.445
√ (Solution on p. 208.)
(Section 3.3) 49
Exercise 3.446
√ (Solution on p. 208.)
(Section 3.3) 3
27
Exercise 3.447
√ (Solution on p. 208.)
(Section 3.3) 8
1
Exercise 3.448 (Solution on p. 208.)
(Section 3.3) 16 + 2 · (8 − 6)
Exercise 3.449 √ (Solution on p. 208.)
(Section 3.3) 53 − 100 + 8 · 2 − 20 ÷ 5
Exercise 3.450 (Solution on p. 208.)
82 −2·32 63 −4·52
(Section 3.3) 3 · 52 −2 · 29
Exercise 3.451 √
(Solution on p. 208.)
(Section 3.3) 2320·2−5·2
+24
· 5·7− 81
7+3·2
Exercise 3.452 (Solution on p. 208.)
h
2 √ i
(Section 3.3) (8 − 3) + 33 − 4 49 − 2 10 − 32 + 9 − 5
  

For problems 16-20, nd the prime factorization of each whole number. If the number is prime, write "prime."
Exercise 3.453 (Solution on p. 208.)
(Section 3.4) 18
Exercise 3.454 (Solution on p. 208.)
(Section 3.4) 68
9 This content is available online at <http://cnx.org/content/m34879/1.2/>.
197

Exercise 3.455 (Solution on p. 209.)


(Section 3.4) 142
Exercise 3.456 (Solution on p. 209.)
(Section 3.4) 151
Exercise 3.457 (Solution on p. 209.)
(Section 3.4) 468
For problems 21 and 22, nd the greatest common factor.
Exercise 3.458 (Solution on p. 209.)
(Section 3.5) 200 and 36
Exercise 3.459 (Solution on p. 209.)
(Section 3.5) 900 and 135
Exercise 3.460 (Solution on p. 209.)
(Section 3.5) Write all the factors of 36.
Exercise 3.461 (Solution on p. 209.)
(Section 3.5) Write all the divisors of 18.
Exercise 3.462 (Solution on p. 209.)
(Section 3.5) Does 7 divide into 52 · 63 · 74 · 8? Explain.
Exercise 3.463 (Solution on p. 209.)
(Section 3.5) Is 3 a factor of 26 · 32 · 53 · 46 ? Explain.
Exercise 3.464 (Solution on p. 209.)
(Section 3.5) Does 13 divide into 113 · 124 · 152 ? Explain.
For problems 28 and 29, nd the least common multiple.
Exercise 3.465 (Solution on p. 209.)
(Section 3.6) 432 and 180
Exercise 3.466 (Solution on p. 209.)
(Section 3.6) 28, 40, and 95
198
CHAPTER 3. EXPONENTS, ROOTS, AND FACTORIZATION OF WHOLE
NUMBERS
Solutions to Exercises in Chapter 3
Solution to Exercise 3.1 (p. 153)
372
Solution to Exercise 3.2 (p. 153)
165
Solution to Exercise 3.3 (p. 153)
910
Solution to Exercise 3.4 (p. 153)
85 · 85 · 85
Solution to Exercise 3.5 (p. 153)
4·4·4·4·4·4·4
Solution to Exercise 3.6 (p. 153)
1, 739 · 1, 739
Solution to Exercise 3.7 (p. 155)
8
Solution to Exercise 3.8 (p. 155)
10
Solution to Exercise 3.9 (p. 155)
4
Solution to Exercise 3.10 (p. 155)
2
Solution to Exercise 3.11 (p. 156)
9
Solution to Exercise 3.12 (p. 156)
54
Solution to Exercise 3.13 (p. 156)
231
Solution to Exercise 3.14 (p. 156)
4
Solution to Exercise 3.15 (p. 156)
42
Solution to Exercise 3.17 (p. 156)
94
Solution to Exercise 3.19 (p. 156)
8263
Solution to Exercise 3.21 (p. 156)
685
Solution to Exercise 3.23 (p. 157)
13008
Solution to Exercise 3.25 (p. 157)
7·7·7·7
Solution to Exercise 3.27 (p. 157)
117 · 117 · 117 · 117 · 117
Solution to Exercise 3.29 (p. 157)
30 · 30
Solution to Exercise 3.31 (p. 157)
4 · 4 = 16
Solution to Exercise 3.33 (p. 157)
10 · 10 = 100
Solution to Exercise 3.35 (p. 157)
12 · 12 = 144
199

Solution to Exercise 3.37 (p. 157)


15 · 15 = 225
Solution to Exercise 3.39 (p. 157)
3 · 3 · 3 · 3 = 81
Solution to Exercise 3.41 (p. 157)
10 · 10 · 10 = 1, 000
Solution to Exercise 3.43 (p. 157)
8 · 8 · 8 = 512
Solution to Exercise 3.45 (p. 158)
9 · 9 · 9 = 729
Solution to Exercise 3.47 (p. 158)
71 = 7
Solution to Exercise 3.49 (p. 158)
2 · 2 · 2 · 2 · 2 · 2 · 2 = 128
Solution to Exercise 3.51 (p. 158)
8 · 8 · 8 · 8 = 4, 096
Solution to Exercise 3.53 (p. 158)
6 · 6 · 6 · 6 · 6 · 6 · 6 · 6 · 6 = 10, 077, 696
Solution to Exercise 3.55 (p. 158)
42 · 42 = 1, 764
Solution to Exercise 3.57 (p. 158)
15 · 15 · 15 · 15 · 15 = 759, 375
Solution to Exercise 3.59 (p. 158)
816 · 816 = 665, 856
Solution to Exercise 3.61 (p. 158)
4
Solution to Exercise 3.63 (p. 158)
8
Solution to Exercise 3.65 (p. 158)
12
Solution to Exercise 3.67 (p. 159)
15
Solution to Exercise 3.69 (p. 159)
2
Solution to Exercise 3.71 (p. 159)
6
Solution to Exercise 3.73 (p. 159)
20
Solution to Exercise 3.75 (p. 159)
100
Solution to Exercise 3.77 (p. 159)
60
Solution to Exercise 3.79 (p. 159)
34
Solution to Exercise 3.81 (p. 159)
4,158
Solution to Exercise 3.83 (p. 159)
24
Solution to Exercise 3.85 (p. 159)
4
200
CHAPTER 3. EXPONENTS, ROOTS, AND FACTORIZATION OF WHOLE
NUMBERS
Solution to Exercise 3.87 (p. 159)
5
Solution to Exercise 3.89 (p. 160)
81
Solution to Exercise 3.91 (p. 160)
8 is the multiplier; 4 is the multiplicand
Solution to Exercise 3.93 (p. 160)
Yes; 0
Solution to Exercise 3.95 (p. 161)
10
Solution to Exercise 3.96 (p. 161)
68
Solution to Exercise 3.97 (p. 161)
24
Solution to Exercise 3.98 (p. 161)
4
Solution to Exercise 3.99 (p. 161)
0
Solution to Exercise 3.100 (p. 161)
not possible (indeterminant)
Solution to Exercise 3.101 (p. 162)
54
Solution to Exercise 3.102 (p. 162)
23
Solution to Exercise 3.103 (p. 162)
4
Solution to Exercise 3.104 (p. 162)
93
Solution to Exercise 3.105 (p. 162)
74
Solution to Exercise 3.106 (p. 162)
27
Solution to Exercise 3.107 (p. 164)
33
Solution to Exercise 3.108 (p. 164)
57
Solution to Exercise 3.109 (p. 164)
0
Solution to Exercise 3.110 (p. 164)
52
Solution to Exercise 3.111 (p. 164)
3
Solution to Exercise 3.112 (p. 164)
125
Solution to Exercise 3.113 (p. 164)
7
Solution to Exercise 3.114 (p. 167)
13,499
Solution to Exercise 3.115 (p. 167)
6,070
201

Solution to Exercise 3.116 (p. 168)


1,107,495
Solution to Exercise 3.117 (p. 168)
This number is too big for a nonscientic calculator. A scientic calculator will probably give you
2.217747109 × 1011
Solution to Exercise 3.118 (p. 168)
26
Solution to Exercise 3.120 (p. 168)
46
Solution to Exercise 3.122 (p. 168)
1
Solution to Exercise 3.124 (p. 168)
0
Solution to Exercise 3.126 (p. 168)
3
Solution to Exercise 3.128 (p. 168)
26
Solution to Exercise 3.130 (p. 168)
97
Solution to Exercise 3.132 (p. 168)
29
Solution to Exercise 3.134 (p. 168)
1
Solution to Exercise 3.136 (p. 169)
0
Solution to Exercise 3.138 (p. 169)
90
Solution to Exercise 3.140 (p. 169)
508
Solution to Exercise 3.142 (p. 169)
19
Solution to Exercise 3.144 (p. 169)
144
Solution to Exercise 3.146 (p. 169)
1
Solution to Exercise 3.148 (p. 169)
52
Solution to Exercise 3.150 (p. 169)
25,001
Solution to Exercise 3.152 (p. 169)
1
25
Solution to Exercise 3.154 (p. 169)
14
Solution to Exercise 3.156 (p. 169)
0
Solution to Exercise 3.158 (p. 170)
152
Solution to Exercise 3.160 (p. 170)
4
5
Solution to Exercise 3.162 (p. 170)
2,690,730
202
CHAPTER 3. EXPONENTS, ROOTS, AND FACTORIZATION OF WHOLE
NUMBERS
Solution to Exercise 3.164 (p. 170)
1
Solution to Exercise 3.166 (p. 171)
1, 2, 3, 6
Solution to Exercise 3.167 (p. 171)
1, 2, 3, 4, 6, 12
Solution to Exercise 3.168 (p. 171)
1, 2, 3, 6, 9, 18
Solution to Exercise 3.169 (p. 171)
1, 5
Solution to Exercise 3.170 (p. 171)
1, 2, 5, 10
Solution to Exercise 3.171 (p. 171)
1, 3, 11, 33
Solution to Exercise 3.172 (p. 172)
1, 19
Solution to Exercise 3.173 (p. 172)
prime
Solution to Exercise 3.174 (p. 172)
composite
Solution to Exercise 3.175 (p. 172)
composite
Solution to Exercise 3.176 (p. 172)
composite
Solution to Exercise 3.177 (p. 172)
prime
Solution to Exercise 3.178 (p. 172)
prime
Solution to Exercise 3.179 (p. 173)
prime
Solution to Exercise 3.180 (p. 173)
composite
Solution to Exercise 3.181 (p. 175)
22 = 2 · 11
Solution to Exercise 3.182 (p. 175)
40 = 23 · 5
Solution to Exercise 3.183 (p. 175)
48 = 24 · 3
Solution to Exercise 3.184 (p. 175)
63 = 32 · 7
Solution to Exercise 3.185 (p. 175)
945 = 33 · 5 · 7
Solution to Exercise 3.186 (p. 175)
1617 = 3 · 72 · 11
Solution to Exercise 3.187 (p. 175)
17 is prime
Solution to Exercise 3.188 (p. 175)
61 is prime
Solution to Exercise 3.189 (p. 175)
2
203

Solution to Exercise 3.191 (p. 176)


4
Solution to Exercise 3.193 (p. 176)
11
Solution to Exercise 3.195 (p. 176)
3·2
Solution to Exercise 3.197 (p. 176)
2·3·5
Solution to Exercise 3.199 (p. 176)
1, 2, 4, 8, 16
Solution to Exercise 3.201 (p. 176)
1, 2, 4, 7, 8, 14, 28, 56
Solution to Exercise 3.203 (p. 176)
1, 2, 4, 5, 10, 11, 20, 22, 44, 55, 110, 220
Solution to Exercise 3.205 (p. 176)
1, 2, 4, 8, 16, 32
Solution to Exercise 3.207 (p. 176)
1, 2, 71, 142
Solution to Exercise 3.209 (p. 176)
prime
Solution to Exercise 3.211 (p. 177)
composite
Solution to Exercise 3.213 (p. 177)
prime
Solution to Exercise 3.215 (p. 177)
prime
Solution to Exercise 3.217 (p. 177)
prime
Solution to Exercise 3.219 (p. 177)
composite ( 5 · 11)
Solution to Exercise 3.221 (p. 177)
composite
Solution to Exercise 3.223 (p. 177)
composite
Solution to Exercise 3.225 (p. 177)
composite ( 11 · 19)
Solution to Exercise 3.227 (p. 177)
composite
Solution to Exercise 3.229 (p. 177)
2 · 13
Solution to Exercise 3.231 (p. 177)
2 · 33
Solution to Exercise 3.233 (p. 178)
23 · 7
Solution to Exercise 3.235 (p. 178)
25 · 3 · 5
Solution to Exercise 3.237 (p. 178)
34 · 52
Solution to Exercise 3.239 (p. 178)
26,580
204
CHAPTER 3. EXPONENTS, ROOTS, AND FACTORIZATION OF WHOLE
NUMBERS
Solution to Exercise 3.241 (p. 178)
true
Solution to Exercise 3.243 (p. 178)
14
Solution to Exercise 3.244 (p. 180)
12
Solution to Exercise 3.245 (p. 180)
24
Solution to Exercise 3.246 (p. 180)
10
Solution to Exercise 3.247 (p. 180)
3
Solution to Exercise 3.248 (p. 180)
30
Solution to Exercise 3.249 (p. 181)
2
Solution to Exercise 3.251 (p. 181)
4
Solution to Exercise 3.253 (p. 181)
4
Solution to Exercise 3.255 (p. 181)
5
Solution to Exercise 3.257 (p. 181)
33
Solution to Exercise 3.259 (p. 181)
9
Solution to Exercise 3.261 (p. 181)
11
Solution to Exercise 3.263 (p. 181)
3
Solution to Exercise 3.265 (p. 181)
7
Solution to Exercise 3.267 (p. 181)
25
Solution to Exercise 3.269 (p. 181)
2
Solution to Exercise 3.271 (p. 182)
11
Solution to Exercise 3.273 (p. 182)
1
Solution to Exercise 3.275 (p. 182)
1
Solution to Exercise 3.277 (p. 182)
53
Solution to Exercise 3.279 (p. 182)
86 = 262, 144
Solution to Exercise 3.281 (p. 183)
4, 8, 12, 16, 20
Solution to Exercise 3.282 (p. 183)
5, 10, 15, 20, 25
205

Solution to Exercise 3.283 (p. 183)


6, 12, 18, 24, 30
Solution to Exercise 3.284 (p. 183)
7, 14, 21, 28, 35
Solution to Exercise 3.285 (p. 183)
9, 18, 27, 36, 45
Solution to Exercise 3.286 (p. 184)
4, 8, 12, 16, 20
Solution to Exercise 3.287 (p. 184)
12, 24, 36, 48, 60
Solution to Exercise 3.288 (p. 184)
10, 20, 30, 40, 50
Solution to Exercise 3.289 (p. 184)
6, 12, 18, 24, 30
Solution to Exercise 3.290 (p. 184)
20, 40, 60, 80, 100
Solution to Exercise 3.291 (p. 186)
540
Solution to Exercise 3.292 (p. 186)
28
Solution to Exercise 3.293 (p. 186)
126
Solution to Exercise 3.294 (p. 186)
1,960
Solution to Exercise 3.295 (p. 186)
6,534,000
Solution to Exercise 3.296 (p. 186)
24
Solution to Exercise 3.298 (p. 186)
40
Solution to Exercise 3.300 (p. 186)
12
Solution to Exercise 3.302 (p. 186)
18
Solution to Exercise 3.304 (p. 186)
30
Solution to Exercise 3.306 (p. 186)
12
Solution to Exercise 3.308 (p. 186)
63
Solution to Exercise 3.310 (p. 187)
72
Solution to Exercise 3.312 (p. 187)
720
Solution to Exercise 3.314 (p. 187)
120
Solution to Exercise 3.316 (p. 187)
216
Solution to Exercise 3.318 (p. 187)
144
206
CHAPTER 3. EXPONENTS, ROOTS, AND FACTORIZATION OF WHOLE
NUMBERS
Solution to Exercise 3.320 (p. 187)
105
Solution to Exercise 3.322 (p. 187)
231
Solution to Exercise 3.324 (p. 187)
126
Solution to Exercise 3.326 (p. 187)
4,410
Solution to Exercise 3.328 (p. 187)
240
Solution to Exercise 3.330 (p. 187)
432
Solution to Exercise 3.332 (p. 187)
144
Solution to Exercise 3.334 (p. 188)
224
Solution to Exercise 3.336 (p. 188)
193,050
Solution to Exercise 3.338 (p. 188)
8
Solution to Exercise 3.340 (p. 188)
36
Solution to Exercise 3.342 (p. 188)
6,552
Solution to Exercise 3.344 (p. 188)
84 · 84 · 84
Solution to Exercise 3.346 (p. 191)
27
Solution to Exercise 3.348 (p. 191)
0
Solution to Exercise 3.350 (p. 191)
144
Solution to Exercise 3.352 (p. 191)
64
Solution to Exercise 3.354 (p. 191)
32
Solution to Exercise 3.356 (p. 191)
225
Solution to Exercise 3.358 (p. 191)
625
Solution to Exercise 3.360 (p. 192)
15
Solution to Exercise 3.362 (p. 192)
2
Solution to Exercise 3.364 (p. 192)
1
Solution to Exercise 3.366 (p. 192)
12
Solution to Exercise 3.368 (p. 192)
1
207

Solution to Exercise 3.370 (p. 192)


2
Solution to Exercise 3.372 (p. 192)
0
Solution to Exercise 3.374 (p. 192)
325
Solution to Exercise 3.376 (p. 192)
64
Solution to Exercise 3.378 (p. 192)
-3
Solution to Exercise 3.380 (p. 193)
9
− 57
Solution to Exercise 3.382 (p. 193)
146
Solution to Exercise 3.384 (p. 193)
5
Solution to Exercise 3.386 (p. 193)
The sum of square roots is not necessarily equal to the square root of the sum.
Solution to Exercise 3.388 (p. 193)
6
Solution to Exercise 3.390 (p. 193)
8
Solution to Exercise 3.392 (p. 193)
1, 2, 3, 6, 9, 18
Solution to Exercise 3.394 (p. 193)
1, 11
Solution to Exercise 3.396 (p. 193)
1, 3, 17, 51,
Solution to Exercise 3.398 (p. 193)
1, 2
Solution to Exercise 3.400 (p. 194)
5 · 11
Solution to Exercise 3.402 (p. 194)
24 · 5
Solution to Exercise 3.404 (p. 194)
22 · 52 · 7
Solution to Exercise 3.406 (p. 194)
2 · 3 · 269
Solution to Exercise 3.408 (p. 194)
29 is a prime number
Solution to Exercise 3.410 (p. 194)
5
Solution to Exercise 3.412 (p. 194)
5
Solution to Exercise 3.414 (p. 194)
6
Solution to Exercise 3.416 (p. 194)
20
Solution to Exercise 3.418 (p. 195)
21
208
CHAPTER 3. EXPONENTS, ROOTS, AND FACTORIZATION OF WHOLE
NUMBERS
Solution to Exercise 3.420 (p. 195)
11
Solution to Exercise 3.422 (p. 195)
42
Solution to Exercise 3.424 (p. 195)
180
Solution to Exercise 3.426 (p. 195)
120
Solution to Exercise 3.428 (p. 195)
79, 380
Solution to Exercise 3.430 (p. 195)
450
Solution to Exercise 3.432 (p. 195)
1, 2, 3, 4, 6, 8, 12, 24
Solution to Exercise 3.434 (p. 195)
1, 2, 4, 5, 7, 8, 10, 14, 20, 25, 35, 40, 50, 56, 70, 100, 140, 175, 200, 280, 700, 1,400
Solution to Exercise 3.436 (p. 195)
yes
Solution to Exercise 3.438 (p. 196)
base; exponent
Solution to Exercise 3.439 (p. 196)
127
Solution to Exercise 3.440 (p. 196)
94 = 9 · 9 · 9 · 9 = 6, 561
Solution to Exercise 3.441 (p. 196)
64
Solution to Exercise 3.442 (p. 196)
1
Solution to Exercise 3.443 (p. 196)
0
Solution to Exercise 3.444 (p. 196)
64
Solution to Exercise 3.445 (p. 196)
7
Solution to Exercise 3.446 (p. 196)
3
Solution to Exercise 3.447 (p. 196)
1
Solution to Exercise 3.448 (p. 196)
20
Solution to Exercise 3.449 (p. 196)
127
Solution to Exercise 3.450 (p. 196)
24
Solution to Exercise 3.451 (p. 196)
8
Solution to Exercise 3.452 (p. 196)
5
Solution to Exercise 3.453 (p. 196)
32 · 2
209

Solution to Exercise 3.454 (p. 196)


22 · 17
Solution to Exercise 3.455 (p. 197)
2 · 71
Solution to Exercise 3.456 (p. 197)
prime
Solution to Exercise 3.457 (p. 197)
22 · 32 · 13
Solution to Exercise 3.458 (p. 197)
4
Solution to Exercise 3.459 (p. 197)
45
Solution to Exercise 3.460 (p. 197)
1, 2, 3, 4, 6, 9, 12, 18, 36
Solution to Exercise 3.461 (p. 197)
1, 2, 3, 6, 9, 18
Solution to Exercise 3.462 (p. 197)
Yes, because one of the (prime) factors of the number is 7.
Solution to Exercise 3.463 (p. 197)
Yes, because it is one of the factors of the number.
Solution to Exercise 3.464 (p. 197)
No, because the prime 13 is not a factor any of the listed factors of the number.
Solution to Exercise 3.465 (p. 197)
2,160
Solution to Exercise 3.466 (p. 197)
5,320
210
CHAPTER 3. EXPONENTS, ROOTS, AND FACTORIZATION OF WHOLE
NUMBERS
Chapter 4

Introduction to Fractions and

Multiplication and Division of Fractions

4.1 Objectives 1

After completing this chapter, you should


Fractions of Whole Numbers (Section 4.2)

• understand the concept of fractions of whole numbers


• be able to recognize the parts of a fraction

Proper Fractions, improper Fractions, and Mixed Numbers (Section 4.3)

• be able to distinguish between proper fractions, improper fractions, and mixed numbers
• be able to convert an improper fraction to a mixed number
• be able to convert a mixed number to an improper fraction

Equivalent Fractions, Reducing Fractions to Lowest Terms, and Raising Fractions to Higher
Terms (Section 4.4)

• be able to recognize equivalent fractions


• be able to reduce a fraction to lowest terms
• be able to raise a fraction to higher terms

Multiplication of Fractions (Section 4.5)

• understand the concept of multiplication of fractions


• be able to multiply one fraction by another
• be able to multiply mixed numbers
• be able to nd powers and roots of various fractions

Division of Fractions (Section 4.6)

• be able to determine the reciprocal of a number


• be able to divide one fraction by another

Applications Involving Fractions (Section 4.7)

• be able to solve missing product statements


• be able to solve missing factor statements
1 This content is available online at <http://cnx.org/content/m18892/1.3/>.

211
212
CHAPTER 4. INTRODUCTION TO FRACTIONS AND MULTIPLICATION
AND DIVISION OF FRACTIONS
4.2 Fractions of Whole Numbers 2
4.2.1 Section Overview
• More Numbers on the Number Line
• Fractions of Whole Numbers
• The Parts of a Fraction
• Reading and Writing Fractions

4.2.2 More Numbers on the Number Line


In Chapters Section 1.1, Section 2.1, and Section 3.1, we studied the whole numbers and methods of combining
them. We noted that we could visually display the whole numbers by drawing a number line and placing
closed circles at whole number locations.

By observing this number line, we can see that the whole numbers do not account for every point on the
line. What numbers, if any, can be associated with these points? In this section we will see that many of the
points on the number line, including the points already associated with whole numbers, can be associated
with numbers called fractions.

4.2.3 Fractions of Whole Numbers


The Nature of the Positive Fractions
We can extend our collection of numbers, which now contains only the whole numbers, by including fractions
of whole numbers. We can determine the nature of these fractions using the number line.

If we place a pencil at some whole number and proceed to travel to the right to the next whole number, we
see that our journey can be broken into dierent types of equal parts as shown in the following examples.

a. 1 part.

b. 2 equal parts.

c. 3 equal parts.

2 This content is available online at <http://cnx.org/content/m34908/1.2/>.


213

d. 4 equal parts.

The Latin Word Fractio


Notice that the number of parts, 2, 3, and 4, that we are breaking the original quantity into is always a
nonzero whole number. The idea of breaking up a whole quantity gives us the word fraction. The word
fraction comes from the Latin word "fractio" which means a breaking, or fracture.

Suppose we break up the interval from some whole number to the next whole number into ve equal parts.

After starting to move from one whole number to the next, we decide to stop after covering only two parts.
We have covered 2 parts of 5 equal parts. This situation is described by writing 25 .

Positive Fraction
A number such as 2
5 is called a positive fraction, or more simply, a fraction.

4.2.4 The Parts of a Fraction


A fraction has three parts.

1. The fraction bar .Fraction Bar


The fraction bar serves as a grouping symbol. It separates a quantity into individual groups. These
groups have names, as noted in 2 and 3 below.
2. The nonzero number below the fraction bar.Denominator
This number is called the denominator of the fraction, and it indicates the number of parts the
whole quantity has been divided into. Notice that the denominator must be a nonzero whole number
since the least number of parts any quantity can have is one.
3. The number above the fraction bar. Numerator
This number is called the numerator of the fraction, and it indicates how many of the specied parts
are being considered. Notice that the numerator can be any whole number (including zero) since any
number of the specied parts can be considered.
whole number numerator
nonzero whole number ↔ denominator

4.2.4.1 Sample Set A

The diagrams in the following problems are illustrations of fractions.


Example 4.1
214
CHAPTER 4. INTRODUCTION TO FRACTIONS AND MULTIPLICATION
AND DIVISION OF FRACTIONS

(a) (b) (c)

Diagrams 4.1: (a) A whole circle (b) The whole circle divided into 3 equal parts (c) 1 of the 3 equal
parts

The fraction 1
3 is read as "one third."
Example 4.2

(a) (b) (c)

Figure 4.2: (a) A whole rectangle (b) The whole rectangle divided into 5 equal parts (c) 3 of the 5
equal parts

The fraction 3
5 "is read as "three fths."
Example 4.3
215

(a)

(b)

(c)

Figure 4.3: (a) The number line between 0 and 1 (b) The number line between 0 and 1 divided into
7 equal parts (c) 6 of the 7 equal parts

The fraction 6
7 is read as "six sevenths."
Example 4.4

(a) (b) (c)

Figure 4.4: (a) A whole circle (b) The whole circle divided into 4 equal parts (c) 4 of the 4 equal parts

When the numerator and denominator are equal, the fraction represents the entire quantity, and
its value is 1.
nonzero whole number
same nonzero whole number = 1

4.2.4.2 Practice Set A

Specify the numerator and denominator of the following fractions.


216
CHAPTER 4. INTRODUCTION TO FRACTIONS AND MULTIPLICATION
AND DIVISION OF FRACTIONS
Exercise 4.1 (Solution on p. 276.)
4
7
Exercise 4.2 (Solution on p. 276.)
5
8
Exercise 4.3 (Solution on p. 276.)
10
15
Exercise 4.4 (Solution on p. 276.)
1
9
Exercise 4.5 (Solution on p. 276.)
0
2

4.2.5 Reading and Writing Fractions


In order to properly translate fractions from word form to number form, or from number form to word form,
it is necessary to understand the use of the hyphen.

Use of the Hyphen


One of the main uses of the hyphen is to tell the reader that two words not ordinarily joined are to be taken
in combination as a unit. Hyphens are always used for numbers between and including 21 and 99 (except
those ending in zero).

4.2.5.1 Sample Set B

Write each fraction using whole numbers.


Example 4.5
Fifty three-hundredths. The hyphen joins the words three and hundredths and tells us to
consider them as a single unit. Therefore,
50
fty three-hundredths translates as 300
Example 4.6
Fifty-three hundredths. The hyphen joins the numbers fty and three and tells us to consider
them as a single unit. Therefore,
53
fty-three hundredths translates as 100
Example 4.7
Four hundred seven-thousandths. The hyphen joins the words seven and thousandths and
tells us to consider them as a single unit. Therefore,
400
four hundred seven-thousandths translates as 7,000
Example 4.8
Four hundred seven thousandths. The absence of hyphens indicates that the words seven and
thousandths are to be considered individually.
407
four hundred seven thousandths translates as 1000
Write each fraction using words.
Example 4.9
21
85 translates as twenty-one eighty-fths.
Example 4.10
200
3,000 translates as two hundred three-thousandths. A hyphen is needed between the words
three and thousandths to tell the reader that these words are to be considered as a single unit.
217

Example 4.11
203
1,000 translates as two hundred three thousandths.

4.2.5.2 Practice Set B

Write the following fractions using whole numbers.


Exercise 4.6 (Solution on p. 276.)
one tenth
Exercise 4.7 (Solution on p. 276.)
eleven fourteenths
Exercise 4.8 (Solution on p. 276.)
sixteen thirty-fths
Exercise 4.9 (Solution on p. 276.)
eight hundred seven-thousandths
Write the following using words.
Exercise 4.10 (Solution on p. 276.)
3
8
Exercise 4.11 (Solution on p. 276.)
1
10
Exercise 4.12 (Solution on p. 276.)
3
250
Exercise 4.13 (Solution on p. 276.)
114
3,190
Name the fraction that describes each shaded portion.
Exercise 4.14 (Solution on p. 276.)

Exercise 4.15 (Solution on p. 276.)

In the following 2 problems, state the numerator and denominator, and write each fraction in words.
Exercise 4.16 (Solution on p. 276.)
The number 5
9 is used in converting from Fahrenheit to Celsius.
Exercise 4.17 (Solution on p. 276.)
A dime is 10
1
of a dollar.
218
CHAPTER 4. INTRODUCTION TO FRACTIONS AND MULTIPLICATION
AND DIVISION OF FRACTIONS
4.2.6 Exercises
For the following 10 problems, specify the numerator and denominator in each fraction.
Exercise 4.18 (Solution on p. 276.)
3
4
Exercise 4.19
9
10
Exercise 4.20 (Solution on p. 276.)
1
5
Exercise 4.21
5
6
Exercise 4.22 (Solution on p. 276.)
7
7
Exercise 4.23
4
6
Exercise 4.24 (Solution on p. 276.)
0
12
Exercise 4.25
25
25
Exercise 4.26 (Solution on p. 276.)
18
1
Exercise 4.27
0
16
For the following 10 problems, write the fractions using whole numbers.
Exercise 4.28 (Solution on p. 276.)
four fths
Exercise 4.29
two ninths
Exercise 4.30 (Solution on p. 276.)
fteen twentieths
Exercise 4.31
forty-seven eighty-thirds
Exercise 4.32 (Solution on p. 276.)
ninety-one one hundred sevenths
Exercise 4.33
twenty-two four hundred elevenths
Exercise 4.34 (Solution on p. 277.)
six hundred ve eight hundred thirty-fourths
Exercise 4.35
three thousand three forty-four ten-thousandths
Exercise 4.36 (Solution on p. 277.)
ninety-two one-millionths
Exercise 4.37
one three-billionths
For the following 10 problems, write the fractions using words.
219

Exercise 4.38 (Solution on p. 277.)


5
9
Exercise 4.39
6
10
Exercise 4.40 (Solution on p. 277.)
8
15
Exercise 4.41
10
13
Exercise 4.42 (Solution on p. 277.)
75
100
Exercise 4.43
86
135
Exercise 4.44 (Solution on p. 277.)
916
1,014
Exercise 4.45
501
10,001
Exercise 4.46 (Solution on p. 277.)
18
31,608
Exercise 4.47
1
500,000
For the following 4 problems, name the fraction corresponding to the shaded portion.
Exercise 4.48 (Solution on p. 277.)

Exercise 4.49

Exercise 4.50 (Solution on p. 277.)

Exercise 4.51

For the following 4 problems, shade the portion corresponding to the given fraction on the given gure.
220
CHAPTER 4. INTRODUCTION TO FRACTIONS AND MULTIPLICATION
AND DIVISION OF FRACTIONS
Exercise 4.52 (Solution on p. 277.)
3
5

Exercise 4.53
1
8

Exercise 4.54 (Solution on p. 277.)


6
6

Exercise 4.55
0
3

State the numerator and denominator and write in words each of the fractions appearing in the statements
for the following 10 problems.
Exercise 4.56 (Solution on p. 277.)
A contractor is selling houses on 1
4 acre lots.
Exercise 4.57
The fraction 22
7 is sometimes used as an approximation to the number π . (The symbol is read
pi.")
Exercise 4.58 (Solution on p. 277.)
The fraction 4
3 is used in nding the volume of a sphere.
Exercise 4.59
One inch is 12
1
of a foot.
Exercise 4.60 (Solution on p. 277.)
About 2
7 of the students in a college statistics class received a B in the course.
Exercise 4.61
The probability of randomly selecting a club when drawing one card from a standard deck of 52
cards is 13
52 .
221

Exercise 4.62 (Solution on p. 277.)


In a box that contains eight computer chips, ve are known to be good and three are known to be
defective. If three chips are selected at random, the probability that all three are defective is 56
1
.
Exercise 4.63
In a room of 25 people, the probability that at least two people have the same birthdate (date and
569 .
month, not year) is 1000
Exercise 4.64 (Solution on p. 277.)
The mean (average) of the numbers 21, 25, 43, and 36 is 125
4 .
Exercise 4.65
32 meters high after
If a rock falls from a height of 20 meters on Jupiter, the rock will be 25 6
seconds.
5

4.2.6.1 Exercises For Review

Exercise 4.66 (Solution on p. 277.)


(Section 1.7) Use the numbers 3 and 11 to illustrate the commutative property of addition.
Exercise 4.67
(Section 2.4) Find the quotient. 676 ÷ 26
Exercise 4.68 (Solution on p. 277.)
(Section 3.2) Write 7 · 7 · 7 · 7 · 7 using exponents.
Exercise 4.69
8·(6+20) 3·(6+16)
(Section 3.3) Find the value of 8 + 22 .
Exercise 4.70 (Solution on p. 277.)
(Section 3.6) Find the least common multiple of 12, 16, and 18.

4.3 Proper Fractions, Improper Fractions, and Mixed Numbers3


4.3.1 Section Overview
• Positive Proper Fractions
• Positive Improper Fractions
• Positive Mixed Numbers
• Relating Positive Improper Fractions and Positive Mixed Numbers
• Converting an Improper Fraction to a Mixed Number
• Converting a Mixed Number to an Improper Fraction

Now that we know what positive fractions are, we consider three types of positive fractions: proper fractions,
improper fractions, and mixed numbers.

4.3.2 Positive Proper Fractions


Positive Proper Fraction
Fractions in which the whole number in the numerator is strictly less than the whole number in the de-
nominator are called positive proper fractions. On the number line, proper fractions are located in the
interval from 0 to 1. Positive proper fractions are always less than one.
3 This content is available online at <http://cnx.org/content/m34912/1.2/>.
222
CHAPTER 4. INTRODUCTION TO FRACTIONS AND MULTIPLICATION
AND DIVISION OF FRACTIONS

The closed circle at 0 indicates that 0 is included, while the open circle at 1 indicates that 1 is not included.

Some examples of positive proper fractions are


20 , and 106
2, 5,
1 3
27 255
Note that 1 < 2, 3 < 5, 20 < 27, and 106 < 225.

4.3.3 Positive Improper Fractions


Positive Improper Fractions
Fractions in which the whole number in the numerator is greater than or equal to the whole number in the
denominator are called positive improper fractions. On the number line, improper fractions lie to the
right of (and including) 1. Positive improper fractions are always greater than or equal to 1.

Some examples of positive improper fractions are

2, 5, 4,
3 8 4
and 105
16
Note that 3 ≥ 2, 8 ≥ 5, 4 ≥ 4, and 105 ≥ 16.

4.3.4 Positive Mixed Numbers


Positive Mixed Numbers
A number of the form

nonzero whole number + proper fraction

is called a positive mixed number. For example, 2 35 is a mixed number. On the number line, mixed
numbers are located in the interval to the right of (and including) 1. Mixed numbers are always greater than
or equal to 1.

4.3.5 Relating Positive Improper Fractions and Positive Mixed Numbers


A relationship between improper fractions and mixed numbers is suggested by two facts. The rst is that
improper fractions and mixed numbers are located in the same interval on the number line. The second fact,
that mixed numbers are the sum of a natural number and a fraction, can be seen by making the following
observations.

Divide a whole quantity into 3 equal parts.


223

Now, consider the following examples by observing the respective shaded areas.

In the shaded region, there are 2 one thirds, or 32 .

2 13 = 32


There are 3 one thirds, or 33 , or 1.

3 13 = 33 or 1


Thus,
3
3 =1

Improper fraction = whole number.

There are 4 one thirds, or 34 , or 1 and 13 .

4 31 = 34 or 1 and 13


The terms 1 and 1


3 can be represented as 1 + 1
3 or 1 31

Thus,
4
3 = 1 13 .

Improper fraction = mixed number.

There are 5 one thirds, or 35 , or 1 and 23 .

5 31 = 35 or 1 and 23


The terms 1 and 2


3 can be represented as 1 + 2
3 or 1 32 .

Thus,
5
3 = 1 23 .

Improper fraction = mixed number.

There are 6 one thirds, or 36 , or 2.

6 13 = 36 = 2

224
CHAPTER 4. INTRODUCTION TO FRACTIONS AND MULTIPLICATION
AND DIVISION OF FRACTIONS
Thus,
6
3 =2

Improper fraction = whole number.

The following important fact is illustrated in the preceding examples.

Mixed Number = Natural Number + Proper Fraction


Mixed numbers are the sum of a natural number and a proper fraction. Mixed number = (natural number)
+ (proper fraction)

For example 1 13 can be expressed as 1 + 1


3 The fraction 5 87 can be expressed as 5 + 78 .

It is important to note that a number such as 5 + 78 does not indicate multiplication. To indicate multipli-
cation, we would need to use a multiplication symbol (such as ·)

note: 5 78 means 5 + 7
8 and not 5 · 78 , which means 5 times 7
8 or 5 multiplied by 78 .

Thus, mixed numbers may be represented by improper fractions, and improper fractions may be represented
by mixed numbers.

4.3.6 Converting Improper Fractions to Mixed Numbers


To understand how we might convert an improper fraction to a mixed number, let's consider the fraction, 34 .

4 1 1 1 1
3 = + + + 3
|3 {z
3 3}
1
1
= 1+ 3
= 1 13

Thus, 4
3 = 1 13 .

We can illustrate a procedure for converting an improper fraction to a mixed number using this example.
However, the conversion is more easily accomplished by dividing the numerator by the denominator and
using the result to write the mixed number.

Converting an Improper Fraction to a Mixed Number


To convert an improper fraction to a mixed number, divide the numerator by the denominator.

1. The whole number part of the mixed number is the quotient.


2. The fractional part of the mixed number is the remainder written over the divisor (the denominator
of the improper fraction).
225

4.3.6.1 Sample Set A

Convert each improper fraction to its corresponding mixed number.


Example 4.12
5
3 Divide 5 by 3.

The improper fraction 5


3 = 1 23 .

Example 4.13
46 . Divide 46 by 9.
9

The improper fraction 46


9 = 59.
1

Example 4.14
83 . Divide 83 by 11.
11

The improper fraction 83


11 = 7 11 .
6

Example 4.15
104 Divide 104 by 4.
4
226
CHAPTER 4. INTRODUCTION TO FRACTIONS AND MULTIPLICATION
AND DIVISION OF FRACTIONS

104 = 26 0 = 26
4 4

The improper fraction 104


4 = 26.

4.3.6.2 Practice Set A

Convert each improper fraction to its corresponding mixed number.


Exercise 4.71 (Solution on p. 277.)
9
2
Exercise 4.72 (Solution on p. 277.)
11
3
Exercise 4.73 (Solution on p. 277.)
14
11
Exercise 4.74 (Solution on p. 277.)
31
13
Exercise 4.75 (Solution on p. 278.)
79
4
Exercise 4.76 (Solution on p. 278.)
496
8

4.3.7 Converting Mixed Numbers to Improper Fractions


To understand how to convert a mixed number to an improper fraction, we'll recall

mixed number = (natural number) + (proper fraction)

and consider the following diagram.


227

Recall that multiplication describes repeated addition.

Notice that 5
3 can be obtained from 1 32 using multiplication in the following way.

Multiply: 3 · 1 = 3

Add: 3 + 2 = 5. Place the 5 over the 3: 5


3

The procedure for converting a mixed number to an improper fraction is illustrated in this example.

Converting a Mixed Number to an Improper Fraction


To convert a mixed number to an improper fraction,

1. Multiply the denominator of the fractional part of the mixed number by the whole number part.
2. To this product, add the numerator of the fractional part.
3. Place this result over the denominator of the fractional part.

4.3.7.1 Sample Set B

Convert each mixed number to an improper fraction.


Example 4.16
5 78

1. Multiply: 8 · 5 = 40.
2. Add: 40 + 7 = 47.
3. Place 47 over 8: 47
8 .

Thus, 5 78 = 47
8 .

Example 4.17
16 23

1. Multiply: 3 · 16 = 48.
2. Add: 48 + 2 = 50.
3. Place 50 over 3: 50
3

Thus, 16 23 = 50
3
228
CHAPTER 4. INTRODUCTION TO FRACTIONS AND MULTIPLICATION
AND DIVISION OF FRACTIONS
4.3.7.2 Practice Set B

Convert each mixed number to its corresponding improper fraction.


Exercise 4.77 (Solution on p. 278.)
8 14
Exercise 4.78 (Solution on p. 278.)
5 35
Exercise 4.79 (Solution on p. 278.)
4
1 15
Exercise 4.80 (Solution on p. 278.)
12 27

4.3.8 Exercises
For the following 15 problems, identify each expression as a proper fraction, an improper fraction, or a mixed
number.
Exercise 4.81 (Solution on p. 278.)
3
2
Exercise 4.82
4
9
Exercise 4.83 (Solution on p. 278.)
5
7
Exercise 4.84
1
8
Exercise 4.85 (Solution on p. 278.)
6 14
Exercise 4.86
11
8
Exercise 4.87 (Solution on p. 278.)
1,001
12
Exercise 4.88
191 54
Exercise 4.89 (Solution on p. 278.)
9
1 13
Exercise 4.90
31 76
Exercise 4.91 (Solution on p. 278.)
1
3 40
Exercise 4.92
55
12
Exercise 4.93 (Solution on p. 278.)
0
9
Exercise 4.94
8
9
Exercise 4.95 (Solution on p. 278.)
101 11
1
229

For the following 15 problems, convert each of the improper fractions to its corresponding mixed number.
Exercise 4.96
11
6
Exercise 4.97 (Solution on p. 278.)
14
3
Exercise 4.98
25
4
Exercise 4.99 (Solution on p. 278.)
35
4
Exercise 4.100
71
8
Exercise 4.101 (Solution on p. 278.)
63
7
Exercise 4.102
121
11
Exercise 4.103 (Solution on p. 278.)
165
12
Exercise 4.104
346
15
Exercise 4.105 (Solution on p. 278.)
5,000
9
Exercise 4.106
23
5
Exercise 4.107 (Solution on p. 278.)
73
2
Exercise 4.108
19
2
Exercise 4.109 (Solution on p. 278.)
316
41
Exercise 4.110
800
3
For the following 15 problems, convert each of the mixed numbers to its corresponding improper fraction.
Exercise 4.111 (Solution on p. 278.)
4 18
Exercise 4.112
5
1 12
Exercise 4.113 (Solution on p. 278.)
6 79
Exercise 4.114
15 41
Exercise 4.115 (Solution on p. 278.)
10 11
5

Exercise 4.116
15 10
3

Exercise 4.117 (Solution on p. 278.)


8 23
230
CHAPTER 4. INTRODUCTION TO FRACTIONS AND MULTIPLICATION
AND DIVISION OF FRACTIONS
Exercise 4.118
4 43
Exercise 4.119 (Solution on p. 279.)
21 25
Exercise 4.120
17 10
9

Exercise 4.121 (Solution on p. 279.)


9 20
21
Exercise 4.122
1
5 16
Exercise 4.123 (Solution on p. 279.)
90 100
1

Exercise 4.124
300 1,43
000
Exercise 4.125 (Solution on p. 279.)
19 78
Exercise 4.126
Why does 0 47 not qualify as a mixed number?

Hint: See the denition of a mixed number.

Exercise 4.127 (Solution on p. 279.)


Why does 5 qualify as a mixed number?

note: See the denition of a mixed number.

Calculator Problems
For the following 8 problems, use a calculator to convert each mixed number to its corresponding improper
fraction.
Exercise 4.128
11
35 12
Exercise 4.129 (Solution on p. 279.)
27 61
5

Exercise 4.130
40
83 41
Exercise 4.131 (Solution on p. 279.)
105 21
23
Exercise 4.132
605
72 606
Exercise 4.133 (Solution on p. 279.)
816 19
25
Exercise 4.134
42
708 51
Exercise 4.135 (Solution on p. 279.)
6, 012 4,216
8,117
231

4.3.8.1 Exercises For Review

Exercise 4.136
(Section 1.4) Round 2,614,000 to the nearest thousand.
Exercise 4.137 (Solution on p. 279.)
(Section 2.2) Find the product. 1,004 · 1,005.
Exercise 4.138
(Section 2.5) Determine if 41,826 is divisible by 2 and 3.
Exercise 4.139 (Solution on p. 279.)
(Section 3.6) Find the least common multiple of 28 and 36.
Exercise 4.140
(Section 4.2) Specify the numerator and denominator of the fraction 12
19 .

4.4 Equivalent Fractions, Reducing Fractions to Lowest Terms, and


Raising Fractions to Higher Terms 4
4.4.1 Section Overview
• Equivalent Fractions
• Reducing Fractions to Lowest Terms
• Raising Fractions to Higher Terms

4.4.2 Equivalent Fractions


Let's examine the following two diagrams.

Notice that both 2


3 and 4
6 represent the same part of the whole, that is, they represent the same number.

Equivalent Fractions
Fractions that have the same value are called equivalent fractions. Equivalent fractions may look dierent,
but they are still the same point on the number line.

There is an interesting property that equivalent fractions satisfy.

A Test for Equivalent Fractions Using the Cross Product


These pairs of products are called cross products.

4 This content is available online at <http://cnx.org/content/m34927/1.3/>.


232
CHAPTER 4. INTRODUCTION TO FRACTIONS AND MULTIPLICATION
AND DIVISION OF FRACTIONS
If the cross products are equal, the fractions are equivalent. If the cross products are not equal, the fractions
are not equivalent.

Thus, 2
3 and 4
6 are equivalent, that is, 2
3 = 46 .

4.4.2.1 Sample Set A

Determine if the following pairs of fractions are equivalent.


Example 4.18
4 and 8 . Test for equality of the cross products.
3 6

The cross products are equals.

The fractions 3
4 and 6
8 are equivalent, so 3
4 = 68 .
Example 4.19
3
8 and 16
9
. Test for equality of the cross products.

The cross products are not equal.

The fractions 3
8 and 16
9
are not equivalent.

4.4.2.2 Practice Set A

Determine if the pairs of fractions are equivalent.


Exercise 4.141 (Solution on p. 279.)
2, 6
1 3

Exercise 4.142 (Solution on p. 279.)


12
5,
4
15
Exercise 4.143 (Solution on p. 279.)
3 , 15
2 8

Exercise 4.144 (Solution on p. 279.)


8 , 40
1 5

Exercise 4.145 (Solution on p. 279.)


12 ,
3 1
4

4.4.3 Reducing Fractions to Lowest Terms


It is often very useful to conver t one fraction to an equivalent fraction that has reduced values in the
numerator and denominator. We can suggest a method for doing so by considering the equivalent fractions
15 and 5 . First, divide both the numerator and denominator of 15 by 3. The fractions 15 and 5 are
9 3 9 9 3

equivalent.
233

(Can you prove this?) So, 15


9
= 53 . We wish to convert 15
9
to 53 . Now divide the numerator and denominator
of 15 by 3, and see what happens.
9

9÷3 3
15÷3 = 5

The fraction 15
9
is converted to 35 .

A natural question is "Why did we choose to divide by 3?" Notice that


9 3·3
15 = 5·3

We can see that the factor 3 is common to both the numerator and denominator.

Reducing a Fraction
From these observations we can suggest the following method for converting one fraction to an equivalent
fraction that has reduced values in the numerator and denominator. The method is called reducing a
fraction.

A fraction can be reduced by dividing both the numerator and denominator by the same nonzero whole
number.

Consider the collection of equivalent fractions

20 , 16 , 12 , 8, 4
5 4 3 2 1

Reduced to Lowest Terms


Notice that each of the rst four fractions can be reduced to the last fraction, 41 , by dividing both the
numerator and denominator by, respectively, 5, 4, 3, and 2. When a fraction is converted to the fraction
that has the smallest numerator and denominator in its collection of equivalent fractions, it is said to be
reduced to lowest terms. The fractions 41 , 38 , 25 , and 10
7
are all reduced to lowest terms.

Observe a very important property of a fraction that has been reduced to lowest terms. The only whole
number that divides both the numerator and denominator without a remainder is the number 1. When 1 is
the only whole number that divides two whole numbers, the two whole numbers are said to be relatively
prime.

Relatively Prime
A fraction is reduced to lowest terms if its numerator and denominator are relatively prime.

4.4.3.1 Methods of Reducing Fractions to Lowest Terms

Method 1: Dividing Out Common Primes

1. Write the numerator and denominator as a product of primes.


2. Divide the numerator and denominator by each of the common prime factors. We often indicate
this division by drawing a slanted line through each divided out factor. This process is also called
cancelling common factors.
3. The product of the remaining factors in the numerator and the product of remaining factors of the
denominator are relatively prime, and this fraction is reduced to lowest terms.
234
CHAPTER 4. INTRODUCTION TO FRACTIONS AND MULTIPLICATION
AND DIVISION OF FRACTIONS
4.4.3.1.1 Sample Set B

Reduce each fraction to lowest terms.


Example 4.20
1 1
)2·)3
6
18 = = 1
3 1 and 3 are relatively prime.
)2·)3·3
1 1

Example 4.21
1 1
16 = )2·)2·2·2
= 4
4 and 5 are relatively prime.
20 )2·)2·5 5
1 1

Example 4.22
1 1 1
56 )2·)2·)2·7 7
7 and 13 are relatively prime (and also truly prime)
104 = )2·)2·)2·13
= 13
1 1 1

Example 4.23
1 1
315 = )3·3·5·)7 15 15 and 16 are relatively prime.
= 16
336 2·2·2·2·)3·)7·
1 1

Example 4.24
8
15 =
2·2·2
3·5 No common prime factors, so 8 and 15 are relatively prime.

The fraction 15
8
is reduced to lowest terms.

4.4.3.1.2 Practice Set B

Reduce each fraction to lowest terms.


Exercise 4.146 (Solution on p. 279.)
4
8
Exercise 4.147 (Solution on p. 279.)
6
15
Exercise 4.148 (Solution on p. 279.)
6
48
Exercise 4.149 (Solution on p. 279.)
21
48
Exercise 4.150 (Solution on p. 279.)
72
42
Exercise 4.151 (Solution on p. 279.)
135
243

Method 2: Dividing Out Common Factors

1. Mentally divide the numerator and the denominator by a factor that is common to each. Write the
quotient above the original number.
2. Continue this process until the numerator and denominator are relatively prime.
235

4.4.3.1.3 Sample Set C

Reduce each fraction to lowest terms.


Example 4.25
25 . 5 divides into both 25 and 30.
30
5
)25
= 5
6 5 and 6 are relatively prime.
)30
6

Example 4.26
18 . Both numbers are even so we can divide by 2.
24
9
)18
Now, both 9 and 12 are divisible by 3.
)24
12

3
)9
)18
= 3
4 3 and 4 are relatively prime.
)24
)12
4

Example 4.27
7
)21
)210
= 75 . 7 and 5 are relatively prime.
)150
)15
5

Example 4.28
36 = 18 = 9 =
8. 3 and 8 are relatively prime.
3
96 48 24

4.4.3.1.4 Practice Set C

Reduce each fraction to lowest terms.


Exercise 4.152 (Solution on p. 279.)
12
16
Exercise 4.153 (Solution on p. 279.)
9
24
Exercise 4.154 (Solution on p. 279.)
21
84
Exercise 4.155 (Solution on p. 280.)
48
64
Exercise 4.156 (Solution on p. 280.)
63
81
Exercise 4.157 (Solution on p. 280.)
150
240
236
CHAPTER 4. INTRODUCTION TO FRACTIONS AND MULTIPLICATION
AND DIVISION OF FRACTIONS
4.4.4 Raising Fractions to Higher Terms
Equally as important as reducing fractions is raising fractions to higher terms. Raising a fraction to higher
terms is the process of constructing an equivalent fraction that has higher values in the numerator and
denominator than the original fraction.

The fractions 3
5 and 15
9
are equivalent, that is, 3
5
9
= 15 . Notice also,
3·3 9
5·3 = 15

Notice that 3
3 = 1 and that 3
5 · 1 = 35 . We are not changing the value of 35 .

From these observations we can suggest the following method for converting one fraction to an equivalent
fraction that has higher values in the numerator and denominator. This method is called raising a fraction
to higher terms.

Raising a Fraction to Higher Terms


A fraction can be raised to an equivalent fraction that has higher terms in the numerator and denominator
by multiplying both the numerator and denominator by the same nonzero whole number.

The fraction 3
4 can be raised to 24
32 by multiplying both the numerator and denominator by 8.

Most often, we will want to convert a given fraction to an equivalent fraction with a higher specied de-
nominator. For example, we may wish to convert 85 to an equivalent fraction that has denominator 32, that
is,
5 ?
8 = 32

This is possible to do because we know the process. We must multiply both the numerator and denominator
of 58 by the same nonzero whole number in order to 8 obtain an equivalent fraction.

We have some information. The denominator 8 was raised to 32 by multiplying it by some nonzero whole
number. Division will give us the proper factor. Divide the original denominator into the new denominator.

32 ÷ 8 = 4

Now, multiply the numerator 5 by 4.

5 · 4 = 20

Thus,
5
= 5·4 20
= 32
8 8·4

So,
5
8 = 20
32

4.4.4.1 Sample Set D

Determine the missing numerator or denominator.


237

Example 4.29
3
7
?
= 35 . Divide the original denominator into the new denominator.

35 ÷ 7 = 5 The quotient is 5. Multiply the original numerator by 5.


3
= 3·5 15 The missing numerator is 15.
= 35
7 7·5
Example 4.30
5
6 = 45
? . Divide the original numerator into the new numerator.

45 ÷ 5 = 9 The quotient is 9. Multiply the original denominator by 9.


5
= 5·9 45 The missing denominator is 45.
= 54
6 6·9

4.4.4.2 Practice Set D

Determine the missing numerator or denominator.


Exercise 4.158 (Solution on p. 280.)
4 ?
5 = 40
Exercise 4.159 (Solution on p. 280.)
3 ?
7 = 28
Exercise 4.160 (Solution on p. 280.)
1 ?
6 = 24
Exercise 4.161 (Solution on p. 280.)
3 45
10 = ?
Exercise 4.162 (Solution on p. 280.)
8 ?
15 = 165

4.4.5 Exercises
For the following problems, determine if the pairs of fractions are equivalent.
Exercise 4.163 (Solution on p. 280.)
1 5
2 , 10
Exercise 4.164
2 8
3 , 12
Exercise 4.165 (Solution on p. 280.)
5 10
12 , 24
Exercise 4.166
1 3
2, 6
Exercise 4.167 (Solution on p. 280.)
3 12
5, 15
Exercise 4.168
1 7
6 , 42
Exercise 4.169 (Solution on p. 280.)
16 49
25 , 75
Exercise 4.170
5 20
28 , 112
238
CHAPTER 4. INTRODUCTION TO FRACTIONS AND MULTIPLICATION
AND DIVISION OF FRACTIONS
Exercise 4.171 (Solution on p. 280.)
3 36
10 , 110
Exercise 4.172
6 18
10 , 32
Exercise 4.173 (Solution on p. 280.)
5 15
8, 24
Exercise 4.174
10 , 15
16 24
Exercise 4.175 (Solution on p. 280.)
4 3
5, 4
Exercise 4.176
5 15
7, 21
Exercise 4.177 (Solution on p. 280.)
9 11
11 , 9
For the following problems, determine the missing numerator or denominator.
Exercise 4.178
1 ?
3 = 12
Exercise 4.179 (Solution on p. 280.)
1 ?
5 = 30
Exercise 4.180
2 ?
3 = 9
Exercise 4.181 (Solution on p. 280.)
1 ?
5 = 30
Exercise 4.182
2 ?
3 = 9
Exercise 4.183 (Solution on p. 280.)
3 ?
4 = 16
Exercise 4.184
5 ?
6 = 18
Exercise 4.185 (Solution on p. 280.)
4 ?
5 = 25
Exercise 4.186
1 4
2 = ?
Exercise 4.187 (Solution on p. 280.)
9 27
25 = ?
Exercise 4.188
3
2 = 18
?
Exercise 4.189 (Solution on p. 280.)
5
3 = 80
?
Exercise 4.190
1 3
8 = ?
Exercise 4.191 (Solution on p. 280.)
4 ?
5 = 100
Exercise 4.192
1
2 = 25
?
239

Exercise 4.193 (Solution on p. 280.)


3 ?
16 = 96
Exercise 4.194
15 = 225
16 ?
Exercise 4.195 (Solution on p. 280.)
11 ?
12 = 168
Exercise 4.196
9 ?
13 = 286
Exercise 4.197 (Solution on p. 281.)
32 ?
33 = 1518
Exercise 4.198
19 = 1045
20 ?
Exercise 4.199 (Solution on p. 281.)
37 1369
50 = ?
For the following problems, reduce, if possible, each of the fractions to lowest terms.
Exercise 4.200
6
8
Exercise 4.201 (Solution on p. 281.)
8
10
Exercise 4.202
5
10
Exercise 4.203 (Solution on p. 281.)
6
14
Exercise 4.204
3
12
Exercise 4.205 (Solution on p. 281.)
4
14
Exercise 4.206
1
6
Exercise 4.207 (Solution on p. 281.)
4
6
Exercise 4.208
18
14
Exercise 4.209 (Solution on p. 281.)
20
8
Exercise 4.210
4
6
Exercise 4.211 (Solution on p. 281.)
10
6
Exercise 4.212
6
14
Exercise 4.213 (Solution on p. 281.)
14
6
Exercise 4.214
10
12
240
CHAPTER 4. INTRODUCTION TO FRACTIONS AND MULTIPLICATION
AND DIVISION OF FRACTIONS
Exercise 4.215 (Solution on p. 281.)
16
70
Exercise 4.216
40
60
Exercise 4.217 (Solution on p. 281.)
20
12
Exercise 4.218
32
28
Exercise 4.219 (Solution on p. 281.)
36
10
Exercise 4.220
36
60
Exercise 4.221 (Solution on p. 281.)
12
18
Exercise 4.222
18
27
Exercise 4.223 (Solution on p. 281.)
18
24
Exercise 4.224
32
40
Exercise 4.225 (Solution on p. 281.)
11
22
Exercise 4.226
27
81
Exercise 4.227 (Solution on p. 281.)
17
51
Exercise 4.228
16
42
Exercise 4.229 (Solution on p. 281.)
39
13
Exercise 4.230
44
11
Exercise 4.231 (Solution on p. 281.)
66
33
Exercise 4.232
15
1
Exercise 4.233 (Solution on p. 281.)
15
16
Exercise 4.234
15
40
Exercise 4.235 (Solution on p. 281.)
36
100
Exercise 4.236
45
32
Exercise 4.237 (Solution on p. 281.)
30
75
241

Exercise 4.238
121
132
Exercise 4.239 (Solution on p. 281.)
72
64
Exercise 4.240
30
105
Exercise 4.241 (Solution on p. 281.)
46
60
Exercise 4.242
75
45
Exercise 4.243 (Solution on p. 281.)
40
18
Exercise 4.244
108
76
Exercise 4.245 (Solution on p. 281.)
7
21
Exercise 4.246
6
51
Exercise 4.247 (Solution on p. 282.)
51
12
Exercise 4.248
8
100
Exercise 4.249 (Solution on p. 282.)
51
54
Exercise 4.250
A ream of paper contains 500 sheets. What fraction of a ream of paper is 200 sheets? Be sure to
reduce.
Exercise 4.251 (Solution on p. 282.)
There are 24 hours in a day. What fraction of a day is 14 hours?
Exercise 4.252
A full box contains 80 calculators. How many calculators are in 1
4 of a box?
Exercise 4.253 (Solution on p. 282.)
There are 48 plants per at. How many plants are there in 1
3 of a at?
Exercise 4.254
A person making $18,000 per year must pay $3,960 in income tax. What fraction of this person's
yearly salary goes to the IRS?
For the following problems, nd the mistake.
Exercise 4.255 (Solution on p. 282.)
3 )3 0
24 = )3·8
= 8 =0
Exercise 4.256
8 )2+6 6 3
10 = )2+8
= 8 = 4

Exercise 4.257 (Solution on p. 282.)


7 )7 1
15 = )7+8
= 8

Exercise 4.258
6 )5+1 1
7 = = 2
)5+2
242
CHAPTER 4. INTRODUCTION TO FRACTIONS AND MULTIPLICATION
AND DIVISION OF FRACTIONS
Exercise 4.259 (Solution on p. 282.)
)9 0
= 0 =0
)9

4.4.5.1 Exercises for Review

Exercise 4.260
(Section 1.4) Round 816 to the nearest thousand.
Exercise 4.261 (Solution on p. 282.)
(Section 2.3) Perform the division: 0 ÷ 6.
Exercise 4.262
(Section 3.4) Find all the factors of 24.
Exercise 4.263 (Solution on p. 282.)
(Section 3.5) Find the greatest common factor of 12 and 18.
Exercise 4.264
(Section 4.3) Convert 15
8 to a mixed number.

4.5 Multiplication of Fractions5


4.5.1 Section Overview
• Fractions of Fractions
• Multiplication of Fractions
• Multiplication of Fractions by Dividing Out Common Factors
• Multiplication of Mixed Numbers
• Powers and Roots of Fractions

4.5.2 Fractions of Fractions


We know that a fraction represents a part of a whole quantity. For example, two fths of one unit can be
represented by

2
5 of the whole is shaded.

A natural question is, what is a fractional part of a fractional quantity, or, what is a fraction of a fraction?
For example, what 32 of 12 ?

We can suggest an answer to this question by using a picture to examine 2


3 of 12 .

First, let's represent 12 .

1
2 of the whole is shaded.

Then divide each of the 1


2 parts into 3 equal parts.
5 This content is available online at <http://cnx.org/content/m34928/1.2/>.
243

Each part is 1
6 of the whole.

Now we'll take 2


3 of the 1
2 unit.

2
3 of 1
2 is 26 , which reduces to 13 .

4.5.3 Multiplication of Fractions


Now we ask, what arithmetic operation (+, , ×, ÷) will produce 2
6 from 2
3 of 21 ?

Notice that, if in the fractions 2


3 and 12 , we multiply the numerators together and the denominators together,
we get precisely 62 .
2·1 2
3·2 = 6

This reduces to 1
3 as before.

Using this observation, we can suggest the following:

1. The word "of" translates to the arithmetic operation "times."


2. To multiply two or more fractions, multiply the numerators together and then multiply the denomina-
tors together. Reduce if necessary.
numerator 1 numerator 2 numerator 1 numerator 2
denominator 1 · denominator 2 = denominator 1 · denominator 2

4.5.3.1 Sample Set A

Perform the following multiplications.


Example 4.31
3
4 · 1
6 = 3·1
4·6
3
= 24 Now, reduce.
1
)3 1
= = 8
)24
8

Thus
3 1 1
4 · 6 = 8

This means that 3


4 of 1
6 is 18 , that is, 3
4 of 1
6 of a unit is 1
8 of the original unit.
244
CHAPTER 4. INTRODUCTION TO FRACTIONS AND MULTIPLICATION
AND DIVISION OF FRACTIONS
Example 4.32
3
8 · 4. Write 4 as a fraction by writing 4
1

3
3
8 · 4
1 = 3·4
8·1 = 12
8 =
)12
= 3
2
)8
2

3 3
8 ·4= 2

This means that 3


8 of 4 whole units is 3
2 of one whole unit.
Example 4.33
1
2 5 1 2·5·1 )10 1
5 · 8 · 4 = 5·8·4 = = 16
)160
16

This means that 2


5 of 5
8 of 1
4 of a whole unit is 16
1
of the original unit.

4.5.3.2 Practice Set A

Perform the following multiplications.


Exercise 4.265 (Solution on p. 282.)
2 1
5 · 6
Exercise 4.266 (Solution on p. 282.)
1 8
4 · 9
Exercise 4.267 (Solution on p. 282.)
4 15
9 · 16
Exercise
  4.268 (Solution on p. 282.)
2 2
3 3
Exercise
  4.269 (Solution on p. 282.)
7 8
4 5
Exercise 4.270 (Solution on p. 282.)
5 7
6 · 8
Exercise 4.271 (Solution on p. 282.)
2
3 ·5
Exercise 4.272 (Solution on p. 282.)
3
(10)

4
Exercise 4.273 (Solution on p. 282.)
3 8 5
4 · 9 · 12

4.5.4 Multiplying Fractions by Dividing Out Common Factors


We have seen that to multiply two fractions together, we multiply numerators together, then denominators
together, then reduce to lowest terms, if necessary. The reduction can be tedious if the numbers in the
fractions are large. For example,
9 10 9·10 90 45 15
16 · 21 = 16·21 = 336 = 168 = 28
We avoid the process of reducing if we divide out common factors before we multiply.
245

3 5
9 10 )9 )10 3·5
= 15
16 · 21 = )16
·
)21
= 8·7 56
8 7

Divide 3 into 9 and 21, and divide 2 into 10 and 16. The product is a fraction that is reduced to lowest
terms.

The Process of Multiplication by Dividing Out Common Factors


To multiply fractions by dividing out common factors, divide out factors that are common to both a numer-
ator and a denominator. The factor being divided out can appear in any numerator and any denominator.

4.5.4.1 Sample Set B

Perform the following multiplications.


Example 4.34
4 5
5 · 6

2 1
)4 )5 2·1 2
· = 1·3 = 3
)5 )6
1 3

Divide 4 and 6 by 2
Divide 5 and 5 by 5
Example 4.35
8 8
12 · 10
4 2
)8 )8 4·2 8
· = 3·5 = 15
)12 )10
3 5

Divide 8 and 10 by 2.
Divide 8 and 12 by 4.
Example 4.36
2
5
8 · 12 = )8
1 · 5
= 2·5
1·3 = 10
3
)12
3

Example 4.37
35 · 63
18 105
1
)7 7
)35 )63 1·7 7
= 2·3 = 6
)18 )105
2 )21
3

Example 4.38
13 · 6 · 1
9 39 12
1
1 )2
)13 )6 1 1·1·1 1
· · = = 54
9 )39 )12 9·1·6
)3 6
1
246
CHAPTER 4. INTRODUCTION TO FRACTIONS AND MULTIPLICATION
AND DIVISION OF FRACTIONS
4.5.4.2 Practice Set B

Perform the following multiplications.


Exercise 4.274 (Solution on p. 282.)
2 7
3 · 8
Exercise 4.275 (Solution on p. 282.)
25 10
12 · 45
Exercise 4.276 (Solution on p. 282.)
40 72
48 · 90
Exercise 4.277 (Solution on p. 282.)
2
7 · 49
Exercise 4.278 (Solution on p. 282.)
12 · 3
8
Exercise 4.279 (Solution on p. 282.)
 13 14
7 26
Exercise 4.280 (Solution on p. 282.)
16 22 21
10 · 6 · 44

4.5.5 Multiplication of Mixed Numbers


Multiplying Mixed Numbers
To perform a multiplication in which there are mixed numbers, it is convenient to rst convert each mixed
number to an improper fraction, then multiply.

4.5.5.1 Sample Set C

Perform the following multiplications. Convert improper fractions to mixed numbers.


Example 4.39
1 18 · 4 23

Convert each mixed number to an improper fraction.

1 81 = 8·1+1
8 = 9
8

4 32 = 4·3+2
3 = 14
3

3 7
)9
· )14
= 3·7
4·1 = 21 1
4 = 54
)8 )3
4 1

Example 4.40
16 · 8 15

Convert 8 15 to an improper fraction.

8 15 = 5·8+1
5 = 41
5

5 .
16 41
1 ·

There are no common factors to divide out.


16 · 41 = 16·41 = 656 = 131 1
1 5 1·5 5 5
247

Example 4.41
9 16 · 12 35

Convert to improper fractions.

9 61 = 6·9+1
6 = 55
6
5·12+3
12 35 = 5 = 63
5

11 21

)55
· )63
= 112·1
·21
= 231
2 = 115 2
1
)6 )5
2 1

Example 4.42
3 5
11 )9 )10
8 · 4 12 · 3 18 = 11
8 · ·
)2 )3
1 1
11·3·5 165
= 8·1·1 = 8 = 20 58

4.5.5.2 Practice Set C

Perform the following multiplications. Convert improper fractions to mixed numbers.


Exercise 4.281 (Solution on p. 283.)
2 23 · 2 14
Exercise 4.282 (Solution on p. 283.)
6 23 · 3 10
3

Exercise 4.283 (Solution on p. 283.)


7 18 · 12
Exercise 4.284 (Solution on p. 283.)
2 25 · 3 34 · 3 13

4.5.6 Powers and Roots of Fractions


4.5.6.1 Sample Set D

Find the value of each of the following.


Example 4.43
1 2 1 1 1·1 1

6 = 6 · 6 = 6·6 = 36
Example
q 4.44

100 . We're looking for a number, call it ?, such that when it is squared, 100 is produced.
9 9

29
(?) = 100

We know that

32 = 9 and 102 = 100

We'll try 10
3
. Since

3 2 3 3 3·3 9

10 = 10 · 10 = 10·10 = 100
248
CHAPTER 4. INTRODUCTION TO FRACTIONS AND MULTIPLICATION
AND DIVISION OF FRACTIONS
q
9 3
100 = 10
Example 4.45
100
q
4 25 · 121
2 2
)22 )10 2·2 4
· = 1·1 = 1 =4
)5 )11
1 1

100 = 4
q
4 52 · 121

4.5.6.2 Practice Set D

Find the value of each of the following.


Exercise 4.285 (Solution on p. 283.)
1 2

8
Exercise 4.286 (Solution on p. 283.)
3
2
10
Exercise
q 4.287 (Solution on p. 283.)
4
9
Exercise
q 4.288 (Solution on p. 283.)
1
4
Exercise
q 4.289 (Solution on p. 283.)
3 1
8 · 9
Exercise
q 4.290 (Solution on p. 283.)
9 13 · 81
100
Exercise
q 4.291 (Solution on p. 283.)
8
2 13 · 169
16

4.5.7 Exercises
For the following six problems, use the diagrams to nd each of the following parts. Use multiplication to
verify your result.
Exercise 4.292 (Solution on p. 283.)
3
4 of 1
3

Exercise 4.293
2
3 of 3
5
249

Exercise 4.294 (Solution on p. 283.)


2
7 of 7
8

Exercise 4.295
5
6 of 3
4

Exercise 4.296 (Solution on p. 283.)


1
8 of 1
8

Exercise 4.297
12 of
7 6
7

For the following problems, nd each part without using a diagram.
Exercise 4.298 (Solution on p. 283.)
1
2 of 4
5
Exercise 4.299
3
5 of 12
5

Exercise 4.300 (Solution on p. 283.)


1
4 of 8
9
Exercise 4.301
12
16 of 15
3
250
CHAPTER 4. INTRODUCTION TO FRACTIONS AND MULTIPLICATION
AND DIVISION OF FRACTIONS
Exercise 4.302 (Solution on p. 284.)
2
9 of 6
5
Exercise 4.303
1
8 of 3
8
Exercise 4.304 (Solution on p. 284.)
2
3 of 10
9

Exercise 4.305
18 of 38
19 54
Exercise 4.306 (Solution on p. 284.)
5
6 of 2 25
Exercise 4.307
3
4 of 3 35
Exercise 4.308 (Solution on p. 284.)
3
2 of 2 29
Exercise 4.309
15 of 4 4
4 5
Exercise 4.310 (Solution on p. 284.)
5 13 of 9 34
Exercise 4.311
1 13
15 of 8 4
3

Exercise 4.312 (Solution on p. 284.)


8
9 of 3
4 of 2
3
Exercise 4.313
1 12 of 26
of 13
6 36
Exercise 4.314 (Solution on p. 284.)
1
2 of 31 of 14
Exercise 4.315
1 37 of 5 15 of 8 13
Exercise 4.316 (Solution on p. 284.)
2 45 of 5 56 of 7 57
For the following problems, nd the products. Be sure to reduce.
Exercise 4.317
1 2
3 · 3
Exercise 4.318 (Solution on p. 284.)
1 1
2 · 2
Exercise 4.319
3 3
4 · 8
Exercise 4.320 (Solution on p. 284.)
2 5
5 · 6
Exercise 4.321
3 8
8 · 9
Exercise 4.322 (Solution on p. 284.)
5 14
· 15
6
Exercise 4.323
4 7
7 · 4
251

Exercise 4.324 (Solution on p. 284.)


3 11
11 · 3
Exercise 4.325
9 20
16 · 27
Exercise 4.326 (Solution on p. 284.)
35 48
36 · 55
Exercise 4.327
21 · 15
25 14
Exercise 4.328 (Solution on p. 284.)
76 66
99 · 38
Exercise 4.329
3 14 ·
· 18 6
7 2
Exercise 4.330 (Solution on p. 284.)
4 10 27
15 · 3 · 2
Exercise 4.331
14 · 21 · 45
15 28 7
Exercise 4.332 (Solution on p. 284.)
8
3 · 15 16
4 · 21
Exercise 4.333
18 · 21 · 36
14 35 7
Exercise 4.334 (Solution on p. 284.)
3
5 · 20
Exercise 4.335
8
9 · 18
Exercise 4.336 (Solution on p. 284.)
11 · 33
6

Exercise 4.337
18 · 38
19
Exercise 4.338 (Solution on p. 284.)
5
6 · 10
Exercise 4.339
1
9 ·3
Exercise 4.340 (Solution on p. 284.)
3
5· 8
Exercise 4.341
16 · 1
4
Exercise 4.342 (Solution on p. 284.)
2
3 · 12 · 3
4
Exercise 4.343
3
8 · 24 · 2
3
Exercise 4.344 (Solution on p. 284.)
18 · 10 ·
5 2
5
Exercise 4.345
16 · 50 · 3
15 10
Exercise 4.346 (Solution on p. 284.)
5 13 · 27
32
252
CHAPTER 4. INTRODUCTION TO FRACTIONS AND MULTIPLICATION
AND DIVISION OF FRACTIONS
Exercise 4.347
2 67 · 5 35
Exercise 4.348 (Solution on p. 284.)
4
6 14 · 2 15
Exercise 4.349
9
9 13 · 16 · 1 13
Exercise 4.350 (Solution on p. 284.)
3 59 · 1 13
14 · 10 2
1

Exercise 4.351
20 14 · 8 23 · 16 45
Exercise 4.352 (Solution on p. 285.)
2 2

3
Exercise 4.353
3 2

8
Exercise 4.354 (Solution on p. 285.)
2
2
11
Exercise 4.355
8 2

9
Exercise 4.356 (Solution on p. 285.)
1 2

2
Exercise 4.357
3 2
· 20

5 3
Exercise 4.358 (Solution on p. 285.)
1 2
· 16

4 15
Exercise 4.359
1 2 8

2 · 9
Exercise 4.360 (Solution on p. 285.)
1 2 2 2
 
2 · 5
Exercise4.361
3 2 1 2

7 · 9
For the following problems, nd each value. Reduce answers to lowest terms or convert to mixed numbers.
Exercise
q 4.362 (Solution on p. 285.)
4
9
Exercise 4.363
16
q
25
Exercise 4.364 (Solution on p. 285.)
81
q
121
Exercise 4.365
36
q
49
Exercise 4.366 (Solution on p. 285.)
144
q
25
Exercise
q 4.367
2 9
3 · 16
253

Exercise 4.368 (Solution on p. 285.)


25
q
3
5 · 81
Exercise
q 4.369
8 2

· 25
5 64
Exercise
q4.370 (Solution on p. 285.)
2
1 34 · 4
49
Exercise
q4.371
36 · 64
2 q
2 23 · 49 81

4.5.7.1 Exercises for Review

Exercise 4.372 (Solution on p. 285.)


(Section 1.2) How many thousands in 342,810?
Exercise 4.373
(Section 1.5) Find the sum of 22, 42, and 101.
Exercise 4.374 (Solution on p. 285.)
(Section 2.5) Is 634,281 divisible by 3?
Exercise 4.375
(Section 3.4) Is the whole number 51 prime or composite?
Exercise 4.376 (Solution on p. 285.)
36 to lowest terms.
(Section 4.4) Reduce 150

4.6 Division of Fractions6


4.6.1 Section Overview
• Reciprocals
• Dividing Fractions

4.6.2 Reciprocals
Reciprocals
Two numbers whose product is 1 are called reciprocals of each other.

4.6.2.1 Sample Set A

The following pairs of numbers are reciprocals.


Example 4.46
3 4
and
|4 {z 3}
3 4
4 · 3 =1

6 This content is available online at <http://cnx.org/content/m34929/1.2/>.


254
CHAPTER 4. INTRODUCTION TO FRACTIONS AND MULTIPLICATION
AND DIVISION OF FRACTIONS
Example 4.47
7 16
and
| {z 7}
16
7 16
16 · 7 = 1

Example 4.48
1 6
and
| {z 1}
6
1 6
6 · 1 =1
Notice that we can nd the reciprocal of a nonzero number in fractional form by inverting it (exchanging
positions of the numerator and denominator).

4.6.2.2 Practice Set A

Find the reciprocal of each number.


Exercise 4.377 (Solution on p. 285.)
3
10
Exercise 4.378 (Solution on p. 285.)
2
3
Exercise 4.379 (Solution on p. 285.)
7
8
Exercise 4.380 (Solution on p. 285.)
1
5
Exercise 4.381 (Solution on p. 285.)
2 27
Hint: Write this number as an improper fraction rst.

Exercise 4.382 (Solution on p. 285.)


5 14
Exercise 4.383 (Solution on p. 285.)
10 16
3

4.6.3 Dividing Fractions


Our concept of division is that it indicates how many times one quantity is contained in another quantity.
For example, using the diagram we can see that there are 6 one-thirds in 2.

There are 6 one-thirds in 2.

Since 2 contains six 31 's we express this as


255

Using these observations, we can suggest the following method for dividing a number by a fraction.

Dividing One Fraction by Another Fraction


To divide a rst fraction by a second, nonzero fraction, multiply the rst traction by the reciprocal of the
second fraction.

Invert and Multiply


This method is commonly referred to as "invert the divisor and multiply."

4.6.3.1 Sample Set B

Perform the following divisions.


Example 4.49
1
3 ÷ 43 . The divisor is 34 . Its reciprocal is 34 . Multiply 1
3 by 43 .
1 4 1·4 4
3 · 3 = 3·3 = 9
1 3 4
3 ÷ 4 = 9
Example 4.50
3
8 ÷ 5
4 The divisor is 45 . Its reciprocal is 45 . Multiply 3
8 by 54 .
1
3 )4 3·1 3
· 5 = 2·5 = 10
)3
2

3 5 3
8 ÷ 4 = 10
Example 4.51
5
6
5
÷ 12 . The divisor is 12
5
. Its reciprocal is 12
5 . Multiply
5
6 by 12
5 .

1 2
)5
· )12 = 1·2
1·1 = 2
1 =2
)6 )5
1 1

5 5
6 ÷ 12 =2
Example 4.52
2 29 ÷ 3 13 . Convert each mixed number to an improper fraction.

2 92 = 9·2+2
9 = 20
9 .

3 31 = 3 .
3·3+1 10
3 =
20 ÷ 10 The divisor is 10 . Its reciprocal is 3 . Multiply 20 by 3 .
9 3 3 10 9 10
2 1
)20 )3 2·1 2
· = 3·1 = 3
)9 )10
3 1
256
CHAPTER 4. INTRODUCTION TO FRACTIONS AND MULTIPLICATION
AND DIVISION OF FRACTIONS
2 92 ÷ 3 13 = 2
3
Example 4.53
12 ÷ 8. First conveniently write 8 as
1.
8
11
12 The divisor is 81 . Its reciprocal is 18 . Multiply 12
11 by 8 .
8 1
11 ÷ 1

3
)12 1 3·1 3
11 · )8
= 11·2 = 22
2

12 ÷ 8 = 3
11 22
Example 4.54
7 21 · 3 . The divisor is 21 . Its reciprocal is 20 .
÷ 20
8 35 20 21
1
1 )5 1
)7
· )20 )3
= 1·1·1 1
= 14
)8 )21 )35 2·1·7
2 )3 7
1

7
8 ÷ 21 3 1
20 · 25 = 14
Example 4.55
How many 2 38 -inch-wide packages can be placed in a box 19 inches wide?

The problem is to determine how many two and three eighths are contained in 19, that is, what is
19 ÷ 2 38 ?

2 83 = 19
8 Convert the divisor 2 8 to an improper fraction.
3

19 = 19 19
1 Write the dividend 19 as 1 .
19 ÷ 19 The divisor is 19 . Its reciprocal is 8 .
1 8 8 19
1
)19 8 1·8 8
· = = =8
1 )19 1·1 1
1

Thus, 8 packages will t into the box.

4.6.3.2 Practice Set B

Perform the following divisions.


Exercise 4.384 (Solution on p. 285.)
1 9
2 ÷ 8
Exercise 4.385 (Solution on p. 285.)
3 9
8 ÷ 24
Exercise 4.386 (Solution on p. 285.)
7 14
15 ÷ 15
Exercise 4.387 (Solution on p. 285.)
8
8 ÷ 15
Exercise 4.388 (Solution on p. 285.)
6 14 ÷ 12
5
257

Exercise 4.389 (Solution on p. 286.)


3 13 ÷ 1 23
Exercise 4.390 (Solution on p. 286.)
5 2 8
6 ÷ 3 · 25
Exercise 4.391 (Solution on p. 286.)
A container will hold 106 ounces of grape juice. How many 6 85 -ounce glasses of grape juice can
be served from this container?
Determine each of the following quotients and then write a rule for this type of division.
Exercise 4.392 (Solution on p. 286.)
2
1÷ 3
Exercise 4.393 (Solution on p. 286.)
3
1÷ 8
Exercise 4.394 (Solution on p. 286.)
3
1÷ 4
Exercise 4.395 (Solution on p. 286.)
5
1÷ 2
Exercise 4.396 (Solution on p. 286.)
When dividing 1 by a fraction, the quotient is the .

4.6.4 Exercises
For the following problems, nd the reciprocal of each number.
Exercise 4.397 (Solution on p. 286.)
4
5
Exercise 4.398
8
11
Exercise 4.399 (Solution on p. 286.)
2
9
Exercise 4.400
1
5
Exercise 4.401 (Solution on p. 286.)
3 14
Exercise 4.402
8 41
Exercise 4.403 (Solution on p. 286.)
3 27
Exercise 4.404
5 43
Exercise 4.405 (Solution on p. 286.)
1
Exercise 4.406
4
For the following problems, nd each value.
Exercise 4.407 (Solution on p. 286.)
3 3
8 ÷ 5
258
CHAPTER 4. INTRODUCTION TO FRACTIONS AND MULTIPLICATION
AND DIVISION OF FRACTIONS
Exercise 4.408
5 5
9 ÷ 6
Exercise 4.409 (Solution on p. 286.)
9 15
16 ÷ 8
Exercise 4.410
4 6
9 ÷ 15
Exercise 4.411 (Solution on p. 286.)
25 4
49 ÷ 9
Exercise 4.412
15 ÷ 27
4 8
Exercise 4.413 (Solution on p. 286.)
24 8
75 ÷ 15
Exercise 4.414
5
7 ÷0
Exercise 4.415 (Solution on p. 286.)
7 7
8 ÷ 8
Exercise 4.416
3
0÷ 5
Exercise 4.417 (Solution on p. 286.)
4 4
11 ÷ 11
Exercise 4.418
2 2
3 ÷ 3
Exercise 4.419 (Solution on p. 286.)
7 10
10 ÷ 7
Exercise 4.420
3
4 ÷6
Exercise 4.421 (Solution on p. 286.)
9
5 ÷3
Exercise 4.422
4 61 ÷ 3 13
Exercise 4.423 (Solution on p. 286.)
7 17 ÷ 8 13
Exercise 4.424
1 21 ÷ 1 15
Exercise 4.425 (Solution on p. 286.)
3 25 ÷ 25
6

Exercise 4.426
5 61 ÷ 31
6
Exercise 4.427 (Solution on p. 286.)
35 ÷ 3 3
6 4
Exercise 4.428
5 19 ÷ 18
1

Exercise 4.429 (Solution on p. 286.)


8 34 ÷ 7
8
Exercise 4.430
12 ÷ 1 1
8 2
259

Exercise 4.431 (Solution on p. 287.)


3 18 ÷ 15
16
Exercise 4.432
11 ÷ 9 5
11 12 8
Exercise 4.433 (Solution on p. 287.)
2 29 ÷ 11 32
Exercise 4.434
16 ÷ 6 2
3 5
Exercise 4.435 (Solution on p. 287.)
3
4 25 ÷ 2 56
75
Exercise 4.436
1 1
1000 ÷ 100
Exercise 4.437 (Solution on p. 287.)
3 9 6
8 ÷ 16 · 5
Exercise 4.438
3 9 6
16 · 8 · 5
Exercise 4.439 (Solution on p. 287.)
4 2 9
15 ÷ 25 · 10
Exercise 4.440
21 · 1 1 ÷ 9
30 4 10
Exercise 4.441 (Solution on p. 287.)
36 ÷ 4
8 13 · 75

4.6.4.1 Exercises for Review

Exercise 4.442
(Section 1.2) What is the value of 5 in the number 504,216?
Exercise 4.443 (Solution on p. 287.)
(Section 2.2) Find the product of 2,010 and 160.
Exercise 4.444
(Section 2.6) Use the numbers 8 and 5 to illustrate the commutative property of multiplication.
Exercise 4.445 (Solution on p. 287.)
(Section 3.6) Find the least common multiple of 6, 16, and 72.
Exercise 4.446
(Section 4.5) Find 8
9 of 6 34 .

4.7 Applications Involving Fractions 7

4.7.1 Section Overview


• Multiplication Statements
• Missing Product Statements
• Missing Factor Statements

7 This content is available online at <http://cnx.org/content/m34930/1.2/>.


260
CHAPTER 4. INTRODUCTION TO FRACTIONS AND MULTIPLICATION
AND DIVISION OF FRACTIONS
4.7.2 Multiplication Statements
Statement, Multiplication Statement
A statement is a sentence that is either true or false. A mathematical statement of the form

product = (factor 1) · (factor 2)

is a multiplication statement. Depending on the numbers that are used, it can be either true or false.

Omitting exactly one of the three numbers in the statement will produce exactly one of the following three
problems. For convenience, we'll represent the omitted (or missing) number with the letter M (M for
Missing).

1. M = (factor 1) · (factor 2) Missing product statement.


2. M · (factor 2) = product Missing factor statement.
3. (factor 1) · M = product Missing factor statement.

We are interested in developing and working with methods to determine the missing number that makes
the statement true. Fundamental to these methods is the ability to translate two words to mathematical
symbols. The word

of translates to times
is translates to equals

4.7.3 Missing Products Statements


The equation M = 8 · 4 is a missing product statement. We can nd the value of M that makes this
statement true by multiplying the known factors.

Missing product statements can be used to determine the answer to a question such as, "What number is
fraction 1 of fraction 2?

4.7.3.1 Sample Set A

Find 34 of 89 . We are being asked the question, "What number is 3


4 of 89 ?" We must translate from words to
mathematical symbols.

1 2

M= )3
· )8 = 1·2
1·3 = 32
)4 )9
1 3

Thus, 3
4 of 8
9 is 23 .
261

M= 3
· )24
1 =
3·6
1·1 = 18
1 = 18
)4
1

Thus, 18 is 3
4 of 24.

4.7.3.2 Practice Set A

Exercise 4.447 (Solution on p. 287.)


Find 3
8 of 16
15 .
Exercise 4.448 (Solution on p. 287.)
What number is 10
9
of 5
6 ?
Exercise 4.449 (Solution on p. 287.)
11
16 of 33 is what number?
8

4.7.4 Missing Factor Statements


The equation 8 · M = 32 is a missing factor statement. We can nd the value of M that makes this
statement true by dividing (since we know that 32 ÷ 8 = 4).

Finding the Missing Factor


To nd the missing factor in a missing factor statement, divide the product by the known factor.
missing factor = (product) ÷ (known factor)

Missing factor statements can be used to answer such questions as

1. 83 of what number is 94 ?
2. What part of 1 72 is 1 13
14 ?

4.7.4.1 Sample Set B

Now, using
262
CHAPTER 4. INTRODUCTION TO FRACTIONS AND MULTIPLICATION
AND DIVISION OF FRACTIONS
missing factor = (product) ÷ (known factor)

We get
3 2
9 3 9 8 )9 )8
M= 4 ÷ 8 = 4 · 3 = ·
)4 )3
1 1
3·2
= 1·1
= 6

Thus, 3
8 of 6 is 94 .

For convenience, let's convert the mixed numbers to improper fractions.

M· 9
7 = 27
14
Now, using

missing factor = (product)÷(known factor)

we get
3 1

M = 27
14 ÷
9
= 27
14 ·
7
= )27 · )7
7 9 )14 )9
2 1
3·1
=2·1
= 23
263

Thus, 3
2 of 1 27 is 1 13
14 .

4.7.4.2 Practice Set B

Exercise 4.450 (Solution on p. 287.)


3
5 of what number is 20
9
?
Exercise 4.451 (Solution on p. 287.)
3 34 of what number is 2 29 ?
Exercise 4.452 (Solution on p. 287.)
What part of 3
5 is 10
9
?
Exercise 4.453 (Solution on p. 287.)
What part of 1 14 is 1 78 ?

4.7.5 Exercises
Exercise 4.454 (Solution on p. 287.)
Find 2
3 of 34 .
Exercise 4.455
Find 5
8 of 10
1
.
Exercise 4.456 (Solution on p. 287.)
Find 12 13
13 of 36 .
Exercise 4.457
Find 1
4 of 47 .
Exercise 4.458 (Solution on p. 287.)
15
10 of 4 is what number?
3

Exercise 4.459
14 of 20 is what number?
15 21
Exercise 4.460 (Solution on p. 287.)
11
44 of 12 is what number?
3

Exercise 4.461
1
3 of 2 is what number?
Exercise 4.462 (Solution on p. 287.)
1
4 of 3 is what number?
Exercise 4.463
10 of 100 is what number?
1 1

Exercise 4.464 (Solution on p. 287.)


100 of 10 is what number?
1 1

Exercise 4.465
1 59 of 2 47 is what number?
Exercise 4.466 (Solution on p. 287.)
7
1 18 of 15
4
is what number?
Exercise 4.467
1 18 of 1 11
16 is what number?
Exercise 4.468 (Solution on p. 287.)
Find 2
3 of 1
6 of 92 .
264
CHAPTER 4. INTRODUCTION TO FRACTIONS AND MULTIPLICATION
AND DIVISION OF FRACTIONS
Exercise 4.469
Find 5
8 of 20
9
of 49 .
Exercise 4.470 (Solution on p. 287.)
12 of what number is 6 ?
5 5

Exercise 4.471
14 of what number is 7?
3 6

Exercise 4.472 (Solution on p. 287.)


10 of what number is
9?
5
3
Exercise 4.473
15 of what number is 20 ?
7 21
Exercise 4.474 (Solution on p. 288.)
8
3 of what number is 1 79 ?
Exercise 4.475
1
3 of what number is 31 ?
Exercise 4.476 (Solution on p. 288.)
1
6 of what number is 61 ?
Exercise 4.477
3
4 of what number is 43 ?
Exercise 4.478 (Solution on p. 288.)
11 of what number is 11 ?
8 8

Exercise 4.479
3
8 of what number is 0?
Exercise 4.480 (Solution on p. 288.)
2
3 of what number is 1?
Exercise 4.481
3 51 of what number is 1?
Exercise 4.482 (Solution on p. 288.)
9
1 12 of what number is 5 14 ?
Exercise 4.483
1
3 25 of what number is 2 15
8
?
Exercise 4.484 (Solution on p. 288.)
What part of 2
3 is 1 19 ?
Exercise 4.485
What part of 10
9
is 3 35 ?
Exercise 4.486 (Solution on p. 288.)
What part of 8
9 is 35 ?
Exercise 4.487
What part of 14
15 is 30 ?
7

Exercise 4.488 (Solution on p. 288.)


What part of 3 is 15 ?
Exercise 4.489
What part of 8 is 23 ?
Exercise 4.490 (Solution on p. 288.)
What part of 24 is 9?
Exercise 4.491
What part of 42 is 26?
265

Exercise 4.492 (Solution on p. 288.)


Find 12 39
13 of 40 .
Exercise 4.493
14 of 12 is what number?
15 21
Exercise 4.494 (Solution on p. 288.)
15 of what number is 2 5 ?
8 2

Exercise 4.495
11 of what number is 22 ?
15 35
Exercise 4.496 (Solution on p. 288.)
11
16 of what number is 1?
Exercise 4.497
What part of 23
40 is 3 20 ?
9

Exercise 4.498 (Solution on p. 288.)


35 of 3 22 is what number?
4 9

4.7.5.1 Exercises for Review

Exercise 4.499
(Section 1.7) Use the numbers 2 and 7 to illustrate the commutative property of addition.
Exercise 4.500 (Solution on p. 288.)
(Section 2.3) Is 4 divisible by 0?
Exercise 4.501
(Section 3.5) Expand 37 . Do not nd the actual value.
Exercise 4.502 (Solution on p. 288.)
(Section 4.3) Convert 3 12
5
to an improper fraction.
Exercise 4.503
(Section 4.4) Find the value of 3
8
9
÷ 16 · 65 .

4.8 Summary of Key Concepts 8


4.8.1 Summary of Key Concepts
Fraction (Section 4.2)
The idea of breaking up a whole quantity into equal parts gives us the word fraction.

Fraction Bar, Denominator, Numerator (Section 4.2)


A fraction has three parts:

1. The fraction bar


2. The nonzero whole number below the fraction bar is the denominator.
3. The whole number above the fraction bar is the numerator.
8 This content is available online at <http://cnx.org/content/m34931/1.2/>.
266
CHAPTER 4. INTRODUCTION TO FRACTIONS AND MULTIPLICATION
AND DIVISION OF FRACTIONS

Proper Fraction (Section 4.3)


Proper fractions are fractions in which the numerator is strictly less than the denominator.
4
5 is a proper fraction

Improper Fraction (Section 4.3)


Improper fractions are fractions in which the numerator is greater than or equal to the denominator.
Also, any nonzero number placed over 1 is an improper fraction.

4, 5, and are improper fractions


5 5 5
1

Mixed Number (Section 4.3)


A mixed number is a number that is the sum of a whole number and a proper fraction.

1 15 is a mixed number 1 15 = 1 + 1

5

Correspondence Between Improper Fractions and Mixed Numbers (Section 4.3)


Each improper fraction corresponds to a particular mixed number, and each mixed number corresponds to
a particular improper fraction.

Converting an Improper Fraction to a Mixed Number (Section 4.3)


A method, based on division, converts an improper fraction to an equivalent mixed number.
5
4 can be converted to 1 14

Converting a Mixed Number to an Improper Fraction (Section 4.3)


A method, based on multiplication, converts a mixed number to an equivalent improper fraction.

5 78 can be converted to 47
8

Equivalent Fractions (Section 4.4)


Fractions that represent the same quantity are equivalent fractions.
3
4 and 6
8 are equivalent fractions

Test for Equivalent Fractions (Section 4.4)


If the cross products of two fractions are equal, then the two fractions are equivalent.

Thus, 3
4 and 6
8 are equivalent.

Relatively Prime (Section 4.4)


Two whole numbers are relatively prime when 1 is the only number that divides both of them.

3 and 4 are relatively prime

Reduced to Lowest Terms (Section 4.4)


A fraction is reduced to lowest terms if its numerator and denominator are relatively prime.
267

The number 3
4 is reduced to lowest terms, since 3 and 4 are relatively prime.

The number 6
8 is not reduced to lowest terms since 6 and 8 are not relatively prime.

Reducing Fractions to Lowest Terms (Section 4.4)


Two methods, one based on dividing out common primes and one based on dividing out any common factors,
are available for reducing a fraction to lowest terms.

Raising Fractions to Higher Terms (Section 4.4)


A fraction can be raised to higher terms by multiplying both the numerator and denominator by the same
nonzero number.
3 3·2 6
4 = 4·2 = 8

The Word OF Means Multiplication (Section 4.5)


In many mathematical applications, the word "of" means multiplication.

Multiplication of Fractions (Section 4.5)


To multiply two or more fractions, multiply the numerators together and multiply the denominators together.
Reduce if possible.
5 4
· 15 = 5·4 20 =
= 120 1
8 8·15 6

Multiplying Fractions by Dividing Out Common Factors (Section 4.5)


Two or more fractions can be multiplied by rst dividing out common factors and then using the rule for
multiplying fractions.
1 1
)5 )4 1·1 1
· = 2·3 = 6
)8 )15
2 3

Multiplication of Mixed Numbers (Section 4.5)


To perform a multiplication in which there are mixed numbers, rst convert each mixed number to an
improper fraction, then multiply. This idea also applies to division of mixed numbers.

Reciprocals (Section 4.6)


Two numbers whose product is 1 are reciprocals.

7 and 1
7 are reciprocals

Division of Fractions (Section 4.6)


To divide one fraction by another fraction, multiply the dividend by the reciprocal of the divisor.
4
5
2
÷ 15 = 4
5 · 15
2

Dividing 1 by a Fraction (Section 4.6)


When dividing 1 by a fraction, the quotient is the reciprocal of the fraction.
1 7
3 = 3
7

Multiplication Statements (Section 4.7)


A mathematical statement of the form

product = (factor 1) (factor 2)

is a multiplication statement.
268
CHAPTER 4. INTRODUCTION TO FRACTIONS AND MULTIPLICATION
AND DIVISION OF FRACTIONS
By omitting one of the three numbers, one of three following problems result:

1. M = (factor 1) · (factor 2) Missing product statement.


2. product = (factor 1) · M Missing factor statement.
3. product = M · (factor 2) Missing factor statement.

Missing products are determined by simply multiplying the known factors. Missing factors are determined
by

missing factor = (product) ÷ (known factor)

4.9 Exercise Supplement 9


4.9.1 Exercise Supplement
4.9.1.1 Fractions of Whole Numbers (Section 4.2)

For Problems 1 and 2, name the suggested fraction.


Exercise 4.504 (Solution on p. 288.)

Exercise 4.505

For problems 3-5, specify the numerator and denominator.


Exercise 4.506 (Solution on p. 288.)
4
5
Exercise 4.507
5
12
Exercise 4.508 (Solution on p. 288.)
1
3
For problems 6-10, write each fraction using digits.
Exercise 4.509
Three fths
Exercise 4.510 (Solution on p. 288.)
Eight elevenths
Exercise 4.511
Sixty-one forty rsts
Exercise 4.512 (Solution on p. 288.)
Two hundred six-thousandths
Exercise 4.513
zero tenths
9 This content is available online at <http://cnx.org/content/m34932/1.2/>.
269

For problems 11-15, write each fraction using words.


Exercise 4.514 (Solution on p. 288.)
10
17
Exercise 4.515
21
38
Exercise 4.516 (Solution on p. 288.)
606
1431
Exercise 4.517
0
8
Exercise 4.518 (Solution on p. 288.)
1
16
For problems 16-18, state each numerator and denominator and write each fraction using digits.
Exercise 4.519
One minute is one sixtieth of an hour.
Exercise 4.520 (Solution on p. 288.)
In a box that contains forty-ve electronic components, eight are known to be defective. If three
components are chosen at random from the box, the probability that all three are defective is
fty-six fourteen thousand one hundred ninetieths.
Exercise 4.521
About three fths of the students in a college algebra class received a B in the course.
For problems 19 and 20, shade the region corresponding to the given fraction.
Exercise 4.522 (Solution on p. 288.)
1
4

Exercise 4.523
3
7

4.9.1.2 Proper Fraction, Improper Fraction, and Mixed Numbers (Section 4.3)

For problems 21-29, convert each improper fraction to a mixed number.


Exercise 4.524 (Solution on p. 289.)
11
4
Exercise 4.525
15
2
Exercise 4.526 (Solution on p. 289.)
51
8
Exercise 4.527
121
15
270
CHAPTER 4. INTRODUCTION TO FRACTIONS AND MULTIPLICATION
AND DIVISION OF FRACTIONS
Exercise 4.528 (Solution on p. 289.)
356
3
Exercise 4.529
3
2
Exercise 4.530 (Solution on p. 289.)
5
4
Exercise 4.531
20
5
Exercise 4.532 (Solution on p. 289.)
9
3
For problems 30-40, convert each mixed number to an improper fraction.
Exercise 4.533
5 32
Exercise 4.534 (Solution on p. 289.)
16 18
Exercise 4.535
18 31
Exercise 4.536 (Solution on p. 289.)
3 15
Exercise 4.537
9
2 16
Exercise 4.538 (Solution on p. 289.)
17 20
21
Exercise 4.539
1 87
Exercise 4.540 (Solution on p. 289.)
1 12
Exercise 4.541
2 21
Exercise 4.542 (Solution on p. 289.)
8 67
Exercise 4.543
2 29
Exercise 4.544 (Solution on p. 289.)
Why does 0 12
1
not qualify as a mixed number?
Exercise 4.545
Why does 8 qualify as a mixed number?
271

4.9.1.3 Equivalent Fractions, Reducing Fractions to Lowest Terms, and Raising Fractions to
Higher Term (Section 4.4)

For problems 43-47, determine if the pairs of fractions are equivalent.


Exercise 4.546 (Solution on p. 289.)
15
2,
1
30
Exercise 4.547
32
9,
8
36
Exercise 4.548 (Solution on p. 289.)
24
14 , 110
3

Exercise 4.549
2 38 , 38
16
Exercise 4.550 (Solution on p. 289.)
108 , 1 5
77 13
For problems 48-60, reduce, if possible, each fraction.
Exercise 4.551
10
25
Exercise 4.552 (Solution on p. 289.)
32
44
Exercise 4.553
102
266
Exercise 4.554 (Solution on p. 289.)
15
33
Exercise 4.555
18
25
Exercise 4.556 (Solution on p. 289.)
21
35
Exercise 4.557
9
16
Exercise 4.558 (Solution on p. 289.)
45
85
Exercise 4.559
24
42
Exercise 4.560 (Solution on p. 289.)
70
136
Exercise 4.561
182
580
Exercise 4.562 (Solution on p. 289.)
325
810
Exercise 4.563
250
1000
For problems 61-72, determine the missing numerator or denominator.
Exercise 4.564 (Solution on p. 289.)
3 ?
7 = 35
Exercise 4.565
4 ?
11 = 99
272
CHAPTER 4. INTRODUCTION TO FRACTIONS AND MULTIPLICATION
AND DIVISION OF FRACTIONS
Exercise 4.566 (Solution on p. 289.)
1 ?
12 = 72
Exercise 4.567
5
8 = 25
?
Exercise 4.568 (Solution on p. 289.)
11 = 33
9 ?
Exercise 4.569
4 24
15 = ?
Exercise 4.570 (Solution on p. 289.)
14 ?
15 = 45
Exercise 4.571
0 ?
5 = 20
Exercise 4.572 (Solution on p. 290.)
12 96
21 = ?
Exercise 4.573
14 = ?
23 253
Exercise 4.574 (Solution on p. 290.)
15 180
16 = ?
Exercise 4.575
21 = 336
22 ?

4.9.1.4 Multiplication and Division of Fractions (Section 4.5, Section 4.6)

For problems 73-95, perform each multiplication and division.


Exercise 4.576 (Solution on p. 290.)
4 15
· 16
5
Exercise 4.577
8 3
9 · 24
Exercise 4.578 (Solution on p. 290.)
1 5
10 · 12
Exercise 4.579
14 · 7
15 5
Exercise 4.580 (Solution on p. 290.)
5 13 · 11
· 22
6 39
Exercise 4.581
2
3 ÷ 15
7 ·
5
6
Exercise 4.582 (Solution on p. 290.)
3 12 ÷ 7
2
Exercise 4.583
2 94 ÷ 11
45
Exercise 4.584 (Solution on p. 290.)
8 3 5
15 · 16 · 24
Exercise 4.585
8 3 9
15 ÷ 3 5 · 16
273

Exercise 4.586 (Solution on p. 290.)


14 8 10
15 ÷ 3 9 · 21
Exercise 4.587
18 · 5 43
Exercise 4.588 (Solution on p. 290.)
1
3 37 · 2 12
Exercise 4.589
4 21 ÷ 2 47
Exercise 4.590 (Solution on p. 290.)
6 12 ÷ 3 14
Exercise 4.591
5 7
3 16 ÷ 2 18
Exercise 4.592 (Solution on p. 290.)
7 ÷ 2 13
Exercise 4.593
17 ÷ 4 14
Exercise 4.594 (Solution on p. 290.)
5
8 ÷ 1 14
Exercise 4.595
2 32 · 3 34
Exercise 4.596 (Solution on p. 290.)
20 · 18
4
Exercise 4.597
0 ÷ 4 18
Exercise 4.598 (Solution on p. 290.)
1 ÷ 6 14 · 25
4

4.9.1.5 Applications Involving Fractions (Section 4.7)

Exercise 4.599
Find 8
9 of 27
2 .
Exercise 4.600 (Solution on p. 290.)
What part of 3
8 is 21
16 ?
Exercise 4.601
What part of 3 15 is 1 25
7
?
Exercise 4.602 (Solution on p. 290.)
Find 6 32 of 15
9
.
Exercise 4.603
14
20 of what number is 35 ?
7

Exercise 4.604 (Solution on p. 290.)


What part of 4 16
1
is 3 34 ?
Exercise 4.605
Find 8 10
3
of 16 23 .
Exercise 4.606 (Solution on p. 290.)
18
20 of what number is 30 ?
3
274
CHAPTER 4. INTRODUCTION TO FRACTIONS AND MULTIPLICATION
AND DIVISION OF FRACTIONS
Exercise 4.607
Find 1
3 of 0.
Exercise 4.608 (Solution on p. 290.)
Find 11
12 of 1.

4.10 Prociency Exam10


4.10.1 Prociency Exam
Exercise 4.609 (Solution on p. 290.)
(Section 4.2) Shade a portion that corresponds to the fraction 85 .

Exercise 4.610 (Solution on p. 290.)


(Section 4.2) Specify the numerator and denominator of the fraction 59 .
Exercise 4.611 (Solution on p. 290.)
(Section 4.2) Write the fraction ve elevenths.
Exercise 4.612 (Solution on p. 290.)
(Section 4.2) Write, in words, 45 .
Exercise 4.613 (Solution on p. 290.)
(Section 4.3) Which of the fractions is a proper fraction? 4 12
1
, 12
5
, 12
5
Exercise 4.614 (Solution on p. 291.)
(Section 4.3) Convert 3 47 to an improper fraction.
Exercise 4.615 (Solution on p. 291.)
(Section 4.3) Convert 16
5 to a mixed number.
Exercise 4.616 (Solution on p. 291.)
(Section 4.4) Determine if 12
5
and 20
48 are equivalent fractions.
For problems 9-11, reduce, if possible, each fraction to lowest terms.
Exercise 4.617 (Solution on p. 291.)
21
(Section 4.4) 35
Exercise 4.618 (Solution on p. 291.)
(Section 4.4) 15
51
Exercise 4.619 (Solution on p. 291.)
(Section 4.4) 104
480
For problems 12 and 13, determine the missing numerator or denominator.
Exercise 4.620 (Solution on p. 291.)
(Section 4.4) 5
9
?
= 36
Exercise 4.621 (Solution on p. 291.)
(Section 4.4) 4
3 = 32
?
For problems 14-25, nd each value.
Exercise 4.622 (Solution on p. 291.)
(Section 4.5) 15 4
16 · 25
10 This content is available online at <http://cnx.org/content/m34933/1.2/>.
275

Exercise 4.623 (Solution on p. 291.)


(Section 4.5) 3 34 · 2 29 · 6 35
Exercise 4.624 (Solution on p. 291.)
25
q
(Section 4.5) 36
Exercise 4.625 (Solution on p. 291.)
81
q q
(Section 4.5) 4
9 · 64
Exercise 4.626 (Solution on p. 291.)
(Section 4.5) 11 225
q
30 · 121
Exercise 4.627 (Solution on p. 291.)
(Section 4.6) 15
4
÷8
Exercise 4.628 (Solution on p. 291.)
(Section 4.6) 15
8 5
· 12 ÷ 2 49
Exercise 4.629 (Solution on p. 291.)
1 11
q
6 3
(Section 4.6)

5 ÷ 25
Exercise 4.630 (Solution on p. 291.)
(Section 4.7) Find 12
5
of 24
25 .
Exercise 4.631 (Solution on p. 291.)
(Section 4.7) 2
9 of what number is 18
1
?
Exercise 4.632 (Solution on p. 291.)
(Section 4.7) 1 57 of 21
20 is what number?
Exercise 4.633 (Solution on p. 291.)
(Section 4.7) What part of 14
9
is 67 ?
276
CHAPTER 4. INTRODUCTION TO FRACTIONS AND MULTIPLICATION
AND DIVISION OF FRACTIONS
Solutions to Exercises in Chapter 4
Solution to Exercise 4.1 (p. 215)
4, 7
Solution to Exercise 4.2 (p. 216)
5, 8
Solution to Exercise 4.3 (p. 216)
10, 15
Solution to Exercise 4.4 (p. 216)
1, 9
Solution to Exercise 4.5 (p. 216)
0, 2
Solution to Exercise 4.6 (p. 217)
1
10
Solution to Exercise 4.7 (p. 217)
11
14
Solution to Exercise 4.8 (p. 217)
16
35
Solution to Exercise 4.9 (p. 217)
800
7,000
Solution to Exercise 4.10 (p. 217)
three eighths
Solution to Exercise 4.11 (p. 217)
one tenth
Solution to Exercise 4.12 (p. 217)
three two hundred ftieths
Solution to Exercise 4.13 (p. 217)
one hundred fourteen three thousand one hundred ninetieths
Solution to Exercise 4.14 (p. 217)
3
8
Solution to Exercise 4.15 (p. 217)
1
16
Solution to Exercise 4.16 (p. 217)
5, 9, ve ninths
Solution to Exercise 4.17 (p. 217)
1, 10, one tenth
Solution to Exercise 4.18 (p. 218)
numerator, 3; denominator, 4
Solution to Exercise 4.20 (p. 218)
numerator, 1; denominator, 5
Solution to Exercise 4.22 (p. 218)
numerator, 7; denominator, 7
Solution to Exercise 4.24 (p. 218)
numerator, 0; denominator, 12
Solution to Exercise 4.26 (p. 218)
numerator, 18; denominator, 1
Solution to Exercise 4.28 (p. 218)
4
5
Solution to Exercise 4.30 (p. 218)
15
20
277

Solution to Exercise 4.32 (p. 218)


91
107
Solution to Exercise 4.34 (p. 218)
605
834
Solution to Exercise 4.36 (p. 218)
92
1,000,000
Solution to Exercise 4.38 (p. 218)
ve ninths
Solution to Exercise 4.40 (p. 219)
eight fteenths
Solution to Exercise 4.42 (p. 219)
seventy-ve one hundredths
Solution to Exercise 4.44 (p. 219)
nine hundred sixteen one thousand fourteenths
Solution to Exercise 4.46 (p. 219)
eighteen thirty-one thousand six hundred eighths
Solution to Exercise 4.48 (p. 219)
1
2
Solution to Exercise 4.50 (p. 219)
4
7
Solution to Exercise 4.52 (p. 219)

Solution to Exercise 4.54 (p. 220)

Solution to Exercise 4.56 (p. 220)


Numerator, 1; denominator, 4; one fourth
Solution to Exercise 4.58 (p. 220)
Numerator, 4; denominator, 3; four thirds
Solution to Exercise 4.60 (p. 220)
Numerator, 2; denominator, 7; two sevenths
Solution to Exercise 4.62 (p. 221)
Numerator, 1; denominator, 56; one fty-sixth
Solution to Exercise 4.64 (p. 221)
Numerator, 125; denominator, 4; one hundred twenty-ve fourths
Solution to Exercise 4.66 (p. 221)
3 + 11 = 11 + 3 = 14
Solution to Exercise 4.68 (p. 221)
75
Solution to Exercise 4.70 (p. 221)
144
Solution to Exercise 4.71 (p. 226)
4 21
Solution to Exercise 4.72 (p. 226)
3 23
Solution to Exercise 4.73 (p. 226)
3
1 11
278
CHAPTER 4. INTRODUCTION TO FRACTIONS AND MULTIPLICATION
AND DIVISION OF FRACTIONS
Solution to Exercise 4.74 (p. 226)
5
2 13
Solution to Exercise 4.75 (p. 226)
19 43
Solution to Exercise 4.76 (p. 226)
62
Solution to Exercise 4.77 (p. 228)
33
4
Solution to Exercise 4.78 (p. 228)
28
5
Solution to Exercise 4.79 (p. 228)
19
15
Solution to Exercise 4.80 (p. 228)
86
7
Solution to Exercise 4.81 (p. 228)
improper fraction
Solution to Exercise 4.83 (p. 228)
proper fraction
Solution to Exercise 4.85 (p. 228)
mixed number
Solution to Exercise 4.87 (p. 228)
improper fraction
Solution to Exercise 4.89 (p. 228)
mixed number
Solution to Exercise 4.91 (p. 228)
mixed number
Solution to Exercise 4.93 (p. 228)
proper fraction
Solution to Exercise 4.95 (p. 228)
mixed number
Solution to Exercise 4.97 (p. 229)
4 23
Solution to Exercise 4.99 (p. 229)
8 34
Solution to Exercise 4.101 (p. 229)
9
Solution to Exercise 4.103 (p. 229)
13 12
9
or 13 34
Solution to Exercise 4.105 (p. 229)
555 59
Solution to Exercise 4.107 (p. 229)
36 21
Solution to Exercise 4.109 (p. 229)
7 29
41
Solution to Exercise 4.111 (p. 229)
33
8
Solution to Exercise 4.113 (p. 229)
61
9
Solution to Exercise 4.115 (p. 229)
115
11
279

Solution to Exercise 4.117 (p. 229)


26
3
Solution to Exercise 4.119 (p. 230)
107
5
Solution to Exercise 4.121 (p. 230)
209
21
Solution to Exercise 4.123 (p. 230)
9001
100
Solution to Exercise 4.125 (p. 230)
159
8
Solution to Exercise 4.127 (p. 230)
. . . because it may be written as 5 n0 , where n is any positive whole number.
Solution to Exercise 4.129 (p. 230)
1,652
61
Solution to Exercise 4.131 (p. 230)
2,436
23
Solution to Exercise 4.133 (p. 230)
20,419
25
Solution to Exercise 4.135 (p. 230)
48,803,620
8,117
Solution to Exercise 4.137 (p. 231)
1,009,020
Solution to Exercise 4.139 (p. 231)
252
Solution to Exercise 4.141 (p. 232)

, yes
Solution to Exercise 4.142 (p. 232)

, yes
Solution to Exercise 4.143 (p. 232)
30 6= 24, no
Solution to Exercise 4.144 (p. 232)
, yes
Solution to Exercise 4.145 (p. 232)
, yes
Solution to Exercise 4.146 (p. 234)
1
2
Solution to Exercise 4.147 (p. 234)
2
5
Solution to Exercise 4.148 (p. 234)
1
8
Solution to Exercise 4.149 (p. 234)
7
16
Solution to Exercise 4.150 (p. 234)
12
7
Solution to Exercise 4.151 (p. 234)
5
9
Solution to Exercise 4.152 (p. 235)
3
4
Solution to Exercise 4.153 (p. 235)
3
8
280
CHAPTER 4. INTRODUCTION TO FRACTIONS AND MULTIPLICATION
AND DIVISION OF FRACTIONS
Solution to Exercise 4.154 (p. 235)
1
4
Solution to Exercise 4.155 (p. 235)
3
4
Solution to Exercise 4.156 (p. 235)
7
9
Solution to Exercise 4.157 (p. 235)
5
8
Solution to Exercise 4.158 (p. 237)
32
Solution to Exercise 4.159 (p. 237)
12
Solution to Exercise 4.160 (p. 237)
4
Solution to Exercise 4.161 (p. 237)
150
Solution to Exercise 4.162 (p. 237)
88
Solution to Exercise 4.163 (p. 237)
equivalent
Solution to Exercise 4.165 (p. 237)
equivalent
Solution to Exercise 4.167 (p. 237)
not equivalent
Solution to Exercise 4.169 (p. 237)
not equivalent
Solution to Exercise 4.171 (p. 238)
not equivalent
Solution to Exercise 4.173 (p. 238)
equivalent
Solution to Exercise 4.175 (p. 238)
not equivalent
Solution to Exercise 4.177 (p. 238)
not equivalent
Solution to Exercise 4.179 (p. 238)
6
Solution to Exercise 4.181 (p. 238)
12
Solution to Exercise 4.183 (p. 238)
12
Solution to Exercise 4.185 (p. 238)
20
Solution to Exercise 4.187 (p. 238)
75
Solution to Exercise 4.189 (p. 238)
48
Solution to Exercise 4.191 (p. 238)
80
Solution to Exercise 4.193 (p. 239)
18
281

Solution to Exercise 4.195 (p. 239)


154
Solution to Exercise 4.197 (p. 239)
1,472
Solution to Exercise 4.199 (p. 239)
1,850
Solution to Exercise 4.201 (p. 239)
4
5
Solution to Exercise 4.203 (p. 239)
3
7
Solution to Exercise 4.205 (p. 239)
2
7
Solution to Exercise 4.207 (p. 239)
2
3
Solution to Exercise 4.209 (p. 239)
5
2
Solution to Exercise 4.211 (p. 239)
5
3
Solution to Exercise 4.213 (p. 239)
7
3
Solution to Exercise 4.215 (p. 240)
8
35
Solution to Exercise 4.217 (p. 240)
5
3
Solution to Exercise 4.219 (p. 240)
18
5
Solution to Exercise 4.221 (p. 240)
2
3
Solution to Exercise 4.223 (p. 240)
3
4
Solution to Exercise 4.225 (p. 240)
1
2
Solution to Exercise 4.227 (p. 240)
1
3
Solution to Exercise 4.229 (p. 240)
3
Solution to Exercise 4.231 (p. 240)
2
Solution to Exercise 4.233 (p. 240)
already reduced
Solution to Exercise 4.235 (p. 240)
9
25
Solution to Exercise 4.237 (p. 240)
2
5
Solution to Exercise 4.239 (p. 241)
9
8
Solution to Exercise 4.241 (p. 241)
23
30
Solution to Exercise 4.243 (p. 241)
20
9
282
CHAPTER 4. INTRODUCTION TO FRACTIONS AND MULTIPLICATION
AND DIVISION OF FRACTIONS
Solution to Exercise 4.245 (p. 241)
1
3
Solution to Exercise 4.247 (p. 241)
17
4
Solution to Exercise 4.249 (p. 241)
17
18
Solution to Exercise 4.251 (p. 241)
7
12
Solution to Exercise 4.253 (p. 241)
16
Solution to Exercise 4.255 (p. 241)
Should be 18 ; the cancellation is division, so the numerator should be 1.
Solution to Exercise 4.257 (p. 241)
Cancel factors only, not addends; 15
7
is already reduced.
Solution to Exercise 4.259 (p. 242)
Same as Exercise 4.255; answer is 1
1 or 1.
Solution to Exercise 4.261 (p. 242)
0
Solution to Exercise 4.263 (p. 242)
6
Solution to Exercise 4.265 (p. 244)
1
15
Solution to Exercise 4.266 (p. 244)
2
9
Solution to Exercise 4.267 (p. 244)
5
12
Solution to Exercise 4.268 (p. 244)
4
9
Solution to Exercise 4.269 (p. 244)
14
5
Solution to Exercise 4.270 (p. 244)
35
48
Solution to Exercise 4.271 (p. 244)
10
3
Solution to Exercise 4.272 (p. 244)
15
2
Solution to Exercise 4.273 (p. 244)
5
18
Solution to Exercise 4.274 (p. 246)
7
12
Solution to Exercise 4.275 (p. 246)
25
54
Solution to Exercise 4.276 (p. 246)
2
3
Solution to Exercise 4.277 (p. 246)
2
7
Solution to Exercise 4.278 (p. 246)
9
2
Solution to Exercise 4.279 (p. 246)
1
283

Solution to Exercise 4.280 (p. 246)


14
5
Solution to Exercise 4.281 (p. 247)
6
Solution to Exercise 4.282 (p. 247)
22
Solution to Exercise 4.283 (p. 247)
85 12
Solution to Exercise 4.284 (p. 247)
30
Solution to Exercise 4.285 (p. 248)
1
64
Solution to Exercise 4.286 (p. 248)
9
100
Solution to Exercise 4.287 (p. 248)
2
3
Solution to Exercise 4.288 (p. 248)
1
2
Solution to Exercise 4.289 (p. 248)
1
8
Solution to Exercise 4.290 (p. 248)
8 25
Solution to Exercise 4.291 (p. 248)
8 12
Solution to Exercise 4.292 (p. 248)
1
4

Solution to Exercise 4.294 (p. 249)


1
4

Solution to Exercise 4.296 (p. 249)


1
64

Solution to Exercise 4.298 (p. 249)


2
5
284
CHAPTER 4. INTRODUCTION TO FRACTIONS AND MULTIPLICATION
AND DIVISION OF FRACTIONS
Solution to Exercise 4.300 (p. 249)
2
9
Solution to Exercise 4.302 (p. 250)
4
15
Solution to Exercise 4.304 (p. 250)
3
5
Solution to Exercise 4.306 (p. 250)
2
Solution to Exercise 4.308 (p. 250)
10 or 3 1
3 3
Solution to Exercise 4.310 (p. 250)
52
Solution to Exercise 4.312 (p. 250)
4
9
Solution to Exercise 4.314 (p. 250)
1
24
Solution to Exercise 4.316 (p. 250)
126
Solution to Exercise 4.318 (p. 250)
1
4
Solution to Exercise 4.320 (p. 250)
1
3
Solution to Exercise 4.322 (p. 250)
7
9
Solution to Exercise 4.324 (p. 251)
1
Solution to Exercise 4.326 (p. 251)
28
33
Solution to Exercise 4.328 (p. 251)
4
3
Solution to Exercise 4.330 (p. 251)
12
Solution to Exercise 4.332 (p. 251)
7 13 160
21 or 21
Solution to Exercise 4.334 (p. 251)
12
Solution to Exercise 4.336 (p. 251)
18
Solution to Exercise 4.338 (p. 251)
25 or 8 1
3 3
Solution to Exercise 4.340 (p. 251)
15 =1 7
8 8
Solution to Exercise 4.342 (p. 251)
6
Solution to Exercise 4.344 (p. 251)
10 =1 1
9 9
Solution to Exercise 4.346 (p. 251)
2 =4 2
9 1

Solution to Exercise 4.348 (p. 252)


85 =14 1
6 6
285

Solution to Exercise 4.350 (p. 252)


72
Solution to Exercise 4.352 (p. 252)
4
9
Solution to Exercise 4.354 (p. 252)
4
121
Solution to Exercise 4.356 (p. 252)
1
4
Solution to Exercise 4.358 (p. 252)
1
15
Solution to Exercise 4.360 (p. 252)
1
25
Solution to Exercise 4.362 (p. 252)
2
3
Solution to Exercise 4.364 (p. 252)
9
11
Solution to Exercise 4.366 (p. 252)
12 = 2 2
5 5
Solution to Exercise 4.368 (p. 253)
1
3
Solution to Exercise 4.370 (p. 253)
7
8
Solution to Exercise 4.372 (p. 253)
2
Solution to Exercise 4.374 (p. 253)
yes
Solution to Exercise 4.376 (p. 253)
6
25
Solution to Exercise 4.377 (p. 254)
10
3
Solution to Exercise 4.378 (p. 254)
3
2
Solution to Exercise 4.379 (p. 254)
8
7
Solution to Exercise 4.380 (p. 254)
5
Solution to Exercise 4.381 (p. 254)
7
16
Solution to Exercise 4.382 (p. 254)
4
21
Solution to Exercise 4.383 (p. 254)
16
163
Solution to Exercise 4.384 (p. 256)
4
9
Solution to Exercise 4.385 (p. 256)
1
Solution to Exercise 4.386 (p. 256)
1
2
Solution to Exercise 4.387 (p. 256)
15
286
CHAPTER 4. INTRODUCTION TO FRACTIONS AND MULTIPLICATION
AND DIVISION OF FRACTIONS
Solution to Exercise 4.388 (p. 256)
15
Solution to Exercise 4.389 (p. 257)
2
Solution to Exercise 4.390 (p. 257)
2
5
Solution to Exercise 4.391 (p. 257)
16 glasses
Solution to Exercise 4.392 (p. 257)
3
2
Solution to Exercise 4.393 (p. 257)
8
3
Solution to Exercise 4.394 (p. 257)
4
3
Solution to Exercise 4.395 (p. 257)
2
5
Solution to Exercise 4.396 (p. 257)
is the reciprocal of the fraction.
Solution to Exercise 4.397 (p. 257)
5
4 or 1 14
Solution to Exercise 4.399 (p. 257)
9
2 or 4 12
Solution to Exercise 4.401 (p. 257)
4
13
Solution to Exercise 4.403 (p. 257)
7
23
Solution to Exercise 4.405 (p. 257)
1
Solution to Exercise 4.407 (p. 257)
5
8
Solution to Exercise 4.409 (p. 258)
3
10
Solution to Exercise 4.411 (p. 258)
225 or 1 29
196 196
Solution to Exercise 4.413 (p. 258)
3
5
Solution to Exercise 4.415 (p. 258)
1
Solution to Exercise 4.417 (p. 258)
1
Solution to Exercise 4.419 (p. 258)
49
100
Solution to Exercise 4.421 (p. 258)
3
5
Solution to Exercise 4.423 (p. 258)
6
7
Solution to Exercise 4.425 (p. 258)
85 or 14 1
6 6
Solution to Exercise 4.427 (p. 258)
28 = 14 or 1 5
18 9 9
287

Solution to Exercise 4.429 (p. 258)


10
Solution to Exercise 4.431 (p. 259)
10 or 3 1
3 3
Solution to Exercise 4.433 (p. 259)
4
21
Solution to Exercise 4.435 (p. 259)
3
2 or 1 12
Solution to Exercise 4.437 (p. 259)
4
5
Solution to Exercise 4.439 (p. 259)
3
Solution to Exercise 4.441 (p. 259)
1
Solution to Exercise 4.443 (p. 259)
321,600
Solution to Exercise 4.445 (p. 259)
144
Solution to Exercise 4.447 (p. 261)
2
5
Solution to Exercise 4.448 (p. 261)
3
4
Solution to Exercise 4.449 (p. 261)
1
6
Solution to Exercise 4.450 (p. 263)
3
4
Solution to Exercise 4.451 (p. 263)
16
27
Solution to Exercise 4.452 (p. 263)
1 12
Solution to Exercise 4.453 (p. 263)
1 12
Solution to Exercise 4.454 (p. 263)
1
2
Solution to Exercise 4.456 (p. 263)
1
3
Solution to Exercise 4.458 (p. 263)
9
8 or 1 18
Solution to Exercise 4.460 (p. 263)
1
16
Solution to Exercise 4.462 (p. 263)
3
4
Solution to Exercise 4.464 (p. 263)
1
1,000
Solution to Exercise 4.466 (p. 263)
10
27
Solution to Exercise 4.468 (p. 263)
1
2
Solution to Exercise 4.470 (p. 264)
2
288
CHAPTER 4. INTRODUCTION TO FRACTIONS AND MULTIPLICATION
AND DIVISION OF FRACTIONS
Solution to Exercise 4.472 (p. 264)
1
6
Solution to Exercise 4.474 (p. 264)
2
3
Solution to Exercise 4.476 (p. 264)
1
Solution to Exercise 4.478 (p. 264)
1
Solution to Exercise 4.480 (p. 264)
3
2 or 1 23
Solution to Exercise 4.482 (p. 264)
3
Solution to Exercise 4.484 (p. 264)
5
3 or 1 23
Solution to Exercise 4.486 (p. 264)
27
40
Solution to Exercise 4.488 (p. 264)
1
15
Solution to Exercise 4.490 (p. 264)
3
8
Solution to Exercise 4.492 (p. 265)
9
10
Solution to Exercise 4.494 (p. 265)
9
2 = 4 12
Solution to Exercise 4.496 (p. 265)
16 or 1 5
11 11
Solution to Exercise 4.498 (p. 265)
30
77
Solution to Exercise 4.500 (p. 265)
no
Solution to Exercise 4.502 (p. 265)
41
12
Solution to Exercise 4.504 (p. 268)
2
6 or 1
3
Solution to Exercise 4.506 (p. 268)
numerator, 4; denominator, 5
Solution to Exercise 4.508 (p. 268)
numerator, 1; denominator, 3
Solution to Exercise 4.510 (p. 268)
8
11
Solution to Exercise 4.512 (p. 268)
200
6,000
Solution to Exercise 4.514 (p. 269)
ten seventeenths
Solution to Exercise 4.516 (p. 269)
six hundred six, one thousand four hundred thirty-rsts
Solution to Exercise 4.518 (p. 269)
one sixteenth
Solution to Exercise 4.520 (p. 269)
numerator, 56; denominator, 14,190
289

Solution to Exercise 4.522 (p. 269)

Solution to Exercise 4.524 (p. 269)


2 34
Solution to Exercise 4.526 (p. 269)
6 38
Solution to Exercise 4.528 (p. 270)
118 23
Solution to Exercise 4.530 (p. 270)
1 14
Solution to Exercise 4.532 (p. 270)
3
Solution to Exercise 4.534 (p. 270)
129
8
Solution to Exercise 4.536 (p. 270)
16
5
Solution to Exercise 4.538 (p. 270)
377
21
Solution to Exercise 4.540 (p. 270)
3
2
Solution to Exercise 4.542 (p. 270)
62
7
Solution to Exercise 4.544 (p. 270)
because the whole number part is zero
Solution to Exercise 4.546 (p. 271)
equivalent
Solution to Exercise 4.548 (p. 271)
not equivalent
Solution to Exercise 4.550 (p. 271)
not equivalent
Solution to Exercise 4.552 (p. 271)
8
11
Solution to Exercise 4.554 (p. 271)
5
11
Solution to Exercise 4.556 (p. 271)
3
5
Solution to Exercise 4.558 (p. 271)
9
17
Solution to Exercise 4.560 (p. 271)
35
68
Solution to Exercise 4.562 (p. 271)
65
162
Solution to Exercise 4.564 (p. 271)
15
Solution to Exercise 4.566 (p. 272)
6
Solution to Exercise 4.568 (p. 272)
27
290
CHAPTER 4. INTRODUCTION TO FRACTIONS AND MULTIPLICATION
AND DIVISION OF FRACTIONS
Solution to Exercise 4.570 (p. 272)
42
Solution to Exercise 4.572 (p. 272)
168
Solution to Exercise 4.574 (p. 272)
192
Solution to Exercise 4.576 (p. 272)
3
4
Solution to Exercise 4.578 (p. 272)
1
24
Solution to Exercise 4.580 (p. 272)
5
36
Solution to Exercise 4.582 (p. 272)
1
Solution to Exercise 4.584 (p. 272)
1
48
Solution to Exercise 4.586 (p. 273)
4
35
Solution to Exercise 4.588 (p. 273)
50 = 7 1
7 7
Solution to Exercise 4.590 (p. 273)
2
Solution to Exercise 4.592 (p. 273)
3
Solution to Exercise 4.594 (p. 273)
1
2
Solution to Exercise 4.596 (p. 273)
90
Solution to Exercise 4.598 (p. 273)
1
Solution to Exercise 4.600 (p. 273)
7
2 or 3 12
Solution to Exercise 4.602 (p. 273)
4
Solution to Exercise 4.604 (p. 273)
12
13
Solution to Exercise 4.606 (p. 273)
4
Solution to Exercise 4.608 (p. 274)
11
12
Solution to Exercise 4.609 (p. 274)

Solution to Exercise 4.610 (p. 274)


Numerator, 5; denominator, 9
Solution to Exercise 4.611 (p. 274)
5
11
Solution to Exercise 4.612 (p. 274)
Four fths
291

Solution to Exercise 4.613 (p. 274)


5
12
Solution to Exercise 4.614 (p. 274)
25
7
Solution to Exercise 4.615 (p. 274)
3 15
Solution to Exercise 4.616 (p. 274)
yes
Solution to Exercise 4.617 (p. 274)
3
5
Solution to Exercise 4.618 (p. 274)
5
17
Solution to Exercise 4.619 (p. 274)
13
60
Solution to Exercise 4.620 (p. 274)
20
Solution to Exercise 4.621 (p. 274)
24
Solution to Exercise 4.622 (p. 274)
3
20
Solution to Exercise 4.623 (p. 275)
55
Solution to Exercise 4.624 (p. 275)
5
6
Solution to Exercise 4.625 (p. 275)
3
4
Solution to Exercise 4.626 (p. 275)
1
2
Solution to Exercise 4.627 (p. 275)
1
30
Solution to Exercise 4.628 (p. 275)
1
11
Solution to Exercise 4.629 (p. 275)
36 = 1 11
25 25
Solution to Exercise 4.630 (p. 275)
2
5
Solution to Exercise 4.631 (p. 275)
1
4
Solution to Exercise 4.632 (p. 275)
9
5 = 1 45
Solution to Exercise 4.633 (p. 275)
4
3 or 1 13
292
CHAPTER 4. INTRODUCTION TO FRACTIONS AND MULTIPLICATION
AND DIVISION OF FRACTIONS
Chapter 5

Addition and Subtraction of Fractions,

Comparing Fractions, and Complex

Fractions

5.1 Objectives1
After completing this chapter, you should
Addition and Subtraction of Fractions with Like Denominators (Section 5.2)

• be able to add and subtract fractions with like denominators

Addition and Subtraction of Fractions with Unlike Denominators (Section 5.3)

• be able to add and subtract fractions with unlike denominators

Addition and Subtraction of Mixed Numbers (Section 5.4)

• be able to add and subtract mixed numbers

Comparing Fractions (Section 5.5)

• understand ordering of numbers and be familiar with grouping symbols


• be able to compare two or more fractions

Complex Fractions (Section 5.6)

• be able to distinguish between simple and complex fractions


• be able to convert a complex fraction to a simple fraction

Combinations of Operations with Fractions (Section 5.7)

• gain a further understanding of the order of operations

1 This content is available online at <http://cnx.org/content/m18893/1.3/>.

293
CHAPTER 5. ADDITION AND SUBTRACTION OF FRACTIONS,
294
COMPARING FRACTIONS, AND COMPLEX FRACTIONS
5.2 Addition and Subtraction of Fractions with Like Denominators2
5.2.1 Section Overview
• Addition of Fraction With Like Denominators
• Subtraction of Fractions With Like Denominators

5.2.2 Addition of Fraction With Like Denominators


Let's examine the following diagram.

2 one-fths and 1 one fth is shaded.

It is shown in the shaded regions of the diagram that

(2 one-fths) + (1 one-fth) = (3 one-fths)

That is,
2 1 3
5 + 5 = 5

From this observation, we can suggest the following rule.

Method of Adding Fractions Having Like Denominators


To add two or more fractions that have the same denominators, add the numerators and place the resulting
sum over the common denominator. Reduce, if necessary.

5.2.2.1 Sample Set A

Find the following sums.


Example 5.1
3
7 + 72 . The denominators are the same. Add the numerators and place that sum over 7.
3 2 3+2 5
7 + 7 = 7 = 7
Example 5.2
1
8 + 83 . The denominators are the same. Add the numerators and place the sum over 8. Reduce.
1 3 1+3 4 1
8 + 8 = 8 = 8 = 2
Example 5.3
4
9 + 95 . The denominators are the same. Add the numerators and place the sum over 9.
4 5 4+5 9
9 + 9 = 9 = 9 =1
Example 5.4
7
8 + 85 . The denominators are the same. Add the numerators and place the sum over 8.
7
8 + 5
8 = 7+5
8 = 12
8 =
3
2
2 This content is available online at <http://cnx.org/content/m34934/1.2/>.
295

Example 5.5
To see what happens if we mistakenly add the denominators as well as the numerators, let's add
1 1
2 + 2

Adding the numerators and mistakenly adding the denominators produces


1 1 1+1 2 1
2 + 2 = 2+2 = 4 = 2

This means that two 12 's is the same as one 12 . Preposterous! We do not add denominators.

5.2.2.2 Practice Set A

Find the following sums.


Exercise 5.1 (Solution on p. 327.)
1 3
10 + 10
Exercise 5.2 (Solution on p. 327.)
1 1
4 + 4
Exercise 5.3 (Solution on p. 327.)
7 4
11 + 11
Exercise 5.4 (Solution on p. 327.)
3 1
5 + 5
Exercise 5.5 (Solution on p. 327.)
Show why adding both the numerators and denominators is preposterous by adding 3
4 and 3
4 and
examining the result.

5.2.3 Subtraction of Fractions With Like Denominators


We can picture the concept of subtraction of fractions in much the same way we pictured addition.

From this observation, we can suggest the following rule for subtracting fractions having like denominators:

Subtraction of Fractions with Like Denominators


To subtract two fractions that have like denominators, subtract the numerators and place the resulting
dierence over the common denominator. Reduce, if possible.

5.2.3.1 Sample Set B

Find the following dierences.


Example 5.6
3
5 − 51 . The denominators are the same. Subtract the numerators. Place the dierence over 5.
3 1 3−1 2
5 − 5 = 5 = 5
CHAPTER 5. ADDITION AND SUBTRACTION OF FRACTIONS,
296
COMPARING FRACTIONS, AND COMPLEX FRACTIONS
Example 5.7
8
6 − 62 . The denominators are the same. Subtract the numerators. Place the dierence over 6.
8 2 8−2 6
6 − 6 = 6 = 6 =1
Example 5.8
16 − 2 . The denominators are the same. Subtract numerators and place the dierence over 9.
9 9
16 − 2
= 169−2 = 14
9 9 9
Example 5.9
To see what happens if we mistakenly subtract the denominators, let's consider
7 4 7−4 3
15 − 15 = 15−15 = 0

We get division by zero, which is undened. We do not subtract denominators.

5.2.3.2 Practice Set B

Find the following dierences.


Exercise 5.6 (Solution on p. 327.)
10 − 8
13 13
Exercise 5.7 (Solution on p. 327.)
51
12 − 12
Exercise 5.8 (Solution on p. 327.)
1 1
2 − 2
Exercise 5.9 (Solution on p. 327.)
26 14
10 − 10
Exercise 5.10 (Solution on p. 327.)
Show why subtracting both the numerators and the denominators is in error by performing the
subtraction 95 − 29 .

5.2.4 Exercises
For the following problems, nd the sums and dierences. Be sure to reduce.
Exercise 5.11 (Solution on p. 327.)
3 2
8 + 8
Exercise 5.12
1 2
6 + 6
Exercise 5.13 (Solution on p. 327.)
9 1
10 + 10
Exercise 5.14
3 4
11 + 11
Exercise 5.15 (Solution on p. 327.)
9 4
15 + 15
Exercise 5.16
3 2
10 + 10
297

Exercise 5.17 (Solution on p. 327.)


5 7
12 + 12
Exercise 5.18
11 − 2
16 16
Exercise 5.19 (Solution on p. 327.)
3 3
16 − 16
Exercise 5.20
15 − 2
23 23
Exercise 5.21 (Solution on p. 327.)
1 1
6 − 6
Exercise 5.22
1 1 1
4 + 4 + 4
Exercise 5.23 (Solution on p. 327.)
3 1 5
11 + 11 + 11
Exercise 5.24
16 + 1 + 2
20 20 20
Exercise 5.25 (Solution on p. 327.)
12 + 2
+ 1
8 8 8
Exercise 5.26
1 8 6
15 + 15 + 15
Exercise 5.27 (Solution on p. 327.)
3
8 + 82 − 18
Exercise 5.28
11 + 9 − 5
16 16 16
Exercise 5.29 (Solution on p. 327.)
4 1 9
20 − 20 + 20
Exercise 5.30
7 3 11
10 − 10 + 10
Exercise 5.31 (Solution on p. 327.)
16 − 1
− 2
5 5 5
Exercise 5.32
21 17 31
35 − 35 + 35
Exercise 5.33 (Solution on p. 327.)
5
2 + 16
2 −
1
2
Exercise 5.34
1 3 1 4 5
18 + 18 + 18 + 18 − 18
Exercise 5.35 (Solution on p. 327.)
6 2 4 1 11
22 − 22 + 22 − 22 + 22
The following rule for addition and subtraction of two fractions is preposterous. Show why by performing
the operations using the rule for the following two problems.

Preposterous Rule
To add or subtract two fractions, simply add or subtract the numerators and place this result over the sum
or dierence of the denominators.
Exercise 5.36
3 3
10 − 10
CHAPTER 5. ADDITION AND SUBTRACTION OF FRACTIONS,
298
COMPARING FRACTIONS, AND COMPLEX FRACTIONS
Exercise 5.37 (Solution on p. 327.)
8 8
15 + 15
Exercise 5.38
Find the total length of the screw.

Exercise 5.39 (Solution on p. 327.)


Two months ago, a woman paid o 24 3
of a loan. One month ago, she paid o 24
5
of the total loan.
This month she will again pay o 24 of the total loan. At the end of the month, how much of her
5

total loan will she have paid o?


Exercise 5.40
Find the inside diameter of the pipe.

5.2.4.1 Exercises for Review

Exercise 5.41 (Solution on p. 327.)


(Section 1.4) Round 2,650 to the nearest hundred.
Exercise 5.42
(Section 2.6) Use the numbers 2, 4, and 8 to illustrate the associative property of addition.
Exercise 5.43 (Solution on p. 328.)
(Section 3.4) Find the prime factors of 495.
Exercise 5.44
(Section 4.5) Find the value of 3
4 · 16
25 · 9 .
5

Exercise 5.45 (Solution on p. 328.)


(Section 4.7) 8
3 of what number is 1 79 ?

5.3 Addition and Subtraction of Fractions with Unlike Denominators3


5.3.1 Section Overview
• A Basic Rule
• Addition and Subtraction of Fractions
3 This content is available online at <http://cnx.org/content/m34935/1.2/>.
299

5.3.2 A Basic Rule


There is a basic rule that must be followed when adding or subtracting fractions.

A Basic Rule
Fractions can only be added or subtracted conveniently if they have like denominators.

To see why this rule makes sense, let's consider the problem of adding a quarter and a dime.

1 quarter + 1 dime = 35 cents

Now,
25
1 quarter = 100
} same denominations
10
1 dime = 100
35
35, = 100
25 10 25+10 35
100 + 100 = 100 = 100
In order to combine a quarter and a dime to produce 35¢, we convert them to quantities of the same
denomination.

Same denomination → same denominator

5.3.3 Addition and Subtraction of Fractions


Least Common Multiple (LCM) and Least Common Denominator (LCD)
In Section 3.6, we examined the least common multiple (LCM) of a collection of numbers. If these numbers
are used as denominators of fractions, we call the least common multiple, the least common denominator
(LCD).

Method of Adding or Subtracting Fractions with Unlike Denominators


To add or subtract fractions having unlike denominators, convert each fraction to an equivalent fraction
having as a denominator the least common denominator ( LCD) of the original denominators.

5.3.3.1 Sample Set A

Find the following sums and dierences.


Example 5.10
1
6 + 43 . The denominators are not the same. Find the LCD of 6 and 4.

6 = 2 ·3
}The LCD = 22 · 3 = 4 · 3 = 12
4 = 22

Write each of the original fractions as a new, equivalent fraction having the common denominator
12.
1 3
6 + 4 = 12 + 12

To nd a new numerator, we divide the original denominator into the LCD. Since the original
denominator is being multiplied by this quotient, we must multiply the original numerator by this
quotient.
CHAPTER 5. ADDITION AND SUBTRACTION OF FRACTIONS,
300
COMPARING FRACTIONS, AND COMPLEX FRACTIONS

12 ÷ 6 = 2

12 ÷ 4 = 3

1 3 1·2 3·3
6 + 4 = 12 + 12
= 2 9
12 + 12 Now the denominators are the same.
= 2+9
12 Add the numerators and place the sum over the common denominator.
= 11
12
Example 5.11
1
2 + 32 . The denominators are not the same. Find the LCD of 2 and 3.

LCD = 2 · 3 = 6

Write each of the original fractions as a new, equivalent fraction having the common denominator
6.
1 2
2 + 3 = 6 + 6

To nd a new numerator, we divide the original denominator into the LCD. Since the original
denominator is being multiplied by this quotient, we must multiply the original numerator by this
quotient.

6 ÷ 2 = 3 Multiply the numerator 1 by 3.

6 ÷ 2 = 3 Multiply the numerator 2 by 2.


1 2 1·3 2·3
2 + 3 = 6 + 6
3 4
= 6 + 6
3+4
= 6
= 7
6 or 1 61
Example 5.12
5
9
5
− 12 . The denominators are not the same. Find the LCD of 9 and 12.

9 = 3 · 3 = 32
} LCD = 22 · 32 = 4 · 9 = 36
12 = 2 · 6 = 2 · 2 · 3 = 22 · 3
301

5 5
9 − 12 = 36 − 36

36 ÷ 9 = 4 Multiply the numerator 5 by 4.

36 ÷ 12 = 3 Multiply the numerator 5 by 3.


5 5 5·4 5·3
9 − 12 = 36 − 36
20 15
= 36 − 36
= 20−15
36
5
= 36
Example 5.13
5
6 − 1
8
7
+ 16 The denominators are not the same. Find the LCD of 6, 8, and 16

6 = 2·3
8 = 2 · 4 = 2 · 2 · 2 = 23 } The LCD is 24 · 3 = 48
16 = 2 · 8 = 2 · 2 · 4 = 2 · 2 · 2 · 2 = 24
5 1 7
6 − 8 + 16 = 48 − 48 + 48

48 ÷ 6 = 8 Multiply the numerator 5 by 8

48 ÷ 8 = 6 Multiply the numerator 1 by 6

48 ÷ 16 = 3 Multiply the numerator 7 by 3


5 1 7 5·8 1·6 7·3
6 − 8 + 16 = 48 − 48 + 48
40 6 21
= 48 − 48 + 48
= 40−6+21
48
= 55 or 1 7
48 48

5.3.3.2 Practice Set A

Find the following sums and dierences.


Exercise 5.46 (Solution on p. 328.)
3 1
4 + 12
Exercise 5.47 (Solution on p. 328.)
1 3
2 − 7
Exercise 5.48 (Solution on p. 328.)
7 5
10 − 8
Exercise 5.49 (Solution on p. 328.)
15 1
− 3
16 + 2 4
Exercise 5.50 (Solution on p. 328.)
1 1
32 − 48
CHAPTER 5. ADDITION AND SUBTRACTION OF FRACTIONS,
302
COMPARING FRACTIONS, AND COMPLEX FRACTIONS
5.3.4 Exercises
Exercise 5.51 (Solution on p. 328.)
A most basic rule of arithmetic states that two fractions may be added or subtracted conveniently
only if they have .
For the following problems, nd the sums and dierences.
Exercise 5.52
1 1
2 + 6
Exercise 5.53 (Solution on p. 328.)
1 1
8 + 2
Exercise 5.54
3 1
4 + 3
Exercise 5.55 (Solution on p. 328.)
5 2
8 + 3
Exercise 5.56
1 1
12 + 3
Exercise 5.57 (Solution on p. 328.)
6 1
7 − 4
Exercise 5.58
9 2
10 − 5
Exercise 5.59 (Solution on p. 328.)
7 1
9 − 4
Exercise 5.60
8 3
15 − 10
Exercise 5.61 (Solution on p. 328.)
8 5
13 − 39
Exercise 5.62
11 − 2
12 5
Exercise 5.63 (Solution on p. 328.)
1 5
15 + 12
Exercise 5.64
13 − 1
88 4
Exercise 5.65 (Solution on p. 328.)
1 1
9 − 81
Exercise 5.66
19 + 5
40 12
Exercise 5.67 (Solution on p. 328.)
25 7
26 − 10
Exercise 5.68
9 4
28 − 45
Exercise 5.69 (Solution on p. 328.)
22 16
45 − 35
Exercise 5.70
56 + 22
63 33
Exercise 5.71 (Solution on p. 328.)
1 3 3
16 + 4 − 8
303

Exercise 5.72
5 1 19
12 − 120 + 20
Exercise 5.73 (Solution on p. 328.)
8 1 7
3 − 4 + 36
Exercise 5.74
11 − 1
+ 16
9 7 63
Exercise 5.75 (Solution on p. 328.)
12 − 2
+ 17
5 3 10
Exercise 5.76
4 13 − 9
+ 21
9 14
Exercise 5.77 (Solution on p. 328.)
3 3 5
4 − 22 + 24
Exercise 5.78
25 7 5
48 − 88 + 24
Exercise 5.79 (Solution on p. 328.)
27 47 119
40 + 48 − 126
Exercise 5.80
41 − 5 − 11
44 99 175
Exercise 5.81 (Solution on p. 328.)
5 1 1
12 + 18 + 24
Exercise 5.82
5 1 7
9 + 6 + 15
Exercise 5.83 (Solution on p. 328.)
21 1 7
25 + 6 + 15
Exercise 5.84
5 1 7
18 − 36 + 9
Exercise 5.85 (Solution on p. 328.)
11 1 1
14 − 36 − 32
Exercise 5.86
21 + 12 + 15
33 22 55
Exercise 5.87 (Solution on p. 329.)
5 2 11
51 + 34 + 68
Exercise 5.88
8 16 + 19
− 14
7 21
Exercise 5.89 (Solution on p. 329.)
7 3 34
15 + 10 − 60
Exercise 5.90
14 − 3 − 6 + 7
15 10 25 20
Exercise 5.91 (Solution on p. 329.)
11 − 5 + 17 + 25
6 12 30 18
Exercise 5.92
1 22 − 5 − 1
+ 21
9 18 45
Exercise 5.93 (Solution on p. 329.)
7 28 51
26 + 65 − 104 + 0
CHAPTER 5. ADDITION AND SUBTRACTION OF FRACTIONS,
304
COMPARING FRACTIONS, AND COMPLEX FRACTIONS
Exercise 5.94
13 hours. The return trip took 57 hours.
A morning trip from San Francisco to Los Angeles took 12 60
How much longer did the morning trip take?
Exercise 5.95 (Solution on p. 329.)
At the beginning of the week, Starlight Publishing Company's stock was selling for 115
8 dollars per
share. At the end of the week, analysts had noted that the stock had gone up 11
4 dollars per share.
What was the price of the stock, per share, at the end of the week?
Exercise 5.96
A recipe for fruit punch calls for 23 15
3 cups of pineapple juice, 4 cup of lemon juice, 2 cups of orange
1

juice, 2 cups of sugar, 6 cups of water, and 8 cups of carbonated non-cola soft drink. How many
cups of ingredients will be in the nal mixture?
Exercise 5.97 (Solution on p. 329.)
The side of a particular type of box measures 8 34 inches in length. Is it possible to place three such
boxes next to each other on a shelf that is 26 51 inches in length? Why or why not?
Exercise 5.98
Four resistors, 38 ohm, 14 ohm, 53 ohm, and 87 ohm, are connected in series in an electrical circuit.
What is the total resistance in the circuit due to these resistors? ("In series" implies addition.)
Exercise 5.99 (Solution on p. 329.)
A copper pipe has an inside diameter of 2 16
3
inches and an outside diameter of 2 34
5
inches. How
thick is the pipe?
Exercise 5.100
The probability of an event was originally thought to be 15
32 . Additional information decreased the
probability by 14
3
. What is the updated probability?

5.3.4.1 Exercises for Review

Exercise 5.101 (Solution on p. 329.)


(Section 1.6) Find the dierence between 867 and 418.
Exercise 5.102
(Section 2.5) Is 81,147 divisible by 3?
Exercise 5.103 (Solution on p. 329.)
(Section 3.6) Find the LCM of 11, 15, and 20.
Exercise 5.104
(Section 4.5) Find 3
4 of 4 29 .
Exercise 5.105 (Solution on p. 329.)
(Section 5.2) Find the value of 15
8 3
− 15 2
+ 15 .

5.4 Addition and Subtraction of Mixed Numbers4


5.4.1 Section Overview
• The Method of Converting to Improper Fractions

To add or subtract mixed numbers, convert each mixed number to an improper fraction, then add or subtract
the resulting improper fractions.
4 This content is available online at <http://cnx.org/content/m34936/1.2/>.
305

5.4.1.1 Sample Set A

Find the following sums and dierences.


Example 5.14
8 35 + 5 14 . Convert each mixed number to an improper fraction.

8 53 = 5·8+3
5 = 405+3 = 43
5

5 41 = 4·5+1
4 = 204+1 = 21 43 21
4 Now add the improper fractions 5 and 4 .
43
5 + 21
4 The LCD = 20.

43 + 21 = 43·4 + 21·5
5 4 20 20
= 172 + 105
20 20
= 172+105
20
= 277 Convert this improper fraction to a mixed number.
20
= 13 17
20

Thus, 8 53 + 5 14 = 13 17
20 .
Example 5.15
3 18 − 56 . Convert the mixed number to an improper fraction.

3 81 = 3·8+1
8 = 248+1 = 25
8
25 − 5
The LCD = 24.
8 6

25 − 5
= 25·3 − 5·4
8 6 24 24
= 75 − 20
24 24
= 75−20
24
= 55 Convert his improper fraction to a mixed number.
24
7
= 2 24

Thus, 3 18 − 5
6
7
= 2 24 .

5.4.1.2 Practice Set A

Find the following sums and dierences.


Exercise 5.106 (Solution on p. 329.)
1 59 + 3 29
Exercise 5.107 (Solution on p. 329.)
10 34 − 2 21
Exercise 5.108 (Solution on p. 329.)
2 78 + 5 14
Exercise 5.109 (Solution on p. 329.)
8 35 − 10
3

Exercise 5.110 (Solution on p. 329.)


16 + 2 16
9
CHAPTER 5. ADDITION AND SUBTRACTION OF FRACTIONS,
306
COMPARING FRACTIONS, AND COMPLEX FRACTIONS

5.4.2 Exercises
For the following problems, perform each indicated operation.
Exercise 5.111 (Solution on p. 329.)
3 18 + 4 38
Exercise 5.112
5 31 + 6 13
Exercise 5.113 (Solution on p. 329.)
10 12
5 1
+ 2 12
Exercise 5.114
15 15 − 11 35
Exercise 5.115 (Solution on p. 329.)
3
9 11 + 12 11
3

Exercise 5.116
1 16 + 3 26 + 8 16
Exercise 5.117 (Solution on p. 329.)
5 38 + 1 18 − 2 58
Exercise 5.118
3
5 + 5 15
Exercise 5.119 (Solution on p. 329.)
2 29 − 5
9
Exercise 5.120
6 + 11 32
Exercise 5.121 (Solution on p. 329.)
17 − 8 14
3

Exercise 5.122
5 13 + 2 14
Exercise 5.123 (Solution on p. 329.)
6 27 − 1 13
Exercise 5.124
8 52 + 4 10
1

Exercise 5.125 (Solution on p. 329.)


1 13 + 12 83
Exercise 5.126
3 14 + 1 13 − 2 12
Exercise 5.127 (Solution on p. 329.)
4 34 − 3 56 + 1 23
Exercise 5.128
1
3 12 + 4 31 + 1 14
Exercise 5.129 (Solution on p. 329.)
1 3
5 15 + 8 10 − 5 45
Exercise 5.130
7 13 + 8 56 − 2 14
307

Exercise 5.131 (Solution on p. 330.)


19 20
21 + 42 7 − 14 + 12 7
6 5 1

Exercise 5.132
16 + 4 4 + 10 8 − 9
1 3 3

Exercise 5.133 (Solution on p. 330.)


11 − 2
9 + 10 13 − 2
3 − 5 16 + 6 18
1

Exercise 5.134
5
2 + 2 16 + 11 13 − 11
6
Exercise 5.135 (Solution on p. 330.)
1 18 + 9
4
1
− 16 1
− 32 + 19
8
Exercise 5.136
22 83 − 16 17
Exercise 5.137 (Solution on p. 330.)
15 49 + 4 16
9

Exercise 5.138
4 17 9
88 + 5 110
Exercise 5.139 (Solution on p. 330.)
6 11
12 +
2
3
Exercise 5.140
9 7
8 16 − 9
Exercise 5.141 (Solution on p. 330.)
2 1
5 11 − 12
Exercise 5.142
18 15 33
16 − 34
Exercise 5.143 (Solution on p. 330.)
89 − 21
1 112 56
Exercise 5.144
11 11 13
24 − 7 18
Exercise 5.145 (Solution on p. 330.)
5 27 5 1
84 − 3 42 + 1 21
Exercise 5.146
16 48
1
− 16 96
1 1
+ 144
Exercise 5.147 (Solution on p. 330.)
A man pours 2 85 gallons of paint from a bucket into a tray. After he nishes pouring, there are 1 14
gallons of paint left in his bucket. How much paint did the man pour into the tray?
Hint: Think about the wording.

Exercise 5.148
A particular computer stock opened at 37 83 and closed at 38 14 . What was the net gain for this
stock?
Exercise 5.149 (Solution on p. 330.)
A particular diet program claims that 4 16
3
pounds can be lost the rst month, 3 14 pounds can be
lost the second month, and 1 2 pounds can be lost the third month. How many pounds does this
1

diet program claim a person can lose over a 3-month period?


Exercise 5.150
If a person who weighs 145 43 pounds goes on the diet program described in the problem above,
how much would he weigh at the end of 3 months?
CHAPTER 5. ADDITION AND SUBTRACTION OF FRACTIONS,
308
COMPARING FRACTIONS, AND COMPLEX FRACTIONS
Exercise 5.151 (Solution on p. 330.)
If the diet program described in the problem above makes the additional claim that from the fourth
month on, a person will lose 1 81 pounds a month, how much will a person who begins the program
weighing 208 43 pounds weight after 8 months?

5.4.2.1 Exercises for Review

Exercise 5.152
(Section 3.2) Use exponents to write 4 · 4 · 4.
Exercise 5.153 (Solution on p. 330.)
(Section 3.5) Find the greatest common factor of 14 and 20.
Exercise 5.154
(Section 4.3) Convert 16
5 to a mixed number.
Exercise 5.155 (Solution on p. 330.)
(Section 5.2) Find the sum. 4
9 + 1
9 + 92 .
Exercise 5.156
(Section 5.3) Find the dierence. 15
26 − 10 .
3

5.5 Comparing Fractions5


5.5.1 Section Overview
• Order and the Inequality Symbols
• Comparing Fractions

5.5.2 Order and the Inequality Symbols


Our number system is called an ordered number system because the numbers in the system can be placed
in order from smaller to larger. This is easily seen on the number line.

On the number line, a number that appears to the right of another number is larger than that other number.
For example, 5 is greater than 2 because 5 is located to the right of 2 on the number line. We may also say
that 2 is less than 5.

To make the inequality phrases "greater than" and "less than" more brief, mathematicians represent them
with the symbols > and <, respectively.

Symbols for Greater Than > and Less Than <


> represents the phrase "greater than."
< represents the phrase "less than."

5 > 2 represents "5 is greater than 2."

2 < 5 represents "2 is less than 5."


5 This content is available online at <http://cnx.org/content/m34937/1.3/>.
309

5.5.3 Comparing Fractions


Recall that the fraction 45 indicates that we have 4 of 5 parts of some whole quantity, and the fraction 35
indicates that we have 3 of 5 parts. Since 4 of 5 parts is more than 3 of 5 parts, 45 is greater than 35 ; that is,
4 3
5 > 5

We have just observed that when two fractions have the same denominator, we can determine which is larger
by comparing the numerators.

Comparing Fractions
If two fractions have the same denominators, the fraction with the larger numerator is the larger fraction.

Thus, to compare the sizes of two or more fractions, we need only convert each of them to equivalent
fractions that have a common denominator. We then compare the numerators. It is convenient if the
common denominator is the LCD. The fraction with the larger numerator is the larger fraction.

5.5.3.1 Sample Set A

Example 5.16
Compare 8
9 and 14
15 .
Convert each fraction to an equivalent fraction with the LCD as the denominator. Find the LCD.

9 = 32
} The LCD = 32 · 5 = 9 · 5 = 45
15 = 3·5

8
= 8·5 40
9 45 = 45
14 = 14·3 = 42
15 45 45

Since 40 < 42,


40 < 42
45 45
Thus 8
9 < 14
15 .
Example 5.17
Write 56 , 10
7
, and 13
15 in order from smallest to largest.
Convert each fraction to an equivalent fraction with the LCD as the denominator.

Find the LCD.

6=2·3
10 = 2 · 5 }The LCD = 2 · 3 · 5 = 30
15 = 3 · 5
5
= 5·5 25
6 30 = 30
7 7·3 21
10 = 30 = 30
13 = 13·2 = 26
15 30 30
Since 21 < 25 < 26,
CHAPTER 5. ADDITION AND SUBTRACTION OF FRACTIONS,
310
COMPARING FRACTIONS, AND COMPLEX FRACTIONS
21 25 26
30 < 30 < 30
7
10 <
5
6 < 13
15
Writing these numbers in order from smallest to largest, we get 10
7
, 65 , 13
15 .
Example 5.18
Compare 8 76 and 6 34 .

To compare mixed numbers that have dierent whole number parts, we need only compare whole
number parts. Since 6 < 8,

6 43 < 8 67
Example 5.19
Compare 4 58 and 4 12
7

To compare mixed numbers that have the same whole number parts, we need only compare frac-
tional parts.

8 = 23
}The LCD = 23 · 3 = 8 · 3 = 24
12 = 22 · 3
5
= 5·3 15
8 24 = 24
7 7·2 14
12 = 24 = 24
Since 14 < 15,
14 < 15
24 24
7 5
12 < 8

Hence, 4 12
7
< 4 58

5.5.3.2 Practice Set A

Exercise 5.157 (Solution on p. 330.)


Compare 3
4 and 45 .
Exercise 5.158 (Solution on p. 330.)
Compare 10
9
and 13
15 .
Exercise 5.159 (Solution on p. 330.)
13 , 17 , and 33 in order from smallest to largest.
Write 16 20 40
Exercise 5.160 (Solution on p. 330.)
Compare 11 16 and 9 25 .
Exercise 5.161 (Solution on p. 330.)
Compare 1 14
9
and 1 11
16 .
311

5.5.4 Exercises
Arrange each collection of numbers in order from smallest to largest.
Exercise 5.162 (Solution on p. 330.)
3 5
5, 8
Exercise 5.163
1 2
6, 7
Exercise 5.164 (Solution on p. 330.)
3 5
4, 6
Exercise 5.165
7 11
9, 12
Exercise 5.166 (Solution on p. 330.)
3 2
8, 5
Exercise 5.167
1 5 7
2, 8, 16
Exercise 5.168 (Solution on p. 330.)
1 3 4
2, 5, 7
Exercise 5.169
3 2 5
4, 3, 6
Exercise 5.170 (Solution on p. 330.)
3 7 5
4, 9, 4
Exercise 5.171
7 15 11
8, 16 , 12
Exercise 5.172 (Solution on p. 330.)
3 2 3
14 , 7, 4
Exercise 5.173
17 , 25 , 13
32 48 16
Exercise 5.174 (Solution on p. 330.)
5 35 , 5 47
Exercise 5.175
11 16
3
, 11 12
1

Exercise 5.176 (Solution on p. 331.)


9 23 , 9 45
Exercise 5.177
7 32 , 8 56
Exercise 5.178 (Solution on p. 331.)
9 1
1 16 , 2 20
Exercise 5.179
15 , 20 23
20 16 24
Exercise 5.180 (Solution on p. 331.)
2 29 , 2 37
Exercise 5.181
8 9
5 13 , 5 20
CHAPTER 5. ADDITION AND SUBTRACTION OF FRACTIONS,
312
COMPARING FRACTIONS, AND COMPLEX FRACTIONS
5.5.4.1 Exercises for Review

Exercise 5.182 (Solution on p. 331.)


(Section 1.4) Round 267,006,428 to the nearest ten million.
Exercise 5.183
(Section 2.5) Is the number 82,644 divisible by 2? by 3? by 4?
Exercise 5.184 (Solution on p. 331.)
(Section 4.3) Convert 3 27 to an improper fraction.
Exercise 5.185
(Section 5.3) Find the value of 5
6
3
+ 10 − 2
5
Exercise 5.186 (Solution on p. 331.)
(Section 5.4) Find the value of 8 38 + 5 41 .

5.6 Complex Fractions 6


5.6.1 Section Overview
• Simple Fractions and Complex Fractions
• Converting Complex Fractions to Simple Fractions

5.6.2 Simple Fractions and Complex Fractions


Simple Fraction
A simple fraction is any fraction in which the numerator is any whole number and the denominator is any
nonzero whole number. Some examples are the following:
1
, 4
, 763
2 3 1,000

Complex Fraction
A complex fraction is any fraction in which the numerator and/or the denominator is a fraction; it is a
fraction of fractions. Some examples of complex fractions are the following:
3 1
6 4+ 38
4
5 , 3
2 , 9 , 7− 65
6 10

5.6.3 Converting Complex Fractions to Simple Fractions


The goal here is to convert a complex fraction to a simple fraction. We can do so by employing the methods
of adding, subtracting, multiplying, and dividing fractions. Recall from Section 4.2 that a fraction bar
serves as a grouping symbol separating the fractional quantity into two individual groups. We proceed in
simplifying a complex fraction to a simple fraction by simplifying the numerator and the denominator of the
complex fraction separately. We will simplify the numerator and denominator completely before removing
the fraction bar by dividing. This technique is illustrated in problems 3, 4, 5, and 6 of Section 5.6.3.1 (Sample
Set A).
6 This content is available online at <http://cnx.org/content/m34941/1.2/>.
313

5.6.3.1 Sample Set A

Convert each of the following complex fractions to a simple fraction.


Example 5.20
3
8
15
16

Convert this complex fraction to a simple fraction by performing the indicated division.

÷ 15 The divisor is 15 15
3

16 .Invert 16 and multiply.


3
8
15 = 8 16
16
1 2
)3 )16 1·2 2
= · = 1·5 = 5
)8 )15
1 5

Example 5.21
4
9
6 Write 6 as 61 and divide.
4
4 6
9
6 = 9 ÷ 1
1
2
)4 1 2·1 2
= 9 · = 9·3 = 27
)6
3

Example 5.22
5+ 43
46 Simplify the numerator.

Write 46 as 46
4·5+3 20+3 23
4
46 = 46
4
= 46
4
1 .

23
4
= 23 ÷ 46
46
1
4 1
1
)23 1 1·1 1
= 4 · = 4·2 = 8
)46
2

Example 5.23
÷ 25
1 3 2 3 2+3 5
4+8 8+8 5
1
+ 13 = 12 13
+ 24
= 12+13
8
= 8
25 = 8 24
2 24 24 24 24

1 3
5
8 ÷ 25
24 =
)5
· )24
= 1·3
1·5 = 3
5
)8 )25
1 5

Example 5.24
29 ÷ 20
4·6+5 29
4+ 56
7− 13
= 6
7·3−1 = 20
6
= 6 3
3 3
1

= 29 · )3 = 29
)6 20 40
2

Example 5.25
11+ 10
= 113 24
3 11·10+3 110+3 113

= = = 10 ÷ 5
10 10 10
20+4 24
4 45 4·5+4
5 5 5

1
113 ÷ 24 = 113 · )5 = 113·1 = 113 = 2 17
10 5 )10 24 2·24 48 48
2
CHAPTER 5. ADDITION AND SUBTRACTION OF FRACTIONS,
314
COMPARING FRACTIONS, AND COMPLEX FRACTIONS
5.6.3.2 Practice Set A

Convert each of the following complex fractions to a simple fraction.


Exercise 5.187 (Solution on p. 331.)
4
9
8
15

Exercise 5.188 (Solution on p. 331.)


7
10
28
Exercise 5.189 (Solution on p. 331.)
5+ 52
3+ 35
Exercise 5.190 (Solution on p. 331.)
1 7
8+8
3
6− 10
Exercise 5.191 (Solution on p. 331.)
1 5
6+8
5 1
9−4

Exercise 5.192 (Solution on p. 331.)


16−10 32
11 56 −7 67

5.6.4 Exercises
Simplify each fraction.
Exercise 5.193 (Solution on p. 331.)
3
5
9
15

Exercise
1
5.194
3
1
9

Exercise 5.195 (Solution on p. 331.)


1
4
5
12

Exercise
8
5.196
9
4
15

Exercise 5.197 (Solution on p. 331.)


6+ 41
11+ 14
Exercise
1
5.198
2+ 2
7+ 12
Exercise 5.199 (Solution on p. 331.)
5+ 13
2
2+ 15
Exercise
1
5.200
9+ 2
8
1+ 11
Exercise 5.201 (Solution on p. 331.)
10
4+ 13
12
39

Exercise
1 2
5.202
3+7
26
21

Exercise 5.203 (Solution on p. 331.)


5 1
6−4
1
12
315

Exercise
3 4
5.204
10
+ 12
19
90

Exercise 5.205 (Solution on p. 331.)


9
16
+7
3
139
48

Exercise
1
5.206
288
8 3
9 − 16

Exercise 5.207 (Solution on p. 331.)


27
429
5 1
11 − 13

Exercise
1 2
5.208
3+5
3 17
5 + 45

Exercise 5.209 (Solution on p. 331.)


9 5
70
+ 42
13 1
30
− 21
Exercise
1 1
5.210
16
+ 14
2 13
3 − 60

Exercise 5.211 (Solution on p. 331.)


3
20
+ 11
12
19
7 −1 11
35

Exercise
2 1
5.212
2 3 −1 2
1 1
4 +1 16

Exercise 5.213 (Solution on p. 331.)


3 51 +3 13
6 15
5 − 63

Exercise 5.214
1 1 +15
2
5 1 −3 5
4 12
8 −4 1
1
3 2
11 2 −5 11
3 12

Exercise 5.215 (Solution on p. 331.)


5 3 +3 1
4 5
2 1 +15 7
5 10
9 −4 1
1
2 6
1 +2 1
8 120

5.6.4.1 Exercises for Review

Exercise 5.216
(Section 3.4) Find the prime factorization of 882.
Exercise 5.217 (Solution on p. 331.)
(Section 4.3) Convert 62
7 to a mixed number.
Exercise 5.218
(Section 4.4) Reduce 114
342 to lowest terms.
Exercise 5.219 (Solution on p. 332.)
(Section 5.4) Find the value of 6 38 − 4 65 .
Exercise 5.220
(Section 5.5) Arrange from smallest to largest: 2, 5, 7.
1 3 4
CHAPTER 5. ADDITION AND SUBTRACTION OF FRACTIONS,
316
COMPARING FRACTIONS, AND COMPLEX FRACTIONS
5.7 Combinations of Operations with Fractions 7
5.7.1 Section Overview
• The Order of Operations

5.7.2 The Order of Operations


To determine the value of a quantity such as
1 5 2
2 + 8 · 15

where we have a combination of operations (more than one operation occurs), we must use the accepted
order of operations.
The Order of Operations:

1. In the order (2), (3), (4) described below, perform all operations inside grouping symbols: ( ), [ ], (
), . Work from the innermost set to the outermost set.
2. Perform exponential and root operations.
3. Perform all multiplications and divisions moving left to right.
4. Perform all additions and subtractions moving left to right.

5.7.2.1 Sample Set A

Determine the value of each of the following quantities.


Example 5.26
1 5 2
4 + 8 · 15

(a) Multiply rst.


1 1
1 )5 )2 1 1·1 1 1
+ · = + = + 12
4 )8 )15 4 4·3 4
4 3

(b) Now perform this addition. Find the LCD.


4 = 22
} The LCD = 22 · 3 = 12.
12 = 22 · 3

1 1 1·3 1 3 1
4 + 12 = 12 + 12 = 12 + 12
3+1 4 1
= 12 = 12 = 3

Thus, 1
4 + 5
8
2
· 15 = 1
3

Example 5.27
3 9 5 1

5 + 44 9 − 4

(a) Operate within the parentheses rst, 5 1


.

9 − 4
9=3 2
} The LCD = 22 · 32 = 4 · 9 = 36.
4 = 22
7 This content is available online at <http://cnx.org/content/m34942/1.3/>.
317

5·4 1·9 20 9 20−9 11


36 − 36 = 36 − 36 = 36 = 36
Now we have
3 9 11 
5 + 44 36
(b) Perform the multiplication.
1 1
3 )9 )11 3 1·1 3 1
+ · = + = + 16
5 )44 )36 5 4·4 5
4 4
(c) Now perform the addition. The LCD=80.
3 1 3·16 1·5 48 + 5 = 48+5 = 53
5 + 16 = 80 + 80 =  80 53 80 80 80
Thus, 5 + 44 9 − 4 = 80
3 9 5 1

Example 5.28
15 2 − 1 4  3 1 + 2 1 
8 − 426 15 5 8

(a) Work within each set of parentheses individually.


4
2 − 1 15 = 2 1·15 +4 19
15 = 2 − 15
= 30 − 19 = 30−19 = 11
15 15 15 15
3 15 + 2 81 = 3·5+1
5 + 2·8+1
8
= 16 + 17 LCD = 40
5 8
= 16·8 + 17·5
40 40
= 128 + 85
40 40
= 128+85
40
= 213
40
Now we have
15 11  213 
8 − 426 15 40
(b) Now multiply.
1 1

8 − )15 · 11 · )213 1·11·1 11


40 = 8 − 2·1·40 = 8 − 80
)426 )15
2 1
(c) Now subtract.
8 − 11 80·8 11
80 = 80 − 80 = 80
640 − 11 = 640−11 = 629 or 7 69
80  80 80 80
Thus, 8 − 426 2 − 1 15 3 51 + 2 18 = 7 69
15 4

80

Example 5.29
3 2 8 5

4 · 9 − 12

(a) Square 34 .
3 2
= 34 · 3 3·3 9

4 4 = 4·4 = 16

Now we have
9 5 8
16 ·
− 12 9
(b) Perform the multiplication.
1 1
)9 )8 5 1·1 5 1 5
· − 12 = − 12 = − 12
)16 )9 2·1 2
2 1
(c) Now perform the subtraction.
1 5 6 5 6−5 1
2 − 12 = 12 − 12 = 12 = 12

4 2
Thus, 8 5 1

3 · 9 − 12 = 12
CHAPTER 5. ADDITION AND SUBTRACTION OF FRACTIONS,
318
COMPARING FRACTIONS, AND COMPLEX FRACTIONS
Example
q 5.30
2 87 + 25 ÷ 2 1 − 1 1 
36 2 3

(a) Begin by operating inside the parentheses.


2 12 − 1 13 = 2·2+1
2 − 1·3+1
3 = 5
2 − 4
3
= 15 − 8 = 15−8 = 7
6 6 6 6
(b) Now simplify the square root.
25 = 5 since 5 2 = 25
q  
36 6 6 36

Now we have

2 78 + 56 ÷ 76
(c) Perform the division.
1
)6
2 87 + 5
· 7 = 2 78 + 5·1
1·7 = 2 87 + 5
7
)6
1
(d) Now perform the addition.
2 78 + 5
= 2·8+7
+ 5 23 5
LCD = 56.
7 8 7 = 8 + 7
= 23·7 5·8 161 40
56 + 56 = 56 + 56
= 161+40 = 201 or 3 33
56 56 56

25 ÷ 2 1 − 1 1  = 3 33
q
Thus, 2 78 + 36 2 3 56

5.7.2.2 Practice Set A

Find the value of each of the following quantities.


Exercise 5.221 (Solution on p. 332.)
5 1 1
16 · 10 − 32
Exercise 5.222 (Solution on p. 332.)
6 21 ÷ 9 + 5 1
· 40
7 10 3
Exercise 5.223 (Solution on p. 332.)
7
− 2 4 12 − 3 23

8 10
Exercise 5.224 (Solution on p. 332.)
17 58 1 3
1 − 13

18 − 30 4 − 32 29
Exercise 
5.225 (Solution on p. 332.)
1 1 4 6

10 + 1 2 ÷ 1 5 − 1 25
Exercise 5.226 (Solution on p. 332.)
2 3 4
3−8·9
7 1 1
16
·1 3 +1 4

Exercise 5.227 (Solution on p. 332.)


3 2 3 1

8 + 4 · 8
Exercise q
5.228 (Solution on p. 332.)
2
3 · 2 41 − 4
25
319

5.7.3 Exercises
Find each value.
Exercise 5.229 (Solution on p. 332.)
4 1 1
3 − 6 · 2
Exercise 5.230
7 4 5
9 − 5 · 36
Exercise 5.231 (Solution on p. 332.)
2 27 + 5
8
5
÷ 16
Exercise 5.232
3 9 12 5
16 ÷ 14 · 21 + 6
Exercise 5.233 (Solution on p. 332.)
4 8 7 1
25 ÷ 15 − 20 ÷ 2 10
Exercise 5.234
2 1 3

5 · 19 + 38
Exercise 5.235 (Solution on p. 332.)
3 3 1

7 · 10 − 15
Exercise 5.236
10 ·  8
− 2 3
+ 25 · 5
+ 1

11 9 5 3 4
Exercise 5.237 (Solution on p. 332.)
2 6 3
+ 5 13 · 1 14 − 1
 
7 · 7 − 28 8
Exercise 5.238
( 11
6
− 13 )·( 21
1
+2 13
42 )
1 15 + 40
7

Exercise 5.239 (Solution on p. 332.)


1 2 1

2 + 8
Exercise 5.240
3 2 3

5 − 10
Exercise 5.241 (Solution on p. 332.)
36 +
q
1 2
81 3 · 9
Exerciseq5.242
49 −
q
9
64 4
Exercise 5.243 (Solution on p. 332.)
− 15 16
q q
2 9
3 · 4 4 · 225
Exerciseq5.244
3 2

+ 25
4 16
Exercise
q 5.245 (Solution on p. 332.)
1 2

· 81 + 1 ÷ 1
3 25 40 8
Exercise 5.246
q 2
4 3
49 + 7 ÷ 1 34
Exercise 5.247 (Solution on p. 332.)
2
100 21
q
121 + (11 )2
Exercise
q 5.248
3 1 1
8 + 64 − 2 ÷ 1 13
CHAPTER 5. ADDITION AND SUBTRACTION OF FRACTIONS,
320
COMPARING FRACTIONS, AND COMPLEX FRACTIONS
Exercise
q 5.249 q (Solution on p. 332.)
1 5 2 9
2 13 1

4 · 6 + 14 · − 81
Exercise
q q 5.250
1 6 38 +2 85 7
9 · 16 + 7 10
Exercise 5.251 (Solution on p. 332.)
3
3 43 + 45 ·( 12 )
67 4
240
+( 13 ) ·( 10
9
)
Exercise
r 5.252
16 +
q
1
81 4 ·6
Exercise
r 5.253 (Solution on p. 332.)
81
q
3 1
256 − 32 · 1 8

5.7.3.1 Exercises for Review

Exercise 5.254
(Section 1.2) True or false: Our number system, the Hindu-Arabic number system, is a positional
number system with base ten.
Exercise 5.255 (Solution on p. 332.)
(Section 2.6) The fact that 1 times any whole number = that particular whole number illustrates
which property of multiplication?
Exercise 5.256
(Section 4.3) Convert 8 76 to an improper fraction.
Exercise 5.257 (Solution on p. 332.)
(Section 5.3) Find the sum. 3
8 + 4
5 + 65 .
Exercise 5.258
6+ 18
(Section 5.6) Simplify 6− 81
.

5.8 Summary of Key Concepts 8


5.8.1 Summary of Key Concepts
Addition and Subtraction of Fractions with Like Denominators (Section 5.2)
To add or subtract two fractions that have the same denominators, add or subtract the numerators and
place the resulting sum or dierence over the common denominator. Reduce, if necessary. Do not add or
subtract the denominators.
1 5 1+5 6 3
8 + 8 = 8 = 8 = 4

Basic Rule for Adding and Subtracting Fractions (Section 5.3)


Fractions can be added or subtracted conveniently only if they have like denominators.

Addition and Subtraction of Fractions with Unlike Denominators (Section 5.3)


To add or subtract fractions having unlike denominators, convert each fraction to an equivalent fraction
having as denominator the LCD of the original denominators.
8 This content is available online at <http://cnx.org/content/m34943/1.2/>.
321

Addition and Subtraction of Mixed Numbers (Section 5.4)


1. To add or subtract mixed numbers, convert each mixed number to an improper fraction, then add or
subtract the fractions.

Ordered Number System (Section 5.5)


Our number system is ordered because the numbers in the system can be placed in order from smaller to
larger.

Inequality Symbols (Section 5.5)


> represents the phrase "greater than."
< represents the phrase "less than."

Comparing Fractions (Section 5.5)


If two fractions have the same denominators, the fraction with the larger numerator is the larger fraction.
5 3
8 > 8

Simple Fractions (Section 5.6)


A simple fraction is any fraction in which the numerator is any whole number and the denominator is any
nonzero whole number.

Complex Fractions (Section 5.6)


A complex fraction is any fraction in which the numerator and/or the denominator is a fraction.

Complex fractions can be converted to simple fractions by employing the methods of adding, subtracting,
multiplying, and dividing fractions.

5.9 Exercise Supplement 9


5.9.1 Exercise Supplement
5.9.1.1 Addition and Subtractions of Fractions with Like and Unlike Denominators, and Ad-
dition and Subtraction of Mixed Numbers (Section 5.2, Section 5.3, Section 5.4)

For problems 1-53, perform each indicated operation and write the result in simplest form.
Exercise 5.259 (Solution on p. 332.)
3 5
4 + 8
Exercise 5.260
9 1
16 + 4
Exercise 5.261 (Solution on p. 333.)
1 3
8 + 8
Exercise 5.262
5 1 5
7 + 14 + 21
Exercise 5.263 (Solution on p. 333.)
5 1 5
6 + 3 + 21
Exercise 5.264
2 1
5 + 8
Exercise 5.265 (Solution on p. 333.)
1 1 1
4 + 8 + 4
9 This content is available online at <http://cnx.org/content/m34944/1.2/>.
CHAPTER 5. ADDITION AND SUBTRACTION OF FRACTIONS,
322
COMPARING FRACTIONS, AND COMPLEX FRACTIONS
Exercise 5.266
1 1
16 + 10
Exercise 5.267 (Solution on p. 333.)
2 1
7 + 3
Exercise 5.268
2 13 + 1
6
Exercise 5.269 (Solution on p. 333.)
3 11
16 +
3
4
Exercise 5.270
1
5 12 + 3 81
Exercise 5.271 (Solution on p. 333.)
16 25 + 8 41
Exercise 5.272
1 17 + 2 47
Exercise 5.273 (Solution on p. 333.)
1 38 + 0
Exercise 5.274
1
3 10 +4
Exercise 5.275 (Solution on p. 333.)
18 23 + 6
Exercise 5.276
1 34 + 5 54
Exercise 5.277 (Solution on p. 333.)
21 + 2
4 3
Exercise 5.278
15 − 1
16 8
Exercise 5.279 (Solution on p. 333.)
9 5
11 − 22
Exercise 5.280
2 3
6 15 − 1 10
Exercise 5.281 (Solution on p. 333.)
5 23 + 8 15 − 2 14
Exercise 5.282
3
8 10 − 4 56 − 3 15
1

Exercise 5.283 (Solution on p. 333.)


11 1 1
− 16
12 + 9
Exercise 5.284
7 92 − 5 56 − 1 13
Exercise 5.285 (Solution on p. 333.)
16 25 − 8 16 − 3 15
2

Exercise 5.286
4 18 + 0 − 32
8
Exercise 5.287 (Solution on p. 333.)
4 18 + 0 − 32
8
Exercise 5.288
8 − 2 13
323

Exercise 5.289 (Solution on p. 333.)


5
4 − 3 16
Exercise 5.290
6 73 + 4
Exercise 5.291 (Solution on p. 333.)
11 11
2
−3
Exercise 5.292
21 58 − 5
8
Exercise 5.293 (Solution on p. 333.)
3 5 4
4 + 16 · 5
Exercise 5.294
11 15 1
12 + 16 ÷ 2 2
Exercise 5.295 (Solution on p. 333.)
3
1 10 + 2 32 ÷ 4
9
Exercise 5.296
8 35 − 1 14
1
· 3
7
Exercise 5.297 (Solution on p. 333.)
2 38 ÷ 3 16
9
− 1
9
Exercise 5.298
15 25 ÷ 50 − 10
1

5.9.1.2 Complex Fractions and Combinations of Operations with Fractions (Section 5.5,Sec-
tion 5.6)

Exercise 5.299 (Solution on p. 333.)


9
16
21
32

Exercise
10
5.300
21
11
14

Exercise 5.301 (Solution on p. 333.)


1 79
5
1 27
Exercise
15
5.302
17
50
51

Exercise 5.303 (Solution on p. 333.)


9
1 16
11
2 12
Exercise
4
5.304
8 15
3
Exercise 5.305 (Solution on p. 333.)
1
9 18
6
Exercise
1 1
5.306
3 4 +2 8
5 16
Exercise 5.307 (Solution on p. 333.)
3+2 21
1 5
4+6
CHAPTER 5. ADDITION AND SUBTRACTION OF FRACTIONS,
324
COMPARING FRACTIONS, AND COMPLEX FRACTIONS
Exercise
7
5.308
4+1 10
9−2 15
Exercise 5.309 (Solution on p. 333.)
1 52
9− 22
Exercise 5.310
1 23 ·( 14 + 15 )
1 12
Exercise 5.311 (Solution on p. 334.)
10
23
·( 65 +2)
8
9

5.9.1.3 Comparing Fractions (Section 5.5)

For problems 54-65, place each collection in order from smallest to largest.
Exercise 5.312
8 , 16
1 3

Exercise 5.313 (Solution on p. 334.)


32 ,
3 1
8
Exercise 5.314
16 , 24
5 3

Exercise 5.315 (Solution on p. 334.)


10 ,
3 5
6
Exercise 5.316
9, 3, 6
2 1 1

Exercise 5.317 (Solution on p. 334.)


3 8 19
8, 3, 6
Exercise 5.318
5 , 10 , 20
3 2 7

Exercise 5.319 (Solution on p. 334.)


7, 9
4 5

Exercise 5.320
5, 7
4 5

Exercise 5.321 (Solution on p. 334.)


12 , 9 , 15
5 4 7

Exercise 5.322
36 , 24 , 12
7 1 5

Exercise 5.323 (Solution on p. 334.)


13 ,
8,
5 3
16 4
325

5.10 Prociency Exam 10

5.10.1 Prociency Exam


For problems 1-12, perform each indicated operation and write the result in simplest form.
Exercise 5.324 (Solution on p. 334.)
(Section 5.3) 16
3
+ 1
8
Exercise 5.325 (Solution on p. 334.)
(Section 5.4) 2 23 + 5 16
Exercise 5.326 (Solution on p. 334.)
(Section 5.7) 15
7
· 20
21 +
5
9
Exercise 5.327 (Solution on p. 334.)
(Section 5.2) 11
3 5
+ 11
Exercise 5.328 (Solution on p. 334.)
(Section 5.7) 6 29 · 1 17 4 21 
28 − 3 17 − 17
Exercise 5.329 (Solution on p. 334.)
(Section 5.4) 5 18 − 2 45
Exercise 5.330 (Solution on p. 334.)
7
(Section 5.6) 12
8
21

Exercise 5.331 (Solution on p. 334.)


1 3
8+4
(Section 5.6) 1 87
Exercise 5.332 (Solution on p. 334.)
(Section 5.4) 4 16
5
+ 1 13 − 2 24
5

Exercise 5.333 (Solution on p. 334.)


(Section 5.7) 18
5
· 15 3

16 − 8
Exercise 5.334 (Solution on p. 334.)
(Section 5.4) 4 + 2 31
Exercise 5.335 (Solution on p. 334.)
(Section 5.4) 8 37 − 5
For problems 13-15, specify the fractions that are equivalent.
Exercise 5.336 (Solution on p. 334.)
12
(Section 5.5) 45 , 15
Exercise 5.337 (Solution on p. 334.)
24
(Section 5.5) 58 , 40
Exercise 5.338 (Solution on p. 334.)
(Section 5.5) 12
5 80
, 192
For problems 16-20, place each collection of fractions in order from smallest to largest.
Exercise 5.339 (Solution on p. 334.)
(Section 5.5) 89 , 6
7
Exercise 5.340 (Solution on p. 334.)
(Section 5.5) 58 , 7
9
Exercise 5.341 (Solution on p. 334.)
(Section 5.5) 11 16
5
, 11 12
5

10 This content is available online at <http://cnx.org/content/m34948/1.2/>.


CHAPTER 5. ADDITION AND SUBTRACTION OF FRACTIONS,
326
COMPARING FRACTIONS, AND COMPLEX FRACTIONS
Exercise 5.342 (Solution on p. 335.)
(Section 5.5) 15
2
, 10
3
, 1
6
Exercise 5.343 (Solution on p. 335.)
(Section 5.5) 19
32 , 16 ,
9 5
8
327

Solutions to Exercises in Chapter 5


Solution to Exercise 5.1 (p. 295)
2
5
Solution to Exercise 5.2 (p. 295)
1
2
Solution to Exercise 5.3 (p. 295)
1
Solution to Exercise 5.4 (p. 295)
4
5
Solution to Exercise 5.5 (p. 295)
3
4 + 3
4 = 3+3
4+4 = 6
8 = 34 , so two 34 's= one 3
4 which is preposterous.
Solution to Exercise 5.6 (p. 296)
2
13
Solution to Exercise 5.7 (p. 296)
1
3
Solution to Exercise 5.8 (p. 296)
0
Solution to Exercise 5.9 (p. 296)
6
5
Solution to Exercise 5.10 (p. 296)
5
9 − 2
9 = 5−2
9−9 = 03 , which is undened
Solution to Exercise 5.11 (p. 296)
5
8
Solution to Exercise 5.13 (p. 296)
1
Solution to Exercise 5.15 (p. 296)
13
15
Solution to Exercise 5.17 (p. 297)
1
Solution to Exercise 5.19 (p. 297)
0
Solution to Exercise 5.21 (p. 297)
0
Solution to Exercise 5.23 (p. 297)
9
11
Solution to Exercise 5.25 (p. 297)
15
8
Solution to Exercise 5.27 (p. 297)
1
2
Solution to Exercise 5.29 (p. 297)
3
5
Solution to Exercise 5.31 (p. 297)
13
5
Solution to Exercise 5.33 (p. 297)
10
Solution to Exercise 5.35 (p. 297)
9
11
Solution to Exercise 5.37 (p. 298)
16 =
5 (using the preposterous rule)
8
30
Solution to Exercise 5.39 (p. 298)
13
24
CHAPTER 5. ADDITION AND SUBTRACTION OF FRACTIONS,
328
COMPARING FRACTIONS, AND COMPLEX FRACTIONS
Solution to Exercise 5.41 (p. 298)
2700
Solution to Exercise 5.43 (p. 298)
32 · 5 · 11
Solution to Exercise 5.45 (p. 298)
2
3
Solution to Exercise 5.46 (p. 301)
5
6
Solution to Exercise 5.47 (p. 301)
1
14
Solution to Exercise 5.48 (p. 301)
3
40
Solution to Exercise 5.49 (p. 301)
11
16
Solution to Exercise 5.50 (p. 301)
1
96
Solution to Exercise 5.51 (p. 302)
The same denominator
Solution to Exercise 5.53 (p. 302)
5
8
Solution to Exercise 5.55 (p. 302)
31
24
Solution to Exercise 5.57 (p. 302)
17
28
Solution to Exercise 5.59 (p. 302)
19
36
Solution to Exercise 5.61 (p. 302)
19
39
Solution to Exercise 5.63 (p. 302)
29
60
Solution to Exercise 5.65 (p. 302)
8
81
Solution to Exercise 5.67 (p. 302)
17
65
Solution to Exercise 5.69 (p. 302)
2
63
Solution to Exercise 5.71 (p. 302)
7
16
Solution to Exercise 5.73 (p. 303)
47
18
Solution to Exercise 5.75 (p. 303)
103
30
Solution to Exercise 5.77 (p. 303)
217
264
Solution to Exercise 5.79 (p. 303)
511
720
Solution to Exercise 5.81 (p. 303)
37
72
Solution to Exercise 5.83 (p. 303)
221
150
329

Solution to Exercise 5.85 (p. 303)


1,465
2,016
Solution to Exercise 5.87 (p. 303)
65
204
Solution to Exercise 5.89 (p. 303)
1
5
Solution to Exercise 5.91 (p. 303)
607
180
Solution to Exercise 5.93 (p. 303)
109
520
Solution to Exercise 5.95 (p. 304)
$ 137
8 or $17 8
1

Solution to Exercise 5.97 (p. 304)


00 00
No; 3 boxes add up to 26 14 , which is larger than 25 15 .
Solution to Exercise 5.99 (p. 304)
No pipe at all; inside diameter is greater than outside diameter
Solution to Exercise 5.101 (p. 304)
449
Solution to Exercise 5.103 (p. 304)
660
Solution to Exercise 5.105 (p. 304)
7
15
Solution to Exercise 5.106 (p. 305)
4 79
Solution to Exercise 5.107 (p. 305)
8 14
Solution to Exercise 5.108 (p. 305)
8 18
Solution to Exercise 5.109 (p. 305)
3
8 10
Solution to Exercise 5.110 (p. 305)
18 16
9

Solution to Exercise 5.111 (p. 306)


7 12
Solution to Exercise 5.113 (p. 306)
12 21
Solution to Exercise 5.115 (p. 306)
21 11
6

Solution to Exercise 5.117 (p. 306)


3 78
Solution to Exercise 5.119 (p. 306)
1 23
Solution to Exercise 5.121 (p. 306)
8 11
14
Solution to Exercise 5.123 (p. 306)
4 20
21
Solution to Exercise 5.125 (p. 306)
17
13 24
Solution to Exercise 5.127 (p. 306)
2 12
7
CHAPTER 5. ADDITION AND SUBTRACTION OF FRACTIONS,
330
COMPARING FRACTIONS, AND COMPLEX FRACTIONS
Solution to Exercise 5.129 (p. 306)
7 17
30
Solution to Exercise 5.131 (p. 307)
25
74 42
Solution to Exercise 5.133 (p. 307)
21 13
Solution to Exercise 5.135 (p. 307)
5 21
32
Solution to Exercise 5.137 (p. 307)
20 144
1

Solution to Exercise 5.139 (p. 307)


7
7 12
Solution to Exercise 5.141 (p. 307)
13
5 132
Solution to Exercise 5.143 (p. 307)
47
1 212
Solution to Exercise 5.145 (p. 307)
3 14
Solution to Exercise 5.147 (p. 307)
2 58 gallons
Solution to Exercise 5.149 (p. 307)
16 pounds
8 15
Solution to Exercise 5.151 (p. 308)
3
194 16 pounds
Solution to Exercise 5.153 (p. 308)
2
Solution to Exercise 5.155 (p. 308)
7
9
Solution to Exercise 5.157 (p. 310)
3 4
4 < 5
Solution to Exercise 5.158 (p. 310)
13 < 9
15 10
Solution to Exercise 5.159 (p. 310)
13 , 33 , 17
16 40 20
Solution to Exercise 5.160 (p. 310)
9 52 < 11 16
Solution to Exercise 5.161 (p. 310)
9
1 14 < 1 11
16
Solution to Exercise 5.162 (p. 311)
3 5
5 < 8
Solution to Exercise 5.164 (p. 311)
3 5
4 < 6
Solution to Exercise 5.166 (p. 311)
3 2
8 < 5
Solution to Exercise 5.168 (p. 311)
1 4 3
2 < 7 < 5
Solution to Exercise 5.170 (p. 311)
3 7 5
4 < 9 < 4
Solution to Exercise 5.172 (p. 311)
3 2 3
14 < 7 < 4
331

Solution to Exercise 5.174 (p. 311)


5 47 < 5 35
Solution to Exercise 5.176 (p. 311)
9 23 < 9 45
Solution to Exercise 5.178 (p. 311)
9 1
1 16 < 2 20
Solution to Exercise 5.180 (p. 311)
2 29 < 2 37
Solution to Exercise 5.182 (p. 312)
270,000,000
Solution to Exercise 5.184 (p. 312)
23
7
Solution to Exercise 5.186 (p. 312)
13 58 or 109
8
Solution to Exercise 5.187 (p. 314)
5
6
Solution to Exercise 5.188 (p. 314)
1
40
Solution to Exercise 5.189 (p. 314)
3
2
Solution to Exercise 5.190 (p. 314)
10
57
Solution to Exercise 5.191 (p. 314)
2 13
22
Solution to Exercise 5.192 (p. 314)
5
1 11
Solution to Exercise 5.193 (p. 314)
1
Solution to Exercise 5.195 (p. 314)
3
5
Solution to Exercise 5.197 (p. 314)
5
9
Solution to Exercise 5.199 (p. 314)
5
2
Solution to Exercise 5.201 (p. 314)
31
2
Solution to Exercise 5.203 (p. 314)
7
Solution to Exercise 5.205 (p. 315)
1
Solution to Exercise 5.207 (p. 315)
1
6
Solution to Exercise 5.209 (p. 315)
52
81
Solution to Exercise 5.211 (p. 315)
16
21
Solution to Exercise 5.213 (p. 315)
686
101
Solution to Exercise 5.215 (p. 315)
1
3
CHAPTER 5. ADDITION AND SUBTRACTION OF FRACTIONS,
332
COMPARING FRACTIONS, AND COMPLEX FRACTIONS
Solution to Exercise 5.217 (p. 315)
8 67
Solution to Exercise 5.219 (p. 315)
1 13 37
24 or 24
Solution to Exercise 5.221 (p. 318)
0
Solution to Exercise 5.222 (p. 318)
35 or 5 5
6 6
Solution to Exercise 5.223 (p. 318)
211 or 7 1
30 30
Solution to Exercise 5.224 (p. 318)
7
9
Solution to Exercise 5.225 (p. 318)
2 67
Solution to Exercise 5.226 (p. 318)
3
11
Solution to Exercise 5.227 (p. 318)
15
64
Solution to Exercise 5.228 (p. 318)
11
10
Solution to Exercise 5.229 (p. 319)
5
4
Solution to Exercise 5.231 (p. 319)
4 27
Solution to Exercise 5.233 (p. 319)
2
15
Solution to Exercise 5.235 (p. 319)
1
10
Solution to Exercise 5.237 (p. 319)
3
6 14
Solution to Exercise 5.239 (p. 319)
3
8
Solution to Exercise 5.241 (p. 319)
20
27
Solution to Exercise 5.243 (p. 319)
0
Solution to Exercise 5.245 (p. 319)
2
5
Solution to Exercise 5.247 (p. 319)
1
Solution to Exercise 5.249 (p. 320)
125
72
Solution to Exercise 5.251 (p. 320)
252
19
Solution to Exercise 5.253 (p. 320)
165
256
Solution to Exercise 5.255 (p. 320)
multiplicative identity
Solution to Exercise 5.257 (p. 320)
241 or 2 1
120 120
333

Solution to Exercise 5.259 (p. 321)


11 or 1 3
8 8
Solution to Exercise 5.261 (p. 321)
1
2
Solution to Exercise 5.263 (p. 321)
59 = 1 17
42 42
Solution to Exercise 5.265 (p. 321)
5
8
Solution to Exercise 5.267 (p. 322)
13
21
Solution to Exercise 5.269 (p. 322)
7
4 16
Solution to Exercise 5.271 (p. 322)
3
8 20
Solution to Exercise 5.273 (p. 322)
1 38
Solution to Exercise 5.275 (p. 322)
24 32
Solution to Exercise 5.277 (p. 322)
71 = 5 11
12 12
Solution to Exercise 5.279 (p. 322)
13
22
Solution to Exercise 5.281 (p. 322)
37
11 60
Solution to Exercise 5.283 (p. 322)
139
144
Solution to Exercise 5.285 (p. 322)
1
5 10
Solution to Exercise 5.287 (p. 322)
1
8
Solution to Exercise 5.289 (p. 323)
11
16
Solution to Exercise 5.291 (p. 323)
2
8 11
Solution to Exercise 5.293 (p. 323)
1
Solution to Exercise 5.295 (p. 323)
3
7 10
Solution to Exercise 5.297 (p. 323)
5
9
Solution to Exercise 5.299 (p. 323)
6
7
Solution to Exercise 5.301 (p. 323)
3
2 or 1 12
Solution to Exercise 5.303 (p. 323)
15
28
Solution to Exercise 5.305 (p. 323)
163 or 1 55
108 108
Solution to Exercise 5.307 (p. 323)
66 or 5 1
13 13
CHAPTER 5. ADDITION AND SUBTRACTION OF FRACTIONS,
334
COMPARING FRACTIONS, AND COMPLEX FRACTIONS
Solution to Exercise 5.309 (p. 324)
7
40
Solution to Exercise 5.311 (p. 324)
255 or 1 71
184 184
Solution to Exercise 5.313 (p. 324)
32 ,
3 1
8
Solution to Exercise 5.315 (p. 324)
10 ,
3 5
6
Solution to Exercise 5.317 (p. 324)
3 8 19
8, 3, 6
Solution to Exercise 5.319 (p. 324)
9, 7
5 4

Solution to Exercise 5.321 (p. 324)


12 , 9 , 15
5 4 7

Solution to Exercise 5.323 (p. 324)


13
8, 4,
5 3
16
Solution to Exercise 5.324 (p. 325)
5
16
Solution to Exercise 5.325 (p. 325)
7 56
Solution to Exercise 5.326 (p. 325)
1
Solution to Exercise 5.327 (p. 325)
8
11
Solution to Exercise 5.328 (p. 325)
8
Solution to Exercise 5.329 (p. 325)
2 13
40
Solution to Exercise 5.330 (p. 325)
49 or 17
32 32
Solution to Exercise 5.331 (p. 325)
7
15
Solution to Exercise 5.332 (p. 325)
7
3 16
Solution to Exercise 5.333 (p. 325)
5
32
Solution to Exercise 5.334 (p. 325)
6 13 or 19
3
Solution to Exercise 5.335 (p. 325)
3 37
Solution to Exercise 5.336 (p. 325)
equivalent
Solution to Exercise 5.337 (p. 325)
not equivalent
Solution to Exercise 5.338 (p. 325)
equivalent
Solution to Exercise 5.339 (p. 325)
7, 9
6 8

Solution to Exercise 5.340 (p. 325)


8, 9
5 7
335

Solution to Exercise 5.341 (p. 325)


11 16
5
, 11 12
5

Solution to Exercise 5.342 (p. 326)


15 , 6 , 10
2 1 3

Solution to Exercise 5.343 (p. 326)


19
16 , 32 ,
9 5
8
CHAPTER 5. ADDITION AND SUBTRACTION OF FRACTIONS,
336
COMPARING FRACTIONS, AND COMPLEX FRACTIONS
Chapter 6

Decimals

6.1 Objectives1
After completing this chapter, you should
Reading and Writing Decimals (Section 6.2)

• understand the meaning of digits occurring to the right of the ones position
• be familiar with the meaning of decimal fractions
• be able to read and write a decimal fraction

Converting a Decimal to a Fraction (Section 6.3)

• be able to convert an ordinary decimal and a complex decimal to a fraction

Rounding Decimals (Section 6.4)

• be able to round a decimal number to a specied position

Addition and Subtraction of Decimals (Section 6.5)

• understand the method used for adding and subtracting decimals


• be able to add and subtract decimals
• be able to use the calculator to add and subtract decimals

Multiplication of Decimals (Section 6.6)

• understand the method used for multiplying decimals


• be able to multiply decimals
• be able to simplify a multiplication of a decimal by a power of 10
• understand how to use the word "of" in multiplication

Division of Decimals (Section 6.7)

• understand the method used for dividing decimals


• be able to divide a decimal number by a nonzero whole number and by another, nonzero, decimal
number
• be able to simplify a division of a decimal by a power of 10

Nonterminating Divisions (Section 6.8)


1 This content is available online at <http://cnx.org/content/m18894/1.3/>.

337
338 CHAPTER 6. DECIMALS

• understand the meaning of a nonterminating division


• be able to recognize a nonterminating number by its notation

Converting a Fraction to a Decimal (Section 6.9)

• be able to convert a fraction to a decimal

Combinations of Operations with Decimals and Fractions (Section 6.10)

• be able to combine operations with decimals

6.2 Reading and Writing Decimals2


6.2.1 Section Overview
• Digits to the Right of the Ones Position
• Decimal Fractions
• Reading Decimal Fractions
• Writing Decimal Fractions

6.2.2 Digits to the Right of the Ones Position


We began our study of arithmetic (Section 1.2) by noting that our number system is called a positional
number system with base ten. We also noted that each position has a particular value. We observed that
each position has ten times the value of the position to its right.

This means that each position has 1


10 the value of the position to its left.

Thus, a digit written to the right of the units position must have a value of 10
1
of 1. Recalling that the word
"of" translates to multiplication (·), we can see that the value of the rst position to the right of the units
digit is 10
1
of 1, or
1 1
10 · 1 = 10
The value of the second position to the right of the units digit is 10
1
of 10
1
, or
1 1 1 1
10 · 10 = 102 = 100

2 This content is available online at <http://cnx.org/content/m34957/1.2/>.


339

The value of the third position to the right of the units digit is 10
1
of 100
1
, or
1 1 1 1
10 · 100 = 103 = 1000
This pattern continues.

We can now see that if we were to write digits in positions to the right of the units positions, those positions
have values that are fractions. Not  only do the positions have fractional values, but the fractional values are
all powers of 10 10, 102 , 103 , . . . .

6.2.3 Decimal Fractions


Decimal Point, Decimal
If we are to write numbers with digits appearing to the right of the units digit, we must have a way of denoting
where the whole number part ends and the fractional part begins. Mathematicians denote the separation
point of the units digit and the tenths digit by writing a decimal point. The word decimal comes from
the Latin prex "deci" which means ten, and we use it because we use a base ten number system. Numbers
written in this form are called decimal fractions, or more simply, decimals.

Notice that decimal numbers have the sux "th."

Decimal Fraction
A decimal fraction is a fraction in which the denominator is a power of 10.

The following numbers are examples of decimals.

1. 42.6
The 6 is in the tenths position.

42.6 = 42 10
6

2. 9.8014
The 8 is in the tenths position.
The 0 is in the hundredths position.
The 1 is in the thousandths position.
The 4 is in the ten thousandths position.

9.8014 = 9 108014
,000
3. 0.93
The 9 is in the tenths position.
The 3 is in the hundredths position.
93
0.93 = 100
note: Quite often a zero is inserted in front of a decimal point (in the units position) of a
decimal fraction that has a value less than one. This zero helps keep us from overlooking the
decimal point.
340 CHAPTER 6. DECIMALS

4. 0.7
The 7 is in the tenths position.

0.7 = 10
7

note: We can insert zeros to the right of the right-most digit in a decimal fraction without
changing the value of the number.
70
10 = 0.7 = 0.70 = 100 = 10
7 7

6.2.4 Reading Decimal Fractions


Reading a Decimal Fraction
To read a decimal fraction,
1. Read the whole number part as usual. (If the whole number is less than 1, omit steps 1 and 2.)
2. Read the decimal point as the word "and."
3. Read the number to the right of the decimal point as if it were a whole number.
4. Say the name of the position of the last digit.

6.2.4.1 Sample Set A

Read the following numbers.


Example 6.1
6.8

note: Some people read this as "six point eight." This phrasing gets the message across, but
technically, "six and eight tenths" is the correct phrasing.

Example 6.2
14.116

Example 6.3
0.0019

Example 6.4
81

Eighty-one

In this problem, the indication is that any whole number is a decimal fraction. Whole numbers are
often called decimal numbers.

81 = 81.0
341

6.2.4.2 Practice Set A

Read the following decimal fractions.


Exercise 6.1 (Solution on p. 402.)
12.9
Exercise 6.2 (Solution on p. 402.)
4.86
Exercise 6.3 (Solution on p. 402.)
7.00002
Exercise 6.4 (Solution on p. 402.)
0.030405

6.2.5 Writing Decimal Fractions


Writing a Decimal Fraction
To write a decimal fraction,

1. Write the whole number part.


2. Write a decimal point for the word "and."
3. Write the decimal part of the number so that the right-most digit appears in the position indicated in
the word name. If necessary, insert zeros to the right of the decimal point in order that the right-most
digit appears in the correct position.

6.2.5.1 Sample Set B

Write each number.


Example 6.5
Thirty-one and twelve hundredths.

The decimal position indicated is the hundredths position.

31.12
Example 6.6
Two and three hundred-thousandths.

The decimal position indicated is the hundred thousandths. We'll need to insert enough zeros to
the immediate right of the decimal point in order to locate the 3 in the correct position.

2.00003
Example 6.7
Six thousand twenty-seven and one hundred four millionths.

The decimal position indicated is the millionths position. We'll need to insert enough zeros to the
immediate right of the decimal point in order to locate the 4 in the correct position.

6,027.000104
Example 6.8
Seventeen hundredths.

The decimal position indicated is the hundredths position.


342 CHAPTER 6. DECIMALS

0.17

6.2.5.2 Practice Set B

Write each decimal fraction.


Exercise 6.5 (Solution on p. 402.)
Three hundred six and forty-nine hundredths.
Exercise 6.6 (Solution on p. 402.)
Nine and four thousandths.
Exercise 6.7 (Solution on p. 402.)
Sixty-one millionths.

6.2.6 Exercises
For the following three problems, give the decimal name of the position of the given number in each decimal
fraction.
Exercise 6.8 (Solution on p. 402.)
1. 3.941
9 is in the position.
4 is in the position.
1 is in the position.
Exercise 6.9
17.1085
1 is in the position.
0 is in the position.
8 is in the position.
5 is in the position.
Exercise 6.10 (Solution on p. 402.)
652.3561927
9 is in the position.
7 is in the position.
For the following 7 problems, read each decimal fraction by writing it.
Exercise 6.11
9.2
Exercise 6.12 (Solution on p. 402.)
8.1
Exercise 6.13
10.15
Exercise 6.14 (Solution on p. 402.)
55.06
Exercise 6.15
0.78
Exercise 6.16 (Solution on p. 402.)
1.904
Exercise 6.17
10.00011
343

For the following 10 problems, write each decimal fraction.


Exercise 6.18 (Solution on p. 402.)
Three and twenty one-hundredths.
Exercise 6.19
Fourteen and sixty seven-hundredths.
Exercise 6.20 (Solution on p. 402.)
One and eight tenths.
Exercise 6.21
Sixty-one and ve tenths.
Exercise 6.22 (Solution on p. 402.)
Five hundred eleven and four thousandths.
Exercise 6.23
Thirty-three and twelve ten-thousandths.
Exercise 6.24 (Solution on p. 402.)
Nine hundred forty-seven thousandths.
Exercise 6.25
Two millionths.
Exercise 6.26 (Solution on p. 402.)
Seventy-one hundred-thousandths.
Exercise 6.27
One and ten ten-millionths.
Calculator Problems
For the following 10 problems, perform each division using a calculator. Then write the resulting decimal
using words.
Exercise 6.28 (Solution on p. 402.)
3÷4
Exercise 6.29
1÷8
Exercise 6.30 (Solution on p. 402.)
4 ÷ 10
Exercise 6.31
2÷5
Exercise 6.32 (Solution on p. 402.)
4 ÷ 25
Exercise 6.33
1 ÷ 50
Exercise 6.34 (Solution on p. 402.)
3 ÷ 16
Exercise 6.35
15 ÷ 8
Exercise 6.36 (Solution on p. 402.)
11 ÷ 20
Exercise 6.37
9 ÷ 40
344 CHAPTER 6. DECIMALS

6.2.6.1 Exercises for Review

Exercise 6.38 (Solution on p. 402.)


(Section 1.4) Round 2,614 to the nearest ten.
Exercise 6.39
(Section 2.5) Is 691,428,471 divisible by 3?
Exercise 6.40 (Solution on p. 402.)
(Section 4.4) Determine the missing numerator.

3 ?
14 = 56
Exercise 6.41
(Section 4.7) Find 16
3
of 32
39
Exercise 6.42 (Solution on p. 402.)
25 +
q
2 2
(Section 5.7) Find the value of 1

81 3 + 9

6.3 Converting a Decimal to a Fraction3


6.3.1 Section Overview
• Converting an Ordinary Decimal to a Fraction
• Converting a Complex Decimal to a Fraction

6.3.2 Converting an Ordinary Decimal to a Fraction


We can convert a decimal fraction to a fraction, essentially, by saying it in words, then writing what we say.
We may have to reduce that fraction.

6.3.2.1 Sample Set A

Convert each decimal fraction to a proper fraction or a mixed number.


Example 6.9

Reading: six tenths→ 10 .


6

Reduce: 5.
3

Example 6.10

903 .
Reading: nine hundred three thousands→ 1000
Example 6.11

3 This content is available online at <http://cnx.org/content/m34958/1.2/>.


345

61 .
Reading: eighteen and sixty-one hundredths→ 18 100
Example 6.12

Reading: ve hundred eight and ve ten thousandths→ 508 10,5000 .

Reduce: 508 2,000


1
.

6.3.2.2 Practice Set A

Convert the following decimals to fractions or mixed numbers. Be sure to reduce.


Exercise 6.43 (Solution on p. 402.)
16.84
Exercise 6.44 (Solution on p. 403.)
0.513
Exercise 6.45 (Solution on p. 403.)
6,646.0107
Exercise 6.46 (Solution on p. 403.)
1.1

6.3.3 Converting A Complex Decimal to a Fraction


Complex Decimals
Numbers such as 0.11 23 are called complex decimals. We can also convert complex decimals to fractions.

6.3.3.1 Sample Set B

Convert the following complex decimals to fractions.


Example 6.13
0.11 23

The 23 appears to occur in the thousands position, but it is referring to 2


3 of a hundredth. So, we
read 0.11 23 as "eleven and two-thirds hundredths."
11 2 11·3+2

0.11 23 = 1003 = 3
100
35

= 3
100
1

= 35 ÷ 100
3 1
7
)35 1
= 3 ·
)100
20

7
= 60
Example 6.14
4.006 14

Note that 4.006 14 = 4 + .006 14


346 CHAPTER 6. DECIMALS

61
4 + .006 14 = 4 + 1000
4

25

= 4 + 1000
4
1
1
)25 1
= 4+ 4 ·
)1000
40

= 4 + 4·1·1
40
1
= 4 + 160
1
= 4 160

6.3.3.2 Practice Set B

Convert each complex decimal to a fraction or mixed number. Be sure to reduce.


Exercise 6.47 (Solution on p. 403.)
0.8 34
Exercise 6.48 (Solution on p. 403.)
0.12 25
Exercise 6.49 (Solution on p. 403.)
6.005 56
Exercise 6.50 (Solution on p. 403.)
18.1 17
3

6.3.4 Exercises
For the following 20 problems, convert each decimal fraction to a proper fraction or a mixed number. Be
sure to reduce.
Exercise 6.51 (Solution on p. 403.)
0.7
Exercise 6.52
0.1
Exercise 6.53 (Solution on p. 403.)
0.53
Exercise 6.54
0.71
Exercise 6.55 (Solution on p. 403.)
0.219
Exercise 6.56
0.811
Exercise 6.57 (Solution on p. 403.)
4.8
Exercise 6.58
2.6
Exercise 6.59 (Solution on p. 403.)
16.12
Exercise 6.60
25.88
347

Exercise 6.61 (Solution on p. 403.)


6.0005
Exercise 6.62
1.355
Exercise 6.63 (Solution on p. 403.)
16.125
Exercise 6.64
0.375
Exercise 6.65 (Solution on p. 403.)
3.04
Exercise 6.66
21.1875
Exercise 6.67 (Solution on p. 403.)
8.225
Exercise 6.68
1.0055
Exercise 6.69 (Solution on p. 403.)
9.99995
Exercise 6.70
22.110
For the following 10 problems, convert each complex decimal to a fraction.
Exercise 6.71 (Solution on p. 403.)
0.7 12
Exercise 6.72
0.012 12
Exercise 6.73 (Solution on p. 403.)
2.16 14
Exercise 6.74
5.18 23
Exercise 6.75 (Solution on p. 403.)
14.112 13
Exercise 6.76
80.0011 37
Exercise 6.77 (Solution on p. 403.)
1.40 16
5

Exercise 6.78
0.8 53
Exercise 6.79 (Solution on p. 403.)
1.9 75
Exercise 6.80
1.7 37
9
348 CHAPTER 6. DECIMALS

6.3.4.1 Exercises for Review

Exercise 6.81 (Solution on p. 403.)


(Section 3.6) Find the greatest common factor of 70, 182, and 154.
Exercise 6.82
(Section 3.6) Find the greatest common multiple of 14, 26, and 60.
Exercise 6.83 (Solution on p. 403.)
(Section 4.5) Find the value of 3
5 · 15
18 ÷ 9 .
5

Exercise 6.84
(Section 5.4) Find the value of 5 23 + 8 12
1
.
Exercise 6.85 (Solution on p. 403.)
(Section 6.2) In the decimal number 26.10742, the digit 7 is in what position?

6.4 Rounding Decimals4


6.4.1 Section Overview
• Rounding Decimal Numbers

6.4.2 Rounding Decimal Numbers


We rst considered the concept of rounding numbers in Section 1.4 where our concern with rounding was
related to whole numbers only. With a few minor changes, we can apply the same rules of rounding to
decimals.

To round a decimal to a particular position:

1. Mark the position of the round-o digit (with an arrow or check).


2. Note whether the digit to the immediate right of the marked digit is
(a) less than 5. If so, leave the round-o digit unchanged.
(b) 5 or greater. If so, add 1 to the round-o digit.
3. If the round-o digit is
(a) to the right of the decimal point, eliminate all the digits to its right.
(b) to the left of the decimal point, replace all the digits between it and the decimal point with zeros
and eliminate the decimal point and all the decimal digits.

6.4.2.1 Sample Set A

Round each decimal to the specied position. (The numbers in parentheses indicate which step is being
used.)
Example 6.15
Round 32.116 to the nearest hundredth.

(1)
4 This content is available online at <http://cnx.org/content/m34959/1.2/>.
349

(2b) The digit immediately to the right is 6, and 6 > 5, so we add 1 to the round-o digit:
1+1 = 2
(3a) The round-o digit is to the right of the decimal point, so we eliminate all digits to its right.
32.12
The number 32.116 rounded to the nearest hundredth is 32.12.
Example 6.16
Round 633.14216 to the nearest hundred.

(1)
(2a) The digit immediately to the right is 3, and 3 < 5 so we leave the round-o digit unchanged.
(3b) The round-o digit is to the left of 0, so we replace all the digits between it and the decimal
point with zeros and eliminate the decimal point and all the decimal digits. 600
The number 633.14216 rounded to the nearest hundred is 600.
Example 6.17
1,729.63 rounded to the nearest ten is 1,730.
Example 6.18
1.0144 rounded to the nearest tenth is 1.0.
Example 6.19
60.98 rounded to the nearest one is 61.
Sometimes we hear a phrase such as "round to three decimal places." This phrase means that the round-o
digit is the third decimal digit (the digit in the thousandths position).
Example 6.20
67.129 rounded to the second decimal place is 67.13.
Example 6.21
67.129558 rounded to 3 decimal places is 67.130.

6.4.2.2 Practice Set A

Round each decimal to the specied position.


Exercise 6.86 (Solution on p. 404.)
4.816 to the nearest hundredth.
Exercise 6.87 (Solution on p. 404.)
0.35928 to the nearest ten thousandths.
Exercise 6.88 (Solution on p. 404.)
82.1 to the nearest one.
Exercise 6.89 (Solution on p. 404.)
753.98 to the nearest hundred.
Exercise 6.90 (Solution on p. 404.)
Round 43.99446 to three decimal places.
Exercise 6.91 (Solution on p. 404.)
Round 105.019997 to four decimal places.
Exercise 6.92 (Solution on p. 404.)
Round 99.9999 to two decimal places.
350 CHAPTER 6. DECIMALS

6.4.3 Exercises
For the rst 10 problems, complete the chart by rounding each decimal to the indicated positions.
Exercise 6.93 (Solution on p. 404.)
20.01071

Tenth Hundredth Thousandth Ten Thousandth

Table 6.1

Exercise 6.94
3.52612

Tenth Hundredth Thousandth Ten Thousandth


3.53

Table 6.2

Exercise 6.95 (Solution on p. 404.)


531.21878

Tenth Hundredth Thousandth Ten Thousandth

Table 6.3

Exercise 6.96
36.109053

Tenth Hundredth Thousandth Ten Thousandth


36.1

Table 6.4

Exercise 6.97 (Solution on p. 404.)


1.999994

Tenth Hundredth Thousandth Ten Thousandth

Table 6.5

Exercise 6.98
7.4141998
351

Tenth Hundredth Thousandth Ten Thousandth


7.414

Table 6.6

Exercise 6.99 (Solution on p. 404.)


0.000007

Tenth Hundredth Thousandth Ten Thousandth

Table 6.7

Exercise 6.100
0.00008

Tenth Hundredth Thousandth Ten Thousandth


0.0001

Table 6.8

Exercise 6.101 (Solution on p. 404.)


9.19191919

Tenth Hundredth Thousandth Ten Thousandth

Table 6.9

Exercise 6.102
0.0876543

Tenth Hundredth Thousandth Ten Thousandth

Table 6.10

Calculator Problems
For the following 5 problems, round 18.4168095 to the indicated place.
Exercise 6.103 (Solution on p. 405.)
3 decimal places.
Exercise 6.104
1 decimal place.
Exercise 6.105 (Solution on p. 405.)
5 decimal places.
352 CHAPTER 6. DECIMALS

Exercise 6.106
6 decimal places.
Exercise 6.107 (Solution on p. 405.)
2 decimal places.
Calculator Problems
For the following problems, perform each division using a calculator.
Exercise 6.108
4 ÷ 3 and round to 2 decimal places.
Exercise 6.109 (Solution on p. 405.)
1 ÷ 8 and round to 1 decimal place.
Exercise 6.110
1 ÷ 27 and round to 6 decimal places.
Exercise 6.111 (Solution on p. 405.)
51 ÷ 61 and round to 5 decimal places.
Exercise 6.112
3 ÷ 16 and round to 3 decimal places.
Exercise 6.113 (Solution on p. 405.)
16 ÷ 3 and round to 3 decimal places.
Exercise 6.114
26 ÷ 7 and round to 5 decimal places.

6.4.3.1 Exercises for Review

Exercise 6.115 (Solution on p. 405.)


(Section 1.2) What is the value of 2 in the number 421,916,017?
Exercise 6.116
(Section 2.4) Perform the division: 378 ÷ 29.
Exercise 6.117 (Solution on p. 405.)
(Section 3.2) Find the value of 44 .
Exercise 6.118
(Section 4.3) Convert 11
3 to a mixed number.
Exercise 6.119 (Solution on p. 405.)
(Section 6.3) Convert 3.16 to a mixed number fraction.

6.5 Addition and Subtraction of Decimals5


6.5.1 Section Overview
• The Logic Behind the Method
• The Method of Adding and Subtracting Decimals
• Calculators

5 This content is available online at <http://cnx.org/content/m34960/1.5/>.


353

6.5.2 The Logic Behind the Method


Consider the sum of 4.37 and 3.22. Changing each decimal to a fraction, we have
37 + 3 22 Performing the addition, we get
4 100 100
37 + 3 22
4.37 + 3.22 = 4 100 = 4·100+37
+ 3·100 +22
100 100 100
= 437 + 322
100 100
= 437+322
100
= 759
100
= 59
7 100
= seven and fty-nine hundredths
= 7.59

Thus, 4.37 + 3.22 = 7.59.

6.5.3 The Method of Adding and Subtracting Decimals


When writing the previous addition, we could have written the numbers in columns.

4.37
+3.22
7.59

This agrees with our previous result. From this observation, we can suggest a method for adding and
subtracting decimal numbers.

Method of Adding and Subtracting Decimals


To add or subtract decimals:

1. Align the numbers vertically so that the decimal points line up under each other and the corresponding
decimal positions are in the same column.
2. Add or subtract the numbers as if they were whole numbers.
3. Place a decimal point in the resulting sum or dierence directly under the other decimal points.

6.5.3.1 Sample Set A

Find the following sums and dierences.


Example 6.22
9.813 + 2.140

9.813 The decimal points are aligned in the same column.


+2.140
11.953
Example 6.23
841.0056 + 47.016 + 19.058
354 CHAPTER 6. DECIMALS

841.0056
47.016
+19.058

To insure that the columns align properly, we can write a 0 in the position at the end of the numbers
47.016 and 19.058 without changing their values.

Example 6.24
1.314 − 0.58

1.314
−0.58 Write a 0 in the thousandths position.

Example 6.25
16.01 − 7.053

16.01
−7.053 Write a 0 in the thousandths position.

Example 6.26
Find the sum of 6.88106 and 3.5219 and round it to three decimal places.

6.88106
+3.5219 Write a 0 in the ten thousandths position.

We need to round the sum to the thousandths position. Since the digit in the position immediately
to the right is 9, and 9>5, we get

10.403
Example 6.27
Wendy has $643.12 in her checking account. She writes a check for $16.92. How much is her new
account balance?
355

To nd the new account balance, we need to nd the dierence between 643.12 and 16.92. We will
subtract 16.92 from 643.12.

After writing a check for $16.92, Wendy now has a balance of $626.20 in her checking account.

6.5.3.2 Practice Set A

Find the following sums and dierences.


Exercise 6.120 (Solution on p. 405.)
3.187 + 2.992
Exercise 6.121 (Solution on p. 405.)
14.987 − 5.341
Exercise 6.122 (Solution on p. 405.)
0.5261 + 1.0783
Exercise 6.123 (Solution on p. 405.)
1.06 − 1.0535
Exercise 6.124 (Solution on p. 405.)
16,521.07 + 9,256.15
Exercise 6.125 (Solution on p. 405.)
Find the sum of 11.6128 and 14.07353, and round it to two decimal places.

6.5.4 Calculators
The calculator can be useful for nding sums and dierences of decimal numbers. However, calculators with
an eight-digit display cannot be used when working with decimal numbers that contain more than eight
digits, or when the sum results in more than eight digits. In practice, an eight-place decimal will seldom be
encountered. There are some inexpensive calculators that can handle 13 decimal places.

6.5.4.1 Sample Set B

Use a calculator to nd each sum or dierence.


Example 6.28
42.0638 + 126.551

Display Reads
Type 42.0638 42.0638
Press + 42.0638
Type 126.551 126.551
Press = 168.6148

Table 6.11
356 CHAPTER 6. DECIMALS

The sum is 168.6148.


Example 6.29
Find the dierence between 305.0627 and 14.29667.

Display Reads
Type 305.0627 305.0627
Press  305.0627
Type 14.29667 14.29667
Press = 290.76603

Table 6.12

The dierence is 290.76603


Example 6.30
51.07 + 3,891.001786

Since 3,891.001786 contains more than eight digits, we will be unable to use an eight-digit display
calculator to perform this addition. We can, however, nd the sum by hand.

51.070000
3891.001786
3942.071786

The sum is 3,942.071786.

6.5.4.2 Practice Set B

Use a calculator to perform each operation.


Exercise 6.126 (Solution on p. 405.)
4.286 + 8.97
Exercise 6.127 (Solution on p. 405.)
452.0092 − 392.558
Exercise 6.128 (Solution on p. 405.)
Find the sum of 0.095 and 0.001862
Exercise 6.129 (Solution on p. 405.)
Find the dierence between 0.5 and 0.025
Exercise 6.130 (Solution on p. 405.)
Find the sum of 2,776.00019 and 2,009.00012.
357

6.5.5 Exercises
For the following 15 problems, perform each addition or subtraction. Use a calculator to check each result.
Exercise 6.131 (Solution on p. 405.)
1.84 + 7.11
Exercise 6.132
15.015 − 6.527
Exercise 6.133 (Solution on p. 405.)
11.842 + 28.004
Exercise 6.134
3.16 − 2.52
Exercise 6.135 (Solution on p. 405.)
3.55267 + 8.19664
Exercise 6.136
0.9162 − 0.0872
Exercise 6.137 (Solution on p. 406.)
65.512 − 8.3005
Exercise 6.138
761.0808 − 53.198
Exercise 6.139 (Solution on p. 406.)
4.305 + 2.119 − 3.817
Exercise 6.140
19.1161 + 27.8014 + 39.3161
Exercise 6.141 (Solution on p. 406.)
0.41276 − 0.0018 − 0.00011
Exercise 6.142
2.181 + 6.05 + 1.167 + 8.101
Exercise 6.143 (Solution on p. 406.)
1.0031+6.013106+0.00018+0.0092+2.11
Exercise 6.144
27 + 42 + 9.16 − 0.1761 + 81.6
Exercise 6.145 (Solution on p. 406.)
10.28 + 11.111 + 0.86 + 5.1
For the following 10 problems, solve as directed. A calculator may be useful.
Exercise 6.146
Add 6.1121 and 4.916 and round to 2 decimal places.
Exercise 6.147 (Solution on p. 406.)
Add 21.66418 and 18.00184 and round to 4 decimal places.
Exercise 6.148
Subtract 5.2121 from 9.6341 and round to 1 decimal place.
Exercise 6.149 (Solution on p. 406.)
Subtract 0.918 from 12.006 and round to 2 decimal places.
Exercise 6.150
Subtract 7.01884 from the sum of 13.11848 and 2.108 and round to 4 decimal places.
358 CHAPTER 6. DECIMALS

Exercise 6.151 (Solution on p. 406.)


A checking account has a balance of $42.51. A check is written for $19.28. What is the new
balance?
Exercise 6.152
A checking account has a balance of $82.97. One check is written for $6.49 and another for $39.95.
What is the new balance?
Exercise 6.153 (Solution on p. 406.)
A person buys $4.29 worth of hamburger and pays for it with a $10 bill. How much change does
this person get?
Exercise 6.154
A man buys $6.43 worth of stationary and pays for it with a $20 bill. After receiving his change,
he realizes he forgot to buy a pen. If the total price of the pen is $2.12, and he buys it, how much
of the $20 bill is left?
Exercise 6.155 (Solution on p. 406.)
A woman starts recording a movie on her video cassette recorder with the tape counter set at
21.93. The movie runs 847.44 tape counter units. What is the nal tape counter reading?

6.5.5.1 Exercises for Review

Exercise 6.156
(Section 1.6) Find the dierence between 11,206 and 10,884.
Exercise 6.157 (Solution on p. 406.)
(Section 2.2) Find the product, 820 · 10,000.
Exercise 6.158 √ √
(Section 3.3) Find the value of 121 − 25 + 82 + 16 ÷ 22 .
Exercise 6.159 (Solution on p. 406.)
(Section 4.6) Find the value of 8 31 · 36
75 ÷ 2 5 .
2

Exercise 6.160
(Section 6.4) Round 1.08196 to the nearest hundredth.

6.6 Multiplication of Decimals6


6.6.1 Section Overview
• The Logic Behind the Method
• The Method of Multiplying Decimals
• Calculators
• Multiplying Decimals By Powers of 10
• Multiplication in Terms of Of

6 This content is available online at <http://cnx.org/content/m34963/1.2/>.


359

6.6.2 The Logic Behind the Method


Consider the product of 3.2 and 1.46. Changing each decimal to a fraction, we have

(3.2) (1.46) = 2
3 10 46
· 1 100
= 32 · 146
10 100
= 32·146
10·100
= 4672
1000
= 672
4 1000
= four and six hundred seventy-two thousandths
= 4.672

Thus, (3.2) (1.46) = 4.672.

Notice that the factor

3.2 has 1 decimal place,


1.46 has 2 decimal places,
}1 + 2 = 3
and the product
4.672 has 3 decimal places.

Using this observation, we can suggest that the sum of the number of decimal places in the factors equals
the number of decimal places in the product.

6.6.3 The Method of Multiplying Decimals


Method of Multiplying Decimals
To multiply decimals,
1. Multiply the numbers as if they were whole numbers.
2. Find the sum of the number of decimal places in the factors.
3. The number of decimal places in the product is the sum found in step 2.

6.6.3.1 Sample Set A

Find the following products.


Example 6.31
6.5 · 4.3
360 CHAPTER 6. DECIMALS

Thus, 6.5 · 4.3 = 27.95.


Example 6.32
23.4 · 1.96

Thus, 23.4 · 1.96 = 45.864.


Example 6.33
Find the product of 0.251 and 0.00113 and round to three decimal places.

Now, rounding to three decimal places, we get

6.6.3.2 Practice Set A

Find the following products.


Exercise 6.161 (Solution on p. 406.)
5.3 · 8.6
Exercise 6.162 (Solution on p. 406.)
2.12 · 4.9
Exercise 6.163 (Solution on p. 406.)
1.054 · 0.16
Exercise 6.164 (Solution on p. 406.)
0.00031 · 0.002
Exercise 6.165 (Solution on p. 406.)
Find the product of 2.33 and 4.01 and round to one decimal place.
Exercise 6.166 (Solution on p. 406.)
10 · 5.394
Exercise 6.167 (Solution on p. 406.)
100 · 5.394
Exercise 6.168 (Solution on p. 406.)
1000 · 5.394
Exercise 6.169 (Solution on p. 406.)
10,000 · 5.394
361

6.6.4 Calculators
Calculators can be used to nd products of decimal numbers. However, a calculator that has only an eight-
digit display may not be able to handle numbers or products that result in more than eight digits. But there
are plenty of inexpensive ($50 - $75) calculators with more than eight-digit displays.

6.6.4.1 Sample Set B

Find the following products, if possible, using a calculator.


Example 6.34
2.58 · 8.61

Display Reads
Type 2.58 2.58
Press × 2.58
Type 8.61 8.61
Press = 22.2138
Table 6.13

The product is 22.2138.


Example 6.35
0.006 · 0.0042

Display Reads
Type .006 .006
Press × .006
Type .0042 0.0042
Press = 0.0000252
Table 6.14

We know that there will be seven decimal places in the product (since 3 + 4 = 7). Since the display
shows 7 decimal places, we can assume the product is correct. Thus, the product is 0.0000252.
Example 6.36
0.0026 · 0.11976

Since we expect 4 + 5 = 9 decimal places in the product, we know that an eight-digit display
calculator will not be able to provide us with the exact value. To obtain the exact value, we must
use "hand technology." Suppose, however, that we agree to round o this product to three decimal
places. We then need only four decimal places on the display.

Display Reads
Type .0026 .0026
Press × .0026
Type .11976 0.11976
Press = 0.0003114
362 CHAPTER 6. DECIMALS

Table 6.15

Rounding 0.0003114 to three decimal places we get 0.000. Thus, 0.0026 · 0.11976 = 0.000 to three
decimal places.

6.6.4.2 Practice Set B

Use a calculator to nd each product. If the calculator will not provide the exact product, round the result
to four decimal places.
Exercise 6.170 (Solution on p. 406.)
5.126 · 4.08
Exercise 6.171 (Solution on p. 406.)
0.00165 · 0.04
Exercise 6.172 (Solution on p. 406.)
0.5598 · 0.4281
Exercise 6.173 (Solution on p. 406.)
0.000002 · 0.06

6.6.5 Multiplying Decimals by Powers of 10


There is an interesting feature of multiplying decimals by powers of 10. Consider the following multiplications.

Multiplication Number of Zeros in the Power of Number of Positions the Decimal


10 Point Has Been Moved to the
Right
10 · 8.315274 = 83.15274 1 1
100 · 8.315274 = 831.5274 2 2
1, 000 · 8.315274 = 8, 315.274 3 3
10, 000 · 8.315274 = 83, 152.74 4 4

Table 6.16

Multiplying a Decimal by a Power of 10


To multiply a decimal by a power of 10, move the decimal place to the right of its current position as many
places as there are zeros in the power of 10. Add zeros if necessary.

6.6.5.1 Sample Set C

Find the following products.


Example 6.37
100 · 34.876. Since there are 2 zeros in 100, Move the decimal point in 34.876 two places to the
right.
363

Example 6.38
1, 000 · 4.8058. Since there are 3 zeros in 1,000, move the decimal point in 4.8058 three places to
the right.

Example 6.39
10, 000 · 56.82. Since there are 4 zeros in 10,000, move the decimal point in 56.82 four places to
the right. We will have to add two zeros in order to obtain the four places.

Since there is no fractional part, we can drop the decimal point.


Example 6.40

Example 6.41

6.6.5.2 Practice Set C

Find the following products.


Exercise 6.174 (Solution on p. 407.)
100 · 4.27
Exercise 6.175 (Solution on p. 407.)
10,000 · 16.52187
Exercise 6.176 (Solution on p. 407.)
(10) (0.0188)
Exercise 6.177 (Solution on p. 407.)
(10,000,000,000) (52.7)

6.6.6 Multiplication in Terms of Of 


Recalling that the word "of" translates to the arithmetic operation of multiplication, let's observe the fol-
lowing multiplications.
364 CHAPTER 6. DECIMALS

6.6.6.1 Sample Set D

Example 6.42
Find 4.1 of 3.8.

Translating "of" to "×", we get

4.1
×3.8
328
123
15.58

Thus, 4.1 of 3.8 is 15.58.


Example 6.43
Find 0.95 of the sum of 2.6 and 0.8.

We rst nd the sum of 2.6 and 0.8.

2.6
+0.8
3.4

Now nd 0.95 of 3.4

3.4
×0.95
170
306
3.230

Thus, 0.95 of (2.6 + 0.8) is 3.230.

6.6.6.2 Practice Set D

Exercise 6.178 (Solution on p. 407.)


Find 2.8 of 6.4.
Exercise 6.179 (Solution on p. 407.)
Find 0.1 of 1.3.
Exercise 6.180 (Solution on p. 407.)
Find 1.01 of 3.6.
Exercise 6.181 (Solution on p. 407.)
Find 0.004 of 0.0009.
Exercise 6.182 (Solution on p. 407.)
Find 0.83 of 12.
Exercise 6.183 (Solution on p. 407.)
Find 1.1 of the sum of 8.6 and 4.2.
365

6.6.7 Exercises
For the following 30 problems, nd each product and check each result with a calculator.
Exercise 6.184 (Solution on p. 407.)
3.4 · 9.2
Exercise 6.185
4.5 · 6.1
Exercise 6.186 (Solution on p. 407.)
8.0 · 5.9
Exercise 6.187
6.1 · 7
Exercise 6.188 (Solution on p. 407.)
(0.1) (1.52)
Exercise 6.189
(1.99) (0.05)
Exercise 6.190 (Solution on p. 407.)
(12.52) (0.37)
Exercise 6.191
(5.116) (1.21)
Exercise 6.192 (Solution on p. 407.)
(31.82) (0.1)
Exercise 6.193
(16.527) (9.16)
Exercise 6.194 (Solution on p. 407.)
0.0021 · 0.013
Exercise 6.195
1.0037 · 1.00037
Exercise 6.196 (Solution on p. 407.)
(1.6) (1.6)
Exercise 6.197
(4.2) (4.2)
Exercise 6.198 (Solution on p. 407.)
0.9 · 0.9
Exercise 6.199
1.11 · 1.11
Exercise 6.200 (Solution on p. 407.)
6.815 · 4.3
Exercise 6.201
9.0168 · 1.2
Exercise 6.202 (Solution on p. 407.)
(3.5162) (0.0000003)
Exercise 6.203
(0.000001) (0.01)
366 CHAPTER 6. DECIMALS

Exercise 6.204 (Solution on p. 407.)


(10) (4.96)
Exercise 6.205
(10) (36.17)
Exercise 6.206 (Solution on p. 407.)
10 · 421.8842
Exercise 6.207
10 · 8.0107
Exercise 6.208 (Solution on p. 407.)
100 · 0.19621
Exercise 6.209
100 · 0.779
Exercise 6.210 (Solution on p. 407.)
1000 · 3.596168
Exercise 6.211
1000 · 42.7125571
Exercise 6.212 (Solution on p. 407.)
1000 · 25.01
Exercise 6.213
100, 000 · 9.923
Exercise 6.214 (Solution on p. 408.)
(4.6) (6.17)

Actual product Tenths Hundreds Thousandths

Table 6.17

Exercise 6.215
(8.09) (7.1)

Actual product Tenths Hundreds Thousandths

Table 6.18

Exercise 6.216 (Solution on p. 408.)


(11.1106) (12.08)

Actual product Tenths Hundreds Thousandths

Table 6.19

Exercise 6.217
0.0083 · 1.090901
367

Actual product Tenths Hundreds Thousandths

Table 6.20

Exercise 6.218 (Solution on p. 408.)


7 · 26.518

Actual product Tenths Hundreds Thousandths

Table 6.21

For the following 15 problems, perform the indicated operations


Exercise 6.219
Find 5.2 of 3.7.
Exercise 6.220 (Solution on p. 408.)
Find 12.03 of 10.1
Exercise 6.221
Find 16 of 1.04
Exercise 6.222 (Solution on p. 408.)
Find 12 of 0.1
Exercise 6.223
Find 0.09 of 0.003
Exercise 6.224 (Solution on p. 408.)
Find 1.02 of 0.9801
Exercise 6.225
Find 0.01 of the sum of 3.6 and 12.18
Exercise 6.226 (Solution on p. 408.)
Find 0.2 of the sum of 0.194 and 1.07
Exercise 6.227
Find the dierence of 6.1 of 2.7 and 2.7 of 4.03
Exercise 6.228 (Solution on p. 408.)
Find the dierence of 0.071 of 42 and 0.003 of 9.2
Exercise 6.229
If a person earns $8.55 an hour, how much does he earn in twenty-ve hundredths of an hour?
Exercise 6.230 (Solution on p. 408.)
A man buys 14 items at $1.16 each. What is the total cost?
Exercise 6.231
In the problem above, how much is the total cost if 0.065 sales tax is added?
Exercise 6.232 (Solution on p. 408.)
A river rafting trip is supposed to last for 10 days and each day 6 miles is to be rafted. On the third
day a person falls out of the raft after only 52 of that day's mileage. If this person gets discouraged
and quits, what fraction of the entire trip did he complete?
368 CHAPTER 6. DECIMALS

Exercise 6.233
A woman starts the day with $42.28. She buys one item for $8.95 and another for $6.68. She then
buys another item for sixty two-hundredths of the remaining amount. How much money does she
have left?
Calculator Problems
For the following 10 problems, use a calculator to determine each product. If the calculator will not provide
the exact product, round the results to ve decimal places.
Exercise 6.234 (Solution on p. 408.)
0.019 · 0.321
Exercise 6.235
0.261 · 1.96
Exercise 6.236 (Solution on p. 408.)
4.826 · 4.827
Exercise 6.237
2
(9.46)
Exercise 6.238 (Solution on p. 408.)
2
(0.012)
Exercise 6.239
0.00037 · 0.0065
Exercise 6.240 (Solution on p. 408.)
0.002 · 0.0009
Exercise 6.241
0.1286 · 0.7699
Exercise 6.242 (Solution on p. 408.)
0.01 · 0.00000471
Exercise 6.243
0.00198709 · 0.03

6.6.7.1 Exercises for Review

Exercise 6.244 (Solution on p. 408.)


(Section 2.3) Find the value, if it exists, of 0 ÷ 15.
Exercise 6.245
(Section 3.5) Find the greatest common factor of 210, 231, and 357.
Exercise 6.246 (Solution on p. 409.)
(Section 4.4) Reduce 2,280
156 to lowest terms.
Exercise 6.247
(Section 6.2) Write "fourteen and one hundred twenty-one ten-thousandths, using digits."
Exercise 6.248 (Solution on p. 409.)
(Section 6.5) Subtract 6.882 from 8.661 and round the result to two decimal places.
369

6.7 Division of Decimals7


6.7.1 Section Overview
• The Logic Behind the Method
• A Method of Dividing a Decimal By a Nonzero Whole Number
• A Method of Dividing a Decimal by a Nonzero Decimal
• Dividing Decimals by Powers of 10

6.7.2 The Logic Behind the Method


As we have done with addition, subtraction, and multiplication of decimals, we will study a method of
division of decimals by converting them to fractions, then we will make a general rule.

We will proceed by using this example: Divide 196.8 by 6.

32
6)196.8
18
16
12
4

We have, up to this point, divided 196.8 by 6 and have gotten a quotient of 32 with a remainder of 4. If we
follow our intuition and bring down the .8, we have the division 4.8 ÷ 6.

4.8 ÷ 6 = 8
4 10 ÷6
= 48 ÷ 6
10 1
8
)48 1
= 10 · )6
1
8
= 10

Thus, 4.8 ÷ 6 = .8.

Now, our intuition and experience with division direct us to place the .8 immediately to the right of 32.

From these observations, we suggest the following method of division.


7 This content is available online at <http://cnx.org/content/m34968/1.2/>.
370 CHAPTER 6. DECIMALS

6.7.3 A Method of Dividing a Decimal by a Nonzero Whole Number


Method of Dividing a Decimal by a Nonzero Whole Number
To divide a decimal by a nonzero whole number:

1. Write a decimal point above the division line and directly over the decimal point of the dividend.
2. Proceed to divide as if both numbers were whole numbers.
3. If, in the quotient, the rst nonzero digit occurs to the right of the decimal point, but not in the tenths
position, place a zero in each position between the decimal point and the rst nonzero digit of the
quotient.

6.7.3.1 Sample Set A

Find the decimal representations of the following quotients.


Example 6.44
114.1 ÷ 7 = 7

16.3
7)114.1
7
44
42
2.1
2.1
0

Thus, 114.1 ÷ 7 = 16.3.

Check: If 114.1 ÷ 7 = 16.3, then 7 · 16.3 should equal 114.1.


4 2
16.3
7
114.1 True.
Example 6.45
0.02068 ÷ 4

Place zeros in the tenths and hundredths positions. (See Step 3.)

Thus, 0.02068 ÷ 4 = 0.00517.


371

6.7.3.2 Practice Set A

Find the following quotients.


Exercise 6.249 (Solution on p. 409.)
184.5 ÷ 3
Exercise 6.250 (Solution on p. 409.)
16.956 ÷ 9
Exercise 6.251 (Solution on p. 409.)
0.2964 ÷ 4
Exercise 6.252 (Solution on p. 409.)
0.000496 ÷ 8

6.7.4 A Method of Dividing a Decimal By a Nonzero Decimal


Now that we can divide decimals by nonzero whole numbers, we are in a position to divide decimals by a
nonzero decimal. We will do so by converting a division by a decimal into a division by a whole number, a
process with which we are already familiar. We'll illustrate the method using this example: Divide 4.32 by
1.8.
32 ÷ 1 8 .
Let's look at this problem as 4 100 10
32 ÷ 1 8
4 100 =
32
4 100
10 8
1 10
432

= 100
18
10

The divisor is 18 18
10 . We can convert 10 into a whole number if we multiply it by 10.
1
18 · 10 = 18 · )10
= 18
10 )10 1
1

But, we know from our experience with fractions, that if we multiply the denominator of a fraction by a
nonzero whole number, we must multiply the numerator by that same nonzero whole number. Thus, when
converting 18 432
10 to a whole number by multiplying it by 10, we must also multiply the numerator 100 by 10.
1
432 432 )10 432·1 = 432
100 · 10 = )100 · 1 = 10·1 10
10

= 43 10
2

= 43.2

We have converted the division 4.32 ÷ 1.8 into the division 43.2 ÷ 18, that is,

1.8)4.32 → 18)43.2

Notice what has occurred.

If we "move" the decimal point of the divisor one digit to the right, we must also "move" the decimal point
of the dividend one place to the right. The word "move" actually indicates the process of multiplication by
a power of 10.
372 CHAPTER 6. DECIMALS

Method of Dividing a Decimal by a Decimal Number


To divide a decimal by a nonzero decimal,
1. Convert the divisor to a whole number by moving the decimal point to the position immediately to
the right of the divisor's last digit.
2. Move the decimal point of the dividend to the right the same number of digits it was moved in the
divisor.
3. Set the decimal point in the quotient by placing a decimal point directly above the newly located
decimal point in the dividend.
4. Divide as usual.

6.7.4.1 Sample Set B

Find the following quotients.


Example 6.46
32.66 ÷ 7.1

7.1)32.66

The divisor has one decimal place.


Move the decimal point of both the divisor and the dividend 1 place to the right.
Set the decimal point.
Divide as usual.

Thus, 32.66 ÷ 7.1 = 4.6.

Check: 32.66 ÷ 7.1 = 4.6 if 4.6 × 7.1 = 32.66

4.6
×7.1
46
322
32.66 True.
Example 6.47
1.0773 ÷ 0.513

The divisor has 3 decimal places.


373

Move the decimal point of both the divisor and the dividend 3 places to the right.
Set the decimal place and divide.

Thus, 1.0773 ÷ 0.513 = 2.1.

Checking by multiplying 2.1 and 0.513 will convince us that we have obtained the correct result.
(Try it.)
Example 6.48
12 ÷ 0.00032

0.00032)12.00000

The divisor has 5 decimal places.


Move the decimal point of both the divisor and the dividend 5 places to the right. We will need
to add 5 zeros to 12.
Set the decimal place and divide.

This is now the same as the division of whole numbers.

37500.
32)1200000.
96
240
224
160
160
000

Checking assures us that 12 ÷ 0.00032 = 37, 500.

6.7.4.2 Practice Set B

Find the decimal representation of each quotient.


Exercise 6.253 (Solution on p. 409.)
9.176 ÷ 3.1
Exercise 6.254 (Solution on p. 409.)
5.0838 ÷ 1.11
Exercise 6.255 (Solution on p. 409.)
16 ÷ 0.0004
Exercise 6.256 (Solution on p. 409.)
8, 162.41 ÷ 10
Exercise 6.257 (Solution on p. 409.)
8, 162.41 ÷ 100
Exercise 6.258 (Solution on p. 409.)
8, 162.41 ÷ 1, 000
374 CHAPTER 6. DECIMALS

Exercise 6.259 (Solution on p. 409.)


8, 162.41 ÷ 10, 000

6.7.5 Calculators
Calculators can be useful for nding quotients of decimal numbers. As we have seen with the other calculator
operations, we can sometimes expect only approximate results. We are alerted to approximate results when
the calculator display is lled with digits. We know it is possible that the operation may produce more digits
than the calculator has the ability to show. For example, the multiplication

| {z } × 0.4567
0.12345 | {z }
5 decimal 4 decimal
places places

produces 5 + 4 = 9 decimal places. An eight-digit display calculator only has the ability to show eight digits,
and an approximation results. The way to recognize a possible approximation is illustrated in problem 3 of
the next sample set.

6.7.5.1 Sample Set C

Find each quotient using a calculator. If the result is an approximation, round to ve decimal places.
Example 6.49
12.596 ÷ 4.7

Display Reads
Type 12.596 12.596
Press ÷ 12.596
Type 4.7 4.7
Press = 2.68

Table 6.22

Since the display is not lled, we expect this to be an accurate result.


Example 6.50
0.5696376 ÷ 0.00123

Display Reads
Type .5696376 0.5696376
Press ÷ 0.5696376
Type .00123 0.00123
Press = 463.12

Table 6.23

Since the display is not lled, we expect this result to be accurate.


Example 6.51
0.8215199 ÷ 4.113
375

Display Reads
Type .8215199 0.8215199
Press ÷ 0.8215199
Type 4.113 4.113
Press = 0.1997373

Table 6.24

There are EIGHT DIGITS  DISPLAY FILLED! BE AWARE OF POSSIBLE APPROXI-


MATIONS.
3
We can check for a possible approximation in the following way. Since the division 4)12 can be
checked by multiplying 4 and 3, we can check our division by performing the multiplication

{z } × 0.1997373
|4.113 | {z }
3 decimal 7 decimal
places places

This multiplication produces 3 + 7 = 10 decimal digits. But our suspected quotient contains only 8
decimal digits. We conclude that the answer is an approximation. Then, rounding to ve decimal
places, we get 0.19974.

6.7.5.2 Practice Set C

Find each quotient using a calculator. If the result is an approximation, round to four decimal places.
Exercise 6.260 (Solution on p. 409.)
42.49778 ÷ 14.261
Exercise 6.261 (Solution on p. 409.)
0.001455 ÷ 0.291
Exercise 6.262 (Solution on p. 409.)
7.459085 ÷ 2.1192

6.7.6 Dividing Decimals By Powers of 10


In problems 4 and 5 of Section 6.7.4.2 (Practice Set B), we found the decimal representations of 8, 162.41 ÷ 10
and 8, 162.41 ÷ 100. Let's look at each of these again and then, from these observations, make a general
statement regarding division of a decimal number by a power of 10.
376 CHAPTER 6. DECIMALS

816.241
10)8162.410
80
16
10
62
60
24
20
41
40
10
10
0

Thus, 8, 162.41 ÷ 10 = 816.241.

Notice that the divisor 10 is composed of one 0 and that the quotient 816.241 can be obtained from the
dividend 8,162.41 by moving the decimal point one place to the left.

81.6241
100)8162.4100
800
162
100
62 4
60 0
2 41
2 00
410
400
100
100
0

Thus, 8, 162.41 ÷ 100 = 81.6241.

Notice that the divisor 100 is composed of two 0's and that the quotient 81.6241 can be obtained from the
dividend by moving the decimal point two places to the left.

Using these observations, we can suggest the following method for dividing decimal numbers by powers of
10.
377

Dividing a Decimal Fraction by a Power of 10


To divide a decimal fraction by a power of 10, move the decimal point of the decimal fraction to the left as
many places as there are zeros in the power of 10. Add zeros if necessary.

6.7.6.1 Sample Set D

Find each quotient.


Example 6.52
9, 248.6 ÷ 100
Since there are 2 zeros in this power of 10, we move the decimal point 2 places to the left.

Example 6.53
3.28 ÷ 10, 000

Since there are 4 zeros in this power of 10, we move the decimal point 4 places to the left. To do
so, we need to add three zeros.

6.7.6.2 Practice Set D

Find the decimal representation of each quotient.


Exercise 6.263 (Solution on p. 409.)
182.5 ÷ 10
Exercise 6.264 (Solution on p. 409.)
182.5 ÷ 100
Exercise 6.265 (Solution on p. 409.)
182.5 ÷ 1, 000
Exercise 6.266 (Solution on p. 409.)
182.5 ÷ 10, 000
Exercise 6.267 (Solution on p. 409.)
646.18 ÷ 100
Exercise 6.268 (Solution on p. 409.)
21.926 ÷ 1, 000

6.7.7 Exercises
For the following 30 problems, nd the decimal representation of each quotient. Use a calculator to check
each result.
Exercise 6.269 (Solution on p. 409.)
4.8 ÷ 3
Exercise 6.270
16.8 ÷ 8
Exercise 6.271 (Solution on p. 409.)
18.5 ÷ 5
378 CHAPTER 6. DECIMALS

Exercise 6.272
12.33 ÷ 3
Exercise 6.273 (Solution on p. 409.)
54.36 ÷ 9
Exercise 6.274
73.56 ÷ 12
Exercise 6.275 (Solution on p. 410.)
159.46 ÷ 17
Exercise 6.276
12.16 ÷ 64
Exercise 6.277 (Solution on p. 410.)
37.26 ÷ 81
Exercise 6.278
439.35 ÷ 435
Exercise 6.279 (Solution on p. 410.)
36.98 ÷ 4.3
Exercise 6.280
46.41 ÷ 9.1
Exercise 6.281 (Solution on p. 410.)
3.6 ÷ 1.5
Exercise 6.282
0.68 ÷ 1.7
Exercise 6.283 (Solution on p. 410.)
50.301 ÷ 8.1
Exercise 6.284
2.832 ÷ 0.4
Exercise 6.285 (Solution on p. 410.)
4.7524 ÷ 2.18
Exercise 6.286
16.2409 ÷ 4.03
Exercise 6.287 (Solution on p. 410.)
1.002001 ÷ 1.001
Exercise 6.288
25.050025 ÷ 5.005
Exercise 6.289 (Solution on p. 410.)
12.4 ÷ 3.1
Exercise 6.290
0.48 ÷ 0.08
Exercise 6.291 (Solution on p. 410.)
30.24 ÷ 2.16
Exercise 6.292
48.87 ÷ 0.87
Exercise 6.293 (Solution on p. 410.)
12.321 ÷ 0.111
Exercise 6.294
64, 351.006 ÷ 10
379

Exercise 6.295 (Solution on p. 410.)


64, 351.006 ÷ 100
Exercise 6.296
64, 351.006 ÷ 1, 000
Exercise 6.297 (Solution on p. 410.)
64, 351.006 ÷ 1, 000, 000
Exercise 6.298
0.43 ÷ 100
For the following 5 problems, nd each quotient. Round to the specied position. A calculator may be used.
Exercise 6.299 (Solution on p. 410.)
11.2944 ÷ 6.24

Actual Quotient Tenths Hundredths Thousandths

Table 6.25

Exercise 6.300
45.32931 ÷ 9.01

Actual Quotient Tenths Hundredths Thousandths

Table 6.26

Exercise 6.301 (Solution on p. 410.)


3.18186 ÷ 0.66

Actual Quotient Tenths Hundredths Thousandths

Table 6.27

Exercise 6.302
4.3636 ÷ 4

Actual Quotient Tenths Hundredths Thousandths

Table 6.28

Exercise 6.303 (Solution on p. 410.)


0.00006318 ÷ 0.018

Actual Quotient Tenths Hundredths Thousandths


380 CHAPTER 6. DECIMALS

Table 6.29

For the following 9 problems, nd each solution.


Exercise 6.304
Divide the product of 7.4 and 4.1 by 2.6.
Exercise 6.305 (Solution on p. 411.)
Divide the product of 11.01 and 0.003 by 2.56 and round to two decimal places.
Exercise 6.306
Divide the dierence of the products of 2.1 and 9.3, and 4.6 and 0.8 by 0.07 and round to one
decimal place.
Exercise 6.307 (Solution on p. 411.)
A ring costing $567.08 is to be paid o in equal monthly payments of $46.84. In how many months
will the ring be paid o?
Exercise 6.308
Six cans of cola cost $2.58. What is the price of one can?
Exercise 6.309 (Solution on p. 411.)
A family traveled 538.56 miles in their car in one day on their vacation. If their car used 19.8
gallons of gas, how many miles per gallon did it get?
Exercise 6.310
Three college students decide to rent an apartment together. The rent is $812.50 per month. How
much must each person contribute toward the rent?
Exercise 6.311 (Solution on p. 411.)
A woman notices that on slow speed her video cassette recorder runs through 296.80 tape units
in 10 minutes and at fast speed through 1098.16 tape units. How many times faster is fast speed
than slow speed?
Exercise 6.312
A class of 34 rst semester business law students pay a total of $1,354.90, disregarding sales tax,
for their law textbooks. What is the cost of each book?
Calculator Problems
For the following problems, use calculator to nd the quotients. If the result is approximate (see Sample Set
C Example 6.51) round the result to three decimal places.
Exercise 6.313 (Solution on p. 411.)
3.8994 ÷ 2.01
Exercise 6.314
0.067444 ÷ 0.052
Exercise 6.315 (Solution on p. 411.)
14, 115.628 ÷ 484.74
Exercise 6.316
219, 709.36 ÷ 9941.6
Exercise 6.317 (Solution on p. 411.)
0.0852092 ÷ 0.49271
Exercise 6.318
2.4858225 ÷ 1.11611
Exercise 6.319 (Solution on p. 411.)
0.123432 ÷ 0.1111
Exercise 6.320
2.102838 ÷ 1.0305
381

6.7.7.1 Exercises for Review

Exercise 6.321 (Solution on p. 411.)


(Section 4.3) Convert 4 78 to an improper fraction.
Exercise 6.322
(Section 4.7) 2
7 of what number is 45 ?
Exercise 6.323 (Solution on p. 411.)
(Section 5.3) Find the sum. 15
4 7
+ 10 + 53 .
Exercise 6.324
(Section 6.4) Round 0.01628 to the nearest ten-thousandths.
Exercise 6.325 (Solution on p. 411.)
(Section 6.6) Find the product (2.06)(1.39)

6.8 Nonterminating Divisions8


6.8.1 Section Overview
• Nonterminating Divisions
• Denoting Nonterminating Quotients

6.8.2 Nonterminating Divisions


Let's consider two divisions:

1. 9.8 ÷ 3.5
2. 4 ÷ 3

Terminating Divisions
Previously, we have considered divisions like example 1, which is an example of a terminating division. A
terminating division is a division in which the quotient terminates after several divisions (the remainder
is zero ).

Exact Divisions
The quotient in this problem terminates in the tenths position. Terminating divisions are also called exact
divisions.

Nonterminating Division
The division in example 2 is an example of a nonterminating division. A non-terminating division is a
division that, regardless of how far we carry it out, always has a remainder.
8 This content is available online at <http://cnx.org/content/m34969/1.2/>.
382 CHAPTER 6. DECIMALS

Repeating Decimal
We can see that the pattern in the brace is repeated endlessly. Such a decimal quotient is called a repeating
decimal.

6.8.3 Denoting Nonterminating Quotients


We use three dots at the end of a number to indicate that a pattern repeats itself endlessly.

4 ÷ 3 = 1.333 . . .

Another way, aside from using three dots, of denoting an endlessly repeating pattern is to write a bar ( )
above the repeating sequence of digits.

4 ÷ 3 = 1.3

The bar indicates the repeated pattern of 3.

Repeating patterns in a division can be discovered in two ways:

1. As the division process progresses, should the remainder ever be the same as the dividend, it can be
concluded that the division is nonterminating and that the pattern in the quotient repeats. This fact
is illustrated in Example 6.54 of Section 6.8.3.1 (Sample Set A).
2. As the division process progresses, should the "product, dierence" pattern ever repeat two consecutive
times, it can be concluded that the division is nonterminating and that the pattern in the quotient
repeats. This fact is illustrated in Example 6.55 and 4 of Section 6.8.3.1 (Sample Set A).

6.8.3.1 Sample Set A

Carry out each division until the repeating pattern can be determined.
Example 6.54
100 ÷ 27
383

3.70370
27)100.00000
81
19 0
18 9
100
81
190
189
When the remainder is identical to the dividend, the division is nonterminating. This implies that
the pattern in the quotient repeats.

100 ÷ 27 = 3.70370370 . . . The repeating block is 703.

100 ÷ 27 = 3.703
Example 6.55
1÷9

We see that this product, dierencepattern repeats. We can conclude that the division is nonter-
minating and that the quotient repeats.

1 ÷ 9 = 0.111 . . . The repeating block is 1.

1 ÷ 9 = 0.1
Example 6.56
Divide 2 by 11 and round to 3 decimal places.

Since we wish to round the quotient to three decimal places, we'll carry out the division so that
the quotient has four decimal places.
384 CHAPTER 6. DECIMALS

.1818
11)2.0000
11
90
88
20
11
90

The number .1818 rounded to three decimal places is .182. Thus, correct to three decimal places,

2 ÷ 11 = 0.182
Example 6.57
Divide 1 by 6.

We see that this product, dierence pattern repeats. We can conclude that the division is non-
terminating and that the quotient repeats at the 6.

1 ÷ 6 = 0.16

6.8.3.2 Practice Set A

Carry out the following divisions until the repeating pattern can be determined.
Exercise 6.326 (Solution on p. 411.)
1÷3
Exercise 6.327 (Solution on p. 411.)
5÷6
Exercise 6.328 (Solution on p. 411.)
11 ÷ 9
Exercise 6.329 (Solution on p. 411.)
17 ÷ 9
Exercise 6.330 (Solution on p. 411.)
Divide 7 by 6 and round to 2 decimal places.
Exercise 6.331 (Solution on p. 411.)
Divide 400 by 11 and round to 4 decimal places.
385

6.8.4 Exercises
For the following 20 problems, carry out each division until the repeating pattern is determined. If a repeating
pattern is not apparent, round the quotient to three decimal places.
Exercise 6.332 (Solution on p. 411.)
4÷9
Exercise 6.333
8 ÷ 11
Exercise 6.334 (Solution on p. 411.)
4 ÷ 25
Exercise 6.335
5÷6
Exercise 6.336 (Solution on p. 411.)
1÷7
Exercise 6.337
3 ÷ 1.1
Exercise 6.338 (Solution on p. 411.)
20 ÷ 1.9
Exercise 6.339
10 ÷ 2.7
Exercise 6.340 (Solution on p. 411.)
1.11 ÷ 9.9
Exercise 6.341
8.08 ÷ 3.1
Exercise 6.342 (Solution on p. 411.)
51 ÷ 8.2
Exercise 6.343
0.213 ÷ 0.31
Exercise 6.344 (Solution on p. 411.)
0.009 ÷ 1.1
Exercise 6.345
6.03 ÷ 1.9
Exercise 6.346 (Solution on p. 411.)
0.518 ÷ 0.62
Exercise 6.347
1.55 ÷ 0.27
Exercise 6.348 (Solution on p. 411.)
0.333 ÷ 0.999
Exercise 6.349
0.444 ÷ 0.999
Exercise 6.350 (Solution on p. 412.)
0.555 ÷ 0.27
Exercise 6.351
3.8 ÷ 0.99
Calculator Problems
For the following 10 problems, use a calculator to perform each division.
386 CHAPTER 6. DECIMALS

Exercise 6.352 (Solution on p. 412.)


7÷9
Exercise 6.353
8 ÷ 11
Exercise 6.354 (Solution on p. 412.)
14 ÷ 27
Exercise 6.355
1 ÷ 44
Exercise 6.356 (Solution on p. 412.)
2 ÷ 44
Exercise 6.357
0.7 ÷ 0.9 (Compare this with Exercise 6.352.)
Exercise 6.358 (Solution on p. 412.)
80 ÷ 110 (Compare this with Exercise 6.353.)
Exercise 6.359
0.0707 ÷ 0.7070
Exercise 6.360 (Solution on p. 412.)
0.1414 ÷ 0.2020
Exercise 6.361
1 ÷ 0.9999999

6.8.4.1 Exercise for Review

Exercise 6.362 (Solution on p. 412.)


(Section 1.2) In the number 411,105, how many ten thousands are there?
Exercise 6.363
(Section 2.3) Find the quotient, if it exists. 17 ÷ 0.
Exercise 6.364 (Solution on p. 412.)
(Section 3.6) Find the least common multiple of 45, 63, and 98.
Exercise 6.365
(Section 6.5) Subtract 8.01629 from 9.00187 and round the result to three decimal places.
Exercise 6.366 (Solution on p. 412.)
(Section 6.7) Find the quotient. 104.06 ÷ 12.1.

6.9 Converting a Fraction to a Decimal9


Now that we have studied and practiced dividing with decimals, we are also able to convert a fraction to a
decimal. To do so we need only recall that a fraction bar can also be a division symbol. Thus, 34 not only
means "3 objects out of 4," but can also mean "3 divided by 4."
9 This content is available online at <http://cnx.org/content/m34970/1.2/>.
387

6.9.1 Sample Set A


Convert the following fractions to decimals. If the division is nonterminating, round to two decimal places.
Example 6.58
4. Divide 3 by 4.
3

.75
4)3.00
28
20
20
0

Thus, 3
4 = 0.75.
Example 6.59
1
5 Divide 1 by 5.

.2
5)1.0
1.0
0

Thus, 1
5 = 0.2
Example 6.60
6. Divide 5 by 6.
5

5
6 = 0.833 · · · We are to round to two decimal places.

Thus, 5
6 = 0.83 to two decimal places.
Example 6.61
5 18 . Note that 5 18 = 5 + 18 .

Convert 1
8 to a decimal.
388 CHAPTER 6. DECIMALS

.125
8)1.000
8
20
16
40
40
0
1
8 = .125

Thus, 5 18 = 5 + 1
8 = 5 + .125 = 5.125.
Example 6.62
0.16 14 . This is a complex decimal.

Note that the 6 is in the hundredths position.

The number 0.16 14 is read as "sixteen and one-fourth hundredths."


13
16 1 )65
= 13 13
16·4+1 65

0.16 14 = 1004 = 100


4
= 4
= · 1 ·1
4·20 = 80
)100
100
1
4
20

Now, convert 13
80 to a decimal.

.1625
80)13.0000
80
5 00
4 80
200
160
400
400
0

Thus, 0.16 14 = 0.1625.

6.9.2 Practice Set A


Convert the following fractions and complex decimals to decimals (in which no proper fractions appear). If
the divison is nonterminating, round to two decimal places.
Exercise 6.367 (Solution on p. 412.)
1
4
389

Exercise 6.368 (Solution on p. 412.)


1
25
Exercise 6.369 (Solution on p. 412.)
1
6
Exercise 6.370 (Solution on p. 412.)
15
16
Exercise 6.371 (Solution on p. 412.)
0.9 21
Exercise 6.372 (Solution on p. 412.)
8.0126 83

6.9.3 Exercises
For the following 30 problems, convert each fraction or complex decimal number to a decimal (in which no
proper fractions appear).
Exercise 6.373 (Solution on p. 412.)
1
2
Exercise 6.374
4
5
Exercise 6.375 (Solution on p. 412.)
7
8
Exercise 6.376
5
8
Exercise 6.377 (Solution on p. 412.)
3
5
Exercise 6.378
2
5
Exercise 6.379 (Solution on p. 412.)
1
25
Exercise 6.380
3
25
Exercise 6.381 (Solution on p. 412.)
1
20
Exercise 6.382
1
15
Exercise 6.383 (Solution on p. 412.)
1
50
Exercise 6.384
1
75
Exercise 6.385 (Solution on p. 412.)
1
3
Exercise 6.386
5
6
Exercise 6.387 (Solution on p. 412.)
3
16
390 CHAPTER 6. DECIMALS

Exercise 6.388
9
16
Exercise 6.389 (Solution on p. 412.)
1
27
Exercise 6.390
5
27
Exercise 6.391 (Solution on p. 412.)
7
13
Exercise 6.392
9
14
Exercise 6.393 (Solution on p. 413.)
7 23
Exercise 6.394
5
8 16
Exercise 6.395 (Solution on p. 413.)
2
1 15
Exercise 6.396
65 22
5

Exercise 6.397 (Solution on p. 413.)


101 25
6

Exercise 6.398
0.1 12
Exercise 6.399 (Solution on p. 413.)
0.24 18
Exercise 6.400
5.66 23
Exercise 6.401 (Solution on p. 413.)
810.3106 16
5

Exercise 6.402
4.1 91
For the following 18 problems, convert each fraction to a decimal. Round to ve decimal places.
Exercise 6.403 (Solution on p. 413.)
1
9
Exercise 6.404
2
9
Exercise 6.405 (Solution on p. 413.)
3
9
Exercise 6.406
4
9
Exercise 6.407 (Solution on p. 413.)
5
9
Exercise 6.408
6
9
Exercise 6.409 (Solution on p. 413.)
7
9
391

Exercise 6.410
8
9
Exercise 6.411 (Solution on p. 413.)
1
11
Exercise 6.412
2
11
Exercise 6.413 (Solution on p. 413.)
3
11
Exercise 6.414
4
11
Exercise 6.415 (Solution on p. 413.)
5
11
Exercise 6.416
6
11
Exercise 6.417 (Solution on p. 413.)
7
11
Exercise 6.418
8
11
Exercise 6.419 (Solution on p. 413.)
9
11
Exercise 6.420
10
11
Calculator Problems
For the following problems, use a calculator to convert each fraction to a decimal. If no repeating pattern
seems to exist, round to four decimal places.
Exercise 6.421 (Solution on p. 413.)
16
125
Exercise 6.422
85
311
Exercise 6.423 (Solution on p. 413.)
192
197
Exercise 6.424
1
1469
Exercise 6.425 (Solution on p. 413.)
4
21,015
Exercise 6.426
81,426
106,001
Exercise 6.427 (Solution on p. 413.)
16,501
426
392 CHAPTER 6. DECIMALS

6.9.3.1 Exercises for Review

Exercise 6.428
(Section 1.4) Round 2,105,106 to the nearest hundred thousand.
Exercise 6.429 (Solution on p. 413.)
(Section 4.7) 8
5 of what number is 32 ?
Exercise 6.430
(Section 5.5) Arrange 1 16
9
, 1 58 , and 1 12
7
in increasing order.
Exercise 6.431 (Solution on p. 413.)
(Section 6.3) Convert the complex decimal 3.6 54 to a fraction.
Exercise 6.432
(Section 6.8) Find the quotient. 30 ÷ 1.1.

6.10 Combinations of Operations with Decimals and Fractions10


Having considered operations with decimals and fractions, we now consider operations that involve both
decimals and fractions.

6.10.1 Sample Set A


Perform the following operations.
Example 6.63
0.38 · 14 . Convert both numbers to decimals or both numbers to fractions. We'll convert to decimals.

.25
4)1.00
8
20
20
0
To convert 1
4 to a decimal, divide 1 by 4.

Now multiply 0.38 and .25.


1
4
.3 8
×.25
190
76
.0950

Thus, 0.38 · 1
4 = 0.095.

10 This content is available online at <http://cnx.org/content/m34971/1.2/>.


393

In the problems that follow, the conversions from fraction to decimal, or decimal to fraction, and
some of the additions, subtraction, multiplications, and divisions will be left to you.
Example 6.64
1.85 + 3
8 · 4.1 Convert 3
8 to a decimal.

1.85 + 0.375 · 4.1 Multiply before adding.

1.85 + 1.5375 Now add.

3.3875
Example 6.65
5 4
− 0.28 Convert 0.28 to a fraction.

13 5

5 4 28 
− 100 = 5 7
− 25
 4
13 5 13 5
5 20 7

= 13 25 − 25
1 1
)5 )13
= ·
)13 )25
1 5
1
= 5

Example 6.66
125
0.125 1 1
1 13
+ 16 − 0.1211 = 1000
4 + 16 − 0.1211
3
1
1
= 8
4 + 16 − 0.1211
3
1 3 1
= 8 · 416 − 0.1211
+
3 1
= 32 + 16 − 0.1211
3 2 5
= 32 + 32 − 0.1211 = 32 − 0.1211
= 0.15625 − 0.1211
= 0.03515 Convert this to fraction form
3515
= 100,000
703
= 20,000

6.10.2 Practice Set A


Perform the following operations.
Exercise 6.433 (Solution on p. 413.)
3
5 + 1.6
Exercise 6.434 (Solution on p. 413.)
8.91 + 1
5 · 1.6
Exercise 6.435 (Solution on p. 413.)
9
1 16 6.12 + 25
7

Exercise 6.436 (Solution on p. 413.)


0.156
1 11
− 0.05
15
394 CHAPTER 6. DECIMALS

6.10.3 Exercises
Exercise 6.437 (Solution on p. 413.)
10 + 0.7
3

Exercise 6.438
1
5 + 0.1
Exercise 6.439 (Solution on p. 414.)
5
8 − 0.513
Exercise 6.440
67
0.418 − 200
Exercise 6.441 (Solution on p. 414.)
0.22 · 1
4
Exercise 6.442
3
5 · 8.4
Exercise 6.443 (Solution on p. 414.)
25 · 3.19
1

Exercise 6.444
20 ÷ 0.05
3

Exercise 6.445 (Solution on p. 414.)


40 ÷ 0.25
7

Exercise 6.446
1
1 15 ÷ 0.9 · 0.12
Exercise 6.447 (Solution on p. 414.)
9.26 + 1
4 · 0.81
Exercise 6.448
0.588 + 40
1
· 0.24
Exercise 6.449 (Solution on p. 414.)
20 + 3.62 ·
1 3
8
Exercise 6.450
7 + 0.15÷ 30
3

Exercise 6.451 (Solution on p. 414.)


15
16 · 10 − 0.5
7


Exercise 6.452
0.2 · 20
7
+ 1.1143


Exercise 6.453 (Solution on p. 414.)


3
· 0.875 + 1

4 8
Exercise 6.454
14 + 0.119
5.198 − 0.26 · 250
Exercise 6.455 (Solution on p. 414.)
2
0.5 14 + (0.3)
Exercise 6.456
2
(1.4) − 1.6 12
Exercise 6.457 (Solution on p. 414.)
3 2 2
− 0.000625 + (1.1)

8
Exercise 6.458
2
(0.6) · 20
1 1

− 25
395

Exercise 6.459 (Solution on p. 414.)


1 2
− 0.125

2
Exercise 6.460
0.75 5
4 21
+ 12
Exercise 6.461 (Solution on p. 414.)
0.375
 
1
1
2 16
− 33
Exercise
 1 6.462
14
8 13 · 2.25
9
+ 25
Exercise 6.463 (Solution on p. 414.)
0.32
12
35
0.35
Exercise
“√ ” 6.464
49
64 −5 0.125
1.375

6.10.3.1 Exercises for Review

Exercise 6.465 (Solution on p. 414.)


(Section 2.5) Is 21,480 divisible by 3?
Exercise 6.466
(Section 3.2) Expand 144 . Do not nd the actual value.
Exercise 6.467 (Solution on p. 414.)
(Section 3.3) Find the prime factorization of 15,400.
Exercise 6.468
(Section 6.3) Convert 8.016 to a fraction.
Exercise 6.469 (Solution on p. 414.)
(Section 6.9) Find the quotient. 16 ÷ 27.

6.11 Summary of Key Concepts11


6.11.1 Summary of Key Concepts
Decimal Point (Section 6.2)
A decimal point is a point that separates the units digit from the tenths digit.

Decimal or Decimal Fraction (Section 6.2)


A decimal fraction is a fraction whose denominator is a power of ten.

Converting a Decimal to a Fraction (Section 6.3)


Decimals can be converted to fractions by saying the decimal number in words, then writing what was said.

Rounding Decimals (Section 6.4)


Decimals are rounded in much the same way whole numbers are rounded.

Addition and Subtraction of Decimals (Section 6.5)


To add or subtract decimals,
11 This content is available online at <http://cnx.org/content/m34972/1.2/>.
396 CHAPTER 6. DECIMALS

1. Align the numbers vertically so that the decimal points line up under each other and the corresponding
decimal positions are in the same column.
2. Add or subtract the numbers as if they were whole numbers.
3. Place a decimal point in the resulting sum directly under the other decimal points.
Multiplication of Decimals (Section 6.6)
To multiply two decimals,
1. Multiply the numbers as if they were whole numbers.
2. Find the sum of the number of decimal places in the factors.
3. The number of decimal places in the product is the number found in step 2.
Multiplying Decimals by Powers of 10 (Section 6.6)
To multiply a decimal by a power of 10, move the decimal point to the right as many places as there are
zeros in the power of ten. Add zeros if necessary.

Division of a Decimal by a Decimal (Section 6.7)


To divide a decimal by a nonzero decimal,
1. Convert the divisor to a whole number by moving the decimal point until it appears to the right of the
divisor's last digit.
2. Move the decimal point of the dividend to the right the same number of digits it was moved in the
divisor.
3. Proceed to divide.
4. Locate the decimal in the answer by bringing it straight up from the dividend.
Dividing Decimals by Powers of 10 (Section 6.7)
To divide a decimal by a power of 10, move the decimal point to the left as many places as there are zeros
in the power of ten. Add zeros if necessary.

Terminating Divisions (Section 6.8)


A terminating division is a division in which the quotient terminates after several divisions. Terminating
divisions are also called exact divisions.

Nonterminating Divisions (Section 6.8)


A nonterminating division is a division that, regardless of how far it is carried out, always has a remainder.
Nonterminating divisions are also called nonexact divisions.

Converting Fractions to Decimals (Section 6.9)


A fraction can be converted to a decimal by dividing the numerator by the denominator.

6.12 Exercise Supplement12


6.12.1 Exercise Supplement
6.12.1.1 Reading and Writing Decimals (Section 6.2)

Exercise 6.470 (Solution on p. 414.)


The decimal digit that appears two places to the right of the decimal point is in the
position.
Exercise 6.471
The decimal digit that appears four places to the right of the decimal point is in the
position.
12 This content is available online at <http://cnx.org/content/m34978/1.2/>.
397

For problems 3-8, read each decimal by writing it in words.


Exercise 6.472 (Solution on p. 414.)
7.2
Exercise 6.473
8.105
Exercise 6.474 (Solution on p. 414.)
16.52
Exercise 6.475
5.9271
Exercise 6.476 (Solution on p. 414.)
0.005
Exercise 6.477
4.01701
For problems 9-13, write each decimal using digits.
Exercise 6.478 (Solution on p. 414.)
Nine and twelve-hundredths.
Exercise 6.479
Two and one hundred seventy-seven thousandths.
Exercise 6.480 (Solution on p. 414.)
Fifty-six and thirty-ve ten-thousandths.
Exercise 6.481
Four tenths.
Exercise 6.482 (Solution on p. 414.)
Four thousand eighty-one millionths.

6.12.1.2 Converting a Decimal to a Fraction (Section 6.3)

For problem 14-20, convert each decimal to a proper fraction or a mixed number.
Exercise 6.483
1.07
Exercise 6.484 (Solution on p. 414.)
58.63
Exercise 6.485
0.05
Exercise 6.486 (Solution on p. 414.)
0.14 23
Exercise 6.487
1.09 18
Exercise 6.488 (Solution on p. 415.)
4.01 27
1

Exercise 6.489
9.11 19
398 CHAPTER 6. DECIMALS

6.12.1.3 Rounding Decimals (Section 6.4)

For problems 21-25, round each decimal to the specied position.


Exercise 6.490 (Solution on p. 415.)
4.087 to the nearest hundredth.
Exercise 6.491
4.087 to the nearest tenth.
Exercise 6.492 (Solution on p. 415.)
16.5218 to the nearest one.
Exercise 6.493
817.42 to the nearest ten.
Exercise 6.494 (Solution on p. 415.)
0.9811602 to the nearest one.

6.12.1.4 Addition, Subtraction, Multiplication and Division of Decimals, and Nonterminating


Divisions (Section 6.5,Section 6.6,Section 6.7,Section 6.8)

For problem 26-45, perform each operation and simplify.


Exercise 6.495
7.10 + 2.98
Exercise 6.496 (Solution on p. 415.)
14.007 − 5.061
Exercise 6.497
1.2 · 8.6
Exercise 6.498 (Solution on p. 415.)
41.8 · 0.19
Exercise 6.499
57.51 ÷ 2.7
Exercise 6.500 (Solution on p. 415.)
0.54003 ÷ 18.001
Exercise 6.501
32,051.3585 ÷ 23, 006.9999
Exercise 6.502 (Solution on p. 415.)
100 · 1, 816.001
Exercise 6.503
1, 000 · 1, 816.001
Exercise 6.504 (Solution on p. 415.)
10.000 · 0.14
Exercise 6.505
0.135888 ÷ 16.986
Exercise 6.506 (Solution on p. 415.)
150.79 ÷ 100
Exercise 6.507
4.119 ÷ 10, 000
Exercise 6.508 (Solution on p. 415.)
42.7 ÷ 18
399

Exercise 6.509
6.9 ÷ 12
Exercise 6.510 (Solution on p. 415.)
0.014 ÷ 47.6. Round to three decimal places.
Exercise 6.511
8.8 ÷ 19. Round to one decimal place.
Exercise 6.512 (Solution on p. 415.)
1.1 ÷ 9
Exercise 6.513
1.1 ÷ 9.9
Exercise 6.514 (Solution on p. 415.)
30 ÷ 11.1

6.12.1.5 Converting a Fraction to a Decimal (Section 6.9)

For problems 46-55, convert each fraction to a decimal.


Exercise 6.515
3
8
Exercise 6.516 (Solution on p. 415.)
43
100
Exercise 6.517
82
1000
Exercise 6.518 (Solution on p. 415.)
9 47
Exercise 6.519
5
8 16
Exercise 6.520 (Solution on p. 415.)
1.3 13
Exercise 6.521
25.6 32
Exercise 6.522 (Solution on p. 415.)
125.125 18
Exercise 6.523
9.11 19
Exercise 6.524 (Solution on p. 415.)
0.0 56
400 CHAPTER 6. DECIMALS

6.12.1.6 Combinations of Operations with Decimals and Fractions (Section 6.10)

For problems 56-62, perform each operation.


Exercise 6.525
5
8 · 0.25
Exercise 6.526 (Solution on p. 415.)
16 · 1.36
3

Exercise 6.527
3 1
+ 1.75

5 · 2
Exercise 6.528 (Solution on p. 415.)
7 5
+ 0.30

2 · 4
Exercise 6.529
19.375 ÷ 4.375 − 1 16
1


Exercise 6.530 (Solution on p. 415.)


15
602 · 2.6 + 3 4
1

Exercise 6.531
4 13 3 5

18 ÷ 5 14 + 3 21

6.13 Prociency Exam13


6.13.1 Prociency Exam
Exercise 6.532 (Solution on p. 415.)
(Section 6.2) The decimal digit that appears three places to the right of the decimal point is in
the position.
Exercise 6.533 (Solution on p. 415.)
(Section 6.2) Write, using words, 15.036.
Exercise 6.534 (Solution on p. 415.)
(Section 6.2) Write eighty-one and twelve hundredths using digits. 81.12
Exercise 6.535 (Solution on p. 416.)
(Section 6.2) Write three thousand seventeen millionths using digits.
Exercise 6.536 (Solution on p. 416.)
(Section 6.3) Convert 0.78 to a fraction. Reduce.
Exercise 6.537 (Solution on p. 416.)
(Section 6.3) Convert 0.875 to a fraction. Reduce.
Exercise 6.538 (Solution on p. 416.)
(Section 6.4) Round 4.8063 to the nearest tenth.
Exercise 6.539 (Solution on p. 416.)
(Section 6.4) Round 187.51 to the nearest hundred.
Exercise 6.540 (Solution on p. 416.)
(Section 6.4) Round 0.0652 to the nearest hundredth.
For problems 10-20, perform each operation.
Exercise 6.541 (Solution on p. 416.)
(Section 6.5) 15.026 + 5.971
13 This content is available online at <http://cnx.org/content/m34979/1.2/>.
401

Exercise 6.542 (Solution on p. 416.)


(Section 6.5) 72.15 − 26.585
Exercise 6.543 (Solution on p. 416.)
(Section 6.6) 16.2 · 4.8
Exercise 6.544 (Solution on p. 416.)
(Section 6.6) 10, 000 · 0.016
Exercise 6.545 (Solution on p. 416.)
(Section 6.7) 44.64 ÷ 18.6
Exercise 6.546 (Solution on p. 416.)
(Section 6.7) 0.21387 ÷ 0.19
Exercise 6.547 (Solution on p. 416.)
(Section 6.8) 0.27 − 11
3

Exercise 6.548 (Solution on p. 416.)


(Section 6.9) Convert 6 11
2
to a decimal.
Exercise 6.549 (Solution on p. 416.)
(Section 6.9) Convert 0.5 169 to a decimal.
Exercise 6.550 (Solution on p. 416.)
(Section 6.10) 3 18 + 2.325
Exercise 6.551 (Solution on p. 416.)
(Section 6.10) 3
8 × 0.5625
402 CHAPTER 6. DECIMALS

Solutions to Exercises in Chapter 6


Solution to Exercise 6.1 (p. 341)
twelve and nine tenths
Solution to Exercise 6.2 (p. 341)
four and eighty-six hundredths
Solution to Exercise 6.3 (p. 341)
seven and two hundred thousandths
Solution to Exercise 6.4 (p. 341)
thirty thousand four hundred ve millionths
Solution to Exercise 6.5 (p. 342)
306.49
Solution to Exercise 6.6 (p. 342)
9.004
Solution to Exercise 6.7 (p. 342)
0.000061
Solution to Exercise 6.8 (p. 342)
Tenths; hundredths, thousandths
Solution to Exercise 6.10 (p. 342)
Hundred thousandths; ten millionths
Solution to Exercise 6.12 (p. 342)
eight and one tenth
Solution to Exercise 6.14 (p. 342)
fty-ve and six hundredths
Solution to Exercise 6.16 (p. 342)
one and nine hundred four thousandths
Solution to Exercise 6.18 (p. 343)
3.20
Solution to Exercise 6.20 (p. 343)
1.8
Solution to Exercise 6.22 (p. 343)
511.004
Solution to Exercise 6.24 (p. 343)
0.947
Solution to Exercise 6.26 (p. 343)
0.00071
Solution to Exercise 6.28 (p. 343)
seventy-ve hundredths
Solution to Exercise 6.30 (p. 343)
four tenths
Solution to Exercise 6.32 (p. 343)
sixteen hundredths
Solution to Exercise 6.34 (p. 343)
one thousand eight hundred seventy-ve ten thousandths
Solution to Exercise 6.36 (p. 343)
fty-ve hundredths
Solution to Exercise 6.38 (p. 344)
2610
Solution to Exercise 6.40 (p. 344)
12
Solution to Exercise 6.42 (p. 344)
10 or 1 19
9
403

Solution to Exercise 6.43 (p. 345)


16 21
25
Solution to Exercise 6.44 (p. 345)
513
1,000
Solution to Exercise 6.45 (p. 345)
6, 646 10107
,000
Solution to Exercise 6.46 (p. 345)
1
1 10
Solution to Exercise 6.47 (p. 346)
7
8
Solution to Exercise 6.48 (p. 346)
31
250
Solution to Exercise 6.49 (p. 346)
7
6 1,200
Solution to Exercise 6.50 (p. 346)
18 17
2

Solution to Exercise 6.51 (p. 346)


7
10
Solution to Exercise 6.53 (p. 346)
53
100
Solution to Exercise 6.55 (p. 346)
219
1,000
Solution to Exercise 6.57 (p. 346)
4 45
Solution to Exercise 6.59 (p. 346)
16 25
3

Solution to Exercise 6.61 (p. 347)


1
6 2,000
Solution to Exercise 6.63 (p. 347)
16 81
Solution to Exercise 6.65 (p. 347)
1
3 25
Solution to Exercise 6.67 (p. 347)
9
8 40
Solution to Exercise 6.69 (p. 347)
9 19,999
20,000
Solution to Exercise 6.71 (p. 347)
3
4
Solution to Exercise 6.73 (p. 347)
2 13
80
Solution to Exercise 6.75 (p. 347)
14 3,337
000
Solution to Exercise 6.77 (p. 347)
1 129
320
Solution to Exercise 6.79 (p. 347)
1
2 25
Solution to Exercise 6.81 (p. 348)
14
Solution to Exercise 6.83 (p. 348)
9
10
404 CHAPTER 6. DECIMALS

Solution to Exercise 6.85 (p. 348)


thousandths
Solution to Exercise 6.86 (p. 349)
4.82
Solution to Exercise 6.87 (p. 349)
0.3593
Solution to Exercise 6.88 (p. 349)
82
Solution to Exercise 6.89 (p. 349)
800
Solution to Exercise 6.90 (p. 349)
43.994
Solution to Exercise 6.91 (p. 349)
105.0200
Solution to Exercise 6.92 (p. 349)
100.00
Solution to Exercise 6.93 (p. 350)

Tenth Hundredth Thousandth Ten Thousandth


20.0 20.01 20.011 20.0107

Table 6.30

Solution to Exercise 6.95 (p. 350)

Tenth Hundredth Thousandth Ten Thousandth


531.2 531.22 531.219 531.2188

Table 6.31

Solution to Exercise 6.97 (p. 350)

Tenth Hundredth Thousandth Ten Thousandth


2.0 2.00 2.000 2.0000

Table 6.32

Solution to Exercise 6.99 (p. 351)

Tenth Hundredth Thousandth Ten Thousandth


0.0 0.00 0.000 0.0000

Table 6.33

Solution to Exercise 6.101 (p. 351)


405

Tenth Hundredth Thousandth Ten Thousandth


9.2 9.19 9.192 9.1919

Table 6.34

Solution to Exercise 6.103 (p. 351)


18.417
Solution to Exercise 6.105 (p. 351)
18.41681
Solution to Exercise 6.107 (p. 352)
18.42
Solution to Exercise 6.109 (p. 352)
0.1
Solution to Exercise 6.111 (p. 352)
0.83607
Solution to Exercise 6.113 (p. 352)
5.333
Solution to Exercise 6.115 (p. 352)
Ten million
Solution to Exercise 6.117 (p. 352)
256
Solution to Exercise 6.119 (p. 352)
4
3 25
Solution to Exercise 6.120 (p. 355)
6.179
Solution to Exercise 6.121 (p. 355)
9.646
Solution to Exercise 6.122 (p. 355)
1.6044
Solution to Exercise 6.123 (p. 355)
0.0065
Solution to Exercise 6.124 (p. 355)
25,777.22
Solution to Exercise 6.125 (p. 355)
25.69
Solution to Exercise 6.126 (p. 356)
13.256
Solution to Exercise 6.127 (p. 356)
59.4512
Solution to Exercise 6.128 (p. 356)
0.096862
Solution to Exercise 6.129 (p. 356)
0.475
Solution to Exercise 6.130 (p. 356)
Since each number contains more than eight digits, using some calculators may not be helpful. Adding these
by hand technology, we get 4,785.00031
Solution to Exercise 6.131 (p. 357)
8.95
Solution to Exercise 6.133 (p. 357)
39.846
406 CHAPTER 6. DECIMALS

Solution to Exercise 6.135 (p. 357)


11.74931
Solution to Exercise 6.137 (p. 357)
57.2115
Solution to Exercise 6.139 (p. 357)
2.607
Solution to Exercise 6.141 (p. 357)
0.41085
Solution to Exercise 6.143 (p. 357)
9.135586
Solution to Exercise 6.145 (p. 357)
27.351
Solution to Exercise 6.147 (p. 357)
39.6660
Solution to Exercise 6.149 (p. 357)
11.09
Solution to Exercise 6.151 (p. 358)
$23.23
Solution to Exercise 6.153 (p. 358)
$5.71
Solution to Exercise 6.155 (p. 358)
869.37
Solution to Exercise 6.157 (p. 358)
8,200,000
Solution to Exercise 6.159 (p. 358)
20 = 5
or 2 29
9 3
Solution to Exercise 6.161 (p. 360)
45.58
Solution to Exercise 6.162 (p. 360)
10.388
Solution to Exercise 6.163 (p. 360)
0.16864
Solution to Exercise 6.164 (p. 360)
0.00000062
Solution to Exercise 6.165 (p. 360)
9.3
Solution to Exercise 6.166 (p. 360)
53.94
Solution to Exercise 6.167 (p. 360)
539.4
Solution to Exercise 6.168 (p. 360)
5,394
Solution to Exercise 6.169 (p. 360)
59,340
Solution to Exercise 6.170 (p. 362)
20.91408
Solution to Exercise 6.171 (p. 362)
0.000066
Solution to Exercise 6.172 (p. 362)
0.2397
407

Solution to Exercise 6.173 (p. 362)


0.0000
Solution to Exercise 6.174 (p. 363)
427
Solution to Exercise 6.175 (p. 363)
165,218.7
Solution to Exercise 6.176 (p. 363)
0.188
Solution to Exercise 6.177 (p. 363)
527,000,000,000
Solution to Exercise 6.178 (p. 364)
17.92
Solution to Exercise 6.179 (p. 364)
0.13
Solution to Exercise 6.180 (p. 364)
3.636
Solution to Exercise 6.181 (p. 364)
0.0000036
Solution to Exercise 6.182 (p. 364)
9.96
Solution to Exercise 6.183 (p. 364)
14.08
Solution to Exercise 6.184 (p. 365)
31.28
Solution to Exercise 6.186 (p. 365)
47.20
Solution to Exercise 6.188 (p. 365)
0.152
Solution to Exercise 6.190 (p. 365)
4.6324
Solution to Exercise 6.192 (p. 365)
3.182
Solution to Exercise 6.194 (p. 365)
0.0000273
Solution to Exercise 6.196 (p. 365)
2.56
Solution to Exercise 6.198 (p. 365)
0.81
Solution to Exercise 6.200 (p. 365)
29.3045
Solution to Exercise 6.202 (p. 365)
0.00000105486
Solution to Exercise 6.204 (p. 366)
49.6
Solution to Exercise 6.206 (p. 366)
4,218.842
Solution to Exercise 6.208 (p. 366)
19.621
Solution to Exercise 6.210 (p. 366)
3,596.168
408 CHAPTER 6. DECIMALS

Solution to Exercise 6.212 (p. 366)


25,010
Solution to Exercise 6.214 (p. 366)

Actual product Tenths Hundreds Thousandths


28.382 28.4 28.38 28.382

Table 6.35

Solution to Exercise 6.216 (p. 366)

Actual product Tenths Hundreds Thousandths


134.216048 134.2 134.22 134.216

Table 6.36

Solution to Exercise 6.218 (p. 367)

Actual product Tenths Hundreds Thousandths


185.626 185.6 185.63 185.626

Table 6.37

Solution to Exercise 6.220 (p. 367)


121.503
Solution to Exercise 6.222 (p. 367)
1.2
Solution to Exercise 6.224 (p. 367)
0.999702
Solution to Exercise 6.226 (p. 367)
0.2528
Solution to Exercise 6.228 (p. 367)
2.9544
Solution to Exercise 6.230 (p. 367)
$16.24
Solution to Exercise 6.232 (p. 367)
0.24
Solution to Exercise 6.234 (p. 368)
0.006099
Solution to Exercise 6.236 (p. 368)
23.295102
Solution to Exercise 6.238 (p. 368)
0.000144
Solution to Exercise 6.240 (p. 368)
0.0000018
Solution to Exercise 6.242 (p. 368)
0.0000000471
409

Solution to Exercise 6.244 (p. 368)


0
Solution to Exercise 6.246 (p. 368)
10
77
Solution to Exercise 6.248 (p. 368)
1.78
Solution to Exercise 6.249 (p. 371)
61.5
Solution to Exercise 6.250 (p. 371)
1.884
Solution to Exercise 6.251 (p. 371)
0.0741
Solution to Exercise 6.252 (p. 371)
0.000062
Solution to Exercise 6.253 (p. 373)
2.96
Solution to Exercise 6.254 (p. 373)
4.58
Solution to Exercise 6.255 (p. 373)
40,000
Solution to Exercise 6.256 (p. 373)
816.241
Solution to Exercise 6.257 (p. 373)
81.6241
Solution to Exercise 6.258 (p. 373)
8.16241
Solution to Exercise 6.259 (p. 374)
0.816241
Solution to Exercise 6.260 (p. 375)
2.98
Solution to Exercise 6.261 (p. 375)
0.005
Solution to Exercise 6.262 (p. 375)
3.5197645 is an approximate result. Rounding to four decimal places, we get 3.5198
Solution to Exercise 6.263 (p. 377)
18.25
Solution to Exercise 6.264 (p. 377)
1.825
Solution to Exercise 6.265 (p. 377)
0.1825
Solution to Exercise 6.266 (p. 377)
0.01825
Solution to Exercise 6.267 (p. 377)
6.4618
Solution to Exercise 6.268 (p. 377)
0.021926
Solution to Exercise 6.269 (p. 377)
1.6
Solution to Exercise 6.271 (p. 377)
3.7
410 CHAPTER 6. DECIMALS

Solution to Exercise 6.273 (p. 378)


6.04
Solution to Exercise 6.275 (p. 378)
9.38
Solution to Exercise 6.277 (p. 378)
0.46
Solution to Exercise 6.279 (p. 378)
8.6
Solution to Exercise 6.281 (p. 378)
2.4
Solution to Exercise 6.283 (p. 378)
6.21
Solution to Exercise 6.285 (p. 378)
2.18
Solution to Exercise 6.287 (p. 378)
1.001
Solution to Exercise 6.289 (p. 378)
4
Solution to Exercise 6.291 (p. 378)
14
Solution to Exercise 6.293 (p. 378)
111
Solution to Exercise 6.295 (p. 379)
643.51006
Solution to Exercise 6.297 (p. 379)
0.064351006
Solution to Exercise 6.299 (p. 379)

Actual Quotient Tenths Hundredths Thousandths


1.81 1.8 1.81 1.810

Table 6.38

Solution to Exercise 6.301 (p. 379)

Actual Quotient Tenths Hundredths Thousandths


4.821 4.8 4.82 4.821

Table 6.39

Solution to Exercise 6.303 (p. 379)

Actual Quotient Tenths Hundredths Thousandths


0.00351 0.0 0.00 0.004

Table 6.40
411

Solution to Exercise 6.305 (p. 380)


0.01
Solution to Exercise 6.307 (p. 380)
12.11 months
Solution to Exercise 6.309 (p. 380)
27.2 miles per gallon
Solution to Exercise 6.311 (p. 380)
3.7
Solution to Exercise 6.313 (p. 380)
1.94
Solution to Exercise 6.315 (p. 380)
29.120
Solution to Exercise 6.317 (p. 380)
0.173
Solution to Exercise 6.319 (p. 380)
1.111
Solution to Exercise 6.321 (p. 381)
39
8
Solution to Exercise 6.323 (p. 381)
47
30 or 1 17
30
Solution to Exercise 6.325 (p. 381)
2.8634
Solution to Exercise 6.326 (p. 384)
0.3
Solution to Exercise 6.327 (p. 384)
0.83
Solution to Exercise 6.328 (p. 384)
1.2
Solution to Exercise 6.329 (p. 384)
1.8
Solution to Exercise 6.330 (p. 384)
1.17
Solution to Exercise 6.331 (p. 384)
36.3636
Solution to Exercise 6.332 (p. 385)
0.4
Solution to Exercise 6.334 (p. 385)
0.16
Solution to Exercise 6.336 (p. 385)
0.142857
Solution to Exercise 6.338 (p. 385)
10.526
Solution to Exercise 6.340 (p. 385)
0.112
Solution to Exercise 6.342 (p. 385)
6.21951
Solution to Exercise 6.344 (p. 385)
0.0081
Solution to Exercise 6.346 (p. 385)
0.835
412 CHAPTER 6. DECIMALS

Solution to Exercise 6.348 (p. 385)


0.3
Solution to Exercise 6.350 (p. 385)
2.05
Solution to Exercise 6.352 (p. 385)
0.7
Solution to Exercise 6.354 (p. 386)
0.518
Solution to Exercise 6.356 (p. 386)
0.045
Solution to Exercise 6.358 (p. 386)
0.72
Solution to Exercise 6.360 (p. 386)
0.7
Solution to Exercise 6.362 (p. 386)
1
Solution to Exercise 6.364 (p. 386)
4410
Solution to Exercise 6.366 (p. 386)
8.6
Solution to Exercise 6.367 (p. 388)
0.25
Solution to Exercise 6.368 (p. 389)
0.04
Solution to Exercise 6.369 (p. 389)
0.17
Solution to Exercise 6.370 (p. 389)
0.9375
Solution to Exercise 6.371 (p. 389)
0.95
Solution to Exercise 6.372 (p. 389)
8.0126375
Solution to Exercise 6.373 (p. 389)
0.5
Solution to Exercise 6.375 (p. 389)
0.875
Solution to Exercise 6.377 (p. 389)
0.6
Solution to Exercise 6.379 (p. 389)
0.04
Solution to Exercise 6.381 (p. 389)
0.05
Solution to Exercise 6.383 (p. 389)
0.02
Solution to Exercise 6.385 (p. 389)
0.3
Solution to Exercise 6.387 (p. 389)
0.1875
Solution to Exercise 6.389 (p. 390)
0.037
413

Solution to Exercise 6.391 (p. 390)


0.538461
Solution to Exercise 6.393 (p. 390)
7.6
Solution to Exercise 6.395 (p. 390)
1.13
Solution to Exercise 6.397 (p. 390)
101.24
Solution to Exercise 6.399 (p. 390)
0.24125
Solution to Exercise 6.401 (p. 390)
810.31063125
Solution to Exercise 6.403 (p. 390)
0.11111
Solution to Exercise 6.405 (p. 390)
0.33333
Solution to Exercise 6.407 (p. 390)
0.55556
Solution to Exercise 6.409 (p. 390)
0.77778
Solution to Exercise 6.411 (p. 391)
0.09091
Solution to Exercise 6.413 (p. 391)
0.27273
Solution to Exercise 6.415 (p. 391)
0.45455
Solution to Exercise 6.417 (p. 391)
0.63636
Solution to Exercise 6.419 (p. 391)
0.81818
Solution to Exercise 6.421 (p. 391)
0.128
Solution to Exercise 6.423 (p. 391)
0.9746
Solution to Exercise 6.425 (p. 391)
0.0002
Solution to Exercise 6.427 (p. 391)
38.7347
Solution to Exercise 6.429 (p. 392)
15
16
Solution to Exercise 6.431 (p. 392)
3 29
40 or 3.725
Solution to Exercise 6.433 (p. 393)
2.2 or 2 15
Solution to Exercise 6.434 (p. 393)
9.23
Solution to Exercise 6.435 (p. 393)
10
Solution to Exercise 6.436 (p. 393)
25 or 0.04
1
414 CHAPTER 6. DECIMALS

Solution to Exercise 6.437 (p. 394)


1
Solution to Exercise 6.439 (p. 394)
0.112
Solution to Exercise 6.441 (p. 394)
0.055
Solution to Exercise 6.443 (p. 394)
0.1276
Solution to Exercise 6.445 (p. 394)
0.7
Solution to Exercise 6.447 (p. 394)
9.4625
Solution to Exercise 6.449 (p. 394)
1.4075
Solution to Exercise 6.451 (p. 394)
0.1875
Solution to Exercise 6.453 (p. 394)
0.75
Solution to Exercise 6.455 (p. 394)
0.615
Solution to Exercise 6.457 (p. 394)
1.35
Solution to Exercise 6.459 (p. 395)
0.125
Solution to Exercise 6.461 (p. 395)
0.15
Solution to Exercise 6.463 (p. 395)
2.6
Solution to Exercise 6.465 (p. 395)
yes
Solution to Exercise 6.467 (p. 395)
23 · 52 · 7 · 11
Solution to Exercise 6.469 (p. 395)
0.592
Solution to Exercise 6.470 (p. 396)
hundredths
Solution to Exercise 6.472 (p. 397)
seven and two tenths
Solution to Exercise 6.474 (p. 397)
sixteen and fty-two hundredths
Solution to Exercise 6.476 (p. 397)
ve thousandths
Solution to Exercise 6.478 (p. 397)
9.12
Solution to Exercise 6.480 (p. 397)
56.0035
Solution to Exercise 6.482 (p. 397)
0.004081
Solution to Exercise 6.484 (p. 397)
63
85 100
415

Solution to Exercise 6.486 (p. 397)


11
75
Solution to Exercise 6.488 (p. 397)
7
4 675
Solution to Exercise 6.490 (p. 398)
4.09
Solution to Exercise 6.492 (p. 398)
17
Solution to Exercise 6.494 (p. 398)
1
Solution to Exercise 6.496 (p. 398)
8.946
Solution to Exercise 6.498 (p. 398)
7.942
Solution to Exercise 6.500 (p. 398)
0.03
Solution to Exercise 6.502 (p. 398)
181,600.1
Solution to Exercise 6.504 (p. 398)
1.4
Solution to Exercise 6.506 (p. 398)
1.5079
Solution to Exercise 6.508 (p. 398)
2.372
Solution to Exercise 6.510 (p. 399)
0.000
Solution to Exercise 6.512 (p. 399)
0.12
Solution to Exercise 6.514 (p. 399)
2.702
Solution to Exercise 6.516 (p. 399)
0.43
Solution to Exercise 6.518 (p. 399)
9.571428
Solution to Exercise 6.520 (p. 399)
1.3
Solution to Exercise 6.522 (p. 399)
125.125125 (not repeating)
Solution to Exercise 6.524 (p. 399)
0.083
Solution to Exercise 6.526 (p. 400)
0.255
Solution to Exercise 6.528 (p. 400)
5.425
Solution to Exercise 6.530 (p. 400)
0.09343
Solution to Exercise 6.532 (p. 400)
thousandth
Solution to Exercise 6.533 (p. 400)
fteen and thirty-six thousandths
416 CHAPTER 6. DECIMALS

Solution to Exercise 6.534 (p. 400)


81.12
Solution to Exercise 6.535 (p. 400)
0.003017
Solution to Exercise 6.536 (p. 400)
39
50
Solution to Exercise 6.537 (p. 400)
7
8
Solution to Exercise 6.538 (p. 400)
4.8
Solution to Exercise 6.539 (p. 400)
200
Solution to Exercise 6.540 (p. 400)
0.07
Solution to Exercise 6.541 (p. 400)
20.997
Solution to Exercise 6.542 (p. 401)
45.565
Solution to Exercise 6.543 (p. 401)
77.76
Solution to Exercise 6.544 (p. 401)
16
Solution to Exercise 6.545 (p. 401)
2.4
Solution to Exercise 6.546 (p. 401)
1.1256
Solution to Exercise 6.547 (p. 401)
0
Solution to Exercise 6.548 (p. 401)
6.18
Solution to Exercise 6.549 (p. 401)
0.055625
Solution to Exercise 6.550 (p. 401)
5.45
Solution to Exercise 6.551 (p. 401)
27
128 or 0.2109375
Chapter 7

Ratios and Rates

7.1 Objectives1
After completing this chapter, you should
Ratios and Rates (Section 7.2)

• be able to distinguish between denominate and pure numbers and between ratios and rates

Proportions (Section 7.3)

• be able to describe proportions and nd the missing factor in a proportion


• be able to work with proportions involving rates

Applications of Proportions (Section 7.4)

• solve proportion problems using the ve-step method

Percent (Section 7.5)

• understand the relationship between ratios and percents


• be able to make conversions between fractions, decimals, and percents

Fractions of One Percent (Section 7.6)

• understand the meaning of a fraction of one percent


• be able to make conversions involving fractions of one percent

Applications of Percents (Section 7.7)

• be able to distinguish between base, percent, and percentage


• be able to nd the percentage, the percent, and the base

7.2 Ratios and Rates2


7.2.1 Section Overview
• Denominate Numbers and Pure Numbers
• Ratios and Rates
1 This content is available online at <http://cnx.org/content/m18895/1.3/>.
2 This content is available online at <http://cnx.org/content/m34980/1.2/>.

417
418 CHAPTER 7. RATIOS AND RATES

7.2.2 Denominate Numbers and Pure Numbers


Denominate Numbers, Like and Unlike Denominate Numbers
It is often necessary or convenient to compare two quantities. Denominate numbersare numbers together
with some specied unit. If the units being compared are alike, the denominate numbers are called like
denominate numbers. If units are not alike, the numbers are called unlike denominate numbers.
Examples of denominate numbers are shown in the diagram:

Pure Numbers
Numbers that exist purely as numbers and do not represent amounts of quantities are called pure numbers.
Examples of pure numbers are 8, 254, 0, 21 85 , 25 , and 0.07.

Numbers can be compared in two ways: subtraction and division.

Comparing Numbers by Subtraction and Division


Comparison of two numbers by subtraction indicates how much more one number is than another.
Comparison by division indicates how many times larger or smaller one number is than another.

Comparing Pure or Like Denominate Numbers by Subtraction


Numbers can be compared by subtraction if and only if they both are like denominate numbers or both pure
numbers.

7.2.2.1 Sample Set A

Example 7.1
Compare 8 miles and 3 miles by subtraction.

8 mile − 3 miles = 5 miles

This means that 8 miles is 5 miles more than 3 miles.

Examples of use : I can now jog 8 miles whereas I used to jog only 3 miles. So, I can now jog 5
miles more than I used to.
Example 7.2
Compare 12 and 5 by subtraction.

12 − 5 = 7

This means that 12 is 7 more than 5.


Example 7.3
Comparing 8 miles and 5 gallons by subtraction makes no sense.

8 miles − 5 gallons =?
Example 7.4
Compare 36 and 4 by division.

36 ÷ 4 = 9

This means that 36 is 9 times as large as 4. Recall that 36 ÷ 4 = 9 can be expressed as 36


4 = 9.
419

Example 7.5
Compare 8 miles and 2 miles by division.
8 miles = 4
2 miles
This means that 8 miles is 4 times as large as 2 miles.

Example of use : I can jog 8 miles to your 2 miles. Or, for every 2 miles that you jog, I jog 8. So, I
jog 4 times as many miles as you jog.

Notice that when like quantities are being compared by division, we drop the units. Another way
of looking at this is that the units divide out (cancel).
Example 7.6
Compare 30 miles and 2 gallons by division.
30 miles 15 miles
2 gallons = 1 gallon
Example of use : A particular car goes 30 miles on 2 gallons of gasoline. This is the same as getting
15 miles to 1 gallon of gasoline.

Notice that when the quantities being compared by division are unlike quantities, we do not drop
the units.

7.2.2.2 Practice Set A

Make the following comparisons and interpret each one.


Exercise 7.1 (Solution on p. 464.)
Compare 10 diskettes to 2 diskettes by
(a) subtraction:
(b) division:

Exercise 7.2 (Solution on p. 464.)


Compare, if possible, 16 bananas and 2 bags by
(a) subtraction:
(b) division:

7.2.3 Ratios and Rates


Ratio
A comparison, by division, of two pure numbers or two like denominate numbers is a ratio.

The comparison by division of the pure numbers 36 8 miles


4 and the like denominate numbers 2 miles are examples
of ratios.

Rate
A comparison, by division, of two unlike denominate numbers is a rate.

The comparison by division of two unlike denominate numbers, such as


55 miles and 40 dollars
1 gallon 5 tickets
420 CHAPTER 7. RATIOS AND RATES

are examples of rates.

Let's agree to represent two numbers (pure or denominate) with the letters a and b. This means that we're
letting a represent some number and b represent some, perhaps dierent, number. With this agreement, we
can write the ratio of the two numbers a and b as
a
b or b
a

The ratio a
b is read as " a to b."

The ratio b
a is read as " b to a."

Since a ratio or a rate can be expressed as a fraction, it may be reducible.

7.2.3.1 Sample Set B

Example 7.7
The ratio 30 to 2 can be expressed as 30 15
2 . Reducing, we get 1 .

The ratio 30 to 2 is equivalent to the ratio 15 to 1.


Example 7.8
The rate "4 televisions to 12 people" can be expressed as 412
televisions . The meaning of this rate is
people
that "for every 4 televisions, there are 12 people."

Reducing, we get 1 3television


people . The meaning of this rate is that "for every 1 television, there are 3
people.

Thus, the rate of "4 televisions to 12 people" is the same as the rate of "1 television to 3 people."

7.2.3.2 Practice Set B

Write the following ratios and rates as fractions.


Exercise 7.3 (Solution on p. 464.)
3 to 2
Exercise 7.4 (Solution on p. 464.)
1 to 9
Exercise 7.5 (Solution on p. 464.)
5 books to 4 people
Exercise 7.6 (Solution on p. 464.)
120 miles to 2 hours
Exercise 7.7 (Solution on p. 464.)
8 liters to 3 liters
Write the following ratios and rates in the form "a to b." Reduce when necessary.
Exercise 7.8 (Solution on p. 464.)
9
5
Exercise 7.9 (Solution on p. 464.)
1
3
Exercise 7.10 (Solution on p. 464.)
25 miles
2 gallons
421

Exercise 7.11 (Solution on p. 464.)


2 mechanics
4 wrenches
Exercise 7.12 (Solution on p. 464.)
15 video tapes
18 video tapes

7.2.4 Exercises
For the following 9 problems, complete the statements.
Exercise 7.13 (Solution on p. 464.)
Two numbers can be compared by subtraction if and only if .
Exercise 7.14
A comparison, by division, of two pure numbers or two like denominate numbers is called a
.
Exercise 7.15 (Solution on p. 464.)
A comparison, by division, of two unlike denominate numbers is called a .
Exercise 7.16
11 is an example of a . (ratio/rate)
6

Exercise 7.17 (Solution on p. 464.)


12 is an example of a . (ratio/rate)
5

Exercise 7.18
7 erasers
12 pencils is an example of a . (ratio/rate)
Exercise 7.19 (Solution on p. 464.)
20 silver coins is an example of a .(ratio/rate)
35 gold coins
Exercise 7.20
3 sprinklers is an example of a . (ratio/rate)
5 sprinklers
Exercise 7.21 (Solution on p. 464.)
18 exhaust valves is an example of a .(ratio/rate)
11 exhaust valves
For the following 7 problems, write each ratio or rate as a verbal phrase.
Exercise 7.22
8
3
Exercise 7.23 (Solution on p. 464.)
2
5
Exercise 7.24
8 feet
3 seconds
Exercise 7.25 (Solution on p. 464.)
29 miles
2 gallons
Exercise 7.26
30,000 stars
300 stars
Exercise 7.27 (Solution on p. 464.)
5 yards
2 yards
Exercise 7.28
164 trees
28 trees
For the following problems, write the simplied fractional form of each ratio or rate.
Exercise 7.29 (Solution on p. 464.)
12 to 5
422 CHAPTER 7. RATIOS AND RATES

Exercise 7.30
81 to 19
Exercise 7.31 (Solution on p. 464.)
42 plants to 5 homes
Exercise 7.32
8 books to 7 desks
Exercise 7.33 (Solution on p. 464.)
16 pints to 1 quart
Exercise 7.34
4 quarts to 1 gallon
Exercise 7.35 (Solution on p. 465.)
2.54 cm to 1 in
Exercise 7.36
80 tables to 18 tables
Exercise 7.37 (Solution on p. 465.)
25 cars to 10 cars
Exercise 7.38
37 wins to 16 losses
Exercise 7.39 (Solution on p. 465.)
105 hits to 315 at bats
Exercise 7.40
510 miles to 22 gallons
Exercise 7.41 (Solution on p. 465.)
1,042 characters to 1 page
Exercise 7.42
1,245 pages to 2 books

7.2.4.1 Exercises for Review

Exercise 7.43 (Solution on p. 465.)


(Section 4.3) Convert 16
3 to a mixed number.
Exercise 7.44
(Section 4.7) 1 59 of 2 47 is what number?
Exercise 7.45 (Solution on p. 465.)
(Section 5.3) Find the dierence. 11
28 − 45 .
7

Exercise 7.46
(Section 6.8) Perform the division. If no repeating patterns seems to exist, round the quotient to
three decimal places: 22.35 ÷ 17
Exercise 7.47 (Solution on p. 465.)
(Section 6.10) Find the value of 1.85 + 3
8 · 4.1
423

7.3 Proportions3
7.3.1 Section Overview
• Ratios, Rates, and Proportions
• Finding the Missing Factor in a Proportion
• Proportions Involving Rates

7.3.2 Ratios, Rates, and Proportions


Ratio, Rate
We have dened a ratio as a comparison, by division, of two pure numbers or two like denominate numbers.
We have dened a rate as a comparison, by division, of two unlike denominate numbers.

Proportion
A proportion is a statement that two ratios or rates are equal. The following two examples show how to
read proportions.

7.3.2.1 Sample Set A

Write or read each proportion.


Example 7.9
3 12
= 20
5

3 is to 5 as 12 is to 20
Example 7.10
10 items 2 items
5 dollars = 1 dollar
10 items is to 5 dollars as 2 items is to 1 dollar
Example 7.11
8 is to 12 as 16 is to 24.
8 16
12 = 24
Example 7.12
50 milligrams of vitamin C is to 1 tablet as 300 milligrams of vitamin C is to 6 tablets.
50 = 300
1 6

3 This content is available online at <http://cnx.org/content/m34981/1.2/>.


424 CHAPTER 7. RATIOS AND RATES

7.3.2.2 Practice Set A

Write or read each proportion.


Exercise 7.48 (Solution on p. 465.)
3 6
8 = 16
Exercise 7.49 (Solution on p. 465.)
2 people 10 people
1 window = 5 windows
Exercise 7.50 (Solution on p. 465.)
15 is to 4 as 75 is to 20.
Exercise 7.51 (Solution on p. 465.)
2 plates are to 1 tray as 20 plates are to 10 trays.

7.3.3 Finding the Missing Factor in a Proportion


Many practical problems can be solved by writing the given information as proportions. Such proportions
will be composed of three specied numbers and one unknown number. It is customary to let a letter, such
as x, represent the unknown number. An example of such a proportion is
x 20
= 16
4

This proportion is read as " x is to 4 as 20 is to 16."

There is a method of solving these proportions that is based on the equality of fractions. Recall that two
fractions are equivalent if and only if their cross products are equal. For example,

Notice that in a proportion that contains three specied numbers and a letter representing an unknown
quantity, that regardless of where the letter appears, the following situation always occurs.

(number) · (letter) = (number) · (number)


| {z }

We recognize this as a multiplication statement. Specically, it is a missing factor statement. (See Section 4.7
for a discussion of multiplication statements.) For example,

x 20
= 16 means that 16 · x = 4 · 20
4
4
x = 16
20 means that 4 · 20 = 16 · x
5
4 = x
16 means that 5 · 16 = 4 · x
5
4 = 20
x means that 5 · x = 4 · 20

Each of these statements is a multiplication statement. Specically, each is a missing factor statement. (The
letter used here is x, whereas M was used in Section 4.7.)

Finding the Missing Factor in a Proportion


The missing factor in a missing factor statement can be determined by dividing the product by the known
factor, that is, if x represents the missing factor, then
x = (product) ÷ (known factor)
425

7.3.3.1 Sample Set B

Find the unknown number in each proportion.


Example 7.13
x 20 . Find the cross product.
= 16
4

16 · x = 20 · 4
16 · x = 80 Divide the product 80 by the known factor 16.
80
x = 16
x = 5 The unknown number is 5.

This mean that 5


4 = 20
16 , or 5 is to 4 as 20 is to 16.
Example 7.14
5 20 . Find the cross product.
= 16
x

5 · 16 = 20 · x
80 = 20 · x Divide the product 80 by the known factor 20.
80
20 = x
4 = x The unknown number is 4.

This means that 5


4 = 20
16 , or, 5 is to 4 as 20 is to 6.
Example 7.15
16 = 64 Find the cross product.
3 x

16 · x = 64 · 3
16 · x = 192 Divide 192 by 16.
192
x = 16
x = 12 The unknown number is 12.

The means that 16 64


3 = 12 , or, 16 is to 3 as 64 is to 12.
Example 7.16
9
8
x
= 40 Find the cross products.

9 · 40 = 8·x
360 = 8·x Divide 360 by 8.
360
8 = x
45 = x The unknown number is 45.

7.3.3.2 Practice Set B

Find the unknown number in each proportion.


Exercise 7.52 (Solution on p. 465.)
x
8 = 12
32
426 CHAPTER 7. RATIOS AND RATES

Exercise 7.53 (Solution on p. 465.)


7
x = 14
10
Exercise 7.54 (Solution on p. 465.)
9 x
11 = 55
Exercise 7.55 (Solution on p. 465.)
1 8
6 = x

7.3.4 Proportions Involving Rates


Recall that a rate is a comparison, by division, of unlike denominate numbers. We must be careful when
setting up proportions that involve rates. The form is important. For example, if a rate involves two types
of units, say unit type 1 and unit type 2, we can write

or

Both cross products produce a statement of the type

(unit type 1) · (unit type 2) = (unit type 1) · (unit type 2)

which we take to mean the comparison

Examples of correctly expressed proportions are the following:

However, if we write the same type of units on dierent sides, such as,
unit type 1 = unit type 2
unit type 2 unit type 1
the cross product produces a statement of the form
427

We can see that this is an incorrect comparison by observing the following example: It is incorrect to write
2 hooks = 6 poles
3 poles 4 hooks
for two reason.

1. The cross product is numerically wrong: (2 · 4 6= 3 · 6).


2. The cross product produces the statement hooks are to hooks as poles are to poles, which makes no
sense.

7.3.5 Exercises
Exercise 7.56 (Solution on p. 465.)
A statement that two ratios or are equal is called a .
For the following 9 problems, write each proportion in fractional form.
Exercise 7.57
3 is to 7 as 18 is to 42.
Exercise 7.58 (Solution on p. 465.)
1 is to 11 as 3 is to 33.
Exercise 7.59
9 is to 14 as 27 is to 42.
Exercise 7.60 (Solution on p. 465.)
6 is to 90 as 3 is to 45.
Exercise 7.61
5 liters is to 1 bottle as 20 liters is to 4 bottles.
Exercise 7.62 (Solution on p. 465.)
18 grams of cobalt is to 10 grams of silver as 36 grams of cobalt is to 20 grams of silver.
Exercise 7.63
4 cups of water is to 1 cup of sugar as 32 cups of water is to 8 cups of sugar.
Exercise 7.64 (Solution on p. 465.)
3 people absent is to 31 people present as 15 people absent is to 155 people present.
Exercise 7.65
6 dollars is to 1 hour as 90 dollars is to 15 hours.
For the following 10 problems, write each proportion as a sentence.
Exercise 7.66 (Solution on p. 465.)
3
4 = 15
20
Exercise 7.67
1 5
8 = 40
Exercise 7.68 (Solution on p. 465.)
3 joggers = 6 joggers
100 feet 200 feet
Exercise 7.69
12 marshmallows = 36 marshmallows
3 sticks 9 sticks
428 CHAPTER 7. RATIOS AND RATES

Exercise 7.70 (Solution on p. 465.)


40 miles 2 gallons
80 miles = 4 gallons
Exercise 7.71
4 couches 2 houses
10 couches = 5 houses
Exercise 7.72 (Solution on p. 465.)
1 person = 8 people
1 job 8 jobs
Exercise 7.73
1 popsicle =
1
2 popsicle
2 children 1 child
Exercise 7.74 (Solution on p. 465.)
2,000 pounds = 60,000 pounds
1 ton 30 tons
Exercise 7.75
1 table 2 people
5 tables = 10 people
For the following 10 problems, solve each proportion.
Exercise 7.76 (Solution on p. 466.)
x 6
5 = 15
Exercise 7.77
x 28
10 = 40
Exercise 7.78 (Solution on p. 466.)
5
x = 10
16
Exercise 7.79
13 = 39
x 60
Exercise 7.80 (Solution on p. 466.)
1 x
3 = 24
Exercise 7.81
7 x
12 = 60
Exercise 7.82 (Solution on p. 466.)
8
3 = 72
x
Exercise 7.83
16 = 48
1 x
Exercise 7.84 (Solution on p. 466.)
x 200
25 = 125
Exercise 7.85
65 = x
30 60
For the following 5 problems, express each sentence as a proportion then solve the proportion.
Exercise 7.86 (Solution on p. 466.)
5 hats are to 4 coats as x hats are to 24 coats.
Exercise 7.87
x cushions are to 2 sofas as 24 cushions are to 16 sofas.
Exercise 7.88 (Solution on p. 466.)
1 spacecraft is to 7 astronauts as 5 spacecraft are to x astronauts.
Exercise 7.89
56 microchips are to x circuit boards as 168 microchips are to 3 circuit boards.
Exercise 7.90 (Solution on p. 466.)
18 calculators are to 90 calculators as x students are to 150 students.
Exercise 7.91
x dollars are to $40,000 as 2 sacks are to 1 sack.
429

Indicate whether the proportion is true or false.


Exercise 7.92 (Solution on p. 466.)
3 12
16 = 64
Exercise 7.93
2 10
15 = 75
Exercise 7.94 (Solution on p. 466.)
1 3
9 = 30
Exercise 7.95
6 knives = 12 knives
7 forks 15 forks
Exercise 7.96 (Solution on p. 466.)
33 miles = 99 miles
1 gallon 3 gallons
Exercise 7.97
320 feet 65 feet
5 seconds = 1 second
Exercise 7.98 (Solution on p. 466.)
35 students 1 class
70 students = 2 classes
Exercise 7.99
9 ml chloride 1 test tube
45 ml chloride = 7 test tubes

7.3.5.1 Exercises for Review

Exercise 7.100 (Solution on p. 466.)


(Section 1.7) Use the number 5 and 7 to illustrate the commutative property of addition.
Exercise 7.101
(Section 2.7) Use the numbers 5 and 7 to illustrate the commutative property of multiplication.
Exercise 7.102 (Solution on p. 466.)
(Section 5.3) Find the dierence. 14
5 3
− 22 .
Exercise 7.103
(Section 6.6) Find the product. 8.06129 · 1, 000.
Exercise 7.104 (Solution on p. 466.)
(Section 7.2) Write the simplied fractional form of the rate sixteen sentences to two paragraphs.

7.4 Applications of Proportions4


7.4.1 Section Overview
• The Five-Step Method
• Problem Solving

4 This content is available online at <http://cnx.org/content/m34982/1.2/>.


430 CHAPTER 7. RATIOS AND RATES

7.4.2 The Five-Step Method


In Section 7.3 we noted that many practical problems can be solved by writing the given information
as proportions. Such proportions will be composed of three specied numbers and one unknown number
represented by a letter.

The rst and most important part of solving a proportion problem is to determine, by careful reading, what
the unknown quantity is and to represent it with some letter.

The Five-Step Method


The ve-step method for solving proportion problems:

1. By careful reading, determine what the unknown quantity is and represent it with some letter. There
will be only one unknown in a problem.
2. Identify the three specied numbers.
3. Determine which comparisons are to be made and set up the proportion.
4. Solve the proportion (using the methods of Section 7.3).
5. Interpret and write a conclusion in a sentence with the appropriate units of measure.

Step 1 is extremely important. Many problems go unsolved because time is not taken to establish what
quantity is to be found.

When solving an applied problem, always begin by determining the unknown quantity and representing it
with a letter.

7.4.3 Problem Solving


7.4.3.1 Sample Set A

Example 7.17
On a map, 2 inches represents 25 miles. How many miles are represented by 8 inches?

Step 1: The unknown quantity is miles.


Let x = number of miles represented by 8 inches
Step 2: The three specied numbers are
2 inches
25 miles
8 inches
Step 3: The comparisons are
2 inches
2 inches to 25 miles → 25 miles
8 inches to x miles → x inches
8
miles
Proportions involving ratios and rates are more readily solved by suspending the units while
doing the computations.
2 8
25 = x
Step 4: 2
25 = 8
x Perform the cross multiplication.
2·x = 8 · 25
2·x = 200 Divide 200 by 2.
200
x = 2
x = 100
In step 1, we let x represent the number of miles. So, x represents 100 miles.
Step 5: If 2 inches represents 25 miles, then 8 inches represents 100 miles.
Try Exercise 7.105 in Section 7.4.3.2 (Practice Set A).
431

Example 7.18
An acid solution is composed of 7 parts water to 2 parts acid. How many parts of water are there
in a solution composed of 20 parts acid?
Step 1: The unknown quantity is the number of parts of water.
Let n = number of parts of water.
Step 2: The three specied numbers are
7 parts water
2 parts acid
20 parts acid
Step 3: The comparisons are
7 parts water to 2 parts acid → 72
n parts water to 20 parts acid → 20n
7 n
2 = 20
Step 4: 7
2 = n
20 Perform the cross multiplication.
7 · 20 = 2·n
140 = 2 · n Divide 140 by 2.
140
2 = n
70 = n
In step 1 we let n represent the number of parts of water. So, n represents 70 parts of water.
Step 5: 7 parts water to 2 parts acid indicates 70 parts water to 20 parts acid.
Try Problem 7.106 in Section 7.4.3.2 (Practice Set A).

Example 7.19
A 5-foot girl casts a 3 13 -foot shadow at a particular time of the day. How tall is a person who casts
a 3-foot shadow at the same time of the day?
Step 1: The unknown quantity is the height of the person.
Let h = height of the person.
Step 2: The three specied numbers are
5 feet ( height of girl)
3 13 feet (length of shadow)
3 feet (length of shadow)
Step 3: The comparisons are
5-foot girl is to 3 13 foot shadow → 351
3
h-foot person is to 3-foot shadow → h
3
5 h
3 13
= 3
5 h
Step 4: =
3 13 3

5·3 = 3 13 · h
15 = 10
3 ·h Divide 15 by 10
3
15
10 = h
3
3
)15 3
1 · = h
)10
2
9
2 = h
h = 4 12
Step 5: A person who casts a 3-foot shadow at this particular time of the day is 4 12 feet tall.
Try Exercise 7.107 in Section 7.4.3.2 (Practice Set A).
432 CHAPTER 7. RATIOS AND RATES

Example 7.20
The ratio of men to women in a particular town is 3 to 5. How many women are there in the town
if there are 19,200 men in town?
Step 1: The unknown quantity is the number of women in town.
Let x = number of women in town.
Step 2: The three specied numbers are
3
5
19,200
Step 3: The comparisons are 3 men to 5 women → 35
19,200 men to x women → 19,x200
3 19,200
5 = x
3 19,200
Step 4: =
5 x

3·x = 19, 200 · 5


3·x = 96, 000
96,000
x = 3
x = 32, 000
Step 5: There are 32,000 women in town.

Example 7.21
The rate of wins to losses of a particular baseball team is 9
2 . How many games did this team lose
if they won 63 games?
Step 1: The unknown quantity is the number of games lost.
Let n = number of games lost.
Step 2: Since 92 → means 9 wins to 2 losses, the three specied numbers are
9 (wins)
2 (losses)
63 (wins)
Step 3: The comparisons are
9 wins to 2 losses→ 92
63 wins to n losses → 63
n
9 63
2 = n
Step 4: 9 63
2 = n

9·n = 2 · 63
9·n = 126
126
n = 9
n = 14
Step 5: This team had 14 losses.
Try Exercise 7.108 in Section 7.4.3.2 (Practice Set A).

7.4.3.2 Practice Set A

Solve each problem.


Exercise 7.105 (Solution on p. 466.)
On a map, 3 inches represents 100 miles. How many miles are represented by 15 inches?
433

Step 1:
Step 2:
Step 3:
Step 4:
Step 5:

Exercise 7.106 (Solution on p. 466.)


An alcohol solution is composed of 14 parts water to 3 parts alcohol. How many parts of alcohol
are in a solution that is composed of 112 parts water?
Step 1:
Step 2:
Step 3:
Step 4:
Step 5:

Exercise 7.107 (Solution on p. 466.)


A 5 12 -foot woman casts a 7-foot shadow at a particular time of the day. How long of a shadow
does a 3-foot boy cast at that same time of day?
Step 1:
Step 2:
Step 3:
Step 4:
Step 5:

Exercise 7.108 (Solution on p. 466.)


The rate of houseplants to outside plants at a nursery is 4 to 9. If there are 384 houseplants in the
nursery, how many outside plants are there?
Step 1:
Step 2:
Step 3:
Step 4:
Step 5:

Exercise 7.109 (Solution on p. 466.)


The odds for a particular event occurring are 11 to 2. (For every 11 times the event does occur, it
will not occur 2 times.) How many times does the event occur if it does not occur 18 times?
Step 1:
Step 2:
Step 3:
Step 4:
Step 5:

Exercise 7.110 (Solution on p. 466.)


The rate of passing grades to failing grades in a particular chemistry class is 7
2 . If there are 21
passing grades, how many failing grades are there?
Step 1:
Step 2:
Step 3:
Step 4:
Step 5:
434 CHAPTER 7. RATIOS AND RATES

7.4.4 Exercises
For the following 20 problems, use the ve-step method to solve each problem.
Exercise 7.111 (Solution on p. 466.)
On a map, 4 inches represents 50 miles. How many inches represent 300 miles?
Exercise 7.112
On a blueprint for a house, 2 inches represents 3 feet. How many inches represent 10 feet?
Exercise 7.113 (Solution on p. 466.)
A model is built to 15
2
scale. If a particular part of the model measures 6 inches, how long is the
actual structure?
Exercise 7.114
An acid solution is composed of 5 parts acid to 9 parts of water. How many parts of acid are there
in a solution that contains 108 parts of water?
Exercise 7.115 (Solution on p. 466.)
An alloy contains 3 parts of nickel to 4 parts of silver. How much nickel is in an alloy that contains
44 parts of silver?
Exercise 7.116
The ratio of water to salt in a test tube is 5 to 2. How much salt is in a test tube that contains 35
ml of water?
Exercise 7.117 (Solution on p. 467.)
The ratio of sulfur to air in a container is 45
4
. How many ml of air are there in a container that
contains 207 ml of sulfur?
Exercise 7.118
A 6-foot man casts a 4-foot shadow at a particular time of the day. How tall is a person that casts
a 3-foot shadow at that same time of the day?
Exercise 7.119 (Solution on p. 467.)
A 5 21 -foot woman casts a 1 12 -foot shadow at a particular time of the day. How long a shadow does
her 3 21 -foot niece cast at the same time of the day?
Exercise 7.120
A man, who is 6 feet tall, casts a 7-foot shadow at a particular time of the day. How tall is a tree
that casts an 84-foot shadow at that same time of the day?
Exercise 7.121 (Solution on p. 467.)
The ratio of books to shelves in a bookstore is 350 to 3. How many books are there in a store that
has 105 shelves?
Exercise 7.122
The ratio of algebra classes to geometry classes at a particular community college is 13 to 2. How
many geometry classes does this college oer if it oers 13 algebra classes?
Exercise 7.123 (Solution on p. 467.)
The odds for a particular event to occur are 16 to 3. If this event occurs 64 times, how many times
would you predict it does not occur?
Exercise 7.124
The odds against a particular event occurring are 8 to 3. If this event does occur 64 times, how
many times would you predict it does not occur?
Exercise 7.125 (Solution on p. 467.)
The owner of a stationery store knows that a 1-inch stack of paper contains 300 sheets. The owner
wishes to stack the paper in units of 550 sheets. How many inches tall should each stack be?
435

Exercise 7.126
A recipe that requires 6 cups of sugar for 15 servings is to be used to make 45 servings. How much
sugar will be needed?
Exercise 7.127 (Solution on p. 467.)
A pond loses 7 12 gallons of water every 2 days due to evaporation. How many gallons of water are
lost, due to evaporation, in 12 day?
Exercise 7.128
A photograph that measures 3 inches wide and 4 21 inches high is to be enlarged so that it is 5
inches wide. How high will it be?
Exercise 7.129 (Solution on p. 467.)
If 25 pounds of fertilizer covers 400 square feet of grass, how many pounds will it take to cover 500
square feet of grass?
Exercise 7.130
Every 1 12 teaspoons of a particular multiple vitamin, in granular form, contains 0.65 the minimum
daily requirement of vitamin C. How many teaspoons of this vitamin are required to supply 1.25
the minimum daily requirement?

7.4.4.1 Exercises for Review

Exercise 7.131 (Solution on p. 467.)


(Section 2.2) Find the product, 818 · 0.
Exercise 7.132
(Section 4.4) Determine the missing numerator: 15
8 N
= 90 .
Exercise 7.133 (Solution on p. 467.)
3 4
10 + 12
(Section 5.6) Find the value of 19 .
20

Exercise 7.134
(Section 6.5) Subtract 0.249 from the sum of 0.344 and 0.612.
Exercise 7.135 (Solution on p. 467.)
(Section 7.3) Solve the proportion: 6
x = 36
30 .

7.5 Percent5
7.5.1 Section Overview
• Ratios and Percents
• The Relationship Between Fractions, Decimals, and Percents  Making Conversions

5 This content is available online at <http://cnx.org/content/m34983/1.2/>.


436 CHAPTER 7. RATIOS AND RATES

7.5.2 Ratios and Percents


Ratio, Percent
We dened a ratio as a comparison, by division, of two pure numbers or two like denominate numbers. A
most convenient number to compare numbers to is 100. Ratios in which one number is compared to 100 are
called percents. The word percent comes from the Latin word "per centum." The word "per" means "for
each" or "for every," and the word "centum" means "hundred." Thus, we have the following denition.

Percent means for each hundred," or "for every hundred."

The symbol % is used to represent the word percent.

7.5.2.1 Sample Set A

Example 7.22
The ratio 26 to 100 can be written as 26%. We read 26% as "twenty-six percent."
Example 7.23
The ratio 165
100 can be written as 165%.
We read 165% as "one hundred sixty-ve percent."
Example 7.24
38 .
The percent 38% can be written as the fraction 100
Example 7.25
210 or the mixed number 2 10 or 2.1.
The percent 210% can be written as the fraction 100 100
Example 7.26
25 of a dollar. This implies that 25 cents is 25% of one
Since one dollar is 100 cents, 25 cents is 100
dollar.

7.5.2.2 Practice Set A

Exercise 7.136 (Solution on p. 467.)


Write the ratio 16 to 100 as a percent.
Exercise 7.137 (Solution on p. 467.)
Write the ratio 195 to 100 as a percent.
Exercise 7.138 (Solution on p. 467.)
Write the percent 83% as a ratio in fractional form.
Exercise 7.139 (Solution on p. 467.)
Write the percent 362% as a ratio in fractional form.

7.5.3 The Relationship Between Fractions, Decimals, and Percents  Making


Conversions
Since a percent is a ratio, and a ratio can be written as a fraction, and a fraction can be written as a decimal,
any of these forms can be converted to any other.

Before we proceed to the problems in Section 7.5.3.1 (Sample Set B) and Section 7.5.3.2 (Practice Set B),
let's summarize the conversion techniques.
437

Conversion Techniques  Fractions, Decimals, Percents


438 CHAPTER 7. RATIOS AND RATES

To Convert a Fraction To Convert a Decimal To Convert a Percent


To a decimal: Divide the numer- To a fraction: Read the decimal To a decimal: Move the decimal
ator by the denominator and reduce the resulting fraction point 2 places to the left and drop
the % symbol
To a percent: Convert the frac- To a percent: Move the decimal To a fraction: Drop the % sign
tion rst to a decimal, then move point 2 places to the right and af- and write the number over 100.
the decimal point 2 places to the x the % symbol Reduce, if possible.
right and ax the % symbol.

Table 7.1

7.5.3.1 Sample Set B

Example 7.27
Convert 12% to a decimal.
12 = 0.12
12% = 100

Note that

The % symbol is dropped, and the decimal point moves 2 places to the left.
Example 7.28
Convert 0.75 to a percent.
75 = 75%
0.75 = 100

Note that

The % symbol is axed, and the decimal point moves 2 units to the right.
Example 7.29
Convert 3
5 to a percent.

We see in Example 7.28 that we can convert a decimal to a percent. We also know that we can
convert a fraction to a decimal. Thus, we can see that if we rst convert the fraction to a decimal,
we can then convert the decimal to a percent.

.6
5)3.0 60 = 60%
3
5 → or 3
5 = 0.6 = 10
6
= 100
30
0
Example 7.30
Convert 42% to a fraction.
42 = 21
42% = 100 50
or
439

42 = 21
42% = 0.42 = 100 50

7.5.3.2 Practice Set B

Exercise 7.140 (Solution on p. 467.)


Convert 21% to a decimal.
Exercise 7.141 (Solution on p. 467.)
Convert 461% to a decimal.
Exercise 7.142 (Solution on p. 467.)
Convert 0.55 to a percent.
Exercise 7.143 (Solution on p. 467.)
Convert 5.64 to a percent.
Exercise 7.144 (Solution on p. 467.)
Convert 20
3
to a percent.
Exercise 7.145 (Solution on p. 467.)
Convert 11
8 to a percent
Exercise 7.146 (Solution on p. 467.)
Convert 11
3
to a percent.

7.5.4 Exercises
For the following 12 problems, convert each decimal to a percent.
Exercise 7.147 (Solution on p. 467.)
0.25
Exercise 7.148
0.36
Exercise 7.149 (Solution on p. 467.)
0.48
Exercise 7.150
0.343
Exercise 7.151 (Solution on p. 467.)
0.771
Exercise 7.152
1.42
Exercise 7.153 (Solution on p. 467.)
2.58
Exercise 7.154
4.976
Exercise 7.155 (Solution on p. 468.)
16.1814
Exercise 7.156
533.01
Exercise 7.157 (Solution on p. 468.)
2
440 CHAPTER 7. RATIOS AND RATES

Exercise 7.158
14
For the following 10 problems, convert each percent to a decimal.
Exercise 7.159 (Solution on p. 468.)
15%
Exercise 7.160
43%
Exercise 7.161 (Solution on p. 468.)
16.2%
Exercise 7.162
53.8%
Exercise 7.163 (Solution on p. 468.)
5.05%
Exercise 7.164
6.11%
Exercise 7.165 (Solution on p. 468.)
0.78%
Exercise 7.166
0.88%
Exercise 7.167 (Solution on p. 468.)
0.09%
Exercise 7.168
0.001%
For the following 14 problems, convert each fraction to a percent.
Exercise 7.169 (Solution on p. 468.)
1
5
Exercise 7.170
3
5
Exercise 7.171 (Solution on p. 468.)
5
8
Exercise 7.172
1
16
Exercise 7.173 (Solution on p. 468.)
7
25
Exercise 7.174
16
45
Exercise 7.175 (Solution on p. 468.)
27
55
Exercise 7.176
15
8
Exercise 7.177 (Solution on p. 468.)
41
25
Exercise 7.178
6 54
Exercise 7.179 (Solution on p. 468.)
9
9 20
441

Exercise 7.180
1
200
Exercise 7.181 (Solution on p. 468.)
6
11
Exercise 7.182
35
27
For the following 14 problems, convert each percent to a fraction.
Exercise 7.183 (Solution on p. 468.)
80%
Exercise 7.184
60%
Exercise 7.185 (Solution on p. 468.)
25%
Exercise 7.186
75%
Exercise 7.187 (Solution on p. 468.)
65%
Exercise 7.188
18%
Exercise 7.189 (Solution on p. 468.)
12.5%
Exercise 7.190
37.5%
Exercise 7.191 (Solution on p. 468.)
512.5%
Exercise 7.192
937.5%
Exercise 7.193 (Solution on p. 468.)
_
9. 9 %
Exercise
_ 7.194
55. 5 %
Exercise 7.195 (Solution on p. 468.)
_
22. 2 %
Exercise
_ 7.196
63. 6 %

7.5.4.1 Exercises for Review

Exercise 7.197 (Solution on p. 468.)


40 ÷ 8 7 .
(Section 4.6) Find the quotient. 54 21
Exercise 7.198
(Section 4.7) 3
8 of what number is 2 23 ?
Exercise 7.199 (Solution on p. 468.)
(Section 5.3) Find the value of 28
15 + 10 − 12 .
7 5
442 CHAPTER 7. RATIOS AND RATES

Exercise 7.200
(Section 6.4) Round 6.99997 to the nearest ten thousandths.
Exercise 7.201 (Solution on p. 468.)
(Section 7.4) On a map, 3 inches represent 40 miles. How many inches represent 480 miles?

7.6 Fractions of One Percent6


7.6.1 Section Overview
• Conversions Involving Fractions of One Percent
• Conversions Involving Nonterminating Fractions

7.6.2 Conversions Involving Fractions of One Percent


Percents such as 21 %, 3
5 %, 5
8 %, and 11
7
%, where 1% has not been attained, are fractions of 1%. This implies
that
1
2% = 1
2 of 1%
3
5% = 3
5 of 1%
5
8% = 5
8 of 1%

11 % = 11 of 1%
7 7

Since "percent" means "for each hundred," and "of" means "times," we have
1
2% = 12 of 1% = 1
2
1
· 100 1
= 200
3
5% = 35 of 1% = 3
5
1
· 100 3
= 500
5
8% = 58 of 1% = 5
8
1
· 100 5
= 800

11 % = 11 of 1% = 11 · 100 = 1100
7 7 7 1 7

7.6.2.1 Sample Set A

Example 7.31
Convert 23 % to a fraction.
1
)2
2
3% = 2
3 of 1% = 3 · 1
)100
50
1·1
= 3·50
1
= 150

Example 7.32
Convert 5
8 % to a decimal.

6 This content is available online at <http://cnx.org/content/m34997/1.2/>.


443

5
8% = 5
8 of 1% = 5
8 · 1
100
= 0.625 · 0.01
= 0.00625

7.6.2.2 Practice Set A

Exercise 7.202 (Solution on p. 468.)


Convert 1
4 % to a fraction.
Exercise 7.203 (Solution on p. 469.)
Convert 3
8 % to a fraction.
Exercise 7.204 (Solution on p. 469.)
Convert 3 13 % to a fraction.

7.6.3 Conversions Involving Nonterminating Fractions


We must be careful when changing a fraction of 1% to a decimal. The number 23 , as we know, has a
nonterminating decimal representation. Therefore, it cannot be expressed exactly as a decimal.

When converting nonterminating fractions of 1% to decimals, it is customary to express the fraction as a


rounded decimal with at least three decimal places.

Converting a Nonterminating Fraction to a Decimal


To convert a nonterminating fraction of 1% to a decimal:

1. Convert the fraction as a rounded decimal.


2. Move the decimal point two digits to the left and remove the percent sign.

7.6.3.1 Sample Set B

Example 7.33
Convert 2
3 % to a three-place decimal.

1. Convert 23 to a decimal.
Since we wish the resulting decimal to have three decimal digits, and removing the percent
sign will account for two of them, we need to round 23 to one place (2 + 1 = 3).

3 % = 0.7% to one decimal place. 3 = 0.6666 . . .


2 2


2. Move the decimal point two digits to the left and remove the % sign. We'll need to add zeros
to locate the decimal point in the correct location.
3 % = 0.007 to 3 decimal places
2

Example 7.34
Convert 5 11
4
% to a four-place decimal.

1. Since we wish the resulting decimal to have four decimal places, and removing the percent
sign will account for two, we to round 11
4
to two places.
5 11 % = 5.36% to two decimal places. 11
4 4
= 0.3636 . . .
2. Move the decimal point two places to the left and drop the percent sign. 5 11
4
% = 0.0536 to
four decimal places.
444 CHAPTER 7. RATIOS AND RATES

Example 7.35
Convert 28 59 % to a decimal rounded to ten thousandths.

1. Since we wish the resulting decimal to be rounded to ten thousandths (four decimal places),
and removing the percent sign will account for two, we need to round 59 to two places.
28 59 % = 28.56% to two decimal places. 95 = 0.5555 . . .
2. Move the decimal point to the left two places and drop the percent sign.
28 59 % = 0.2856 correct to ten thousandths.

7.6.3.2 Practice Set B

Exercise 7.205 (Solution on p. 469.)


Convert 97 % to a three-place decimal.
Exercise 7.206 (Solution on p. 469.)
Convert 51 11
5
% to a decimal rounded to ten thousandths.

7.6.4 Exercises
Make the conversions as indicated.
Exercise 7.207 (Solution on p. 469.)
Convert 3
4 % to a fraction.
Exercise 7.208
Convert 56 % to a fraction.
Exercise 7.209 (Solution on p. 469.)
Convert 91 % to a fraction.
Exercise 7.210
Convert 15
19 % to a fraction.
Exercise 7.211 (Solution on p. 469.)
Convert 5
4 % to a fraction.
Exercise 7.212
Convert 73 % to a fraction.
Exercise 7.213 (Solution on p. 469.)
Convert 1 67 % to a fraction.
Exercise 7.214
Convert 2 16
5
% to a fraction.
Exercise 7.215 (Solution on p. 469.)
Convert 25 14 % to a fraction.
Exercise 7.216
Convert 50 12 % to a fraction.
Exercise 7.217 (Solution on p. 469.)
Convert 72 35 % to a fraction.
Exercise 7.218
Convert 99 18 % to a fraction.
Exercise 7.219 (Solution on p. 469.)
Convert 136 32 % to a fraction.
445

Exercise 7.220
Convert 521 34 % to a fraction.
Exercise 7.221 (Solution on p. 469.)
Convert 10 15 % to a decimal.
Exercise 7.222
Convert 12 34 % to a decimal.
Exercise 7.223 (Solution on p. 469.)
Convert 3 78 % to a decimal.
Exercise 7.224
Convert 7 16
1
% to a decimal.
Exercise 7.225 (Solution on p. 469.)
Convert 73 % to a three-place decimal.
Exercise 7.226
Convert 19 % to a three-place decimal.
Exercise 7.227 (Solution on p. 469.)
Convert 6 11
3
% to a four-place decimal.
Exercise 7.228
Convert 9 27 % to a four-place decimal.
Exercise 7.229 (Solution on p. 469.)
Convert 24 21
5
% to a three-place decimal.
Exercise 7.230
Convert 45 27
8
% to a three-place decimal.
Exercise 7.231 (Solution on p. 469.)
Convert 11 16
17 % to a four-place decimal.
Exercise 7.232
Convert 5 17 % to a three-place decimal.

7.6.4.1 Exercises for Review

Exercise 7.233 (Solution on p. 469.)


(Section 3.2) Write 8 · 8 · 8 · 8 · 8 using exponents.
Exercise 7.234
(Section 4.3) Convert 4 87 to an improper fraction.
Exercise 7.235 (Solution on p. 469.)
(Section 5.4) Find the sum. 10
7 2
+ 21 + 71 .
Exercise 7.236
(Section 6.6) Find the product. (4.21) (0.006).
Exercise 7.237 (Solution on p. 469.)
(Section 7.5) Convert 8.062 to a percent.
446 CHAPTER 7. RATIOS AND RATES

7.7 Applications of Percents7


7.7.1 Section Overview
• Base, Percent, and Percentage
• Finding the Percentage
• Finding the Percent
• Finding the Base

7.7.2 Base, Percent, and Percentage


There are three basic types of percent problems. Each type involves a base, a percent, and a percentage,
and when they are translated from words to mathematical symbols each becomes a multiplication statement.
Examples of these types of problems are the following:

1. What number is 30% of 50? (Missing product statement.)


2. 15 is what percent of 50? (Missing factor statement.)
3. 15 is 30% of what number? (Missing factor statement.)

In problem 1 (list, p. 446), the product is missing. To solve the problem, we represent the missing product
with P .

P = 30% · 50

Percentage
The missing product P is called the percentage. Percentage means part, or portion. In P = 30% · 50, P
represents a particular part of 50.

In problem 2 (list, p. 446), one of the factors is missing. Here we represent the missing factor with Q.

15 = Q · 50

Percent
The missing factor is the percent. Percent, we know, means per 100, or part of 100. In 15 = Q · 50, Q
indicates what part of 50 is being taken or considered. Specically, 15 = Q · 50 means that if 50 was to be
divided into 100 equal parts, then Q indicates 15 are being considered.

In problem 3 (list, p. 446), one of the factors is missing. Represent the missing factor with B .

15 = 30% · B

Base
The missing factor is the base. Some meanings of base are a source of supply, or a starting place. In
15 = 30% · B , B indicates the amount of supply. Specically, 15 = 30% · B indicates that 15 represents
30% of the total supply.

Each of these three types of problems is of the form

(percentage) = (percent) · (base)

We can determine any one of the three values given the other two using the methods discussed in Section 4.7.
7 This content is available online at <http://cnx.org/content/m35007/1.2/>.
447

7.7.3 Finding the Percentage


7.7.3.1 Sample Set A

Example 7.36
What number} is 30% of 50? Missing product statement.
| {z ↓ ↓ ↓ ↓

(percentage) = (percent) · (base)


↓ ↓ ↓ ↓ ↓

P = 30% · 50 Convert 30% to a decimal.


P = .30 · 50 Multiply.
P = 15

Thus, 15 is 30% of 50.

Do Section 7.7.3.2 (Practice Set A), Exercise 7.238.


Example 7.37
What number} is 36% of 95? Missing product statement.
| {z ↓ ↓ ↓ ↓

(percentage) = (percent) · (base)


↓ ↓ ↓ ↓ ↓

P = 36% · 95 Convert 36% to a decimal.


P = .36 · 95 Multiply
P = 34.2

Thus, 34.2 is 36% of 95.

Do Section 7.7.3.2 (Practice Set A), Exercise 7.238.


Example 7.38
A salesperson, who gets a commission of 12% of each sale she makes, makes a sale of $8,400.00.
How much is her commission?

We need to determine what part of $8,400.00 is to be taken. What part indicates percentage.

What number} is 12% of 8, 400.00? Missing product statement.


| {z ↓ ↓ ↓ ↓

(percentage) = (percent) · (base)


↓ ↓ ↓ ↓ ↓

P = 12% · 8, 400.00 Convert to decimals.


P = .12 · 8, 400.00 Multiply.
P = 1008.00

Thus, the salesperson's commission is $1,008.00.

Do Section 7.7.3.2 (Practice Set A), Exercise 7.239.


Example 7.39
A girl, by practicing typing on her home computer, has been able to increase her typing speed by
110%. If she originally typed 16 words per minute, by how many words per minute was she able to
increase her speed?

We need to determine what part of 16 has been taken. What part indicates percentage.
448 CHAPTER 7. RATIOS AND RATES

What number} is 110% of 16? Missing product statement.


| {z ↓ ↓ ↓ ↓

(percentage) = (percent) · (base)


↓ ↓ ↓ ↓ ↓

P = 110% · 16 Convert to decimals.


P = 1.10 · 16 Multiply.
P = 17.6

Thus, the girl has increased her typing speed by 17.6 words per minute. Her new speed is 16 + 17.6 =
33.6 words per minute.

Do Section 7.7.3.2 (Practice Set A), Exercise 7.240.


Example 7.40
A student who makes $125 a month working part-time receives a 4% salary raise. What is the
student's new monthly salary?

With a 4% raise, this student will make 100% of the original salary + 4% of the original salary.
This means the new salary will be 104% of the original salary. We need to determine what part of
$125 is to be taken. What part indicates percentage.

What number} is 104% of 125 Missing product statement.


| {z ↓ ↓ ↓ ↓

(percentage) = (percent) · (base)


↓ ↓ ↓ ↓ ↓

P = 104% · 125 Convert to decimals.


P = 1.04 · 125 Multiply.
P = 130

Thus, this student's new monthly salary is $130.

Do Section 7.7.3.2 (Practice Set A), Exercise 7.241.


Example 7.41
An article of clothing is on sale at 15% o the marked price. If the marked price is $24.95, what
is the sale price?

Since the item is discounted 15%, the new price will be 100% − 15% = 85% of the marked price.
We need to determine what part of 24.95 is to be taken. What part indicates percentage.

What number} is 85% of $24.95. Missing product statement.


| {z ↓ ↓ ↓ ↓

(percentage) = (percent) · (base)


↓ ↓ ↓ ↓ ↓

P = 85% · 24.95 Convert to decimals.


P = .85 · 24.95 Multiply.
Since this number represents money,
P = 21.2075
we'll round to 2 decimal places
P = 21.21

Thus, the sale price of the item is $21.21.


449

7.7.3.2 Practice Set A

Exercise 7.238 (Solution on p. 469.)


What number is 42% of 85?
Exercise 7.239 (Solution on p. 469.)
A sales person makes a commission of 16% on each sale he makes. How much is his commission if
he makes a sale of $8,500?
Exercise 7.240 (Solution on p. 469.)
An assembly line worker can assemble 14 parts of a product in one hour. If he can increase his
assembly speed by 35%, by how many parts per hour would he increase his assembly of products?
Exercise 7.241 (Solution on p. 469.)
A computer scientist in the Silicon Valley makes $42,000 annually. What would this scientist's
new annual salary be if she were to receive an 8% raise?

7.7.4 Finding the Percent


7.7.4.1 Sample Set B

Example 7.42
15 is what percent of 50? Missing factor statement.
↓ ↓ | {z } ↓ ↓

(percentage) = (percent) · (base) [(product) = (factor) · (factor)]


↓ ↓ ↓ ↓ ↓

15 = Q · 50

Recall that (missing factor) = (product) ÷ (known factor).

Q = 15 ÷ 50 Divide.
Q = 0.3 Convert to a percent.
Q = 30%

Thus, 15 is 30% of 50.

Do Section 7.7.4.2 (Practice Set B), Exercise 7.242.


Example 7.43
4.32 is what percent of 72? Missing factor statement.
↓ ↓ | {z } ↓ ↓

(percentage) = (percent) · (base) [(product) = (factor) · (factor)]


↓ ↓ ↓ ↓ ↓

4.32 = Q · 72

Q = 4.32 ÷ 72 Divide.
Q = 0.06 Convert to a percent.
Q = 6%

Thus, 4.32 is 6% of 72.


450 CHAPTER 7. RATIOS AND RATES

Do Section 7.7.4.2 (Practice Set B), Exercise 7.242.


Example 7.44
On a 160 question exam, a student got 125 correct answers. What percent is this? Round the
result to two decimal places.

We need to determine the percent.

125 is what percent of 160? Missing factor statement.


↓ ↓ | {z } ↓ ↓

(percentage) = (percent) · (base) [(product) = (factor) · (factor)]


↓ ↓ ↓ ↓ ↓

125 = Q · 160

Q = 125 ÷ 160 Divide.


Q = 0.78125 Round to two decimal places.
Q = .78

Thus, this student received a 78% on the exam.

Do Section 7.7.4.2 (Practice Set B), Exercise 7.243.


Example 7.45
A bottle contains 80 milliliters of hydrochloric acid (HCl) and 30 milliliters of water. What percent
of HCl does the bottle contain? Round the result to two decimal places.

We need to determine the percent. The total amount of liquid in the bottle is

80 milliliters + 30 milliliters = 110 milliliters.

80 is what percent of 110? Missing factor statement.


↓ ↓ | {z } ↓ ↓

(percentage) = (percent) · (base) [(product) = (factor) · (factor)]


↓ ↓ ↓ ↓ ↓

80 = Q · 110

Q = 80 ÷ 110 Divide.
Q = 0.727272. . . Round to two decimal places.
Q ≈ 73% The symbol "≈" is read as "approximately."

Thus, this bottle contains approximately 73% HCl.

Do Section 7.7.4.2 (Practice Set B), Exercise 7.244.


Example 7.46
Five years ago a woman had an annual income of $19,200. She presently earns $42,000 annually.
By what percent has her salary increased? Round the result to two decimal places.

We need to determine the percent.


451

42, 000 is what percent of 19, 200? Missing factor statement.


↓ ↓ | {z } ↓ ↓

(percentage) = (percent) · (base)


↓ ↓ ↓ ↓ ↓

42, 000 = Q · 19, 200

Q = 42, 000 ÷ 19, 200 Divide.


Q = 2.1875 Round to two decimal places.
Q = 2.19 Convert to a percent.
Q = 219% Convert to a percent.

Thus, this woman's annual salary has increased 219%.

7.7.4.2 Practice Set B

Exercise 7.242 (Solution on p. 469.)


99.13 is what percent of 431?
Exercise 7.243 (Solution on p. 470.)
On an 80 question exam, a student got 72 correct answers. What percent did the student get on
the exam?
Exercise 7.244 (Solution on p. 470.)
A bottle contains 45 milliliters of sugar and 67 milliliters of water. What fraction of sugar does
the bottle contain? Round the result to two decimal places (then express as a percent).

7.7.5 Finding the Base


7.7.5.1 Sample Set C

Example 7.47
15 is 30% of what number? Missing factor statement.
↓ ↓ ↓ ↓ | {z }
(percentage) = (percent) · (base) [(percentage) = (factor) · (factor)]
↓ ↓ ↓ ↓ ↓

15 = 30% · B Convert to decimals.


15 = .30 · B [(missing factor) = (product) ÷ (known factor)]

B = 15 ÷ .30
B = 50

Thus, 15 is 30% of 50.

Try Exercise 7.245 in Section 7.7.5.2 (Practice Set C).


452 CHAPTER 7. RATIOS AND RATES

Example 7.48
56.43 is 33% of what } Missing factor statement.
number?
↓ ↓ ↓ ↓ | {z
(percentage) = (percent) · (base)
↓ ↓ ↓ ↓ ↓

56.43 = 33% · B Convert to decimals.


56.43 = .33 · B Divide.

B = 56.43 ÷ .33
B = 171

Thus, 56.43 is 33% of 171.

Try Exercise 7.245 in Section 7.7.5.2 (Practice Set C).


Example 7.49
Fifteen milliliters of water represents 2% of a hydrochloric acid (HCl) solution. How many milliliters
of solution are there?

We need to determine the total supply. The word supply indicates base.

15 is 2% of what number?
} Missing factor statement.
↓ ↓ ↓ ↓ | {z
(percentage) = (percent) · (base)
↓ ↓ ↓ ↓ ↓

15 = 2% · B Convert to decimals.
15 = .02 · B Divide.

B = 15 ÷ .02
B = 750

Thus, there are 750 milliliters of solution in the bottle.

Try Exercise 7.246 in Section 7.7.5.2 (Practice Set C).


Example 7.50
In a particular city, a sales tax of 6 12 % is charged on items purchased in local stores. If the tax on
an item is $2.99, what is the price of the item?

We need to determine the price of the item. We can think of price as the starting place. Starting
place indicates base. We need to determine the base.

2.99 is 6 12 % of what number? Missing factor statement.


↓ ↓ ↓ ↓ | {z }
(percentage) = (percent) · (base)
↓ ↓ ↓ ↓ ↓

2.99 = 6 12 % · B Convert to decimals.


2.99 = 6.5% · B
2.99 = .065 · B [(missing factor) = (product) ÷ (known factor)]
453

B = 2.99 ÷ .065 Divide.


B = 46

Thus, the price of the item is $46.00.

Try Exercise 7.247 in Section 7.7.5.2 (Practice Set C).


Example 7.51
A clothing item is priced at $20.40. This marked price includes a 15% discount. What is the
original price?

We need to determine the original price. We can think of the original price as the starting place.
Starting place indicates base. We need to determine the base. The new price, $20.40, represents
100% − 15% = 85% of the original price.

20.40 is 85% of what number? Missing factor statement.


↓ ↓ ↓ ↓ | {z }
(percentage) = (percent) · (base)
↓ ↓ ↓ ↓ ↓

20.40 = 85% · B Convert to decimals.


20.40 = .85 · B [(missing factor) = (product) ÷ (known factor)]

B = 20.40 ÷ .85 Divide.


B = 24

Thus, the original price of the item is $24.00.

Try Exercise 7.248 in Section 7.7.5.2 (Practice Set C).

7.7.5.2 Practice Set C

Exercise 7.245 (Solution on p. 470.)


1.98 is 2% of what number?
Exercise 7.246 (Solution on p. 470.)
3.3 milliliters of HCl represents 25% of an HCl solution. How many milliliters of solution are there?
Exercise 7.247 (Solution on p. 470.)
A salesman, who makes a commission of 18 14 % on each sale, makes a commission of $152.39 on a
particular sale. Rounded to the nearest dollar, what is the amount of the sale?
Exercise 7.248 (Solution on p. 470.)
At "super-long play," 2 12 hours of play of a video cassette recorder represents 31.25% of the total
playing time. What is the total playing time?

7.7.6 Exercises
For the following 25 problems, nd each indicated quantity.
Exercise 7.249 (Solution on p. 470.)
What is 21% of 104?
454 CHAPTER 7. RATIOS AND RATES

Exercise 7.250
What is 8% of 36?
Exercise 7.251 (Solution on p. 470.)
What is 98% of 545?
Exercise 7.252
What is 143% of 33?
Exercise 7.253 (Solution on p. 470.)
What is 10 12 % of 20?
Exercise 7.254
3.25 is what percent of 88?
Exercise 7.255 (Solution on p. 470.)
22.44 is what percent of 44?
Exercise 7.256
0.0036 is what percent of 0.03?
Exercise 7.257 (Solution on p. 470.)
31.2 is what percent of 26?
Exercise 7.258
266.4 is what percent of 74?
Exercise 7.259 (Solution on p. 470.)
0.0101 is what percent of 0.0505?
Exercise 7.260
2.4 is 24% of what number?
Exercise 7.261 (Solution on p. 470.)
24.19 is 41% of what number?
Exercise 7.262
61.12 is 16% of what number?
Exercise 7.263 (Solution on p. 470.)
82.81 is 91% of what number?
Exercise 7.264
115.5 is 20% of what number?
Exercise 7.265 (Solution on p. 470.)
43.92 is 480% of what number?
Exercise 7.266
What is 85% of 62?
Exercise 7.267 (Solution on p. 470.)
29.14 is what percent of 5.13?
Exercise 7.268
0.6156 is what percent of 5.13?
Exercise 7.269 (Solution on p. 470.)
What is 0.41% of 291.1?
Exercise 7.270
26.136 is 121% of what number?
Exercise 7.271 (Solution on p. 470.)
1,937.5 is what percent of 775?
Exercise 7.272
1 is what percent of 2,000?
455

Exercise 7.273 (Solution on p. 470.)


0 is what percent of 59?
Exercise 7.274
An item of clothing is on sale for 10% o the marked price. If the marked price is $14.95, what is
the sale price? (Round to two decimal places.)
Exercise 7.275 (Solution on p. 470.)
A grocery clerk, who makes $365 per month, receives a 7% raise. How much is her new monthly
salary?
Exercise 7.276
An item of clothing which originally sells for $55.00 is marked down to $46.75. What percent has
it been marked down?
Exercise 7.277 (Solution on p. 470.)
On a 25 question exam, a student gets 21 correct. What percent is this?
Exercise 7.278
On a 45 question exam, a student gets 40%. How many questions did this student get correct?
Exercise 7.279 (Solution on p. 470.)
A vitamin tablet, which weighs 250 milligrams, contains 35 milligrams of vitamin C. What percent
of the weight of this tablet is vitamin C?
Exercise 7.280
Five years ago a secretary made $11,200 annually. The secretary now makes $17,920 annually. By
what percent has this secretary's salary been increased?
Exercise 7.281 (Solution on p. 470.)
A baseball team wins 48 43 % of all their games. If they won 78 games, how many games did they
play?
Exercise 7.282
A typist was able to increase his speed by 120% to 42 words per minute. What was his original
typing speed?
Exercise 7.283 (Solution on p. 470.)
A salesperson makes a commission of 12% on the total amount of each sale. If, in one month, she
makes a total of $8,520 in sales, how much has she made in commission?
Exercise 7.284
A salesperson receives a salary of $850 per month plus a commission of 8 12 % of her sales. If, in a
particular month, she sells $22,800 worth of merchandise, what will be her monthly earnings?
Exercise 7.285 (Solution on p. 470.)
A man borrows $1150.00 from a loan company. If he makes 12 equal monthly payments of $130.60,
what percent of the loan is he paying in interest?
Exercise 7.286
The distance from the sun to the earth is approximately 93,000,000 miles. The distance from the
sun to Pluto is approximately 860.2% of the distance from the sun to the Earth. Approximately,
how many miles is Pluto from the sun?
Exercise 7.287 (Solution on p. 471.)
The number of people on food stamps in Maine in 1975 was 151,000. By 1980, the number had
decreased to 59,200. By what percent did the number of people on food stamps decrease? (Round
the result to the nearest percent.)
Exercise 7.288
In Nebraska, in 1960, there were 734,000 motor-vehicle registrations. By 1979, the total had in-
creased by about 165.6%. About how many motor-vehicle registrations were there in Nebraska in
1979?
456 CHAPTER 7. RATIOS AND RATES

Exercise 7.289 (Solution on p. 471.)


From 1973 to 1979, in the United States, there was an increase of 166.6% of Ph.D. social scientists
to 52,000. How many were there in 1973?
Exercise 7.290
In 1950, in the United States, there were 1,894 daily newspapers. That number decreased to 1,747
by 1981. What percent did the number of daily newspapers decrease?
Exercise 7.291 (Solution on p. 471.)
A particular alloy is 27% copper. How many pounds of copper are there in 55 pounds of the alloy?
Exercise 7.292
A bottle containing a solution of hydrochloric acid (HCl) is marked 15% (meaning that 15% of the
HCl solution is acid). If a bottle contains 65 milliliters of solution, how many milliliters of water
does it contain?
Exercise 7.293 (Solution on p. 471.)
A bottle containing a solution of HCl is marked 45%. A test shows that 36 of the 80 milliliters
contained in the bottle are hydrochloric acid. Is the bottle marked correctly? If not, how should it
be remarked?

7.7.6.1 Exercises For Review

Exercise 7.294
(Section 2.6) Use the numbers 4 and 7 to illustrate the commutative property of multiplication.
Exercise 7.295 (Solution on p. 471.)
(Section 4.3) Convert 14
5 to a mixed number.
Exercise 7.296
(Section 5.5) Arrange the numbers 12
7
, 5
9 and 4
7 in increasing order.
Exercise 7.297 (Solution on p. 471.)
(Section 6.3) Convert 4.006 to a mixed number.
Exercise 7.298
(Section 7.6) Convert 7
8 % to a fraction.

7.8 Summary of Key Concepts8


7.8.1 Summary of Key Concepts
Denominate Numbers (Section 7.2)
Numbers that appear along with units are denominate numbers. The amounts 6 dollars and 4 pints are
examples of denominate numbers.

Like and Unlike Denominate Numbers (Section 7.2)


Like denominate numbers are denominate numbers with like units. If the units are not the same, the numbers
are unlike denominate numbers.

Pure Numbers (Section 7.2)


Numbers appearing without a unit are pure numbers.

8 This content is available online at <http://cnx.org/content/m35008/1.2/>.


457

Comparing Numbers by Subtraction and Division (Section 7.2)


Comparison of two numbers by subtraction indicates how much more one number is than another. Compar-
ison by division indicates how many times larger or smaller one number is than another.

Comparing Pure or Like Denominate Numbers by Subtraction (Section 7.2)


Numbers can be compared by subtraction if and only if they are pure numbers or like denominate numbers.

Ratio Rate (Section 7.2)


A comparison, by division, of two like denominate numbers is a ratio. A comparison, by division, of two
unlike denominate numbers is a rate.

Proportion (Section 7.3)


A proportion is a statement that two ratios or rates are equal.
people people
4 jobs is a proportion.
3 6
2 jobs =

Solving a Proportion (Section 7.3)


To solve a proportion that contains three known numbers and a letter that represents an unknown quantity,
perform the cross multiplication, then divide the product of the two numbers by the number that multiplies
the letter.

Proportions Involving Rates (Section 7.3)


When writing a proportion involving rates it is very important to write it so that the same type of units
appears on the same side of either the equal sign or the fraction bar.
unit type 1 = unit type 1 or unit type 1 = unit type 2
unit type 2 unit type 2 unit type 1 unit type 2

Five-Step Method for Solving Proportions (Section 7.4)

1. By careful reading, determine what the unknown quantity is and represent it with some letter. There
will be only one unknown in a problem.
2. Identify the three specied numbers.
3. Determine which comparisons are to be made and set up the proportion.
4. Solve the proportion.
5. Interpret and write a conclusion.

When solving applied problems, ALWAYS begin by determining the unknown quantity and representing it
with a letter.

Percents (Section 7.5)


A ratio in which one number is compared to 100 is a percent. Percent means "for each hundred."

Conversion of Fractions, Decimals, and Percents (Section 7.5)


It is possible to convert decimals to percents, fractions to percents, percents to decimals, and percents to
fractions.

Applications of Percents:
The three basic types of percent problems involve a base, a percentage, and a percent.

Base (Section 7.7)


The base is the number used for comparison.

Percentage (Section 7.7)


The percentage is the number being compared to the base.
458 CHAPTER 7. RATIOS AND RATES

Percent (Section 7.7)


By its denition, percent means part of.

Solving Problems (Section 7.7)


Percentage = (percent) × (base)
Percent = percentage
base
Base = percentage
percent

7.9 Exercise Supplement9


7.9.1 Exercise Supplement
7.9.1.1 Ratios and Rates (Section 7.2)

Exercise 7.299 (Solution on p. 471.)


Compare 250 watts to 100 watts by subtraction.
Exercise 7.300
Compare 126 and 48 by subtraction.
Exercise 7.301 (Solution on p. 471.)
Compare 98 radishes to 41 radishes by division.
Exercise 7.302
Compare 144 to 9 by division.
Exercise 7.303 (Solution on p. 471.)
Compare 100 tents to 5 tents by division.
Exercise 7.304
Compare 28 feet to 7 feet by division.
Exercise 7.305 (Solution on p. 471.)
Comparison, by division, of two pure numbers or two like denominate numbers is called a
.
Exercise 7.306
A comparison, by division, of two unlike denominate numbers is called a .
For problems 9-12, express each ratio or rate as a fraction.
Exercise 7.307 (Solution on p. 471.)
15 to 5
Exercise 7.308
72 to 12
Exercise 7.309 (Solution on p. 471.)
8 millimeters to 5 milliliters
Exercise 7.310
106 tablets to 52 tablets
For problems 13-16, write each ratio in the form "a to b".
Exercise 7.311 (Solution on p. 471.)
9
16
Exercise 7.312
5
11

9 This content is available online at <http://cnx.org/content/m35009/1.2/>.


459

Exercise 7.313 (Solution on p. 471.)


1 diskette
8 diskettes
Exercise 7.314
5 papers
3 pens
For problems 17-21, write each ratio or rate using words.
Exercise 7.315 (Solution on p. 471.)
9 18
16 = 32
Exercise 7.316
1 12
4 = 48
Exercise 7.317 (Solution on p. 471.)
8 items 2 items
4 dollars = 1 dollar
Exercise 7.318
150 milligrams of niacin is to 2 tablets as 300 milligrams of niacin is to 4 tablets.
Exercise 7.319 (Solution on p. 471.)
20 people is to 4 seats as 5 people is to 1 seat.

7.9.1.2 Proportions (Section 7.3)

For problems 22-27, determine the missing number in each proportion.


Exercise 7.320
x 24
3 = 9
Exercise 7.321 (Solution on p. 471.)
15 60
7 = x
Exercise 7.322
1 x
1 = 44
Exercise 7.323 (Solution on p. 471.)
3 15
x = 50
Exercise 7.324
15 bats x bats
16 balls = 128 balls
Exercise 7.325 (Solution on p. 471.)
36 rooms 504 rooms
29 fans = x fans

7.9.1.3 Applications of Proportions (Section 7.4)

Exercise 7.326
On a map, 3 inches represents 20 miles. How many miles does 27 inches represent?
Exercise 7.327 (Solution on p. 471.)
A salt solution is composed of 8 parts of salt to 5 parts of water. How many parts of salt are there
in a solution that contains 50 parts of water?
Exercise 7.328
A model is built to 15
4
scale. If a particular part of the model measures 8 inches in length, how long
is the actual structure?
Exercise 7.329 (Solution on p. 471.)
The ratio of ammonia to air in a container is 3
40 How many milliliters of air should be in a container
that contains 8 milliliters of ammonia?
460 CHAPTER 7. RATIOS AND RATES

Exercise 7.330
A 4-foot girl casts a 9-foot shadow at a particular time of the day. How tall is a pole that casts a
144-foot shadow at the same time of the day?
Exercise 7.331 (Solution on p. 471.)
The odds that a particular event will occur are 11 to 2. If this event occurs 55 times, how many
times would you predict it does not occur?
Exercise 7.332
Every 1 43 teaspoon of a multiple vitamin, in granular form, contains 0.85 the minimum daily
requirement of vitamin A. How many teaspoons of this vitamin are required to supply 2.25 the
minimum daily requirement?

7.9.1.4 Percent and Fractions of One Percent (Section 7.5,Section 7.6)

For problems 35-39, convert each decimal to a percent.


Exercise 7.333 (Solution on p. 471.)
0.16
Exercise 7.334
0.818
Exercise 7.335 (Solution on p. 471.)
5.3536
Exercise 7.336
0.50
Exercise 7.337 (Solution on p. 472.)
3
For problems 40-48, convert each percent to a decimal.
Exercise 7.338
62%
Exercise 7.339 (Solution on p. 472.)
1.58%
Exercise 7.340
9.15%
Exercise 7.341 (Solution on p. 472.)
0.06%
Exercise 7.342
0.003%
Exercise 7.343 (Solution on p. 472.)
3
5 11 % to a three-place decimal
Exercise 7.344
9
13 % to a three-place decimal
Exercise 7.345 (Solution on p. 472.)
82 25
29 % to a four-place decimal
Exercise 7.346
18 71 % to a four-place decimal
For problems 49-55, convert each fraction or mixed number to a percent.
Exercise 7.347 (Solution on p. 472.)
3
5
461

Exercise 7.348
2
10
Exercise 7.349 (Solution on p. 472.)
5
16
Exercise 7.350
35
8
Exercise 7.351 (Solution on p. 472.)
105
16
Exercise 7.352
45 1
11
Exercise 7.353 (Solution on p. 472.)
6 278
9
For problems 56-64, convert each percent to a fraction or mixed number.
Exercise 7.354
95%
Exercise 7.355 (Solution on p. 472.)
12%
Exercise 7.356
83%
Exercise 7.357 (Solution on p. 472.)
38.125%
Exercise
_ 7.358
61. 2 %
Exercise 7.359 (Solution on p. 472.)
5
8%
Exercise 7.360
9
6 20 %
Exercise 7.361 (Solution on p. 472.)
3
15 22 %
Exercise 7.362
106 19
45 %

7.9.1.5 Applications of Percents (Section 7.7)

For problems 65-72, nd each solution.


Exercise 7.363 (Solution on p. 472.)
What is 16% of 40?
Exercise 7.364
29.4 is what percent of 105?
Exercise 7.365 (Solution on p. 472.)
21
3 50 is 547.2% of what number?
Exercise 7.366
0.09378 is what percent of 52.1?
Exercise 7.367 (Solution on p. 472.)
What is 680% of 1.41?
462 CHAPTER 7. RATIOS AND RATES

Exercise 7.368
A kitchen knife is on sale for 15% o the marked price. If the marked price is $ 39.50, what is the
sale price?
Exercise 7.369 (Solution on p. 472.)
On an 80 question geology exam, a student gets 68 correct. What percent is correct?
Exercise 7.370
A salesperson makes a commission of 18% of her monthly sales total. She also receives a monthly
salary of $1,600.00. If, in a particular month, she sells $4,000.00 worth of merchandise, how much
will she make that month?

7.10 Prociency Exam10


7.10.1 Prociency Exam
Exercise 7.371 (Solution on p. 472.)
(Section 7.2) Compare 4 cassette tapes to 7 dollars.
Exercise 7.372 (Solution on p. 472.)
(Section 7.2) What do we call a comparison, by division, of two unlike denominate numbers?
For problems 3 and 4, express each ratio or rate as a fraction.
Exercise 7.373 (Solution on p. 472.)
(Section 7.2) 11 to 9
Exercise 7.374 (Solution on p. 472.)
(Section 7.2) 5 televisions to 2 radios
For problems 5 and 6, write each ratio or rate in the form "a to b."
Exercise 7.375 (Solution on p. 472.)
(Section 7.2) 38 people
maps

Exercise 7.376 (Solution on p. 472.)


(Section 7.2) 2 psychologists
75 people
For problems 7-9, solve each proportion.
Exercise 7.377 (Solution on p. 472.)
(Section 7.3) 8
x = 48
90
Exercise 7.378 (Solution on p. 472.)
(Section 7.3) x
7
4
= 28
Exercise 7.379 (Solution on p. 473.)
(Section 7.3) 38computers 24 computers
students = x students
Exercise 7.380 (Solution on p. 473.)
(Section 7.4) On a map, 4 inches represents 50 miles. How many miles does 3 inches represent?
Exercise 7.381 (Solution on p. 473.)
(Section 7.4) An acid solution is composed of 6 milliliters of acid to 10 milliliters of water. How
many milliliters of acid are there in an acid solution that is composed of 3 milliliters of water?
Exercise 7.382 (Solution on p. 473.)
(Section 7.4) The odds that a particular event will occur are 9 to 7. If the event occurs 27 times,
how many times would you predict it will it not occur?
For problems 13 and 14, convert each decimal to a percent.
10 This content is available online at <http://cnx.org/content/m35010/1.2/>.
463

Exercise 7.383 (Solution on p. 473.)


(Section 7.5) 0.82
Exercise 7.384 (Solution on p. 473.)
_
(Section 7.5) 3. 7
For problems 15 and 16, convert each percent to a decimal.
Exercise 7.385 (Solution on p. 473.)
(Section 7.5) 2.813%
Exercise 7.386 (Solution on p. 473.)
(Section 7.5) 0.006%
For problems 17-19, convert each fraction to a percent.
Exercise 7.387 (Solution on p. 473.)
(Section 7.5) 42
5
Exercise 7.388 (Solution on p. 473.)
(Section 7.5) 1
8
Exercise 7.389 (Solution on p. 473.)
(Section 7.5) 800
80
For problems 20 and 21, convert each percent to a fraction.
Exercise 7.390 (Solution on p. 473.)
(Section 7.5) 15%
Exercise 7.391 (Solution on p. 473.)
(Section 7.5) 27
4
%
For problems 22-25, nd each indicated quantity.
Exercise 7.392 (Solution on p. 473.)
(Section 7.7) What is 18% of 26?
Exercise 7.393 (Solution on p. 473.)
(Section 7.7) 0.618 is what percent of 0.3?
Exercise 7.394 (Solution on p. 473.)
(Section 7.7) 0.1 is 1.1% of what number?
Exercise 7.395 (Solution on p. 473.)
(Section 7.7) A salesperson makes a monthly salary of $1,000.00. He also gets a commission of
12% of his total monthly sales. If, in a particular month, he sells $5,500.00 worth of merchandise,
what is his income that month?
464 CHAPTER 7. RATIOS AND RATES

Solutions to Exercises in Chapter 7


Solution to Exercise 7.1 (p. 419)

(a) 8 diskettes; 10 diskettes is 8 diskettes more than 2 diskettes.


(b) 5; 10 diskettes is 5 times as many diskettes as 2 diskettes.

Solution to Exercise 7.2 (p. 419)

(a) Comparison by subtraction makes no sense.


(b) 16 2bananas
bags =
8 bananas , 8 bananas per bag.
bag
Solution to Exercise 7.3 (p. 420)
3
2
Solution to Exercise 7.4 (p. 420)
1
9
Solution to Exercise 7.5 (p. 420)
5 books
4 people
Solution to Exercise 7.6 (p. 420)
60 miles
1 hour
Solution to Exercise 7.7 (p. 420)
8
3
Solution to Exercise 7.8 (p. 420)
9 to 5
Solution to Exercise 7.9 (p. 420)
1 to 3
Solution to Exercise 7.10 (p. 420)
25 miles to 2 gallons
Solution to Exercise 7.11 (p. 421)
1 mechanic to 2 wrenches
Solution to Exercise 7.12 (p. 421)
5 to 6
Solution to Exercise 7.13 (p. 421)
They are pure numbers or like denominate numbers.
Solution to Exercise 7.15 (p. 421)
rate
Solution to Exercise 7.17 (p. 421)
ratio
Solution to Exercise 7.19 (p. 421)
rate
Solution to Exercise 7.21 (p. 421)
ratio
Solution to Exercise 7.23 (p. 421)
two to ve
Solution to Exercise 7.25 (p. 421)
29 mile per 2 gallons or 14 12 miles per 1 gallon
Solution to Exercise 7.27 (p. 421)
5 to 2
Solution to Exercise 7.29 (p. 421)
12
5
Solution to Exercise 7.31 (p. 422)
42 plants
5 homes
465

Solution to Exercise 7.33 (p. 422)


16 pints
1 quart
Solution to Exercise 7.35 (p. 422)
2.54 cm
1 inch
Solution to Exercise 7.37 (p. 422)
5
2
Solution to Exercise 7.39 (p. 422)
1 hit
3 at bats
Solution to Exercise 7.41 (p. 422)
1,042 characters
1 page
Solution to Exercise 7.43 (p. 422)
5 13
Solution to Exercise 7.45 (p. 422)
299
1260
Solution to Exercise 7.47 (p. 422)
3.3875
Solution to Exercise 7.48 (p. 424)
3 is to 8 as 6 is to 16
Solution to Exercise 7.49 (p. 424)
2 people are to 1 window as 10 people are to 5 windows
Solution to Exercise 7.50 (p. 424)
15 = 75
4 20
Solution to Exercise 7.51 (p. 424)
2 plates = 20 plates
1 tray 10 trays
Solution to Exercise 7.52 (p. 425)
x=3
Solution to Exercise 7.53 (p. 426)
x=5
Solution to Exercise 7.54 (p. 426)
x = 45
Solution to Exercise 7.55 (p. 426)
x = 48
Solution to Exercise 7.56 (p. 427)
rates, proportion
Solution to Exercise 7.58 (p. 427)
1 3
11 = 33
Solution to Exercise 7.60 (p. 427)
6 3
90 = 45
Solution to Exercise 7.62 (p. 427)
18 gr cobalt = 36 gr cobalt
10 gr silver 20 gr silver
Solution to Exercise 7.64 (p. 427)
3 people absent 15 people absent
31 people present = 155 people present
Solution to Exercise 7.66 (p. 427)
3 is to 4 as 15 is to 20
Solution to Exercise 7.68 (p. 427)
3 joggers are to 100 feet as 6 joggers are to 200 feet
Solution to Exercise 7.70 (p. 428)
40 miles are to 80 miles as 2 gallons are to 4 gallons
Solution to Exercise 7.72 (p. 428)
1 person is to 1 job as 8 people are to 8 jobs
466 CHAPTER 7. RATIOS AND RATES

Solution to Exercise 7.74 (p. 428)


2,000 pounds are to 1 ton as 60,000 pounds are to 30 tons
Solution to Exercise 7.76 (p. 428)
x=2
Solution to Exercise 7.78 (p. 428)
x=8
Solution to Exercise 7.80 (p. 428)
x=8
Solution to Exercise 7.82 (p. 428)
x = 27
Solution to Exercise 7.84 (p. 428)
x = 40
Solution to Exercise 7.86 (p. 428)
x = 30
Solution to Exercise 7.88 (p. 428)
x = 35
Solution to Exercise 7.90 (p. 428)
x = 30
Solution to Exercise 7.92 (p. 429)
true
Solution to Exercise 7.94 (p. 429)
false
Solution to Exercise 7.96 (p. 429)
true
Solution to Exercise 7.98 (p. 429)
true
Solution to Exercise 7.100 (p. 429)
5+7=12
7+5=12
Solution to Exercise 7.102 (p. 429)
17
77
Solution to Exercise 7.104 (p. 429)
8 sentences
1 paragraph
Solution to Exercise 7.105 (p. 432)
500 miles
Solution to Exercise 7.106 (p. 433)
24 parts of alcohol
Solution to Exercise 7.107 (p. 433)
9
3 11 feet
Solution to Exercise 7.108 (p. 433)
864 outside plants
Solution to Exercise 7.109 (p. 433)
The event occurs 99 times.
Solution to Exercise 7.110 (p. 433)
6 failing grades
Solution to Exercise 7.111 (p. 434)
24
Solution to Exercise 7.113 (p. 434)
45 inches
467

Solution to Exercise 7.115 (p. 434)


33 parts
Solution to Exercise 7.117 (p. 434)
2328.75
Solution to Exercise 7.119 (p. 434)
21 feet
22
Solution to Exercise 7.121 (p. 434)
12,250
Solution to Exercise 7.123 (p. 434)
12
Solution to Exercise 7.125 (p. 434)
1 56
Solution to Exercise 7.127 (p. 435)
1 78
Solution to Exercise 7.129 (p. 435)
31 41
Solution to Exercise 7.131 (p. 435)
0
Solution to Exercise 7.133 (p. 435)
2
3
Solution to Exercise 7.135 (p. 435)
5
Solution to Exercise 7.136 (p. 436)
16%
Solution to Exercise 7.137 (p. 436)
195%
Solution to Exercise 7.138 (p. 436)
83
100
Solution to Exercise 7.139 (p. 436)
362 or 181
100 50
Solution to Exercise 7.140 (p. 439)
0.21
Solution to Exercise 7.141 (p. 439)
4.61
Solution to Exercise 7.142 (p. 439)
55%
Solution to Exercise 7.143 (p. 439)
564%
Solution to Exercise 7.144 (p. 439)
15%
Solution to Exercise 7.145 (p. 439)
137.5%
Solution to Exercise 7.146 (p. 439)
27.27%
Solution to Exercise 7.147 (p. 439)
25%
Solution to Exercise 7.149 (p. 439)
48%
Solution to Exercise 7.151 (p. 439)
77.1%
468 CHAPTER 7. RATIOS AND RATES

Solution to Exercise 7.153 (p. 439)


258%
Solution to Exercise 7.155 (p. 439)
1,618.14%
Solution to Exercise 7.157 (p. 439)
200%
Solution to Exercise 7.159 (p. 440)
0.15
Solution to Exercise 7.161 (p. 440)
0.162
Solution to Exercise 7.163 (p. 440)
0.0505
Solution to Exercise 7.165 (p. 440)
0.0078
Solution to Exercise 7.167 (p. 440)
0.0009
Solution to Exercise 7.169 (p. 440)
20%
Solution to Exercise 7.171 (p. 440)
62.5%
Solution to Exercise 7.173 (p. 440)
28%
Solution to Exercise 7.175 (p. 440)
49.09%
Solution to Exercise 7.177 (p. 440)
164%
Solution to Exercise 7.179 (p. 440)
945%
Solution to Exercise 7.181 (p. 441)
54.54%
Solution to Exercise 7.183 (p. 441)
4
5
Solution to Exercise 7.185 (p. 441)
1
4
Solution to Exercise 7.187 (p. 441)
13
20
Solution to Exercise 7.189 (p. 441)
1
8
Solution to Exercise 7.191 (p. 441)
41 or 5 1
8 8
Solution to Exercise 7.193 (p. 441)
1
10
Solution to Exercise 7.195 (p. 441)
2
9
Solution to Exercise 7.197 (p. 441)
4
45
Solution to Exercise 7.199 (p. 441)
129 or 2 9 = 2 3
60 60 20
Solution to Exercise 7.201 (p. 442)
36 inches
469

Solution to Exercise 7.202 (p. 443)


1
400
Solution to Exercise 7.203 (p. 443)
3
800
Solution to Exercise 7.204 (p. 443)
1
30
Solution to Exercise 7.205 (p. 444)
0.008
Solution to Exercise 7.206 (p. 444)
0.5145
Solution to Exercise 7.207 (p. 444)
3
400
Solution to Exercise 7.209 (p. 444)
1
900
Solution to Exercise 7.211 (p. 444)
400 or 80
5 1

Solution to Exercise 7.213 (p. 444)


13
700
Solution to Exercise 7.215 (p. 444)
101
400
Solution to Exercise 7.217 (p. 444)
363
500
Solution to Exercise 7.219 (p. 444)
41
30
Solution to Exercise 7.221 (p. 445)
51
500 = 0.102
Solution to Exercise 7.223 (p. 445)
31
800 = 0.03875
Solution to Exercise 7.225 (p. 445)
0.004
Solution to Exercise 7.227 (p. 445)
0.0627
Solution to Exercise 7.229 (p. 445)
0.242
Solution to Exercise 7.231 (p. 445)
0.1194
Solution to Exercise 7.233 (p. 445)
85
Solution to Exercise 7.235 (p. 445)
197
210
Solution to Exercise 7.237 (p. 445)
806.2%
Solution to Exercise 7.238 (p. 449)
35.7
Solution to Exercise 7.239 (p. 449)
$1,360
Solution to Exercise 7.240 (p. 449)
4.9
Solution to Exercise 7.241 (p. 449)
$45,360
470 CHAPTER 7. RATIOS AND RATES

Solution to Exercise 7.242 (p. 451)


23%
Solution to Exercise 7.243 (p. 451)
90%
Solution to Exercise 7.244 (p. 451)
40%
Solution to Exercise 7.245 (p. 453)
99
Solution to Exercise 7.246 (p. 453)
13.2ml
Solution to Exercise 7.247 (p. 453)
$835
Solution to Exercise 7.248 (p. 453)
8 hours
Solution to Exercise 7.249 (p. 453)
21.84
Solution to Exercise 7.251 (p. 454)
534.1
Solution to Exercise 7.253 (p. 454)
2.1
Solution to Exercise 7.255 (p. 454)
51
Solution to Exercise 7.257 (p. 454)
120
Solution to Exercise 7.259 (p. 454)
20
Solution to Exercise 7.261 (p. 454)
59
Solution to Exercise 7.263 (p. 454)
91
Solution to Exercise 7.265 (p. 454)
9.15
Solution to Exercise 7.267 (p. 454)
568
Solution to Exercise 7.269 (p. 454)
1.19351
Solution to Exercise 7.271 (p. 454)
250
Solution to Exercise 7.273 (p. 455)
0
Solution to Exercise 7.275 (p. 455)
390.55
Solution to Exercise 7.277 (p. 455)
84
Solution to Exercise 7.279 (p. 455)
14
Solution to Exercise 7.281 (p. 455)
160
Solution to Exercise 7.283 (p. 455)
$1,022.40
471

Solution to Exercise 7.285 (p. 455)


36.28%
Solution to Exercise 7.287 (p. 455)
61
Solution to Exercise 7.289 (p. 456)
19,500
Solution to Exercise 7.291 (p. 456)
14.85
Solution to Exercise 7.293 (p. 456)
Marked correctly
Solution to Exercise 7.295 (p. 456)
2 45
Solution to Exercise 7.297 (p. 456)
3
4 500
Solution to Exercise 7.299 (p. 458)
250 watts are 150 watts more than 100 watts
Solution to Exercise 7.301 (p. 458)
98 radishes are 2.39 times as many radishes as 41 radishes
Solution to Exercise 7.303 (p. 458)
100 tents are 20 times as many tents as 5 tents
Solution to Exercise 7.305 (p. 458)
ratio
Solution to Exercise 7.307 (p. 458)
3
1
Solution to Exercise 7.309 (p. 458)
8ml
5ml
Solution to Exercise 7.311 (p. 458)
9 to 16
Solution to Exercise 7.313 (p. 458)
1 diskette to 8 diskettes
Solution to Exercise 7.315 (p. 459)
9 is to 16 as 18 is to 32
Solution to Exercise 7.317 (p. 459)
8 items are to 4 dollars as 2 items are to 1 dollar
Solution to Exercise 7.319 (p. 459)
20
4 = 51
20 people are to 4 seats as 5 people are to 1 seat
Solution to Exercise 7.321 (p. 459)
28
Solution to Exercise 7.323 (p. 459)
10
Solution to Exercise 7.325 (p. 459)
406
Solution to Exercise 7.327 (p. 459)
80
Solution to Exercise 7.329 (p. 459)
320
3 or 106 23
Solution to Exercise 7.331 (p. 460)
10
Solution to Exercise 7.333 (p. 460)
16%
472 CHAPTER 7. RATIOS AND RATES

Solution to Exercise 7.335 (p. 460)


535.36%
Solution to Exercise 7.337 (p. 460)
300%
Solution to Exercise 7.339 (p. 460)
0.0158
Solution to Exercise 7.341 (p. 460)
0.0006
Solution to Exercise 7.343 (p. 460)
0.053
Solution to Exercise 7.345 (p. 460)
0.8286
Solution to Exercise 7.347 (p. 460)
60%
Solution to Exercise 7.349 (p. 461)
31.25%
Solution to Exercise 7.351 (p. 461)
656.25%
Solution to Exercise 7.353 (p. 461)
_
3688. 8 %
Solution to Exercise 7.355 (p. 461)
3
25
Solution to Exercise 7.357 (p. 461)
61
160
Solution to Exercise 7.359 (p. 461)
1
160
Solution to Exercise 7.361 (p. 461)
2977
19800
Solution to Exercise 7.363 (p. 461)
6.4
Solution to Exercise 7.365 (p. 461)
0.625 or 5
8
Solution to Exercise 7.367 (p. 461)
9.588
Solution to Exercise 7.369 (p. 462)
85
Solution to Exercise 7.371 (p. 462)
4 cassette tapes
7 dollars
Solution to Exercise 7.372 (p. 462)
Rate
Solution to Exercise 7.373 (p. 462)
11
9
Solution to Exercise 7.374 (p. 462)
5 televisions
2 radios
Solution to Exercise 7.375 (p. 462)
8 maps to 3 people
Solution to Exercise 7.376 (p. 462)
two psychologists to seventy-ve people
Solution to Exercise 7.377 (p. 462)
15
473

Solution to Exercise 7.378 (p. 462)


1
Solution to Exercise 7.379 (p. 462)
64
Solution to Exercise 7.380 (p. 462)
37 12
Solution to Exercise 7.381 (p. 462)
1.8
Solution to Exercise 7.382 (p. 462)
21
Solution to Exercise 7.383 (p. 462)
82%
Solution to Exercise 7.384 (p. 463)
377 79 %
Solution to Exercise 7.385 (p. 463)
0.02813
Solution to Exercise 7.386 (p. 463)
0.00006
Solution to Exercise 7.387 (p. 463)
840%
Solution to Exercise 7.388 (p. 463)
12.5%
Solution to Exercise 7.389 (p. 463)
1,000%
Solution to Exercise 7.390 (p. 463)
3
20
Solution to Exercise 7.391 (p. 463)
4
2,700 or 675
1

Solution to Exercise 7.392 (p. 463)


4.68
Solution to Exercise 7.393 (p. 463)
206
Solution to Exercise 7.394 (p. 463)
9.09
Solution to Exercise 7.395 (p. 463)
$1,660
474 CHAPTER 7. RATIOS AND RATES
Chapter 8

Techniques of Estimation

8.1 Objectives1
After completing this chapter, you should
Estimation by Rounding (Section 8.2)

• understand the reason for estimation


• be able to estimate the result of an addition, multiplication, subtraction, or division using the rounding
technique

Estimation by Clustering (Section 8.3)

• understand the concept of clustering


• be able to estimate the result of adding more than two numbers when clustering occurs using the
clustering technique

Mental ArithmeticUsing the Distributive Property (Section 8.4)

• understand the distributive property


• be able to obtain the exact result of a multiplication using the distributive property

Estimation by Rounding Fractions (Section 8.5)

• be able to estimate the sum of two or more fractions using the technique of rounding fractions

8.2 Estimation by Rounding2


8.2.1 Section Overview
• Estimation By Rounding

When beginning a computation, it is valuable to have an idea of what value to expect for the result. When
a computation is completed, it is valuable to know if the result is reasonable.

In the rounding process, it is important to note two facts:


1 This content is available online at <http://cnx.org/content/m18896/1.3/>.
2 This content is available online at <http://cnx.org/content/m35011/1.2/>.

475
476 CHAPTER 8. TECHNIQUES OF ESTIMATION

1. The rounding that is done in estimation does not always follow the rules of rounding discussed in
Section 1.4 (Rounding Whole Numbers). Since estimation is concerned with the expected value of a
computation, rounding is done using convenience as the guide rather than using hard-and-fast rounding
rules. For example, if we wish to estimate the result of the division 80 ÷ 26, we might round 26 to 20
rather than to 30 since 80 is more conveniently divided by 20 than by 30.
2. Since rounding may occur out of convenience, and dierent people have dierent ideas of what may
be convenient, results of an estimation done by rounding may vary. For a particular computation,
dierent people may get dierent estimated results. Results may vary.

Estimation
Estimation is the process of determining an expected value of a computation.

Common words used in estimation are about, near, and between.

8.2.2 Estimation by Rounding


The rounding technique estimates the result of a computation by rounding the numbers involved in the
computation to one or two nonzero digits.

8.2.2.1 Sample Set A

Example 8.1
Estimate the sum: 2,357 + 6,106.

Notice that 2,357 is near 2, 400, and that 6,106 is near 6, 100.
| {z } | {z }
two nonzero two nonzero
digits digits

The sum can be estimated by 2,400 + 6,100 = 8,500. (It is quick and easy to add 24 and 61.)

Thus, 2,357 + 6,106 is about 8,400. In fact, 2,357 + 6,106 = 8,463.

8.2.2.2 Practice Set A

Exercise 8.1 (Solution on p. 502.)


Estimate the sum: 4,216 + 3,942.
Exercise 8.2 (Solution on p. 502.)
Estimate the sum: 812 + 514.
Exercise 8.3 (Solution on p. 502.)
Estimate the sum: 43,892 + 92,106.

8.2.2.3 Sample Set B

Example 8.2
Estimate the dierence: 5, 203 − 3, 015.

Notice that 5,203 is near 5, 200, and that 3,015 is near 3, 000.
| {z } | {z }
two nonzero one nonzero
digits digit

The dierence can be estimated by 5, 200 − 3, 000 = 2,200.


477

Thus, 5, 203 − 3, 015 is about 2,200. In fact, 5, 203 − 3, 015 = 2,188.

We could make a less accurate estimation by observing that 5,203 is near 5,000. The number 5,000
has only one nonzero digit rather than two (as does 5,200). This fact makes the estimation quicker
(but a little less accurate). We then estimate the dierence by 5, 000 − 3, 000 = 2,000, and conclude
that 5, 203 − 3, 015 is about 2,000. This is why we say "answers may vary."

8.2.2.4 Practice Set B

Exercise 8.4 (Solution on p. 502.)


Estimate the dierence: 628 − 413.
Exercise 8.5 (Solution on p. 502.)
Estimate the dierence: 7, 842 − 5, 209.
Exercise 8.6 (Solution on p. 502.)
Estimate the dierence: 73, 812 − 28, 492.

8.2.2.5 Sample Set C

Example 8.3
Estimate the product: 73 · 46.

Notice that 73 is near 70, and that 46 is near 50.


|{z}
one nonzero
|{z}
one nonzero
digit digit

The product can be estimated by 70 · 50 = 3,500. (Recall that to multiply numbers ending in
zeros, we multiply the nonzero digits and ax to this product the total number of ending zeros in
the factors. See Section 2.2 for a review of this technique.)

Thus, 73 · 46 is about 3,500. In fact, 73 · 46 = 3,358.


Example 8.4
Estimate the product: 87 · 4,316.

Notice that 87 is close to 90, and that 4,316 is close to 4, 000.


|{z} | {z }
one nonzero one nonzero
digit digit

The product can be estimated by 90 · 4,000 = 360,000.

Thus, 87 · 4,316 is about 360,000. In fact, 87 · 4,316 = 375,492.

8.2.2.6 Practice Set C

Exercise 8.7 (Solution on p. 502.)


Estimate the product: 31 · 87.
Exercise 8.8 (Solution on p. 502.)
Estimate the product: 18 · 42.
Exercise 8.9 (Solution on p. 502.)
Estimate the product: 16 · 94.
478 CHAPTER 8. TECHNIQUES OF ESTIMATION

8.2.2.7 Sample Set D

Example 8.5
Estimate the quotient: 153 ÷ 17.

Notice that 153 is close to 150, and that 17 is close to 15.


|{z}
two nonzero
|{z}
two nonzero
digits digits

The quotient can be estimated by 150 ÷ 15 = 10.

Thus, 153 ÷ 17 is about 10. In fact, 153 ÷ 17 = 9.


Example 8.6
Estimate the quotient: 742,000 ÷ 2,400.

Notice that 742,000 is close to 700, 000 , and that 2,400 is close to 2, 000.
| {z } | {z }
one nonzero one nonzero
digit digit

The quotient can be estimated by 700,000 ÷ 2,000 = 350.

Thus, 742,000 ÷ 2,400 is about 350. In fact, 742,000 ÷ 2,400 = 309.16.

8.2.2.8 Practice Set D

Exercise 8.10 (Solution on p. 502.)


Estimate the quotient: 221 ÷ 18.
Exercise 8.11 (Solution on p. 502.)
Estimate the quotient: 4,079 ÷ 381.
Exercise 8.12 (Solution on p. 502.)
Estimate the quotient: 609,000 ÷ 16,000.

8.2.2.9 Sample Set E

Example 8.7
Estimate the sum: 53.82 + 41.6.

Notice that 53.82 is close to 54, and that 41.6 is close to 42.
|{z}
two nonzero
|{z}
two nonzero
digits digits

The sum can be estimated by 54 + 42 = 96.

Thus, 53.82 + 41.6 is about 96. In fact, 53.82 + 41.6 = 95.42.


479

8.2.2.10 Practice Set E

Exercise 8.13 (Solution on p. 502.)


Estimate the sum: 61.02 + 26.8.
Exercise 8.14 (Solution on p. 502.)
Estimate the sum: 109.12 + 137.88.

8.2.2.11 Sample Set F

Example 8.8
Estimate the product: (31.28) (14.2).

Notice that 31.28 is close to 30, and that 14.2 is close to 15.
|{z}
two nonzero
|{z}
one nonzero
digit digits

The product can be estimated by 30 · 15 = 450. ( 3 · 15 = 45, then ax one zero.)

Thus, (31.28) (14.2) is about 450. In fact, (31.28) (14.2) = 444.176.


Example 8.9
Estimate 21% of 5.42.

Notice that 21% = .21 as a decimal, and that .21 is close to .2.
|{z}
one nonzero
digit

Notice also that 5.42 is close to 5.


|{z}
one nonzero
digit

Then, 21% of 5.42 can be estimated by (.2) (5) = 1.

Thus, 21% of 5.42 is about 1. In fact, 21% of 5.42 is 1.1382.

8.2.2.12 Practice Set F

Exercise 8.15 (Solution on p. 502.)


Estimate the product: (47.8) (21.1).
Exercise 8.16 (Solution on p. 502.)
Estimate 32% of 14.88.

8.2.3 Exercises
Estimate each calculation using the method of rounding. After you have made an estimate, nd the exact
value and compare this to the estimated result to see if your estimated value is reasonable. Results may
vary.
Exercise 8.17 (Solution on p. 502.)
1, 402 + 2, 198
Exercise 8.18
3,481 + 4,216
480 CHAPTER 8. TECHNIQUES OF ESTIMATION

Exercise 8.19 (Solution on p. 502.)


921 + 796
Exercise 8.20
611 + 806
Exercise 8.21 (Solution on p. 502.)
4, 681 + 9, 325
Exercise 8.22
6, 476 + 7, 814
Exercise 8.23 (Solution on p. 502.)
7, 805 − 4, 266
Exercise 8.24
8, 427 − 5, 342
Exercise 8.25 (Solution on p. 502.)
14, 106 − 8, 412
Exercise 8.26
26, 486 − 18, 931
Exercise 8.27 (Solution on p. 502.)
32 · 53
Exercise 8.28
67 · 42
Exercise 8.29 (Solution on p. 502.)
628 · 891
Exercise 8.30
426 · 741
Exercise 8.31 (Solution on p. 502.)
18, 012 · 32, 416
Exercise 8.32
22, 481 · 51, 076
Exercise 8.33 (Solution on p. 502.)
287 ÷ 19
Exercise 8.34
884 ÷ 33
Exercise 8.35 (Solution on p. 503.)
1, 254 ÷ 57
Exercise 8.36
2, 189 ÷ 42
Exercise 8.37 (Solution on p. 503.)
8, 092 ÷ 239
Exercise 8.38
2, 688 ÷ 48
Exercise 8.39 (Solution on p. 503.)
72.14 + 21.08
Exercise 8.40
43.016 + 47.58
Exercise 8.41 (Solution on p. 503.)
96.53 − 26.91
481

Exercise 8.42
115.0012 − 25.018
Exercise 8.43 (Solution on p. 503.)
206.19 + 142.38
Exercise 8.44
592.131 + 211.6
Exercise 8.45 (Solution on p. 503.)
(32.12) (48.7)
Exercise 8.46
(87.013) (21.07)
Exercise 8.47 (Solution on p. 503.)
(3.003) (16.52)
Exercise 8.48
(6.032) (14.091)
Exercise 8.49 (Solution on p. 503.)
(114.06) (384.3)
Exercise 8.50
(5, 137.118) (263.56)
Exercise 8.51 (Solution on p. 503.)
(6.92) (0.88)
Exercise 8.52
(83.04) (1.03)
Exercise 8.53 (Solution on p. 503.)
(17.31) (.003)
Exercise 8.54
(14.016) (.016)
Exercise 8.55 (Solution on p. 503.)
93% of 7.01
Exercise 8.56
107% of 12.6
Exercise 8.57 (Solution on p. 503.)
32% of 15.3
Exercise 8.58
74% of 21.93
Exercise 8.59 (Solution on p. 503.)
18% of 4.118
Exercise 8.60
4% of .863
Exercise 8.61 (Solution on p. 503.)
2% of .0039
482 CHAPTER 8. TECHNIQUES OF ESTIMATION

8.2.3.1 Exercises for Review

Exercise 8.62
(Section 5.3) Find the dierence: 10
7 5
− 16 .
Exercise 8.63 (Solution on p. 503.)
6− 14
(Section 5.6) Find the value 6+ 14
.
Exercise 8.64
(Section 6.3) Convert the complex decimal 1.11 14 to a decimal.
Exercise 8.65 (Solution on p. 503.)
(Section 7.4) A woman 5 foot tall casts an 8-foot shadow at a particular time of the day. How tall
is a tree that casts a 96-foot shadow at the same time of the day?
Exercise 8.66
(Section 7.7) 11.62 is 83% of what number?

8.3 Estimation by Clustering3


8.3.1 Section Overview
• Estimation by Clustering

Cluster
When more than two numbers are to be added, the sum may be estimated using the clustering technique.
The rounding technique could also be used, but if several of the numbers are seen to cluster (are seen to be
close to) one particular number, the clustering technique provides a quicker estimate. Consider a sum such
as

32 + 68 + 29 + 73

Notice two things:

1. There are more than two numbers to be added.


2. Clustering occurs.
(a) Both 68 and 73 cluster around 70, so 68 + 73 is close to 80 + 70 = 2 (70) = 140.

(b) Both 32 and 29 cluster around 30, so 32 + 29 is close to 30 + 30 = 2 (30) = 60.

The sum may be estimated by

(2 · 30) + (2 · 70) = 6 + 140


= 200

In fact, 32 + 68 + 29 + 73 = 202.
3 This content is available online at <http://cnx.org/content/m35012/1.2/>.
483

8.3.1.1 Sample Set A

Estimate each sum. Results may vary.


Example 8.10
27 + 48 + 31 + 52.

27 and 31 cluster near 30. Their sum is about 2 · 30 = 60.

48 and 52 cluster near 50. Their sum is about 2 · 50 = 100.

(2 · 30) + (2 · 50) = 60 + 100


Thus, 27 + 48 + 31 + 52 is about
= 160

In fact, 27 + 48 + 31 + 52 = 158.
Example 8.11
88 + 21 + 19 + 91.

88 and 91 cluster near 90. Their sum is about 2 · 90 = 180.

21 and 19 cluster near 20. Their sum is about 2 · 20 = 40.

(2 · 90) + (2 · 20) = 180 + 40


Thus, 88 + 21 + 19 + 91 is about
= 220

In fact, 88 + 21 + 19 + 91 = 219.
Example 8.12
17 + 21 + 48 + 18.

17, 21, and 18 cluster near 20. Their sum is about 3 · 20 = 60.

48 is about 50.

(3 · 20) + 50 = 60 + 50
Thus, 17 + 21 + 48 + 18 is about
= 110

In fact, 17 + 21 + 48 + 18 = 104.
Example 8.13
61 + 48 + 49 + 57 + 52.

61 and 57 cluster near 60. Their sum is about 2 · 60 = 120.

48, 49, and 52 cluster near 50. Their sum is about 3 · 50 = 150.

(2 · 60) + (3 · 50) = 120 + 150


Thus, 61 + 48 + 49 + 57 + 52 is about
= 270

In fact, 61 + 48 + 49 + 57 + 52 = 267.
Example 8.14
706 + 321 + 293 + 684.

706 and 684 cluster near 700. Their sum is about 2 · 700 = 1,400.
484 CHAPTER 8. TECHNIQUES OF ESTIMATION

321 and 293 cluster near 300. Their sum is about 2 · 300 = 600.

(2 · 700) + (2 · 300) = 1, 400 + 600


Thus, 706 + 321 + 293 + 684 is about
= 2, 000

In fact, 706 + 321 + 293 + 684 = 2,004.

8.3.1.2 Practice Set A

Use the clustering method to estimate each sum.


Exercise 8.67 (Solution on p. 503.)
28 + 51 + 31 + 47
Exercise 8.68 (Solution on p. 503.)
42 + 39 + 68 + 41
Exercise 8.69 (Solution on p. 503.)
37 + 39 + 83 + 42 + 79
Exercise 8.70 (Solution on p. 503.)
612 + 585 + 830 + 794

8.3.2 Exercises
Use the clustering method to estimate each sum. Results may vary.
Exercise 8.71 (Solution on p. 503.)
28 + 51 + 31 + 47
Exercise 8.72
42 + 19 + 39 + 23
Exercise 8.73 (Solution on p. 503.)
88 + 62 + 59 + 90
Exercise 8.74
76 + 29 + 33 + 82
Exercise 8.75 (Solution on p. 503.)
19 + 23 + 87 + 21
Exercise 8.76
41 + 28 + 42 + 37
Exercise 8.77 (Solution on p. 503.)
89 + 32 + 89 + 93
Exercise 8.78
73 + 72 + 27 + 71
Exercise 8.79 (Solution on p. 503.)
43 + 62 + 61 + 55
Exercise 8.80
31 + 77 + 31 + 27
Exercise 8.81 (Solution on p. 504.)
57 + 34 + 28 + 61 + 62
485

Exercise 8.82
94 + 18 + 23 + 91 + 19
Exercise 8.83 (Solution on p. 504.)
103 + 72 + 66 + 97 + 99
Exercise 8.84
42 + 121 + 119 + 124 + 41
Exercise 8.85 (Solution on p. 504.)
19 + 24 + 87 + 23 + 91 + 93
Exercise 8.86
108 + 61 + 63 + 96 + 57 + 99
Exercise 8.87 (Solution on p. 504.)
518 + 721 + 493 + 689
Exercise 8.88
981 + 1208 + 1214 + 1006
Exercise 8.89 (Solution on p. 504.)
23 + 81 + 77 + 79 + 19 + 81
Exercise 8.90
94 + 68 + 66 + 101 + 106 + 71 + 110

8.3.2.1 Exercises for Review

Exercise 8.91 (Solution on p. 504.)


(Section 1.2) Specify all the digits greater than 6.
Exercise 8.92
(Section 4.5) Find the product: 2
3
9
· 14 7
· 12 .
Exercise 8.93 (Solution on p. 504.)
(Section 6.3) Convert 0.06 to a fraction.
Exercise 8.94
(Section 7.3) Write the proportion in fractional form: "5 is to 8 as 25 is to 40."
Exercise 8.95 (Solution on p. 504.)
(Section 8.2) Estimate the sum using the method of rounding: 4,882 + 2,704.

8.4 Mental Arithmetic-Using the Distributive Property4


8.4.1 Section Overview
• The Distributive Property
• Estimation Using the Distributive Property

4 This content is available online at <http://cnx.org/content/m35013/1.2/>.


486 CHAPTER 8. TECHNIQUES OF ESTIMATION

8.4.2 The Distributive Property


Distributive Property
The distributive property is a characteristic of numbers that involves both addition and multiplication.
It is used often in algebra, and we can use it now to obtain exact results for a multiplication.

Suppose we wish to compute 3 (2 + 5). We can proceed in either of two ways, one way which is known to
us already (the order of operations), and a new way (the distributive property).

1. Compute 3 (2 + 5) using the order of operations.


3 (2 + 5)

Operate inside the parentheses rst: 2 + 5 = 7.

3 (2 + 5) = 3 · 7

Now multiply 3 and 7.

3 (2 + 5) = 3 · 7 = 21

Thus, 3 (2 + 5) = 21.
2. Compute 3 (2 + 5) using the distributive property.
We know that multiplication describes repeated addition. Thus,

3 (2 + 5) = 2+5+2+5+2+5
| {z }
2 + 5 appears 3 times
= 2+2+2+5+5+5 (by the commutative property of addition)
= 3·2+3·5 (since multiplication describes repeated addition)
= 6 + 15
= 21

Thus, 3 (2 + 5) = 21.

Let's look again at this use of the distributive property.

3 (2 + 5) = 2+5+2+5+2+5
| {z }
2 + 5 appears 3 times

3 (2 + 5) = 2+2+2 + 5+5+5
| {z } | {z }
2 appears 3 times 5 appears 3 times

The 3 has been distributed to the 2 and 5.

This is the distributive property. We distribute the factor to each addend in the parentheses. The
distributive property works for both sums and dierences.
487

8.4.2.1 Sample Set A

Example 8.15

Using the order of operations, we get

4 (6 + 2) = 4·8
= 32
Example 8.16

Using the order of operations, we get

8 (9 + 6) = 8 · 15
= 120
Example 8.17

Example 8.18

8.4.2.2 Practice Set A

Use the distributive property to compute each value.


Exercise 8.96 (Solution on p. 504.)
6 (8 + 4)
Exercise 8.97 (Solution on p. 504.)
4 (4 + 7)
Exercise 8.98 (Solution on p. 504.)
8 (2 + 9)
Exercise 8.99 (Solution on p. 504.)
12 (10 + 3)
Exercise 8.100 (Solution on p. 504.)
6 (11 − 3)
Exercise 8.101 (Solution on p. 504.)
8 (9 − 7)
Exercise 8.102 (Solution on p. 504.)
15 (30 − 8)
488 CHAPTER 8. TECHNIQUES OF ESTIMATION

8.4.3 Estimation Using the Distributive Property


We can use the distributive property to obtain exact results for products such as 25 · 23. The distributive
property works best for products when one of the factors ends in 0 or 5. We shall restrict our attention to
only such products.

8.4.3.1 Sample Set B

Use the distributive property to compute each value.


Example 8.19
25 · 23

Notice that 23 = 20 + 3. We now write

Thus, 25 · 23 = 575

We could have proceeded by writing 23 as 30 − 7.

Example 8.20
15 · 37

Notice that 37 = 30 + 7. We now write

Thus, 15 · 37 = 555

We could have proceeded by writing 37 as 40 − 3.

Example 8.21
15 · 86

Notice that 86 = 80 + 6. We now write


489

We could have proceeded by writing 86 as 90 − 4.

8.4.3.2 Practice Set B

Use the distributive property to compute each value.


Exercise 8.103 (Solution on p. 504.)
25 · 12
Exercise 8.104 (Solution on p. 504.)
35 · 14
Exercise 8.105 (Solution on p. 504.)
80 · 58
Exercise 8.106 (Solution on p. 504.)
65 · 62

8.4.4 Exercises
Use the distributive property to compute each product.
Exercise 8.107 (Solution on p. 504.)
15 · 13
Exercise 8.108
15 · 14
Exercise 8.109 (Solution on p. 504.)
25 · 11
Exercise 8.110
25 · 16
Exercise 8.111 (Solution on p. 504.)
15 · 16
Exercise 8.112
35 · 12
Exercise 8.113 (Solution on p. 504.)
45 · 83
Exercise 8.114
45 · 38
Exercise 8.115 (Solution on p. 504.)
25 · 38
490 CHAPTER 8. TECHNIQUES OF ESTIMATION

Exercise 8.116
25 · 96
Exercise 8.117 (Solution on p. 504.)
75 · 14
Exercise 8.118
85 · 34
Exercise 8.119 (Solution on p. 505.)
65 · 26
Exercise 8.120
55 · 51
Exercise 8.121 (Solution on p. 505.)
15 · 107
Exercise 8.122
25 · 208
Exercise 8.123 (Solution on p. 505.)
35 · 402
Exercise 8.124
85 · 110
Exercise 8.125 (Solution on p. 505.)
95 · 12
Exercise 8.126
65 · 40
Exercise 8.127 (Solution on p. 505.)
80 · 32
Exercise 8.128
30 · 47
Exercise 8.129 (Solution on p. 505.)
50 · 63
Exercise 8.130
90 · 78
Exercise 8.131 (Solution on p. 505.)
40 · 89

8.4.4.1 Exercises for Review

Exercise 8.132
(Section 3.5) Find the greatest common factor of 360 and 3,780.
Exercise 8.133 (Solution on p. 505.)
(Section 4.5) Reduce 5,594
148 to lowest terms.
Exercise 8.134
(Section 4.7) 1 59 of 2 47 is what number?
Exercise 8.135 (Solution on p. 505.)
(Section 7.3) Solve the proportion: 15
7 x
= 90 .
Exercise 8.136
(Section 8.3) Use the clustering method to estimate the sum: 88 + 106 + 91 + 114.
491

8.5 Estimation by Rounding Fractions5


8.5.1 Section Overview
• Estimation by Rounding Fractions

Estimation by rounding fractions is a useful technique for estimating the result of a computation involving
fractions. Fractions are commonly rounded to 14 , 12 , 34 , 0, and 1. Remember that rounding may cause
estimates to vary.

8.5.1.1 Sample Set A

Make each estimate remembering that results may vary.


Example 8.22
Estimate 3
5
5
+ 12 .

Notice that 3
5 is about 12 , and that 12
5
is about 21 .

Thus, 3
5
5
+ 12 is about 1
2 + 1
2 = 1. In fact, 3
5
5
+ 12 = 61
60 , a little more than 1.
Example 8.23
Estimate 5 38 + 4 10
9
+ 11 15 .

Adding the whole number parts, we get 20. Notice that 3


8 is close to 4 , 10
1 9
is close to 1, and 1
5 is
close to 14 . Then 38 + 10
9
+ 15 is close to 14 + 1 + 14 = 1 21 .

Thus, 5 38 + 4 10
9
+ 11 15 is close to 20 + 1 21 = 21 12 .

In fact, 5 38 + 4 10
9
+ 11 15 = 21 19
40 , a little less than 21 2 .
1

8.5.1.2 Practice Set A

Use the method of rounding fractions to estimate the result of each computation. Results may vary.
Exercise 8.137 (Solution on p. 505.)
5 5
8 + 12
Exercise 8.138 (Solution on p. 505.)
7 3
9 + 5
Exercise 8.139 (Solution on p. 505.)
4 7
8 15 + 3 10
Exercise 8.140 (Solution on p. 505.)
16 20
1
+ 4 78

5 This content is available online at <http://cnx.org/content/m35014/1.2/>.


492 CHAPTER 8. TECHNIQUES OF ESTIMATION

8.5.2 Exercises
Estimate each sum or dierence using the method of rounding. After you have made an estimate, nd the
exact value of the sum or dierence and compare this result to the estimated value. Result may vary.
Exercise 8.141 (Solution on p. 505.)
5 7
6 + 8
Exercise 8.142
3 11
+ 12
8
Exercise 8.143 (Solution on p. 505.)
9 3
10 + 5
Exercise 8.144
13 + 1
15 20
Exercise 8.145 (Solution on p. 505.)
3 6
20 + 25
Exercise 8.146
1 4
12 + 5
Exercise 8.147 (Solution on p. 505.)
15 1
16 + 12
Exercise 8.148
29 + 11
30 20
Exercise 8.149 (Solution on p. 505.)
5 4
12 + 6 11
Exercise 8.150
3 4
7 + 8 15
Exercise 8.151 (Solution on p. 505.)
9 3
10 + 2 8
Exercise 8.152
19 + 15 5
20 9
Exercise 8.153 (Solution on p. 505.)
8 35 + 4 20
1

Exercise 8.154
3 8
5 20 + 2 15
Exercise 8.155 (Solution on p. 505.)
1
9 15 + 6 54
Exercise 8.156
5
7 12 + 10 16
1

Exercise 8.157 (Solution on p. 505.)


3 11 13 7
20 + 2 25 + 1 8
Exercise 8.158
1 1
6 12 + 1 10 + 5 56
Exercise 8.159 (Solution on p. 505.)
15 7
16 − 8
Exercise 8.160
12 − 9
25 20
493

8.5.2.1 Exercises for Review

Exercise 8.161 (Solution on p. 505.)


(Section 2.6) The fact that
(a rst number · a second number) · a third number = a rst number ·
(a second number · a third number)
is an example of which property of multiplication?
Exercise 8.162
(Section 4.6) Find the quotient: 14
15 ÷ 45 .
4

Exercise 8.163 (Solution on p. 505.)


(Section 5.4) Find the dierence: 3 59 − 2 23 .
Exercise 8.164
(Section 6.8) Find the quotient: 4.6 ÷ 0.11.
Exercise 8.165 (Solution on p. 506.)
(Section 8.4) Use the distributive property to compute the product: 25 · 37.

8.6 Summary of Key Concepts6


8.6.1 Summary of Key Concepts
Estimation (Section 8.2)
Estimation is the process of determining an expected value of a computation.

Estimation By Rounding (Section 8.2)


The rounding technique estimates the result of a computation by rounding the numbers involved in the
computation to one or two nonzero digits. For example, 512 + 896 can be estimated by 500 + 900 = 1,400.

Cluster (Section 8.3)


When several numbers are close to one particular number, they are said to cluster near that particular
number.

Estimation By Clustering (Section 8.3)


The clustering technique of estimation can be used when

1. there are more than two numbers to be added, and


2. clustering occurs.

For example, 31 + 62 + 28 + 59 can be estimated by


(2 · 30) + (2 · 60) = 60 + 120 = 180

Distributive Property (Section 8.4)


The distributive property is a characteristic of numbers that involves both addition and multiplication.
For example,
3 (4 + 6) = 3 · 4 + 3 · 6 = 12 + 18 = 30

Estimation Using the Distributive Property (Section 8.4)


The distributive property can be used to obtain exact results for a multiplication.
For example,
15 · 23 = 15 · (20 + 3) = 15 · 20 + 15 · 3 = 300 + 45 = 345

6 This content is available online at <http://cnx.org/content/m35015/1.2/>.


494 CHAPTER 8. TECHNIQUES OF ESTIMATION

Estimation by Rounding Fractions (Section 8.5)


Estimation by rounding fractions commonly rounds fractions to 41 , 12 , 34 , 0, and 1.
For example,
12 + 16 can be estimated by
5 5 1 1 3
2 + 4 = 4

8.7 Exercise Supplement7


8.7.1 Exercise Supplement
8.7.1.1 Estimation by Rounding (Section 8.2)

For problems 1-70, estimate each value using the method of rounding. After you have made an estimate,
nd the exact value. Compare the exact and estimated values. Results may vary.
Exercise 8.166 (Solution on p. 506.)
286 + 312
Exercise 8.167
419 + 582
Exercise 8.168 (Solution on p. 506.)
689 + 511
Exercise 8.169
926 + 1, 105
Exercise 8.170 (Solution on p. 506.)
1, 927 + 3, 017
Exercise 8.171
5, 026 + 2, 814
Exercise 8.172 (Solution on p. 506.)
1, 408 + 2, 352
Exercise 8.173
1, 186 + 4, 228
Exercise 8.174 (Solution on p. 506.)
5, 771 + 246
Exercise 8.175
8, 305 + 484
Exercise 8.176 (Solution on p. 506.)
3, 812 + 2, 906
Exercise 8.177
5, 293 + 8, 007
Exercise 8.178 (Solution on p. 506.)
28, 481 + 32, 856
Exercise 8.179
92, 512 + 26, 071
Exercise 8.180 (Solution on p. 506.)
87, 612 + 2, 106
Exercise 8.181
42, 612 + 4, 861
7 This content is available online at <http://cnx.org/content/m35016/1.2/>.
495

Exercise 8.182 (Solution on p. 506.)


212, 413 + 609
Exercise 8.183
487, 235 + 494
Exercise 8.184 (Solution on p. 506.)
2, 409 + 1, 526
Exercise 8.185
3, 704 + 4, 704
Exercise 8.186 (Solution on p. 506.)
41 · 63
Exercise 8.187
38 · 81
Exercise 8.188 (Solution on p. 506.)
18 · 28
Exercise 8.189
52 · 21
Exercise 8.190 (Solution on p. 506.)
307 · 489
Exercise 8.191
412 · 807
Exercise 8.192 (Solution on p. 506.)
77 · 614
Exercise 8.193
62 · 596
Exercise 8.194 (Solution on p. 506.)
27 · 473
Exercise 8.195
92 · 336
Exercise 8.196 (Solution on p. 506.)
12 · 814
Exercise 8.197
8 · 2, 106
Exercise 8.198 (Solution on p. 506.)
192 · 452
Exercise 8.199
374 · 816
Exercise 8.200 (Solution on p. 506.)
88 · 4, 392
Exercise 8.201
126 · 2, 834
Exercise 8.202 (Solution on p. 506.)
3, 896 · 413
Exercise 8.203
5, 794 · 837
Exercise 8.204 (Solution on p. 506.)
6, 311 · 3, 512
496 CHAPTER 8. TECHNIQUES OF ESTIMATION

Exercise 8.205
7, 471 · 5, 782
Exercise 8.206 (Solution on p. 506.)
180 ÷ 12
Exercise 8.207
309 ÷ 16
Exercise 8.208 (Solution on p. 506.)
286 ÷ 22
Exercise 8.209
527 ÷ 17
Exercise 8.210 (Solution on p. 506.)
1, 007 ÷ 19
Exercise 8.211
1, 728 ÷ 36
Exercise 8.212 (Solution on p. 506.)
2, 703 ÷ 53
Exercise 8.213
2, 562 ÷ 61
Exercise 8.214 (Solution on p. 507.)
1, 260 ÷ 12
Exercise 8.215
3, 618 ÷ 18
Exercise 8.216 (Solution on p. 507.)
3, 344 ÷ 76
Exercise 8.217
7, 476 ÷ 356
Exercise 8.218 (Solution on p. 507.)
20, 984 ÷ 488
Exercise 8.219
43, 776 ÷ 608
Exercise 8.220 (Solution on p. 507.)
7, 196 ÷ 514
Exercise 8.221
51, 492 ÷ 514
Exercise 8.222 (Solution on p. 507.)
26, 962 ÷ 442
Exercise 8.223
33, 712 ÷ 112
Exercise 8.224 (Solution on p. 507.)
105, 152 ÷ 106
Exercise 8.225
176, 978 ÷ 214
Exercise 8.226 (Solution on p. 507.)
48.06 + 23.11
Exercise 8.227
73.73 + 72.9
497

Exercise 8.228 (Solution on p. 507.)


62.91 + 56.4
Exercise 8.229
87.865 + 46.772
Exercise 8.230 (Solution on p. 507.)
174.6 + 97.2
Exercise 8.231
(48.3) (29.6)
Exercise 8.232 (Solution on p. 507.)
(87.11) (23.2)
Exercise 8.233
(107.02) (48.7)
Exercise 8.234 (Solution on p. 507.)
(0.76) (5.21)
Exercise 8.235
(1.07) (13.89)

8.7.1.2 Estimation by Clustering (Section 8.3)

For problems 71-90, estimate each value using the method of clustering. After you have made an estimate,
nd the exact value. Compare the exact and estimated values. Results may vary.
Exercise 8.236 (Solution on p. 507.)
38 + 51 + 41 + 48
Exercise 8.237
19 + 73 + 23 + 71
Exercise 8.238 (Solution on p. 507.)
27 + 62 + 59 + 31
Exercise 8.239
18 + 73 + 69 + 19
Exercise 8.240 (Solution on p. 507.)
83 + 49 + 79 + 52
Exercise 8.241
67 + 71 + 84 + 81
Exercise 8.242 (Solution on p. 507.)
16 + 13 + 24 + 26
Exercise 8.243
34 + 56 + 36 + 55
Exercise 8.244 (Solution on p. 507.)
14 + 17 + 83 + 87
Exercise 8.245
93 + 108 + 96 + 111
Exercise 8.246 (Solution on p. 507.)
18 + 20 + 31 + 29 + 24 + 38
Exercise 8.247
32 + 27 + 48 + 51 + 72 + 69
498 CHAPTER 8. TECHNIQUES OF ESTIMATION

Exercise 8.248 (Solution on p. 507.)


64 + 17 + 27 + 59 + 31 + 21
Exercise 8.249
81 + 41 + 92 + 38 + 88 + 80
Exercise 8.250 (Solution on p. 507.)
87 + 22 + 91
Exercise 8.251
44 + 38 + 87
Exercise 8.252 (Solution on p. 507.)
19 + 18 + 39 + 22 + 42
Exercise 8.253
31 + 28 + 49 + 29
Exercise 8.254 (Solution on p. 507.)
88 + 86 + 27 + 91 + 29
Exercise 8.255
57 + 62 + 18 + 23 + 61 + 21

8.7.1.3 Mental Arithmetic- Using the Distributive Property (Section 8.4)

For problems 91-110, compute each product using the distributive property.
Exercise 8.256 (Solution on p. 507.)
15 · 33
Exercise 8.257
15 · 42
Exercise 8.258 (Solution on p. 507.)
35 · 36
Exercise 8.259
35 · 28
Exercise 8.260 (Solution on p. 507.)
85 · 23
Exercise 8.261
95 · 11
Exercise 8.262 (Solution on p. 507.)
30 · 14
Exercise 8.263
60 · 18
Exercise 8.264 (Solution on p. 508.)
75 · 23
Exercise 8.265
65 · 31
Exercise 8.266 (Solution on p. 508.)
17 · 15
Exercise 8.267
38 · 25
499

Exercise 8.268 (Solution on p. 508.)


14 · 65
Exercise 8.269
19 · 85
Exercise 8.270 (Solution on p. 508.)
42 · 60
Exercise 8.271
81 · 40
Exercise 8.272 (Solution on p. 508.)
15 · 105
Exercise 8.273
35 · 202
Exercise 8.274 (Solution on p. 508.)
45 · 306
Exercise 8.275
85 · 97

8.7.1.4 Estimation by Rounding Fractions (Section 8.5)

For problems 111-125, estimate each sum using the method of rounding fractions. After you have made an
estimate, nd the exact value. Compare the exact and estimated values. Results may vary.
Exercise 8.276 (Solution on p. 508.)
3 5
8 + 6
Exercise 8.277
7 1
16 + 24
Exercise 8.278 (Solution on p. 508.)
7 13
15 + 30
Exercise 8.279
14 + 19
15 20
Exercise 8.280 (Solution on p. 508.)
13 7
25 + 30
Exercise 8.281
11 + 7
12 8
Exercise 8.282 (Solution on p. 508.)
9 15
32 + 16
Exercise 8.283
5 1
8 + 32
Exercise 8.284 (Solution on p. 508.)
2 34 + 6 35
Exercise 8.285
4 95 + 8 27
1

Exercise 8.286 (Solution on p. 508.)


11 18
5 22
+ 7 45
Exercise 8.287
14 19 7
36 + 2 18
500 CHAPTER 8. TECHNIQUES OF ESTIMATION

Exercise 8.288 (Solution on p. 508.)


1
6 20 1
+ 2 10 + 8 13
60
Exercise 8.289
5 78 + 1 14 + 12 12
5

Exercise 8.290 (Solution on p. 508.)


15 + 8 19
10 12 + 6 16 80

8.8 Prociency Exam8


8.8.1 Prociency Exam
For problems 1 - 16, estimate each value. After you have made an estimate, nd the exact value. Results
may vary.
Exercise 8.291 (Solution on p. 508.)
(Section 8.2) 3,716 + 6,789
Exercise 8.292 (Solution on p. 508.)
(Section 8.2) 8,821 + 9,217
Exercise 8.293 (Solution on p. 508.)
(Section 8.2) 7, 316 − 2, 305
Exercise 8.294 (Solution on p. 508.)
(Section 8.2) 110, 812 − 83, 406
Exercise 8.295 (Solution on p. 508.)
(Section 8.2) 82 · 38
Exercise 8.296 (Solution on p. 508.)
(Section 8.2) 51 · 92
Exercise 8.297 (Solution on p. 508.)
(Section 8.2) 48 · 6,012
Exercise 8.298 (Solution on p. 508.)
(Section 8.2) 238 ÷ 17
Exercise 8.299 (Solution on p. 508.)
(Section 8.2) 2,660 ÷ 28
Exercise 8.300 (Solution on p. 508.)
(Section 8.2) 43.06 + 37.94
Exercise 8.301 (Solution on p. 508.)
(Section 8.2) 307.006 + 198.0005
Exercise 8.302 (Solution on p. 509.)
(Section 8.2) (47.2) (92.8)
Exercise 8.303 (Solution on p. 509.)
(Section 8.3) 58 + 91 + 61 + 88
Exercise 8.304 (Solution on p. 509.)
(Section 8.3) 43 + 39 + 89 + 92
Exercise 8.305 (Solution on p. 509.)
(Section 8.3) 81 + 78 + 27 + 79
8 This content is available online at <http://cnx.org/content/m35017/1.2/>.
501

Exercise 8.306 (Solution on p. 509.)


(Section 8.3) 804 + 612 + 801 + 795 + 606
For problems 17-21, use the distributive property to obtain the exact result.
Exercise 8.307 (Solution on p. 509.)
(Section 8.4) 25 · 14
Exercise 8.308 (Solution on p. 509.)
(Section 8.4) 15 · 83
Exercise 8.309 (Solution on p. 509.)
(Section 8.4) 65 · 98
Exercise 8.310 (Solution on p. 509.)
(Section 8.4) 80 · 107
Exercise 8.311 (Solution on p. 509.)
(Section 8.4) 400 · 215
For problems 22-25, estimate each value. After you have made an estimate, nd the exact value. Results
may vary.
Exercise 8.312 (Solution on p. 509.)
(Section 8.5) 15
16 +
5
8
Exercise 8.313 (Solution on p. 509.)
(Section 8.5 ) 25
1
+ 11 17
20 + 30
Exercise 8.314 (Solution on p. 509.)
(Section 8.5) 8 16
9
+ 14 12
1

Exercise 8.315 (Solution on p. 509.)


(Section 8.5) 5 49 + 1 17 5
36 + 6 12
502 CHAPTER 8. TECHNIQUES OF ESTIMATION

Solutions to Exercises in Chapter 8


Solution to Exercise 8.1 (p. 476)
4, 216 + 3, 942 : 4, 200 + 3, 900. About 8,100. In fact, 8,158.
Solution to Exercise 8.2 (p. 476)
812 + 514 : 800 + 500. About 1,300. In fact, 1,326.
Solution to Exercise 8.3 (p. 476)
43, 892 + 92, 106 : 44, 000 + 92, 000. About 136,000. In fact, 135,998.
Solution to Exercise 8.4 (p. 477)
628 − 413 : 600 − 400. About 200. In fact, 215.
Solution to Exercise 8.5 (p. 477)
7, 842 − 5, 209 : 7, 800 − 5, 200. About 2,600. In fact, 2,633.
Solution to Exercise 8.6 (p. 477)
73, 812 − 28, 492 : 74, 000 − 28, 000. About 46,000. In fact, 45,320.
Solution to Exercise 8.7 (p. 477)
31 · 87 : 30 · 90. About 2,700. In fact, 2,697.
Solution to Exercise 8.8 (p. 477)
18 · 42 : 20 · 40. About 800. In fact, 756.
Solution to Exercise 8.9 (p. 477)
16 · 94 : 15 · 100. About 1,500. In fact, 1,504.
Solution to Exercise 8.10 (p. 478)
221 ÷ 18 : 200 ÷ 20. About 10. In fact, 12.27.
Solution to Exercise 8.11 (p. 478)
4, 079 ÷ 381 : 4, 000 ÷ 400. About 10. In fact, 10.70603675...
Solution to Exercise 8.12 (p. 478)
609, 000 ÷ 16, 000 : 600, 000 ÷ 15, 000. About 40. In fact, 38.0625.
Solution to Exercise 8.13 (p. 479)
61.02 + 26.8 : 61 + 27. About 88. In fact, 87.82.
Solution to Exercise 8.14 (p. 479)
109.12 + 137.88 : 110 + 138. About 248. In fact, 247. We could have estimated 137.88 with 140. Then
110 + 140 is an easy mental addition. We would conclude then that 109.12 + 137.88 is about 250.
Solution to Exercise 8.15 (p. 479)
(47.8) (21.1) : (50) (20). About 1,000. In fact, 1,008.58.
Solution to Exercise 8.16 (p. 479)
32% of 14.88 : (.3) (15). About 4.5. In fact, 4.7616.
Solution to Exercise 8.17 (p. 479)
about 3,600; in fact 3,600
Solution to Exercise 8.19 (p. 480)
about 1,700; in fact 1,717
Solution to Exercise 8.21 (p. 480)
about 14,000; in fact 14,006
Solution to Exercise 8.23 (p. 480)
about 3,500; in fact 3,539
Solution to Exercise 8.25 (p. 480)
about 5,700; in fact 5,694
Solution to Exercise 8.27 (p. 480)
about 1,500; in fact 1,696
Solution to Exercise 8.29 (p. 480)
about 540,000; in fact 559,548
Solution to Exercise 8.31 (p. 480)
about 583,200,000; in fact 583,876,992
503

Solution to Exercise 8.33 (p. 480)


about 15; in fact 15.11
Solution to Exercise 8.35 (p. 480)
about 20; in fact 22
Solution to Exercise 8.37 (p. 480)
about 33; in fact 33.86
Solution to Exercise 8.39 (p. 480)
about 93.2; in fact 93.22
Solution to Exercise 8.41 (p. 480)
about 70; in fact 69.62
Solution to Exercise 8.43 (p. 481)
about 348.6; in fact 348.57
Solution to Exercise 8.45 (p. 481)
about 1,568.0; in fact 1,564.244
Solution to Exercise 8.47 (p. 481)
about 49.5; in fact 49.60956
Solution to Exercise 8.49 (p. 481)
about 43,776; in fact 43,833.258
Solution to Exercise 8.51 (p. 481)
about 6.21; in fact 6.0896
Solution to Exercise 8.53 (p. 481)
about 0.0519; in fact 0.05193
Solution to Exercise 8.55 (p. 481)
about 6.3; in fact 6.5193
Solution to Exercise 8.57 (p. 481)
about 4.5; in fact 4.896
Solution to Exercise 8.59 (p. 481)
about 0.8; in fact 0.74124
Solution to Exercise 8.61 (p. 481)
about 0.00008; in fact 0.000078
Solution to Exercise 8.63 (p. 482)
23
25
Solution to Exercise 8.65 (p. 482)
60 feet tall
Solution to Exercise 8.67 (p. 484)
(2 · 30) + (2 · 50) = 60 + 100 = 160
Solution to Exercise 8.68 (p. 484)
(3 · 40) + 70 = 120 + 70 = 190
Solution to Exercise 8.69 (p. 484)
(3 · 40) + (2 · 80) = 120 + 160 = 280
Solution to Exercise 8.70 (p. 484)
(2 · 600) + (2 · 800) = 1, 200 + 1, 600 = 2, 800
Solution to Exercise 8.71 (p. 484)
2 (30) + 2 (50) = 160 (157)
Solution to Exercise 8.73 (p. 484)
2 (90) + 2 (60) = 300 (299)
Solution to Exercise 8.75 (p. 484)
3 (20) + 90 = 150 (150)
Solution to Exercise 8.77 (p. 484)
3 (90) + 30 = 300 (303)
504 CHAPTER 8. TECHNIQUES OF ESTIMATION

Solution to Exercise 8.79 (p. 484)


40 + 3 (60) = 220 (221)
Solution to Exercise 8.81 (p. 484)
3 (60) + 2 (30) = 240 (242)
Solution to Exercise 8.83 (p. 485)
3 (100) + 2 (70) = 440 (437)
Solution to Exercise 8.85 (p. 485)
3 (20) + 3 (90) = 330 (337)
Solution to Exercise 8.87 (p. 485)
2 (500) + 2 (700) = 2, 400 (2, 421)
Solution to Exercise 8.89 (p. 485)
2 (20) + 4 (80) = 360 (360)
Solution to Exercise 8.91 (p. 485)
7, 8, 9
Solution to Exercise 8.93 (p. 485)
3
50
Solution to Exercise 8.95 (p. 485)
4, 900 + 2, 700 = 7, 600 (7, 586)
Solution to Exercise 8.96 (p. 487)
6 · 8 + 6 · 4 = 48 + 24 = 72
Solution to Exercise 8.97 (p. 487)
4 · 4 + 4 · 7 = 16 + 28 = 44
Solution to Exercise 8.98 (p. 487)
8 · 2 + 8 · 9 = 16 + 72 = 88
Solution to Exercise 8.99 (p. 487)
12 · 10 + 12 · 3 = 120 + 36 = 156
Solution to Exercise 8.100 (p. 487)
6 · 11 − 6 · 3 = 66 − 18 = 48
Solution to Exercise 8.101 (p. 487)
8 · 9 − −8 · 7 = 72 − −56 = 16
Solution to Exercise 8.102 (p. 487)
15 · 30 − 15 · 8 = 450 − 120 = 330
Solution to Exercise 8.103 (p. 489)
25 (10 + 2) = 25 · 10 + 25 · 2 = 250 + 50 = 300
Solution to Exercise 8.104 (p. 489)
35 (10 + 4) = 35 · 10 + 35 · 4 = 350 + 140 = 490
Solution to Exercise 8.105 (p. 489)
80 (50 + 8) = 80 · 50 + 80 · 8 = 4, 000 + 640 = 4, 640
Solution to Exercise 8.106 (p. 489)
65 (60 + 2) = 65 · 60 + 65 · 2 = 3, 900 + 130 = 4, 030
Solution to Exercise 8.107 (p. 489)
15 (10 + 3) = 150 + 45 = 195
Solution to Exercise 8.109 (p. 489)
25 (10 + 1) = 250 + 25 = 275
Solution to Exercise 8.111 (p. 489)
15 (20 − 4) = 300 − 60 = 240
Solution to Exercise 8.113 (p. 489)
45 (80 + 3) = 3600 + 135 = 3735
Solution to Exercise 8.115 (p. 489)
25 (40 − 2) = 1, 000 − 50 = 950
505

Solution to Exercise 8.117 (p. 490)


75 (10 + 4) = 750 + 300 = 1, 050
Solution to Exercise 8.119 (p. 490)
65 (20 + 6) = 1, 300 + 390 = 1, 690 or 65 (30 − 4) = 1, 950 − 260 = 1, 690
Solution to Exercise 8.121 (p. 490)
15 (100 + 7) = 1, 500 + 105 = 1, 605
Solution to Exercise 8.123 (p. 490)
35 (400 + 2) = 14, 000 + 70 = 14, 070
Solution to Exercise 8.125 (p. 490)
95 (10 + 2) = 950 + 190 = 1, 140
Solution to Exercise 8.127 (p. 490)
80 (30 + 2) = 2, 400 + 160 = 2, 560
Solution to Exercise 8.129 (p. 490)
50 (60 + 3) = 3, 000 + 150 = 3, 150
Solution to Exercise 8.131 (p. 490)
40 (90 − 1) = 3, 600 − 40 = 3, 560
Solution to Exercise 8.133 (p. 490)
3
26
Solution to Exercise 8.135 (p. 490)
x = 42
Solution to Exercise 8.137 (p. 491)
Results may vary. 1
+ 1
= 1. In fact, 5 5
+ 12 25 = 1 1
= 24
2 2 8 24
Solution to Exercise 8.138 (p. 491)
Results may vary. 1 + 1
2 = 1 12 . In fact, 7
9 + 3
5 = 1 17
45
Solution to Exercise 8.139 (p. 491)
Results may vary. 8 14 + 3 34 = 11 + 1 = 12. In fact, 8 15
4 7
+ 3 10 29
= 11 30
Solution to Exercise 8.140 (p. 491)
Results may vary. (16 + 0) + (4 + 1) = 16 + 5 = 21. In fact, 16 20
1 37
+ 4 78 = 20 40
Solution to Exercise 8.141 (p. 492)
1 + 1 = 2 1 17

24
Solution to Exercise 8.143 (p. 492)
1
= 1 12 1 12

1+ 2
Solution to Exercise 8.145 (p. 492)
1
+ 1
=
 1 39
4 4 2 100
Solution to Exercise 8.147 (p. 492)
1

1 + 0 = 1 1 48
Solution to Exercise 8.149 (p. 492)
1
+ 6 12 = 7 6 103

2 132
Solution to Exercise 8.151 (p. 492)
1 + 2 21 = 3 12 3 11

40
Solution to Exercise 8.153 (p. 492)
13
8 21 + 4 = 12 12 12 20


Solution to Exercise 8.155 (p. 492)


9 + 7 = 16 15 15 13 
Solution to Exercise 8.157 (p. 492)
3 12 + 2 12 + 2 = 8 7 189
200
Solution to Exercise 8.159 (p. 492)
1

1 − 1 = 0 16
Solution to Exercise 8.161 (p. 493)
associative
506 CHAPTER 8. TECHNIQUES OF ESTIMATION

Solution to Exercise 8.163 (p. 493)


8
9
Solution to Exercise 8.165 (p. 493)
25 (40 − 3) = 1000 − 75 = 925
Solution to Exercise 8.166 (p. 494)
600 (598)
Solution to Exercise 8.168 (p. 494)
(1,200)
Solution to Exercise 8.170 (p. 494)
4,900 (4,944)
Solution to Exercise 8.172 (p. 494)
3,800 (3,760)
Solution to Exercise 8.174 (p. 494)
6,050 (6,017)
Solution to Exercise 8.176 (p. 494)
6,700 (6,718)
Solution to Exercise 8.178 (p. 494)
61,400 (61,337)
Solution to Exercise 8.180 (p. 494)
89,700 (89,718)
Solution to Exercise 8.182 (p. 495)
213,000 (213,022)
Solution to Exercise 8.184 (p. 495)
3,900 (3,935)
Solution to Exercise 8.186 (p. 495)
2,400 (2,583)
Solution to Exercise 8.188 (p. 495)
600 (504)
Solution to Exercise 8.190 (p. 495)
150,123 147,000 (150,123)
Solution to Exercise 8.192 (p. 495)
47,278 48,000 (47,278)
Solution to Exercise 8.194 (p. 495)
12,771 14,100 (12,711)
Solution to Exercise 8.196 (p. 495)
8,100 (9,768)
Solution to Exercise 8.198 (p. 495)
90,000 (86,784)
Solution to Exercise 8.200 (p. 495)
396,000 (386,496)
Solution to Exercise 8.202 (p. 495)
1,609,048 1,560,000 (1,609,048)
Solution to Exercise 8.204 (p. 495)
22,050,000 (22,164,232)
Solution to Exercise 8.206 (p. 496)
18 (15)
Solution to Exercise 8.208 (p. 496)
14 12 (13)
Solution to Exercise 8.210 (p. 496)
50 (53)
507

Solution to Exercise 8.212 (p. 496)


54 (51)
Solution to Exercise 8.214 (p. 496)
130 (105)
Solution to Exercise 8.216 (p. 496)
41.25 (44)
Solution to Exercise 8.218 (p. 496)
42 (43)
Solution to Exercise 8.220 (p. 496)
14.4 (14)
Solution to Exercise 8.222 (p. 496)
60 (61)
Solution to Exercise 8.224 (p. 496)
1,000 (992)
Solution to Exercise 8.226 (p. 496)
71.1 (71.17)
Solution to Exercise 8.228 (p. 497)
119.4 (119.31)
Solution to Exercise 8.230 (p. 497)
272 (271.8)
Solution to Exercise 8.232 (p. 497)
2,001 (2,020.952)
Solution to Exercise 8.234 (p. 497)
4.16 (3.9596)
Solution to Exercise 8.236 (p. 497)
2 (40) + 2 (50) = 180 (178)
Solution to Exercise 8.238 (p. 497)
2 (30) + 2 (60) = 180 (179)
Solution to Exercise 8.240 (p. 497)
2 (80) + 2 (50) = 260 (263)
Solution to Exercise 8.242 (p. 497)
3 (20) + 1 (10) = 70 (79)
Solution to Exercise 8.244 (p. 497)
2 (15) + 2 (80) = 190 (201)
Solution to Exercise 8.246 (p. 497)
3 (20) + 2 (30) + 40 = 160 (160)
Solution to Exercise 8.248 (p. 498)
2 (60) + 2 (20) + 2 (30) = 220 (219)
Solution to Exercise 8.250 (p. 498)
2 (90) + 20 = 200 (200)
Solution to Exercise 8.252 (p. 498)
3 (20) + 2 (40) = 140 (140)
Solution to Exercise 8.254 (p. 498)
3 (90) + 2 (30) = 330 (321)
Solution to Exercise 8.256 (p. 498)
15 (30 + 3) = 450 + 45 = 495
Solution to Exercise 8.258 (p. 498)
35 (40 − 4) = 1400 − 140 = 1, 260
Solution to Exercise 8.260 (p. 498)
85 (20 + 3) = 1, 700 + 225 = 1, 955
508 CHAPTER 8. TECHNIQUES OF ESTIMATION

Solution to Exercise 8.262 (p. 498)


30 (10 + 4) = 300 + 120 = 420
Solution to Exercise 8.264 (p. 498)
75 (20 + 3) = 1, 500 + 225 = 1, 725
Solution to Exercise 8.266 (p. 498)
15 (20 − 3) = 300 − 45 = 255
Solution to Exercise 8.268 (p. 499)
65 (10 + 4) = 650 + 260 = 910
Solution to Exercise 8.270 (p. 499)
60 (40 + 2) = 2, 400 + 120 = 2, 520
Solution to Exercise 8.272 (p. 499)
15 (100 + 5) = 1, 500 + 75 = 1, 575
Solution to Exercise 8.274 (p. 499)
45 (300 + 6) = 13, 500 + 270 = 13, 770
Solution to Exercise 8.276 (p. 499)
1
+ 1 = 1 12 1 24
5

2
Solution to Exercise
 8.278 (p. 499)
1
+ 1
=1 27 or 9
2 2 30 10
Solution to Exercise 8.280 (p. 499)
1
+ 1
=
 3 113
2 4 4 150
Solution to Exercise 8.282 (p. 499)
1
4 + 1 = 1 14 3932 or 1 32
7

Solution to Exercise 8.284 (p. 499)


2 34 + 6 12 = 9 14 9 20
7


Solution to Exercise
 8.286 (p. 499)
11 41 + 7 12 = 18 34 18 23
30
Solution to Exercise 8.288 (p. 500)
11
6 + 2 + 8 14 = 16 14 16 30
Solution to Exercise 8.290 (p. 500)
10 12 + 7 + 8 14 = 25 43 25 27

40
Solution to Exercise 8.291 (p. 500)
10,500 (10,505)
Solution to Exercise 8.292 (p. 500)
18,000 (18,038)
Solution to Exercise 8.293 (p. 500)
5,000 (5,011)
Solution to Exercise 8.294 (p. 500)
28,000 (27,406)
Solution to Exercise 8.295 (p. 500)
3,200 (3,116)
Solution to Exercise 8.296 (p. 500)
4,500 (4,692)
Solution to Exercise 8.297 (p. 500)
300,000 (288,576)
Solution to Exercise 8.298 (p. 500)
12 (14)
Solution to Exercise 8.299 (p. 500)
90 (95)
Solution to Exercise 8.300 (p. 500)
81 (81.00)
509

Solution to Exercise 8.301 (p. 500)


505 (505.0065)
Solution to Exercise 8.302 (p. 500)
4,371 (4,380.16)
Solution to Exercise 8.303 (p. 500)
2 (60) + 2 (90) = 300 (298)
Solution to Exercise 8.304 (p. 500)
2 (40) + 2 (90) = 260 (263)
Solution to Exercise 8.305 (p. 500)
30 + 3 (80) = 270 (265)
Solution to Exercise 8.306 (p. 501)
3 (800) + 2 (600) = 3, 600 (3,618)
Solution to Exercise 8.307 (p. 501)
25 (10 + 4) = 250 + 100 = 350
Solution to Exercise 8.308 (p. 501)
15 (80 + 3) = 1, 200 + 45 = 1, 245
Solution to Exercise 8.309 (p. 501)
65 (100 − 2) = 6, 500 − 130 = 6, 370
Solution to Exercise 8.310 (p. 501)
80 (100 + 7) = 8, 000 + 560 = 8, 560
Solution to Exercise 8.311 (p. 501)
400 (200 + 15) = 80, 000 + 6, 000 = 86, 000
Solution to Exercise 8.312 (p. 501)
1
= 1 12 1 16
9

1+ 2
Solution to Exercise 8.313 (p. 501)
0+ 1
+ 1
 47
= 1 1 300
2 2
Solution to Exercise
 8.314 (p. 501)
8 21 + 14 = 22 12 22 31
48
Solution to Exercise 8.315 (p. 501)
5 12 + 1 12 + 6 12 = 13 12 13 31

510 CHAPTER 8. TECHNIQUES OF ESTIMATION
Chapter 9

Measurement and Geometry

9.1 Objectives1
After completing this chapter, you should
Measurement and the United States System (Section 9.2)

• know what the word measurement means


• be familiar with United States system of measurement
• be able to convert from one unit of measure in the United States system to another unit of measure

The Metric System of Measurement (Section 9.3)

• be more familiar with some of the advantages of the base ten number system
• know the prexes of the metric measures
• be familiar with the metric system of measurement
• be able to convert from one unit of measure in the metric system to another unit of measure

Simplication of Denominate Numbers (Section 9.4)

• be able to convert an unsimplied unit of measure to a simplied unit of measure


• be able to add and subtract denominate numbers
• be able to multiply and divide a denominate number by a whole number

Perimeter and Circumference of Geometric Figures (Section 9.5)

• know what a polygon is


• know what perimeter is and how to nd it
• know what the circumference, diameter, and radius of a circle is and how to nd each one
• know the meaning of the symbol π and its approximating value
• know what a formula is and four versions of the circumference formula of a circle

Area and Volume of Geometric Figures and Objects (Section 9.6)

• know the meaning and notation for area


• know the area formulas for some common geometric gures
• be able to nd the areas of some common geometric gures
• know the meaning and notation for volume
• know the volume formulas for some common geometric objects
• be able to nd the volume of some common geometric objects

1 This content is available online at <http://cnx.org/content/m18897/1.3/>.

511
512 CHAPTER 9. MEASUREMENT AND GEOMETRY

9.2 Measurement and the United States System2


9.2.1 Section Overview
• Measurement
• The United States System of Measurement
• Conversions in the United States System

9.2.2 Measurement
There are two major systems of measurement in use today. They are the United States system and the metric
system. Before we describe these systems, let's gain a clear understanding of the concept of measurement.
Measurement
Measurement is comparison to some standard.

Standard Unit of Measure


The concept of measurement is based on the idea of direct comparison. This means that measurement is the
result of the comparison of two quantities. The quantity that is used for comparison is called the standard
unit of measure.

Over the years, standards have changed. Quite some time in the past, the standard unit of measure was
determined by a king. For example,

1 inch was the distance between the tip of the thumb and the knuckle of the king.
1 inch was also the length of 16 barley grains placed end to end.

Today, standard units of measure rarely change. Standard units of measure are the responsibility of the
Bureau of Standards in Washington D.C.

Some desirable properties of a standard are the following:

1. Accessibility. We should have access to the standard so we can make comparisons.


2. Invariance. We should be condent that the standard is not subject to change.
3. Reproducibility. We should be able to reproduce the standard so that measurements are convenient
and accessible to many people.

9.2.3 The United States System of Measurement


Some of the common units (along with their abbreviations) for the United States system of measurement
are listed in the following table.

Unit Conversion Table


Length 1 foot (ft) = 12 inches (in.)
1 yard (yd) = 3 feet (ft)
1 mile (mi) = 5,280 feet
continued on next page

2 This content is available online at <http://cnx.org/content/m35018/1.2/>.


513

Weight 1 pound (lb) =16 ounces (oz)


1 ton (T) = 2,000 pounds
Liquid Volume 1 tablespoon (tbsp) = 3 teaspoons (tsp)
1 uid ounce ( oz) = 2 tablespoons
1 cup (c) = 8 uid ounces
1 pint (pt) = 2 cups
1 quart (qt) = 2 pints
1 gallon (gal) = 4 quarts
Time 1 minute (min) = 60 seconds (sec)
1 hour ( hr) = 60 minutes
1 day (da) = 24 hours
1 week (wk) = 7 days

Table 9.1

9.2.4 Conversions in the United States System


It is often convenient or necessary to convert from one unit of measure to another. For example, it may
be convenient to convert a measurement of length that is given in feet to one that is given in inches. Such
conversions can be made using unit fractions.

Unit Fraction
A unit fraction is a fraction with a value of 1.

Unit fractions are formed by using two equal measurements. One measurement is placed in the numerator
of the fraction, and the other in the denominator. Placement depends on the desired conversion.

Placement of Units
Place the unit being converted to in the numerator.
Place the unit being converted from in the denominator.

For example,

Equal Measurements Unit Fraction


1ft = 12in. 1ft 12in.
12in. or 1ft
1pt = 16  oz 1pt 16  oz
16  oz or 1pt
1wk = 7da 7da 1wk
1wk or 7da
Table 9.2

9.2.4.1 Sample Set A

Make the following conversions. If a fraction occurs, convert it to a decimal rounded to two decimal places.
Example 9.1
Convert 11 yards to feet.

Looking in the unit conversion table under length, we see that 1 yd = 3 ft. There are two corre-
sponding unit fractions, 13 yd 3 ft
ft and 1 yd . Which one should we use? Look to see which unit we wish
to convert to. Choose the unit fraction with this unit in the numerator. We will choose 13 yd
ft since
514 CHAPTER 9. MEASUREMENT AND GEOMETRY

this unit fraction has feet in the numerator. Now, multiply 11 yd by the unit fraction. Notice that
since the unit fraction has the value of 1, multiplying by it does not change the value of 11 yd.

11 yd = 11yd · 3ft Divide out common units.


1 1yd
11)yd
= · 3ft (Units can be added, subtracted, multiplied, and divided, just as numbers can.)
1 1)yd
11·3ft
= 1
= 33ft

Thus, 11 yd = 33ft.
Example 9.2
Convert 36  oz to pints.

Looking in the unit conversion table under liquid volume, we see that 1 pt = 16  oz. Since we
are to convert to pints, we will construct a unit fraction with pints in the numerator.
36  oz 1 pt
36 oz = 1 · 16  oz Divide out common units.
36)  oz 1 pt
= ·
1 16)  oz
36·1 pt
= 16
= 36 pt Reduce.
16

4 pt Convert to decimals: 94 = 2.25.


9
=

Thus, 36  oz = 2.25 pt.


Example 9.3
Convert 2,016 hr to weeks.

Looking in the unit conversion table under time, we see that 1wk = 7da and that 1 da = 24 hr.
To convert from hours to weeks, we must rst convert from hours to days and then from days to
weeks. We need two unit fractions.
1 da . The unit fraction needed for
The unit fraction needed for converting from hours to days is 24 hr
1 wk
converting from days to weeks is 7 da .

2,016 hr 1 da 1 wk
2, 016 hr = 1 ·
24 hr · 7 da Divide out common units.
2,016) hr
= · 1) da · 1 wk
1 24) hr 7) da
2,016·1wk
= 24·7 Reduce.
= 12wk

Thus, 2,016 hr = 12 wk.

9.2.4.2 Practice Set A

Make the following conversions. If a fraction occurs, convert it to a decimal rounded to two decimal places.
Exercise 9.1 (Solution on p. 567.)
Convert 18 ft to yards.
515

Exercise 9.2 (Solution on p. 567.)


Convert 2 mi to feet.
Exercise 9.3 (Solution on p. 567.)
Convert 26 ft to yards.
Exercise 9.4 (Solution on p. 567.)
Convert 9 qt to pints.
Exercise 9.5 (Solution on p. 567.)
Convert 52 min to hours.
Exercise 9.6 (Solution on p. 567.)
Convert 412 hr to weeks.

9.2.5 Exercises
Make each conversion using unit fractions. If fractions occur, convert them to decimals rounded to two
decimal places.
Exercise 9.7 (Solution on p. 567.)
14 yd to feet
Exercise 9.8
3 mi to yards
Exercise 9.9 (Solution on p. 567.)
8 mi to inches
Exercise 9.10
2 mi to inches
Exercise 9.11 (Solution on p. 567.)
18 in. to feet
Exercise 9.12
84 in. to yards
Exercise 9.13 (Solution on p. 567.)
5 in. to yards
Exercise 9.14
106 ft to miles
Exercise 9.15 (Solution on p. 567.)
62 in. to miles
Exercise 9.16
0.4 in. to yards
Exercise 9.17 (Solution on p. 567.)
3 qt to pints
Exercise 9.18
5 lb to ounces
Exercise 9.19 (Solution on p. 567.)
6 T to ounces
Exercise 9.20
4 oz to pounds
Exercise 9.21 (Solution on p. 567.)
15,000 oz to pounds
516 CHAPTER 9. MEASUREMENT AND GEOMETRY

Exercise 9.22
15,000 oz to tons
Exercise 9.23 (Solution on p. 567.)
9 tbsp to teaspoons
Exercise 9.24
3 c to tablespoons
Exercise 9.25 (Solution on p. 567.)
5 pt to uid ounces
Exercise 9.26
16 tsp to cups
Exercise 9.27 (Solution on p. 567.)
5  oz to quarts
Exercise 9.28
3 qt to gallons
Exercise 9.29 (Solution on p. 567.)
5 pt to teaspoons
Exercise 9.30
3 qt to tablespoons
Exercise 9.31 (Solution on p. 567.)
18 min to seconds
Exercise 9.32
4 days to hours
Exercise 9.33 (Solution on p. 567.)
3 hr to days
Exercise 9.34
1
2 hr to days
Exercise 9.35 (Solution on p. 567.)
1
2 da to weeks
Exercise 9.36
3 17 wk to seconds

9.2.5.1 Exercises for Review

Exercise 9.37 (Solution on p. 567.)


(Section 2.5) Specify the digits by which 23,840 is divisible.
Exercise 9.38
(Section 4.5) Find 2 45 of 5 56 of 7 57 .
Exercise 9.39 (Solution on p. 567.)
() Convert 0.3 23 to a fraction.
Exercise 9.40
(Section 8.3) Use the clustering method to estimate the sum: 53 + 82 + 79 + 49.
Exercise 9.41 (Solution on p. 567.)
(Section 8.4) Use the distributive property to compute the product: 60 · 46.
517

9.3 The Metric System of Measurement3


9.3.1 Section Overview
• The Advantages of the Base Ten Number System
• Prexes
• Conversion from One Unit to Another Unit
• Conversion Table

9.3.2 The Advantages of the Base Ten Number System


The metric system of measurement takes advantage of our base ten number system. The advantage of the
metric system over the United States system is that in the metric system it is possible to convert from one
unit of measure to another simply by multiplying or dividing the given number by a power of 10. This means
we can make a conversion simply by moving the decimal point to the right or the left.

9.3.3 Prexes
Common units of measure in the metric system are the meter (for length), the liter (for volume), and the
gram (for mass). To each of the units can be attached a prex. The metric prexes along with their
meaning are listed below.
Metric Prexes

kilo  thousand
deci  tenth
hecto  hundred
centi  hundredth
deka  ten
milli  thousandth

For example, if length is being measured,

1 kilometer is equivalent to 1000 meters.


1 centimeter is equivalent to one hundredth of a meter.
1 millimeter is equivalent to one thousandth of a meter.

9.3.4 Conversion from One Unit to Another Unit


Let's note three characteristics of the metric system that occur in the metric table of measurements.

1. In each category, the prexes are the same.


2. We can move from a larger to a smaller unit of measure by moving the decimal point to the right.
3. We can move from a smaller to a larger unit of measure by moving the decimal point to the left.

The following table provides a summary of the relationship between the basic unit of measure (meter, gram,
liter) and each prex, and how many places the decimal point is moved and in what direction.

kilo hecto deka unit deci centi milli


3 This content is available online at <http://cnx.org/content/m35019/1.2/>.
518 CHAPTER 9. MEASUREMENT AND GEOMETRY

Basic Unit to Prex Move the Decimal Point


unit to deka 1 to 10 1 place to the left
unit to hector 1 to 100 2 places to the left
unit to kilo 1 to 1,000 3 places to the left
unit to deci 1 to 0.1 1 place to the right
unit to centi 1 to 0.01 2 places to the right
unit to milli 1 to 0.001 3 places to the right

Table 9.3

9.3.5 Conversion Table


Listed below, in the unit conversion table, are some of the common metric units of measure.

Unit Conversion Table


1 kilometer (km) = 1, 000 × 1 m
1,000 meters (m)
1 hectometer (hm) = 100 × 1 m
100 meters
Length
1 dekameter (dam) = 10 × 1m
10 meters
1 meter (m) 1 × 1m
1 decimeter (dm) = 10 meter
1
.1 × 1 m
1 centimeter (cm) = 100
1
meter .01 × 1 m
1 millimeter (mm) = .001 × 1 m
1,000 meter
1

1 kilogram (kg) = 1, 000 × 1 g


1,000 grams (g)
1 hectogram (hg) = 100 grams 100 × 1 g

Mass 1 dekagram (dag) = 10 grams 10 × 1 g


1 gram (g) 1 × 1g
1 decigram (dg) = 10 gram
1
.1 × 1 g
continued on next page
519

1 centigram (cg) = 100


1
gram .01 × 1 g
1 milligram (mg) = .001 × 1 g
1
1,000 gram
1 kiloliter (kL) = 1, 000 × 1 L
1, 000 liters (L)
1 hectoliter (hL) = 100 liters 100 × 1 L

Volume 1 dekaliter (daL) = 10 liters 10 × 1 L


1 liter (L) 1 × 1L
1 deciliter (dL) = 10 liter
1
.1 × 1 L
1 centiliter (cL) = 100
1
liter .01 × 1 L
1 milliliter (mL) = 1
1,000 liter .001 × 1 L
Time Same as the United States system

Table 9.4

Distinction Between Mass and Weight


There is a distinction between mass and weight. The weight of a body is related to gravity whereas the
mass of a body is not. For example, your weight on the earth is dierent than it is on the moon, but your
mass is the same in both places. Mass is a measure of a body's resistance to motion. The more massive a
body, the more resistant it is to motion. Also, more massive bodies weigh more than less massive bodies.

Converting Metric Units


To convert from one metric unit to another metric unit:

1. Determine the location of the original number on the metric scale (pictured in each of the following
examples).
2. Move the decimal point of the original number in the same direction and same number of places as is
necessary to move to the metric unit you wish to go to.

We can also convert from one metric unit to another using unit fractions. Both methods are shown in
Example 9.4 of Section 9.3.5.1 (Sample Set A).

9.3.5.1 Sample Set A

Example 9.4
Convert 3 kilograms to grams.

(a) 3 kg can be written as 3.0 kg. Then,

Thus, 3kg = 3,000 g.


(b) We can also use unit fractions to make this conversion. Since we are converting to grams, and
1, 000 g = 1 kg, we choose the unit fraction 1,000 g
1 kg since grams is in the numerator.
520 CHAPTER 9. MEASUREMENT AND GEOMETRY

3 kg = 3 kg · 1,000
1 kg
g

= 3 ) kg · 1,000 g
1 )kg
= 3 · 1, 000 g
= 3,000 g

Example 9.5
Convert 67.2 hectoliters to milliliters.

Thus, 67.2 hL = 6,720,000 mL.


Example 9.6
Convert 100.07 centimeters to meters.

Thus, 100.07 cm = 1.0007 m.


Example 9.7
Convert 0.16 milligrams to grams.

Thus, 0.16 mg = 0.00016 g.

9.3.5.2 Practice Set A

Exercise 9.42 (Solution on p. 567.)


Convert 411 kilograms to grams.
Exercise 9.43 (Solution on p. 567.)
Convert 5.626 liters to centiliters.
Exercise 9.44 (Solution on p. 568.)
Convert 80 milliliters to kiloliters.
Exercise 9.45 (Solution on p. 568.)
Convert 150 milligrams to centigrams.
Exercise 9.46 (Solution on p. 568.)
Convert 2.5 centimeters to meters.
521

9.3.6 Exercises
Make each conversion.
Exercise 9.47 (Solution on p. 568.)
87 m to cm
Exercise 9.48
905 L to mL
Exercise 9.49 (Solution on p. 568.)
16,005 mg to g
Exercise 9.50
48.66 L to dL
Exercise 9.51 (Solution on p. 568.)
11.161 kL to L
Exercise 9.52
521.85 cm to mm
Exercise 9.53 (Solution on p. 568.)
1.26 dag to dg
Exercise 9.54
99.04 dam to cm
Exercise 9.55 (Solution on p. 568.)
0.51 kL to daL
Exercise 9.56
0.17 kL to daL
Exercise 9.57 (Solution on p. 568.)
0.05 m to dm
Exercise 9.58
0.001 km to mm
Exercise 9.59 (Solution on p. 568.)
8.106 hg to cg
Exercise 9.60
17.0186 kL to mL
Exercise 9.61 (Solution on p. 568.)
3 cm to m
Exercise 9.62
9 mm to m
Exercise 9.63 (Solution on p. 568.)
4 g to mg
Exercise 9.64
2 L to kL
Exercise 9.65 (Solution on p. 568.)
6 kg to mg
Exercise 9.66
7 daL to mL
522 CHAPTER 9. MEASUREMENT AND GEOMETRY

9.3.6.1 Exercises for Review

Exercise 9.67 (Solution on p. 568.)


(Section 5.3) Find the value of 5
8 − 1
3 + 43 .
Exercise 9.68
(Section 7.3) Solve the proportion: 9
x = 27
60 .
Exercise 9.69 (Solution on p. 568.)
(Section 8.2) Use the method of rounding to estimate the sum: 8, 226 + 4, 118.
Exercise 9.70
(Section 8.3) Use the clustering method to estimate the sum: 87 + 121 + 118 + 91 + 92.
Exercise 9.71 (Solution on p. 568.)
(Section 9.2) Convert 3 in. to yd.

9.4 Simplication of Denominate Numbers4


9.4.1 Section Overview
• Converting to Multiple Units
• Adding and Subtracting Denominate Numbers
• Multiplying a Denominate Number by a Whole Number
• Dividing a Denominate Number by a Whole Number

9.4.2 Converting to Multiple Units


Denominate Numbers
Numbers that have units of measure associated with them are called denominate numbers. It is often
convenient, or even necessary, to simplify a denominate number.

Simplied Denominate Number


A denominate number is simplied when the number of standard units of measure associated With it does
not exceed the next higher type of unit.

The denominate number 55 min is simplied since it is smaller than the next higher type of unit, 1 hr. The
denominate number 65 min is not simplied since it is not smaller than the next higher type of unit, 1 hr.
The denominate number 65 min can be simplied to 1 hr 5 min. The denominate number 1 hr 5 min is
simplied since the next higher type of unit is day, and 1 hr does not exceed 1 day.

9.4.2.1 Sample Set A

Example 9.8
Simplify 19 in.

Since 12 in. = 1 ft, and 19 = 12 + 7,

19 in. = 12 in. + 7 in.


= 1 ft + 7 in.
= 1 ft 7 in.

4 This content is available online at <http://cnx.org/content/m35021/1.2/>.


523

Example 9.9
Simplify 4 gal 5 qt.

Since 4 qt = 1 gal, and 5 = 4 + 1,

4 gal 5 qt = 4 gal + 4 qt + 1qt


= 4 gal + 1 gal + 1 qt
= 5 gal + 1 qt
= 5 gal 1 qt
Example 9.10
Simplify 2 hr 75 min.

Since 60 min = 1 hr, and 75 = 60 + 15,

2 hr 75 min = 2 hr + 60 min + 15 min


= 2 hr + 1 hr + 15 min
= 3 hr + 15 min
= 3 hr 15 min
Example 9.11
Simplify 43  oz.

Since 8  oz = 1 c (1 cup), and 43 ÷ 8 = 5 R3,

43  oz = 40  oz + 3  oz
= 5 · 8  oz + 3  oz
= 5 · 1 c + 3  oz
= 5 c + 3  oz

But, 2 c = 1 pt and 5 ÷ 2 = 2 R1. So,

5 c + 3  oz = 2 · 2 c + 1 c + 3  oz
= 2 · 1 pt + 1 c + 3  oz
= 2 pt + 1 c + 3  oz

But, 2 pt = 1 qt, so

2 pt + 1 c + 3  oz = 1 qt 1 c 3  oz

9.4.2.2 Practice Set A

Simplify each denominate number. Refer to the conversion tables given in Section 9.2, if necessary.
Exercise 9.72 (Solution on p. 568.)
18 in.
Exercise 9.73 (Solution on p. 568.)
8 gal 9 qt
524 CHAPTER 9. MEASUREMENT AND GEOMETRY

Exercise 9.74 (Solution on p. 568.)


5 hr 80 min
Exercise 9.75 (Solution on p. 568.)
8 wk 11 da
Exercise 9.76 (Solution on p. 568.)
86 da

9.4.3 Adding and Subtracting Denominate Numbers


Adding and Subtracting Denominate Numbers
Denominate numbers can be added or subtracted by:

1. writing the numbers vertically so that the like units appear in the same column.
2. adding or subtracting the number parts, carrying along the unit.
3. simplifying the sum or dierence.

9.4.3.1 Sample Set B

Example 9.12
Add 6 ft 8 in. to 2 ft 9 in.

6 ft 8 in.
+2 ft 9 in.
8 ft 17 in. Simplify this denominate number.

Since 12 in. = 1 ft,

8 ft + 12 in. + 5 in. = 8 ft + 1 ft + 5 in.


= 9 ft + 5 in.
= 9 ft 5 in.
Example 9.13
Subtract 5 da 3 hr from 8 da 11 hr.

8 da 11 hr
−5 da 3 hr
3 da 8 hr
Example 9.14
Subtract 3 lb 14 oz from 5 lb 3 oz.

5 lb 3 oz
−3 lb 14 oz

We cannot directly subtract 14 oz from 3 oz, so we must borrow 16 oz from the pounds.
525

5 lb 3 oz = 5 lb + 3 oz
= 4 lb + 1 lb + 3 oz
= 4 lb + 16 oz + 3 oz (Since 1 lb = 16 oz.)
= 4 lb + 19 oz
= 4 lb 19 oz

4 lb 19 oz
−3 lb 14 oz
1 lb 5 oz
Example 9.15
Subtract 4 da 9 hr 21 min from 7 da 10 min.

7 da 0 hr 10 min
Borrow 1 da from the 7 da.
−4 da 9 hr 21 min

6 da 24 hr 10 min
Borrow 1 hr from the 24 hr.
−4 da 9 hr 21 min

6 da 23 hr 70 min
−4 da 9 hr 21 min
2 da 14 hr 49 min

9.4.3.2 Practice Set B

Perform each operation. Simplify when possible.


Exercise 9.77 (Solution on p. 568.)
Add 4 gal 3 qt to 1 gal 2 qt.
Exercise 9.78 (Solution on p. 568.)
Add 9 hr 48 min to 4 hr 26 min.
Exercise 9.79 (Solution on p. 568.)
Subtract 2 ft 5 in. from 8 ft 7 in.
Exercise 9.80 (Solution on p. 568.)
Subtract 15 km 460 m from 27 km 800 m.
Exercise 9.81 (Solution on p. 569.)
Subtract 8 min 35 sec from 12 min 10 sec.
Exercise 9.82 (Solution on p. 569.)
Add 4 yd 2 ft 7 in. to 9 yd 2 ft 8 in.
Exercise 9.83 (Solution on p. 569.)
Subtract 11 min 55 sec from 25 min 8 sec.
526 CHAPTER 9. MEASUREMENT AND GEOMETRY

9.4.4 Multiplying a Denominate Number by a Whole Number


Let's examine the repeated sum

4 ft 9 in. + 4 ft 9 in. + 4 ft 9 in. = 12 ft 27 in.


| {z }
3 times
Recalling that multiplication is a description of repeated addition, by the distributive property we have

3 (4 ft 9 in.) = 3 (4 ft + 9 in.)
= 3 · 4 ft + 3 · 9 in.
= 12 ft + 27 in. Now, 27 in. = 2 ft 3 in.
= 12 ft + 2 ft + 3 in.
= 14 ft + 3 in.
= 14 ft 3 in.

From these observations, we can suggest the following rule.

Multiplying a Denominate Number by a Whole Number


To multiply a denominate number by a whole number, multiply the number part of each unit by the whole
number and ax the unit to this product.

9.4.4.1 Sample Set C

Perform the following multiplications. Simplify if necessary.


Example 9.16
6 · (2 ft 4 in.) = 6 · 2 ft + 6 · 4 in.
= 12 ft + 24 in.

Since 3 ft = 1 yd and 12 in. = 1 ft,

12 ft + 24 in. = 4 yd + 2 ft
= 4 yd 2 ft
Example 9.17
8 · (5 hr 21 min 55 sec) = 8 · 5 hr + 8 · 21 min + 8 · 55 sec
= 40 hr + 168 min + 440sec
= 40 hr + 168 min + 7 min + 20 sec
= 40 hr + 175 min + 20 sec
= 40 hr + 2 hr + 55 min + 20 sec
= 42 hr + 55 min + 20 sec
= 24hr + 18hr + 55 min + 20 sec
= 1 da + 18 hr + 55 min + 20 sec
= 1 da 18 hr 55 min 20 sec
527

9.4.4.2 Practice Set C

Perform the following multiplications. Simplify.


Exercise 9.84 (Solution on p. 569.)
2 · (10 min)
Exercise 9.85 (Solution on p. 569.)
5 · (3 qt)
Exercise 9.86 (Solution on p. 569.)
4 · (5 ft 8 in.)
Exercise 9.87 (Solution on p. 569.)
10 · (2 hr 15 min 40 sec)

9.4.5 Dividing a Denominate Number by a Whole Number


Dividing a Denominate Number by a Whole Number
To divide a denominate number by a whole number, divide the number part of each unit by the whole
number beginning with the largest unit. Ax the unit to this quotient. Carry any remainder to the next
unit.

9.4.5.1 Sample Set D

Perform the following divisions. Simplify if necessary.


Example 9.18
(12 min 40 sec) ÷ 4

Thus (12 min 40 sec) ÷ 4 = 3 min 10 sec


Example 9.19
(5 yd 2 ft 9 in.) ÷ 3

Convert to feet: 2 yd 2 ft = 8 ft .
Convert to inches: 2 ft 9 in . = 33 in .

Thus (5 yd 2 ft 9 in.) ÷ 3 = 1 yd 2 ft 11 in.


528 CHAPTER 9. MEASUREMENT AND GEOMETRY

9.4.5.2 Practice Set D

Perform the following divisions. Simplify if necessary.


Exercise 9.88 (Solution on p. 569.)
(18 hr 36 min) ÷ 9
Exercise 9.89 (Solution on p. 569.)
(34 hr 8 min.) ÷ 8
Exercise 9.90 (Solution on p. 569.)
(13 yd 7 in.) ÷ 5
Exercise 9.91 (Solution on p. 569.)
(47 gal 2 qt 1 pt) ÷ 3

9.4.6 Exercises
For the following 15 problems, simplify the denominate numbers.
Exercise 9.92 (Solution on p. 569.)
16 in.
Exercise 9.93
19 ft
Exercise 9.94 (Solution on p. 569.)
85 min
Exercise 9.95
90 min
Exercise 9.96 (Solution on p. 569.)
17 da
Exercise 9.97
25 oz
Exercise 9.98 (Solution on p. 569.)
240 oz
Exercise 9.99
3,500 lb
Exercise 9.100 (Solution on p. 569.)
26 qt
Exercise 9.101
300 sec
Exercise 9.102 (Solution on p. 569.)
135 oz
Exercise 9.103
14 tsp
Exercise 9.104 (Solution on p. 569.)
18 pt
Exercise 9.105
3,500 m
Exercise 9.106 (Solution on p. 569.)
16,300 mL
For the following 15 problems, perform the indicated operations and simplify the answers if possible.
529

Exercise 9.107
Add 6 min 12 sec to 5 min 15 sec.
Exercise 9.108 (Solution on p. 569.)
Add 14 da 6 hr to 1 da 5 hr.
Exercise 9.109
Add 9 gal 3 qt to 2 gal 3 qt.
Exercise 9.110 (Solution on p. 569.)
Add 16 lb 10 oz to 42 lb 15 oz.
Exercise 9.111
Subtract 3 gal 1 qt from 8 gal 3 qt.
Exercise 9.112 (Solution on p. 569.)
Subtract 3 ft 10 in. from 5 ft 8 in.
Exercise 9.113
Subtract 5 lb 9 oz from 12 lb 5 oz.
Exercise 9.114 (Solution on p. 569.)
Subtract 10 hr 10 min from 11 hr 28 min.
Exercise 9.115
Add 3  oz 1 tbsp 2 tsp to 5  oz 1 tbsp 2 tsp.
Exercise 9.116 (Solution on p. 569.)
Add 4 da 7 hr 12 min to 1 da 8 hr 53 min.
Exercise 9.117
Subtract 5 hr 21 sec from 11 hr 2 min 14 sec.
Exercise 9.118 (Solution on p. 569.)
Subtract 6 T 1,300 lb 10 oz from 8 T 400 lb 10 oz.
Exercise 9.119
Subtract 15 mi 10 in. from 27 mi 800 ft 7 in.
Exercise 9.120 (Solution on p. 570.)
Subtract 3 wk 5 da 50 min 12 sec from 5 wk 6 da 20 min 5 sec.
Exercise 9.121
Subtract 3 gal 3 qt 1 pt 1 oz from 10 gal 2 qt 2 oz.

9.4.6.1 Exercises for Review

Exercise 9.122 (Solution on p. 570.)


(Section 4.6) Find the value: 5 2
+ 39
64 .

8
Exercise 9.123
(Section 5.4) Find the sum: 8 + 6 53 .
Exercise 9.124 (Solution on p. 570.)
(Section 6.3) Convert 2.05 11
1
to a fraction.
Exercise 9.125
(Section 7.4) An acid solution is composed of 3 parts acid to 7 parts water. How many parts of
acid are there in a solution that contains 126 parts water?
Exercise 9.126 (Solution on p. 570.)
(Section 9.3) Convert 126 kg to grams.
530 CHAPTER 9. MEASUREMENT AND GEOMETRY

9.5 Perimeter and Circumference of Geometric Figures5


9.5.1 Section Overview
• Polygons
• Perimeter
• Circumference/Diameter/Radius
• The Number π
• Formulas

9.5.2 Polygons
We can make use of conversion skills with denominate numbers to make measurements of geometric gures
such as rectangles, triangles, and circles. To make these measurements we need to be familiar with several
denitions.

Polygon
A polygon is a closed plane (at) gure whose sides are line segments (portions of straight lines).

Polygons

Not polygons

9.5.3 Perimeter
Perimeter
The perimeter of a polygon is the distance around the polygon.

To nd the perimeter of a polygon, we simply add up the lengths of all the sides.

9.5.3.1 Sample Set A

Find the perimeter of each polygon.


Example 9.20

5 This content is available online at <http://cnx.org/content/m35022/1.2/>.


531

Perimeter = 2 cm + 5 cm + 2 cm + 5 cm
= 14 cm
Example 9.21

Perimeter = 3.1 mm
4.2 mm
4.3 mm
1.52 mm
5.4 mm
+9.2 mm
27.72 mm
Example 9.22

Our rst observation is that three of the dimensions are missing. However, we can determine
the missing measurements using the following process. Let A, B, and C represent the missing
measurements. Visualize

A = 12m − 2m = 10m
B = 9m + 1m − 2m = 8m
C = 12m − 1m = 11m
532 CHAPTER 9. MEASUREMENT AND GEOMETRY

Perimeter = 8m
10 m
2m
2m
9m
11 m
1m
+ 1m
44 m

9.5.3.2 Practice Set A

Find the perimeter of each polygon.


Exercise 9.127 (Solution on p. 570.)

Exercise 9.128 (Solution on p. 570.)

Exercise 9.129 (Solution on p. 570.)

9.5.4 Circumference/Diameter/Radius
Circumference
The circumference of a circle is the distance around the circle.

Diameter
A diameter of a circle is any line segment that passes through the center of the circle and has its endpoints
on the circle.
533

Radius
A radius of a circle is any line segment having as its endpoints the center of the circle and a point on the
circle.
The radius is one half the diameter.

9.5.5 The Number π


The symbol π , read "pi," represents the nonterminating, nonrepeating decimal number 3.14159 . . . . This
number has been computed to millions of decimal places without the appearance of a repeating block of
digits.

For computational purposes, π is often approximated as 3.14. We will write π ≈ 3.14 to denote that π is
approximately equal to 3.14. The symbol "≈" means "approximately equal to."

9.5.6 Formulas
To nd the circumference of a circle, we need only know its diameter or radius. We then use a formula for
computing the circumference of the circle.

Formula
A formula is a rule or method for performing a task. In mathematics, a formula is a rule that directs us in
computations.

Formulas are usually composed of letters that represent important, but possibly unknown, quantities.

If C , d, and r represent, respectively, the circumference, diameter, and radius of a circle, then the following
two formulas give us directions for computing the circumference of the circle.
Circumference Formulas

1. C = πd or C ≈ (3.14) d
2. C = 2πr or C ≈ 2 (3.14) r

9.5.6.1 Sample Set B

Example 9.23
Find the exact circumference of the circle.

Use the formula C = πd.


534 CHAPTER 9. MEASUREMENT AND GEOMETRY

C = π · 7in.

By commutativity of multiplication,

C = 7in. · π

C = 7π in., exactly

This result is exact since π has not been approximated.


Example 9.24
Find the approximate circumference of the circle.

Use the formula C = πd.

C ≈ (3.14) (6.2)

C ≈ 19.648 mm

This result is approximate since π has been approximated by 3.14.


Example 9.25
Find the approximate circumference of a circle with radius 18 inches.

Since we're given that the radius, r, is 18 in., we'll use the formula C = 2πr.

C ≈ (2) (3.14) (18 in.)

C ≈ 113.04 in.
Example 9.26
Find the approximate perimeter of the gure.

We notice that we have two semicircles (half circles).

The larger radius is 6.2 cm.

The smaller radius is 6.2 cm - 2.0 cm = 4.2 cm.

The width of the bottom part of the rectangle is 2.0 cm.


535

Perimeter = 2.0 cm
5.1 cm
2.0 cm
5.1 cm
(0.5) · (2) · (3.14) · (6.2 cm) Circumference of outer semicircle.
+ (0.5) · (2) · (3.14) · (4.2 cm) Circumference of inner semicircle
6.2 cm − 2.0 cm = 4.2 cm
The 0.5 appears because we want the
perimeter of only half a circle.

Perimeter ≈ 2.0 cm
5.1 cm
2.0 cm
5.1 cm
19.468 cm
+13.188 cm
48.856 cm

9.5.6.2 Practice Set B

Exercise 9.130 (Solution on p. 570.)


Find the exact circumference of the circle.

Exercise 9.131 (Solution on p. 570.)


Find the approximate circumference of the circle.

Exercise 9.132 (Solution on p. 570.)


Find the approximate circumference of the circle with radius 20.1 m.
Exercise 9.133 (Solution on p. 570.)
Find the approximate outside perimeter of
536 CHAPTER 9. MEASUREMENT AND GEOMETRY

9.5.7 Exercises
Find each perimeter or approximate circumference. Use π = 3.14.
Exercise 9.134 (Solution on p. 570.)

Exercise 9.135

Exercise 9.136 (Solution on p. 570.)

Exercise 9.137

Exercise 9.138 (Solution on p. 570.)

Exercise 9.139
537

Exercise 9.140 (Solution on p. 570.)

Exercise 9.141

Exercise 9.142 (Solution on p. 570.)

Exercise 9.143

Exercise 9.144 (Solution on p. 570.)

Exercise 9.145
538 CHAPTER 9. MEASUREMENT AND GEOMETRY

Exercise 9.146 (Solution on p. 570.)

Exercise 9.147

Exercise 9.148 (Solution on p. 570.)

Exercise 9.149
539

Exercise 9.150 (Solution on p. 570.)

Exercise 9.151

Exercise 9.152 (Solution on p. 570.)

Exercise 9.153
540 CHAPTER 9. MEASUREMENT AND GEOMETRY

9.5.7.1 Exercises for Review

Exercise 9.154 q (Solution on p. 570.)


(Section 4.5) Find the value of 2 13 · 8
10 16 .
9

Exercise 9.155
(Section 5.3) Find the value of 15
8 7
+ 10 21 .
+ 60
Exercise 9.156 (Solution on p. 570.)
(Section 6.9) Convert 7
8 to a decimal.
Exercise 9.157
(Section 9.2) What is the name given to a quantity that is used as a comparison to determine the
measure of another quantity?
Exercise 9.158 (Solution on p. 570.)
(Section 9.4) Add 42 min 26 sec to 53 min 40 sec and simplify the result.

9.6 Area and Volume of Geometric Figures and Objects6


9.6.1 Section Overview
• The Meaning and Notation for Area
• Area Formulas
• Finding Areas of Some Common Geometric Figures
• The Meaning and Notation for Volume
• Volume Formulas
• Finding Volumes of Some Common Geometric Objects

Quite often it is necessary to multiply one denominate number by another. To do so, we multiply the number
parts together and the unit parts together. For example,

8 in. · 8 in. = 8 · 8 · in. · in.


= 64 in.2

4 mm · 4 mm · 4 mm = 4 · 4 · 4 · mm · mm · mm
= 64 mm3

Sometimes the product of units has a physical meaning. In this section, we will examine the meaning of the
2 3
products (length unit) and (length unit) .

9.6.2 The Meaning and Notation for Area


2
The product (length unit) · (length unit) = (length unit) , or, square length unit (sq length unit), can be
interpreted physically as the area of a surface.

Area
The area of a surface is the amount of square length units contained in the surface.

For example, 3 sq in. means that 3 squares, 1 inch on each side, can be placed precisely on some surface.
(The squares may have to be cut and rearranged so they match the shape of the surface.)
6 This content is available online at <http://cnx.org/content/m35023/1.2/>.
541

We will examine the area of the following geometric gures.

9.6.3 Area Formulas


We can determine the areas of these geometric gures using the following formulas.

Figure Area Formula Statement


Triangle AT = 1
2 ·b·h Area of a triangle is one
half the base times the
height.

Rectangle AR = l · w Area of a rectangle is


the length times the
width.

continued on next page


542 CHAPTER 9. MEASUREMENT AND GEOMETRY

Parallelogram AP = b · h Area of a parallelogram


is base times the height.

Trapezoid ATrap = 1
2 · (b1 + b2 ) · h Area of a trapezoid is
one half the sum of
the two bases times the
height.
Circle AC = πr2 Area of a circle is π
times the square of the
radius.

Table 9.5

9.6.4 Finding Areas of Some Common Geometric Figures


9.6.4.1 Sample Set A

Example 9.27
Find the area of the triangle.

1
AT = 2 ·b·h
= 1
2 · 20 · 6 sq ft
= 10 · 6 sq ft
= 60 sq ft
= 60 ft2

The area of this triangle is 60 sq ft, which is often written as 60 ft2 .


Example 9.28
Find the area of the rectangle.

Let's rst convert 4 ft 2 in. to inches. Since we wish to convert to inches, we'll use the unit fraction
12 in. since it has inches in the numerator. Then,
1 ft

4 ft = 4 ft · 12 in.
1 1 ft
4) ft 12 in.
= 1 · 1) ft

= 48 in.
543

Thus, 4 ft 2 in. = 48 in. + 2 in. = 50 in.

AR = l·w
= 50 in. · 8 in.
= 400 sq in.

The area of this rectangle is 400 sq in.


Example 9.29
Find the area of the parallelogram.

AP = b·h
= 10.3 cm · 6.2 cm
= 63.86 sq cm

The area of this parallelogram is 63.86 sq cm.


Example 9.30
Find the area of the trapezoid.

1
ATrap = 2 · (b1 + b2 ) · h
= 1
2 · (14.5 mm, +, 20.4 mm) · (4.1 mm)
= 1
2 · (34.9 mm) · (4.1 mm)
= 1
2 · (143.09 sq mm)
= 71.545 sq mm

The area of this trapezoid is 71.545 sq mm.


Example 9.31
Find the approximate area of the circle.
544 CHAPTER 9. MEASUREMENT AND GEOMETRY

Ac = π · r2
2
≈ (3.14) · (16.8 ft)
≈ (3.14) · (282.24 sq ft)
≈ 888.23 sq ft

The area of this circle is approximately 886.23 sq ft.

9.6.4.2 Practice Set A

Find the area of each of the following geometric gures.


Exercise 9.159 (Solution on p. 570.)

Exercise 9.160 (Solution on p. 571.)

Exercise 9.161 (Solution on p. 571.)

Exercise 9.162 (Solution on p. 571.)

Exercise 9.163 (Solution on p. 571.)


545

Exercise 9.164 (Solution on p. 571.)

9.6.5 The Meaning and Notation for Volume


3
The product (length unit) (length unit) (length unit) = (length unit) , or cubic length unit (cu length unit),
can be interpreted physically as the volume of a three-dimensional object.

Volume
The volume of an object is the amount of cubic length units contained in the object.

For example, 4 cu mm means that 4 cubes, 1 mm on each side, would precisely ll some three-dimensional
object. (The cubes may have to be cut and rearranged so they match the shape of the object.)
546 CHAPTER 9. MEASUREMENT AND GEOMETRY

9.6.6 Volume Formulas

Figure Volume Formula Statement


VR = l·w·h
Rectangular solid The volume of a rectan-
= (area of base) · (height
gular )solid is the length
times the width times
the height.

Sphere VS = 4
3 · π · r3 The volume of a sphere
is 34 times π times the
cube of the radius.

continued on next page


547

VCyl = π · r2 · h
Cylinder The volume of a cylin-
= (area of base) · der
(height
is π )times the square
of the radius times the
height.

1
Vc = · π · r2 · h
Cone 3
The volume of a cone
= (area of base) · (height )
is 31 times π times the
square of the radius
times the height.

Table 9.6

9.6.7 Finding Volumes of Some Common Geometric Objects


9.6.7.1 Sample Set B

Example 9.32
Find the volume of the rectangular solid.

VR = l·w·h
= 9 in. · 10 in. · 3 in.
= 270 cu in.
= 270 in.3

The volume of this rectangular solid is 270 cu in.


Example 9.33
Find the approximate volume of the sphere.
548 CHAPTER 9. MEASUREMENT AND GEOMETRY

4
VS = 3 · π · r3
3
4
· (3.14) · (6 cm)

≈ 3
4
· (3.14) · (216 cu cm)

≈ 3
≈ 904.32 cu cm

The approximate volume of this sphere is 904.32 cu cm, which is often written as 904.32 cm3 .
Example 9.34
Find the approximate volume of the cylinder.

VCyl = π · r2 · h
2
≈ (3.14) · (4.9 ft) · (7.8 ft)
≈ (3.14) · (24.01 sq ft) · (7.8 ft)
≈ (3.14) · (187.278 cu ft)
≈ 588.05292 cu ft

The volume of this cylinder is approximately 588.05292 cu ft. The volume is approximate because
we approximated π with 3.14.
Example 9.35
Find the approximate volume of the cone. Round to two decimal places.

1
Vc = 3 · π · r2 · h
2
1
· (3.14) · (2 mm) · (5 mm)

≈ 3
1
· (3.14) · (4 sq mm) · (5 mm)

≈ 3

3 · (3.14) · (20 cu mm)


1


≈ 20.93 cu mm
≈ 20.93 cu mm
549

The volume of this cone is approximately 20.93 cu mm. The volume is approximate because we
approximated π with 3.14.

9.6.7.2 Practice Set B

Find the volume of each geometric object. If π is required, approximate it with 3.14 and nd the approximate
volume.
Exercise 9.165 (Solution on p. 571.)

Exercise 9.166 (Solution on p. 571.)


Sphere

Exercise 9.167 (Solution on p. 571.)

Exercise 9.168 (Solution on p. 571.)

9.6.8 Exercises
Find each indicated measurement.
550 CHAPTER 9. MEASUREMENT AND GEOMETRY

Exercise 9.169 (Solution on p. 571.)


Area

Exercise 9.170
Area

Exercise 9.171 (Solution on p. 571.)


Area

Exercise 9.172
Area

Exercise 9.173 (Solution on p. 571.)


Area

Exercise 9.174
Area

Exercise 9.175 (Solution on p. 571.)


Exact area
551

Exercise 9.176
Approximate area

Exercise 9.177 (Solution on p. 571.)


Area

Exercise 9.178
Area

Exercise 9.179 (Solution on p. 571.)


Approximate area

Exercise 9.180
Exact area

Exercise 9.181 (Solution on p. 571.)


Approximate area
552 CHAPTER 9. MEASUREMENT AND GEOMETRY

Exercise 9.182
Exact area

Exercise 9.183 (Solution on p. 571.)


Approximate area

Exercise 9.184
Area

Exercise 9.185 (Solution on p. 571.)


Approximate area

Exercise 9.186
Volume

Exercise 9.187 (Solution on p. 571.)


Volume
553

Exercise 9.188
Exact volume

Exercise 9.189 (Solution on p. 571.)


Approximate volume

Exercise 9.190
Approximate volume

Exercise 9.191 (Solution on p. 571.)


Exact volume

Exercise 9.192
Approximate volume
554 CHAPTER 9. MEASUREMENT AND GEOMETRY

Exercise 9.193 (Solution on p. 571.)


Approximate volume

Exercise 9.194
Approximate volume

9.6.8.1 Exercises for Review

Exercise 9.195 (Solution on p. 571.)


(Section 1.2) In the number 23,426, how many hundreds are there?
Exercise 9.196
(Section 3.4) List all the factors of 32.
Exercise 9.197 (Solution on p. 571.)
(Section 5.4) Find the value of 4 34 − 3 65 + 1 23 .
Exercise 9.198
5+ 13
(Section 5.6) Find the value of 2
2+ 15
.
Exercise 9.199 (Solution on p. 571.)
(Section 9.5) Find the perimeter.
555

9.7 Summary of Key Concepts7


9.7.1 Summary of Key Concepts
Measurement (Section 9.2)
Measurement is comparison to some standard.

Standard Unit of Measure (Section 9.2)


A quantity that is used for comparison is called a standard unit of measure.

Two Types of Measurement Systems (Section 9.2)


There are two major types of measurement systems in use today. They are the United States system and
the metric system.

Unit Fraction (Section 9.2)


A unit fraction is a fraction that has a value of 1. Unit fractions can be used to convert from one unit of
measure to another.

Meter, Liter, Gram, and associated prexes (Section 9.3)


Common units of measure in the metric system are the meter (m), for length, the liter (L), for volume,
and the gram (g), for mass. To each of these units, a prex can be attached.

kilo  thousand
deci  tenth
hecto  hundred
centi  hundredth
deka  ten
milli  thousandth

Metric Conversions (Section 9.3)


To convert from one metric unit to another:

1. Determine the location of the original number on the metric scale.


2. Move the decimal point of the original number in the same direction and the same number of places
as is necessary to move to the metric unit you wish to convert to.

Denominate Numbers (Section 9.4)


Numbers that have units of measure associated with them are denominate numbers. The number 25 mg
is a denominate number since the mg unit is associated with the pure number 25. The number 82 is not a
denominate number since it has no unit of measure associated with it.

Simplied Denominate Number (Section 9.4)


A denominate number is simplied when the number of standard units of measure associated with it does
not exceed the next higher type of unit. 55 min is simplied, whereas 65 min is not simplied

Addition and Subtraction of Denominate Numbers (Section 9.4)


Denominate numbers can be added or subtracted by

1. writing the numbers vertically so that the like units appear in the same column.
2. adding or subtracting the number parts, carrying along the unit.
3. simplifying the sum or dierence.
7 This content is available online at <http://cnx.org/content/m35024/1.2/>.
556 CHAPTER 9. MEASUREMENT AND GEOMETRY

Multiplying a Denominate Number by a Whole Number (Section 9.4)


To multiply a denominate number by a whole number, multiply the number part of each unit by the whole
number and ax the unit to the product.

Dividing a Denominate Number by a Whole Number (Section 9.4)


To divide a denominate number by a whole number, divide the number part of each unit by the whole
number beginning with the largest unit. Ax the unit to this quotient. Carry the remainder to the next
unit.

Polygon (Section 9.5)


A polygon is a closed plane (at) gure whose sides are line segments (portions of straight lines).

Perimeter (Section 9.5)


The perimeter of a polygon is the distance around the polygon.

Circumference, Diameter, Radius (Section 9.5)


The circumference of a circle is the distance around the circle. The diameter of a circle is any line segment
that passes through the center of the circle and has its endpoints on the circle. The radius of a circle is one
half the diameter of the circle.

The number π (Section 9.5)


The symbol π , read "pi," represents the nonterminating, nonrepeating decimal number 3.14159... . For
computational purposes, π is often approximated by the number 3.14.

Formula (Section 9.5)


A formula is a rule for performing a task. In mathematics, a formula is a rule that directs us in computations.

Circumference Formulas (Section 9.5)


C =π·d C ≈ (3.14) d
C =2·π·r C ≈ 2 (3.14) r

Area (Section 9.6)


The area of a surface is the amount of square length units contained in the surface.

Volume (Section 9.6)


The volume of an object is a measure of the amount of cubic length units contained in the object.

Area Formulas (Section 9.6)


Triangle: A = 21 · b · h
Rectangle: A = l · w
Parallelogram: A = b · h
Trapezoid: A = 12 · (b1 + b2 ) · h
Circle: A = π · r 2

Volume Formulas (Section 9.6)


Rectangle solid: V = l · w · h
Sphere: V = 43 · π · r 3
Cylinder: V = π · r 2 · h
Cone: V = 13 · π · r 2 · h
557

9.8 Exercise Supplement8


9.8.1 Exercise Supplement
9.8.1.1 Measurement and the United States System (Section 9.2)

Exercise 9.200 (Solution on p. 572.)


What is measurement?
For problems 2-6, make each conversion. Use the conversion table given in Section 9.1.
Exercise 9.201
9 ft= yd
Exercise 9.202 (Solution on p. 572.)
32 oz= lb
Exercise 9.203
1,500 mg = g
Exercise 9.204 (Solution on p. 572.)
12,000 lb = T
Exercise 9.205
5,280 ft= mi
For problems 7-23, make each conversion.
Exercise 9.206 (Solution on p. 572.)
23 yd to ft
Exercise 9.207
2 21 mi to yd
Exercise 9.208 (Solution on p. 572.)
8 in. to ft
Exercise 9.209
51 in. to mi
Exercise 9.210 (Solution on p. 572.)
3 qt to pt
Exercise 9.211
8 lb to oz
Exercise 9.212 (Solution on p. 572.)
5 cups to tbsp
Exercise 9.213
9 da to hr
Exercise 9.214 (Solution on p. 572.)
3 12 min to sec
Exercise 9.215
3
4 wk to min

8 This content is available online at <http://cnx.org/content/m35025/1.2/>.


558 CHAPTER 9. MEASUREMENT AND GEOMETRY

9.8.1.2 The Metric System of Measurement (Section 9.3)

Exercise 9.216 (Solution on p. 572.)


250 mL to L
Exercise 9.217
18.57 cm to m
Exercise 9.218 (Solution on p. 572.)
0.01961 kg to mg
Exercise 9.219
52,211 mg to kg
Exercise 9.220 (Solution on p. 572.)
54.006 dag to g
Exercise 9.221
1.181 hg to mg
Exercise 9.222 (Solution on p. 572.)
3.5 kL to mL

9.8.1.3 Simplication of Denominate Numbers (Section 9.4)

For problems 24-31, perform the indicated operations. Simplify, if possible.


Exercise 9.223
Add 8 min 50 sec to 5 min 25 sec.
Exercise 9.224 (Solution on p. 572.)
Add 3 wk 3 da to 2 wk 5 da
Exercise 9.225
Subtract 4 gal 3 qt from 5 gal 2 qt.
Exercise 9.226 (Solution on p. 572.)
Subtract 2 gal 3 qt 1pt from 8 gal 2 qt.
Exercise 9.227
Subtract 5 wk 4 da 21 hr from 12 wk 3 da 14 hr.
Exercise 9.228 (Solution on p. 572.)
Subtract 2 T 1,850 lb from 10 T 1,700 lb.
Exercise 9.229
Subtract the sum of 2 wk 3 da 15 hr and 5 wk 2 da 9 hr from 10 wk.
Exercise 9.230 (Solution on p. 572.)
Subtract the sum of 20 hr 15 min and 18 hr 18 min from the sum of 8 da 1 hr 16 min 5 sec.
For problems 32-43, simplify, if necessary.
Exercise 9.231
18 in.
Exercise 9.232 (Solution on p. 572.)
4 ft
Exercise 9.233
23 da
Exercise 9.234 (Solution on p. 572.)
3,100 lb
559

Exercise 9.235
135 min
Exercise 9.236 (Solution on p. 572.)
4 tsp
Exercise 9.237
10  oz
Exercise 9.238 (Solution on p. 572.)
7 pt
Exercise 9.239
9 qt
Exercise 9.240 (Solution on p. 572.)
2,300 mm
Exercise 9.241
14,780 mL
Exercise 9.242 (Solution on p. 572.)
1,050 m

9.8.1.4 Perimeter, Circumference, Area and Volume of Geometric Figures and Objects (Sec-
tion 9.5,Section 9.6)

For problems 44-58, nd the perimeter, circumference, area or volume.


Exercise 9.243
Perimeter, area

Exercise 9.244 (Solution on p. 572.)


Approximate circumference

Exercise 9.245
Approximate volume
560 CHAPTER 9. MEASUREMENT AND GEOMETRY

Exercise 9.246 (Solution on p. 572.)


Approximate volume

Exercise 9.247
Exact area

Exercise 9.248 (Solution on p. 572.)


Exact area

Exercise 9.249
Exact volume
561

Exercise 9.250 (Solution on p. 573.)


Approximate volume

Exercise 9.251
Area

Exercise 9.252 (Solution on p. 573.)


Volume

Exercise 9.253
Exact area

Exercise 9.254 (Solution on p. 573.)


Approximate area
562 CHAPTER 9. MEASUREMENT AND GEOMETRY

Exercise 9.255
Exact area

Exercise 9.256 (Solution on p. 573.)


Approximate area

Exercise 9.257
Approximate area

9.9 Prociency Exam9


9.9.1 Prociency Exam
Exercise 9.258 (Solution on p. 573.)
(Section 9.2) The process of determining, by comparison to some standard, the size of something
is called .
9 This content is available online at <http://cnx.org/content/m35026/1.2/>.
563

For problems 2-9, make each conversion.


Exercise 9.259 (Solution on p. 573.)
(Section 9.2) 14 yards to feet
Exercise 9.260 (Solution on p. 573.)
(Section 9.2) 51 feet to inches
Exercise 9.261 (Solution on p. 573.)
(Section 9.2) 1
3 yard to feet
Exercise 9.262 (Solution on p. 573.)
(Section 9.2) 2 14 minutes to seconds
Exercise 9.263 (Solution on p. 573.)
(Section 9.3) 8,500 mg to cg
Exercise 9.264 (Solution on p. 573.)
(Section 9.3) 5.8623 L to kL
Exercise 9.265 (Solution on p. 573.)
(Section 9.3) 213.1062 mm to m
Exercise 9.266 (Solution on p. 573.)
(Section 9.3) 100,001 kL to mL
For problems 10-13, simplify each number.
Exercise 9.267 (Solution on p. 573.)
(Section 9.4) 23 da
Exercise 9.268 (Solution on p. 573.)
(Section 9.4) 88 ft
Exercise 9.269 (Solution on p. 573.)
(Section 9.4) 4216 lb
Exercise 9.270 (Solution on p. 573.)
(Section 9.4) 7 qt
For problems 14-18, perform the indicated operations. Simplify answers if possible.
Exercise 9.271 (Solution on p. 573.)
(Section 9.4) Add 6 wk 3 da to 2 wk 2 da.
Exercise 9.272 (Solution on p. 573.)
(Section 9.4) Add 9 gal 3 qt to 4 gal 3 qt.
Exercise 9.273 (Solution on p. 573.)
(Section 9.4) Subtract 3 yd 2 ft 5 in. from 5 yd 8 ft 2 in.
Exercise 9.274 (Solution on p. 573.)
(Section 9.4) Subtract 2 hr 50 min 12 sec from 3 hr 20 min 8 sec.
Exercise 9.275 (Solution on p. 573.)
(Section 9.4) Subtract the sum of 3 wk 6 da and 2 wk 3 da from 10 wk.
For problems 19-30, nd either the perimeter, circumference, area, or volume.
Exercise 9.276 (Solution on p. 573.)
(Section 9.5) Perimeter
564 CHAPTER 9. MEASUREMENT AND GEOMETRY

Exercise 9.277 (Solution on p. 573.)


(Section 9.5) Perimeter

Exercise 9.278 (Solution on p. 573.)


(Section 9.5) Approximate circumference

Exercise 9.279 (Solution on p. 574.)


(Section 9.5) Approximate perimeter

Exercise 9.280 (Solution on p. 574.)


(Section 9.6) Area

Exercise 9.281 (Solution on p. 574.)


(Section 9.6) Approximate area
565

Exercise 9.282 (Solution on p. 574.)


(Section 9.6) Approximate area

Exercise 9.283 (Solution on p. 574.)


(Section 9.6) Area

Exercise 9.284 (Solution on p. 574.)


(Section 9.6) Exact area

Exercise 9.285 (Solution on p. 574.)


(Section 9.6) Approximate volume

Exercise 9.286 (Solution on p. 574.)


(Section 9.6) Exact volume
566 CHAPTER 9. MEASUREMENT AND GEOMETRY

Exercise 9.287 (Solution on p. 574.)


(Section 9.6) Approximate volume
567

Solutions to Exercises in Chapter 9


Solution to Exercise 9.1 (p. 514)
6 yd
Solution to Exercise 9.2 (p. 515)
10,560 ft
Solution to Exercise 9.3 (p. 515)
8.67 yd
Solution to Exercise 9.4 (p. 515)
18 pt
Solution to Exercise 9.5 (p. 515)
0.87 hr
Solution to Exercise 9.6 (p. 515)
2.45 wk
Solution to Exercise 9.7 (p. 515)
42 feet
Solution to Exercise 9.9 (p. 515)
506,880 inches
Solution to Exercise 9.11 (p. 515)
1.5 feet
Solution to Exercise 9.13 (p. 515)
0.14 yard
Solution to Exercise 9.15 (p. 515)
0.00 miles (to two decimal places)
Solution to Exercise 9.17 (p. 515)
6 pints
Solution to Exercise 9.19 (p. 515)
192,000 ounces
Solution to Exercise 9.21 (p. 515)
937.5 pounds
Solution to Exercise 9.23 (p. 516)
27 teaspoons
Solution to Exercise 9.25 (p. 516)
80 uid ounces
Solution to Exercise 9.27 (p. 516)
0.16 quart
Solution to Exercise 9.29 (p. 516)
480 teaspoons
Solution to Exercise 9.31 (p. 516)
1,080 seconds
Solution to Exercise 9.33 (p. 516)
1
8 = 0.125 day
Solution to Exercise 9.35 (p. 516)
14 = 0.0714 week
1

Solution to Exercise 9.37 (p. 516)


1,2,4,5,8
Solution to Exercise 9.39 (p. 516)
11
30
Solution to Exercise 9.41 (p. 516)
60 (50 − 4) = 3, 000 − 240 = 2, 760
Solution to Exercise 9.42 (p. 520)
411,000 g
568 CHAPTER 9. MEASUREMENT AND GEOMETRY

Solution to Exercise 9.43 (p. 520)


562.6 cL
Solution to Exercise 9.44 (p. 520)
0.00008 kL
Solution to Exercise 9.45 (p. 520)
15 cg
Solution to Exercise 9.46 (p. 520)
0.025 m
Solution to Exercise 9.47 (p. 521)
8,700 cm
Solution to Exercise 9.49 (p. 521)
16.005 g
Solution to Exercise 9.51 (p. 521)
11,161 L
Solution to Exercise 9.53 (p. 521)
126 dg
Solution to Exercise 9.55 (p. 521)
5.1 daL
Solution to Exercise 9.57 (p. 521)
0.5 dm
Solution to Exercise 9.59 (p. 521)
81,060 cg
Solution to Exercise 9.61 (p. 521)
0.03 m
Solution to Exercise 9.63 (p. 521)
4,000 mg
Solution to Exercise 9.65 (p. 521)
6,000,000 mg
Solution to Exercise 9.67 (p. 522)
25 1
24 = 1 24
Solution to Exercise 9.69 (p. 522)
12,300 (12,344)
Solution to Exercise 9.71 (p. 522)
0.083 yard
Solution to Exercise 9.72 (p. 523)
1 ft 6 in.
Solution to Exercise 9.73 (p. 523)
10 gal 1 qt
Solution to Exercise 9.74 (p. 524)
6 hr 20 min
Solution to Exercise 9.75 (p. 524)
9 wk 4 da
Solution to Exercise 9.76 (p. 524)
12 wk 2 da
Solution to Exercise 9.77 (p. 525)
6 gal 1 qt
Solution to Exercise 9.78 (p. 525)
14 hr 14 min
Solution to Exercise 9.79 (p. 525)
6 ft 2in.
569

Solution to Exercise 9.80 (p. 525)


12 km 340 m
Solution to Exercise 9.81 (p. 525)
3 min 35 sec
Solution to Exercise 9.82 (p. 525)
14 yd 2 ft 3 in
Solution to Exercise 9.83 (p. 525)
13 min 13 sec
Solution to Exercise 9.84 (p. 527)
20 min
Solution to Exercise 9.85 (p. 527)
15 qt = 3 gal 3 qt
Solution to Exercise 9.86 (p. 527)
20 ft 32 in. = 7 yd 1 ft 8 in.
Solution to Exercise 9.87 (p. 527)
20 hr 150 min 400 sec = 22 hr 36 min 40 sec
Solution to Exercise 9.88 (p. 528)
2 hr 4 min
Solution to Exercise 9.89 (p. 528)
4 hr 16 min
Solution to Exercise 9.90 (p. 528)
2 yd 1 ft 11 in
Solution to Exercise 9.91 (p. 528)
15 gal 3 qt 1 pt
Solution to Exercise 9.92 (p. 528)
1 foot 4 inches
Solution to Exercise 9.94 (p. 528)
1 hour 25 minutes
Solution to Exercise 9.96 (p. 528)
2 weeks 3 days
Solution to Exercise 9.98 (p. 528)
15 pounds
Solution to Exercise 9.100 (p. 528)
6 gallons 2 quarts
Solution to Exercise 9.102 (p. 528)
8 pounds 7 ounces
Solution to Exercise 9.104 (p. 528)
2 gallons 1 quart
Solution to Exercise 9.106 (p. 528)
16 liters 300 milliliters (or 1daL 6 L 3dL)
Solution to Exercise 9.108 (p. 529)
15 days 11 hours
Solution to Exercise 9.110 (p. 529)
59 pounds 9 ounces
Solution to Exercise 9.112 (p. 529)
1 foot 10 inches
Solution to Exercise 9.114 (p. 529)
1 hour 18 minutes
Solution to Exercise 9.116 (p. 529)
5 days 16 hours 5 minutes
570 CHAPTER 9. MEASUREMENT AND GEOMETRY

Solution to Exercise 9.118 (p. 529)


1 ton 1,100 pounds (or 1T 1,100 lb)
Solution to Exercise 9.120 (p. 529)
2 weeks 23 hours 29 minutes 53 seconds
Solution to Exercise 9.122 (p. 529)
1
Solution to Exercise 9.124 (p. 529)
14
2 275
Solution to Exercise 9.126 (p. 529)
126,000 g
Solution to Exercise 9.127 (p. 532)
20 ft
Solution to Exercise 9.128 (p. 532)
26.8 m
Solution to Exercise 9.129 (p. 532)
49.89 mi
Solution to Exercise 9.130 (p. 535)
9.1π in.
Solution to Exercise 9.131 (p. 535)
5.652 mm
Solution to Exercise 9.132 (p. 535)
126.228 m
Solution to Exercise 9.133 (p. 535)
41.634 mm
Solution to Exercise 9.134 (p. 536)
21.8 cm
Solution to Exercise 9.136 (p. 536)
38.14 inches
Solution to Exercise 9.138 (p. 536)
0.86 m
Solution to Exercise 9.140 (p. 537)
87.92 m
Solution to Exercise 9.142 (p. 537)
16.328 cm
Solution to Exercise 9.144 (p. 537)
0.0771 cm
Solution to Exercise 9.146 (p. 538)
120.78 m
Solution to Exercise 9.148 (p. 538)
21.71 inches
Solution to Exercise 9.150 (p. 539)
43.7 mm
Solution to Exercise 9.152 (p. 539)
45.68 cm
Solution to Exercise 9.154 (p. 540)
8.5 or 17
2 or 8 2
1

Solution to Exercise 9.156 (p. 540)


0.875
Solution to Exercise 9.158 (p. 540)
1 hour 36 minutes 6 seconds
571

Solution to Exercise 9.159 (p. 544)


36 sq cm
Solution to Exercise 9.160 (p. 544)
37.503 sq mm
Solution to Exercise 9.161 (p. 544)
13.26 sq in.
Solution to Exercise 9.162 (p. 544)
367.5 sq mi
Solution to Exercise 9.163 (p. 544)
452.16 sq ft
Solution to Exercise 9.164 (p. 545)
44.28 sq cm
Solution to Exercise 9.165 (p. 549)
21 cu in.
Solution to Exercise 9.166 (p. 549)
904.32 cu ft
Solution to Exercise 9.167 (p. 549)
157 cu m
Solution to Exercise 9.168 (p. 549)
0.00942 cu in.
Solution to Exercise 9.169 (p. 549)
16 sq m
Solution to Exercise 9.171 (p. 550)
1.21 sq mm
Solution to Exercise 9.173 (p. 550)
18 sq in.
Solution to Exercise 9.175 (p. 550)
(60.5π + 132) sq ft
Solution to Exercise 9.177 (p. 551)
40.8 sq in.
Solution to Exercise 9.179 (p. 551)
31.0132 sq in.
Solution to Exercise 9.181 (p. 551)
158.2874 sq mm
Solution to Exercise 9.183 (p. 552)
64.2668 sq in.
Solution to Exercise 9.185 (p. 552)
43.96 sq ft
Solution to Exercise 9.187 (p. 552)
512 cu cm
Solution to Exercise 9.189 (p. 553)
11.49 cu cm
Solution to Exercise 9.191 (p. 553)
1024 π cu ft
3
Solution to Exercise 9.193 (p. 554)
22.08 cu in.
Solution to Exercise 9.195 (p. 554)
4
Solution to Exercise 9.197 (p. 554)
31 = 2 7 = 2.58
12 12
572 CHAPTER 9. MEASUREMENT AND GEOMETRY

Solution to Exercise 9.199 (p. 554)


27.9m
Solution to Exercise 9.200 (p. 557)
Measurement is comparison to a standard (unit of measure).
Solution to Exercise 9.202 (p. 557)
2 pounds
Solution to Exercise 9.204 (p. 557)
6 tons
Solution to Exercise 9.206 (p. 557)
69 feet
Solution to Exercise 9.208 (p. 557)
2
3 = 0.666 feet
Solution to Exercise 9.210 (p. 557)
6 pints
Solution to Exercise 9.212 (p. 557)
80 tablespoons
Solution to Exercise 9.214 (p. 557)
210 seconds
Solution to Exercise 9.216 (p. 558)
1
4 = 0.25L
Solution to Exercise 9.218 (p. 558)
19,610 mg
Solution to Exercise 9.220 (p. 558)
540.06 g
Solution to Exercise 9.222 (p. 558)
3,500,000 mL
Solution to Exercise 9.224 (p. 558)
6 weeks 1 day
Solution to Exercise 9.226 (p. 558)
5 gallons 2 quarts 1 pint
Solution to Exercise 9.228 (p. 558)
7 T 1,850 pounds
Solution to Exercise 9.230 (p. 558)
7 days, 11 hours, 56 minutes, 7 seconds
Solution to Exercise 9.232 (p. 558)
1 yard 1 foot
Solution to Exercise 9.234 (p. 558)
1 ton 1,100 pounds
Solution to Exercise 9.236 (p. 559)
1 tablespoon 1 teaspoon
Solution to Exercise 9.238 (p. 559)
3 quarts 1 pint
Solution to Exercise 9.240 (p. 559)
2.3 meters
Solution to Exercise 9.242 (p. 559)
1.05 km
Solution to Exercise 9.244 (p. 559)
5.652 sq cm
Solution to Exercise 9.246 (p. 560)
104.28568 cu ft
573

Solution to Exercise 9.248 (p. 560)


0.18π sq in.
Solution to Exercise 9.250 (p. 561)
267.94667 cu mm
Solution to Exercise 9.252 (p. 561)
32 cu cm
Solution to Exercise 9.254 (p. 561)
39.48 sq in.
Solution to Exercise 9.256 (p. 562)
56.52 sq ft
Solution to Exercise 9.258 (p. 562)
measurement
Solution to Exercise 9.259 (p. 563)
42 feet
Solution to Exercise 9.260 (p. 563)
612 inches
Solution to Exercise 9.261 (p. 563)
1 foot
Solution to Exercise 9.262 (p. 563)
135 seconds
Solution to Exercise 9.263 (p. 563)
850 cg
Solution to Exercise 9.264 (p. 563)
0.0058623 kL
Solution to Exercise 9.265 (p. 563)
0.2132062 m
Solution to Exercise 9.266 (p. 563)
100,001,000,000 mL
Solution to Exercise 9.267 (p. 563)
3 weeks 2 days
Solution to Exercise 9.268 (p. 563)
29 yards 1 foot
Solution to Exercise 9.269 (p. 563)
2 tons 216 pounds
Solution to Exercise 9.270 (p. 563)
1 gallon 3 quarts
Solution to Exercise 9.271 (p. 563)
8 weeks 5 days
Solution to Exercise 9.272 (p. 563)
14 gallons 2 quarts
Solution to Exercise 9.273 (p. 563)
2 yards 5 feet 9 inches
Solution to Exercise 9.274 (p. 563)
29 minutes 56 seconds
Solution to Exercise 9.275 (p. 563)
3 weeks 5 days
Solution to Exercise 9.276 (p. 563)
34.44 m
Solution to Exercise 9.277 (p. 564)
36 mm
574 CHAPTER 9. MEASUREMENT AND GEOMETRY

Solution to Exercise 9.278 (p. 564)


87.92 feet
Solution to Exercise 9.279 (p. 564)
55.14 miles
Solution to Exercise 9.280 (p. 564)
3.75 sq in.
Solution to Exercise 9.281 (p. 564)
6.002826 sq cm
Solution to Exercise 9.282 (p. 565)
6.28 sq miles
Solution to Exercise 9.283 (p. 565)
13 sq in.
Solution to Exercise 9.284 (p. 565)
84.64π sq in.
Solution to Exercise 9.285 (p. 565)
25.12 cu mm
Solution to Exercise 9.286 (p. 565)
4.608π cu ft
Solution to Exercise 9.287 (p. 566)
340.48 cu mm
Chapter 10

Signed Numbers

10.1 Objectives1
After completing this chapter, you should
Variables, Constants, and Real Numbers (Section 10.2)

• be able to distinguish between variables and constants


• be able to recognize a real number and particular subsets of the real numbers
• understand the ordering of the real numbers

Signed Numbers (Section 10.3)

• be able to distinguish between positive and negative real numbers


• be able to read signed numbers
• understand the origin and use of the double-negative product property

Absolute Value (Section 10.4)

• understand the geometric and algebraic denitions of absolute value

Addition of Signed Numbers (Section 10.5)

• be able to add numbers with like signs and with unlike signs
• be able to use the calculator for addition of signed numbers

Subtraction of Signed Numbers (Section 10.6)

• understand the denition of subtraction


• be able to subtract signed numbers
• be able to use a calculator to subtract signed numbers

Multiplication and Division of Signed Numbers (Section 10.7)

• be able to multiply and divide signed numbers


• be able to multiply and divide signed numbers using a calculator

1 This content is available online at <http://cnx.org/content/m18898/1.3/>.

575
576 CHAPTER 10. SIGNED NUMBERS

10.2 Variables, Constants, and Real Numbers 2

10.2.1 Section Overview


• Variables and Constants
• Real Numbers
• Subsets of Real Numbers
• Ordering Real Numbers

10.2.2 Variables and Constants


A basic distinction between algebra and arithmetic is the use of symbols (usually letters) in algebra to
represent numbers. So, algebra is a generalization of arithmetic. Let us look at two examples of situations
in which letters are substituted for numbers:

1. Suppose that a student is taking four college classes, and each class can have at most 1 exam per week.
In any 1-week period, the student may have 0, 1, 2, 3, or 4 exams. In algebra, we can let the letter x
represent the number of exams this student may have in a 1-week period. The letter x may assume
any of the various values 0, 1, 2, 3, 4.
2. Suppose that in writing a term paper for a biology class a student needs to specify the average lifetime,
in days, of a male housey. If she does not know this number o the top of her head, she might
represent it (at least temporarily) on her paper with the letter t (which reminds her of time ). Later,
she could look up the average time in a reference book and nd it to be 17 days. The letter t can
assume only the one value, 17, and no other values. The value t is constant.

Variable, Constant

1. A letter or symbol that represents any member of a collection of two or more numbers is called a
variable.
2. A letter or symbol that represents one specic number, known or unknown, is called a constant.

In example 1, the letter x is a variable since it can represent any of the numbers 0, 1, 2, 3, 4. The letter t
example 2 is a constant since it can only have the value 17.

10.2.3 Real Numbers


Real Number Line
The study of mathematics requires the use of several collections of numbers. The real number line allows
us to visually display (graph) the numbers in which we are interested.

A line is composed of innitely many points. To each point we can associate a unique number, and with
each number, we can associate a particular point.

Coordinate
The number associated with a point on the number line is called the coordinate of the point.

Graph
The point on a number line that is associated with a particular number is called the graph of that number.

Constructing a Real Number Line


We construct a real number line as follows:
2 This content is available online at <http://cnx.org/content/m35027/1.2/>.
577

1. Draw a horizontal line.

2. Origin
Choose any point on the line and label it 0. This point is called the origin.

3. Choose a convenient length. Starting at 0, mark this length o in both directions, being careful to
have the lengths look like they are about the same.

We now dene a real number.

Real Number
A real number is any number that is the coordinate of a point on the real number line.

Positive Numbers, Negative Numbers


Real numbers whose graphs are to the right of 0 are called positive real numbers, or more simply, positive
numbers. Real numbers whose graphs appear to the left of 0 are called negative real numbers, or more
simply, negative numbers.

The number 0 is neither positive nor negative.

10.2.4 Subsets of Real Numbers


The set of real numbers has many subsets. Some of the subsets that are of interest in the study of algebra
are listed below along with their notations and graphs.

Natural Numbers, Counting Numbers


The natural or counting numbers (N ): 1, 2, 3, 4, . . . Read and so on.

Whole Numbers
The whole numbers (W ): 0, 1, 2, 3, 4, . . .

Notice that every natural number is a whole number.

Integers
The integers (Z ): . . . -3, -2, -1, 0, 1, 2, 3, . . .

Notice that every whole number is an integer.

Rational Numbers (Fractions)


The rational numbers (Q): Rational numbers are sometimes called fractions. They are numbers that
can be written as the quotient of two integers. They have decimal representations that either terminate or
do not terminate but contain a repeating block of digits. Some examples are
−3 11
= −0.75 8 = 8.407407407...
| 4 {z } | 27 {z }
Terminating Nonterminating, but repeating
578 CHAPTER 10. SIGNED NUMBERS

Some rational numbers are graphed below.

Notice that every integer is a rational number.

Notice that there are still a great many points on the number line that have not yet been assigned a type
of number. We will not examine these other types of numbers in this text. They are examined in detail in
algebra. An example of these numbers is the number π , whose decimal representation does not terminate
nor contain a repeating block of digits. An approximation for π is 3.14.

10.2.4.1 Sample Set A

Example 10.1
Is every whole number a natural number?

No. The number 0 is a whole number but it is not a natural number.


Example 10.2
Is there an integer that is not a natural number?

Yes. Some examples are 0, -1, -2, -3, and -4.


Example 10.3
Is there an integer that is a whole number?

Yes. In fact, every whole number is an integer.

10.2.4.2 Practice Set A

Exercise 10.1 (Solution on p. 618.)


Is every natural number a whole number?
Exercise 10.2 (Solution on p. 618.)
Is every whole number an integer?
Exercise 10.3 (Solution on p. 618.)
Is every integer a real number?
Exercise 10.4 (Solution on p. 618.)
Is there an integer that is a whole number?
Exercise 10.5 (Solution on p. 618.)
Is there an integer that is not a natural number?

10.2.5 Ordering Real Numbers


Ordering Real Numbers
A real number b is said to be greater than a real number a, denoted b > a, if b is to the right of a on the
number line. Thus, as we would expect, 5 > 2 since 5 is to the right of 2 on the number line. Also, −2 > − 5
since -2 is to the right of -5 on the number line.

If we let a and b represent two numbers, then a and b are related in exactly one of three ways: Either
579

Equality Symbol
a=b a and b are equal (8 = 8)

Inequality Symbols
a>b a is greater than b (8 > 5)
a<b a is less than b (5 < 8)
Some variations of these symbols are
{
a 6= b a is not equal to b (8 6= 5)
a≥b a is greater than or equal to b (a ≥ 8)
a≤b a is less than or equal to b (a ≤ 8)

10.2.5.1 Sample Set B

Example 10.4
What integers can replace x so that the following statement is true?

−3 ≤ x < 2

The integers are -3, -2, -1, 0, 1.


Example 10.5
Draw a number line that extends from -3 to 5. Place points at all whole numbers between and
including -1 and 3.

-1 is not a whole number

10.2.5.2 Practice Set B

Exercise 10.6 (Solution on p. 618.)


What integers can replace x so that the following statement is true? −5 ≤ x < 2
Exercise 10.7 (Solution on p. 618.)
Draw a number line that extends from -4 to 3. Place points at all natural numbers between, but
not including, -2 to 2.

10.2.6 Exercises
For the following 8problems, next to each real number, note all collections to which it belongs by writing N
for natural number, W for whole number, or Z for integer. Some numbers may belong to more than one
collection.
Exercise 10.8 (Solution on p. 618.)
6
Exercise 10.9
12
580 CHAPTER 10. SIGNED NUMBERS

Exercise 10.10 (Solution on p. 618.)


0
Exercise 10.11
1
Exercise 10.12 (Solution on p. 618.)
-3
Exercise 10.13
-7
Exercise 10.14 (Solution on p. 618.)
-805
Exercise 10.15
-900
Exercise 10.16 (Solution on p. 618.)
Is the number 0 a positive number, a negative number, neither, or both?
Exercise 10.17
An integer is an even integer if it is evenly divisible by 2. Draw a number line that extends from
-5 to 5 and place points at all negative even integers and all positive odd integers.
Exercise 10.18 (Solution on p. 618.)
Draw a number line that extends from -5 to 5. Place points at all integers that satisfy −3 ≤ x < 4.
Exercise 10.19
Is there a largest two digit number? If so, what is it?
Exercise 10.20 (Solution on p. 618.)
Is there a smallest two digit number? If so, what is it?
For the pairs of real numbers in the following 5 problems, write the appropriate symbol (<, >, =) in place
of the .
Exercise 10.21
-7  -2
Exercise 10.22 (Solution on p. 618.)
-5  0
Exercise 10.23
-1  4
Exercise 10.24 (Solution on p. 618.)
6  -1
Exercise 10.25
10  10
For the following 5 problems, what numbers can replace m so that the following statements are true?
Exercise 10.26 (Solution on p. 618.)
−1 ≤ m ≤ −5, m an integer.
Exercise 10.27
−7 < m < − 1, m an integer.
Exercise 10.28 (Solution on p. 618.)
−3 ≤ m < 2, m a natural number.
Exercise 10.29
−15 < m ≤ −1, m a natural number.
Exercise 10.30 (Solution on p. 618.)
−5 ≤ m < 5, m a whole number.
581

For the following 10 problems, on the number line, how many units are there between the given pair of
numbers?
Exercise 10.31
0 and 3
Exercise 10.32 (Solution on p. 618.)
-4 and 0
Exercise 10.33
-1 and 6
Exercise 10.34 (Solution on p. 618.)
-6 and 2
Exercise 10.35
-3 and 3
Exercise 10.36 (Solution on p. 618.)
Are all positive numbers greater than zero?
Exercise 10.37
Are all positive numbers greater than all negative numbers?
Exercise 10.38 (Solution on p. 618.)
Is 0 greater than all negative number?
Exercise 10.39
Is there a largest natural number?
Exercise 10.40 (Solution on p. 618.)
Is there a largest negative integer?

10.2.6.1 Exercises for Review

Exercise 10.41
(Section 4.3) Convert 6 58 to an improper fraction.
Exercise 10.42 (Solution on p. 619.)
(Section 4.5) Find the value: 11
3
of 33
5 .
Exercise 10.43
(Section 5.3) Find the sum of 4
5 + 38 .
Exercise 10.44 (Solution on p. 619.)
(Section 9.3) Convert 30.06 cm to m.
Exercise 10.45
(Section 9.6) Find the area of the triangle.
582 CHAPTER 10. SIGNED NUMBERS

10.3 Signed Numbers 3

10.3.1 Section Overview


• Positive and Negative Numbers
• Reading Signed Numbers
• Opposites
• The Double-Negative Property

10.3.2 Positive and Negative Numbers


Positive and Negative Numbers
Each real number other than zero has a sign associated with it. A real number is said to be a positive
number if it is to the right of 0 on the number line and negative if it is to the left of 0 on the number line.

note: + and − Notation


A number is denoted as positive if it is directly preceded by a plus sign or no sign at all.
A number is denoted as negative if it is directly preceded by a minus sign.

10.3.3 Reading Signed Numbers


The plus and minus signs now have two meanings :

The plus sign can denote the operation of addition or a positive number.

The minus sign can denote the operation of subtraction or a negative number.

To avoid any confusion between "sign" and "operation," it is preferable to read the sign of a number as
"positive" or "negative." When "+" is used as an operation sign, it is read as "plus." When "−" is used as
an operation sign, it is read as "minus."

10.3.3.1 Sample Set A

Read each expression so as to avoid confusion between "operation" and "sign."


Example 10.6
−8 should be read as "negative eight" rather than "minus eight."
Example 10.7
4 + (−2) should be read as "four plus negative two" rather than "four plus minus two."
Example 10.8
−6 + (−3)should be read as "negative six plus negative three" rather than "minus six plus minus
three."
Example 10.9
−15−(−6)should be read as "negative fteen minus negative six" rather than "minus fteen minus
minus six."
Example 10.10
−5 + 7 should be read as "negative ve plus seven" rather than "minus ve plus seven."
Example 10.11
0 − 2 should be read as "zero minus two."

3 This content is available online at <http://cnx.org/content/m35029/1.3/>.


583

10.3.3.2 Practice Set A

Write each expression in words.


Exercise 10.46 (Solution on p. 619.)
6+1
Exercise 10.47 (Solution on p. 619.)
2 + (−8)
Exercise 10.48 (Solution on p. 619.)
−7 + 5
Exercise 10.49 (Solution on p. 619.)
−10 − (+3)
Exercise 10.50 (Solution on p. 619.)
−1 − (−8)
Exercise 10.51 (Solution on p. 619.)
0 + (−11)

10.3.4 Opposites
Opposites
On the number line, each real number, other than zero, has an image on the opposite side of 0. For this
reason, we say that each real number has an opposite. Opposites are the same distance from zero but have
opposite signs.

The opposite of a real number is denoted by placing a negative sign directly in front of the number. Thus,
if a is any real number, then −a is its opposite.

note: The letter "a" is a variable. Thus, "a" need not be positive, and "−a" need not be negative.

If a is any real number, −a is opposite a on the number line.

10.3.5 The Double-Negative Property


The number a is opposite −a on the number line. Therefore, − (−a) is opposite −a on the number line.
This means that

− (−a) = a

From this property of opposites, we can suggest the double-negative property for real numbers.

Double-Negative Property: − (−a) = a


If a is a real number, then
− (−a) = a
584 CHAPTER 10. SIGNED NUMBERS

10.3.5.1 Sample Set B

Find the opposite of each number.


Example 10.12
If a = 2, then −a = −2. Also, − (−a) = − (−2) = 2.

Example 10.13
If a = −4, then −a = − (−4) = 4. Also, − (−a) = a = − 4.

10.3.5.2 Practice Set B

Find the opposite of each number.


Exercise 10.52 (Solution on p. 619.)
8
Exercise 10.53 (Solution on p. 619.)
17
Exercise 10.54 (Solution on p. 619.)
-6
Exercise 10.55 (Solution on p. 619.)
-15
Exercise 10.56 (Solution on p. 619.)
-(-1)
Exercise 10.57 (Solution on p. 619.)
− [− (−7)]
Exercise 10.58 (Solution on p. 619.)
Suppose a is a positive number. Is −a positive or negative?
Exercise 10.59 (Solution on p. 619.)
Suppose a is a negative number. Is −a positive or negative?
Exercise 10.60 (Solution on p. 619.)
Suppose we do not know the sign of the number k . Is −k positive, negative, or do we not know?

10.3.6 Exercises
Exercise 10.61 (Solution on p. 619.)
A number is denoted as positive if it is directly preceded by .
Exercise 10.62
A number is denoted as negative if it is directly preceded by .
How should the number in the following 6 problems be read? (Write in words.)
Exercise 10.63 (Solution on p. 619.)
−7
585

Exercise 10.64
−5
Exercise 10.65 (Solution on p. 619.)
15
Exercise 10.66
11
Exercise 10.67 (Solution on p. 619.)
− (−1)
Exercise 10.68
− (−5)
For the following 6 problems, write each expression in words.
Exercise 10.69 (Solution on p. 619.)
5+3
Exercise 10.70
3+8
Exercise 10.71 (Solution on p. 619.)
15 + (−3)
Exercise 10.72
1 + (−9)
Exercise 10.73 (Solution on p. 619.)
−7 − (−2)
Exercise 10.74
0 − (−12)
For the following 6 problems, rewrite each number in simpler form.
Exercise 10.75 (Solution on p. 619.)
− (−2)
Exercise 10.76
− (−16)
Exercise 10.77 (Solution on p. 620.)
− [− (−8)]
Exercise 10.78
− [− (−20)]
Exercise 10.79 (Solution on p. 620.)
7 − (−3)
Exercise 10.80
6 − (−4)
586 CHAPTER 10. SIGNED NUMBERS

10.3.6.1 Exercises for Review

Exercise 10.81 (Solution on p. 620.)


(Section 6.7) Find the quotient; 8 ÷ 27.
Exercise 10.82
(Section 7.3) Solve the proportion: 5
9 = 60
x
Exercise 10.83 (Solution on p. 620.)
(Section 8.2) Use the method of rounding to estimate the sum: 5829 + 8767
Exercise 10.84
(Section 9.2) Use a unit fraction to convert 4 yd to feet.
Exercise 10.85 (Solution on p. 620.)
(Section 9.3) Convert 25 cm to hm.

10.4 Absolute Value4


10.4.1 Section Overview
• Geometric Denition of Absolute Value
• Algebraic Denition of Absolute Value

10.4.2 Geometric Denition of Absolute Value


Absolute Value-Geometric Approach
Geometric denition of absolute value:
The absolute value of a number a, denoted | a |, is the distance from a to 0 on the number line.

Absolute value answers the question of "how far," and not "which way." The phrase "how far" implies
"length" and length is always a nonnegative quantity. Thus, the absolute value of a number is a nonnegative
number.

10.4.2.1 Sample Set A

Determine each value.


Example 10.14
| 4 |= 4

Example 10.15
| −4 |= 4

Example 10.16
| 0 |= 0
4 This content is available online at <http://cnx.org/content/m35030/1.3/>.
587

Example 10.17
− | 5 |= −5. The quantity on the left side of the equal sign is read as "negative the absolute value
of 5." The absolute value of 5 is 5. Hence, negative the absolute value of 5 is -5.
Example 10.18
− | −3 |= −3. The quantity on the left side of the equal sign is read as "negative the absolute
value of -3." The absolute value of -3 is 3. Hence, negative the absolute value of -3 is − (3) = −3.

10.4.2.2 Practice Set A

By reasoning geometrically, determine each absolute value.


Exercise 10.86 (Solution on p. 620.)
|7|
Exercise 10.87 (Solution on p. 620.)
| −3 |
Exercise 10.88 (Solution on p. 620.)
| 12 |
Exercise 10.89 (Solution on p. 620.)
|0|
Exercise 10.90 (Solution on p. 620.)
−|9|
Exercise 10.91 (Solution on p. 620.)
− | −6 |

10.4.3 Algebraic Denition of Absolute Value


From the problems in Section 10.4.2.1 (Sample Set A), we can suggest the following algebraic denition of
absolute value. Note that the denition has two parts.

Absolute ValueAlgebraic Approach


Algebraic denition of absolute value
The absolute value of a number a is
a, if a ≥ 0
|a| = {
−a, if < 0

The algebraic denition takes into account the fact that the number a could be either positive or zero (a ≥ 0)
or negative (a < 0).

1. If the number a is positive or zero (a ≥ 0), the upper part of the denition applies. The upper part of
the denition tells us that if the number enclosed in the absolute value bars is a nonnegative number,
the absolute value of the number is the number itself.
2. The lower part of the denition tells us that if the number enclosed within the absolute value bars is
a negative number, the absolute value of the number is the opposite of the number. The opposite of a
negative number is a positive number.

note: The denition says that the vertical absolute value lines may be eliminated only if we know
whether the number inside is positive or negative.
588 CHAPTER 10. SIGNED NUMBERS

10.4.3.1 Sample Set B

Use the algebraic denition of absolute value to nd the following values.
Example 10.19
| 8 |. The number enclosed within the absolute value bars is a nonnegative number, so the upper
part of the denition applies. This part says that the absolute value of 8 is 8 itself.

| 8 |= 8
Example 10.20
| −3 |. The number enclosed within absolute value bars is a negative number, so the lower part of
the denition applies. This part says that the absolute value of -3 is the opposite of -3, which is
− (−3). By the denition of absolute value and the double-negative property,

| −3 |= − (−3) = 3

10.4.3.2 Practice Set B

Use the algebraic denition of absolute value to nd the following values.
Exercise 10.92 (Solution on p. 620.)
|7|
Exercise 10.93 (Solution on p. 620.)
|9|
Exercise 10.94 (Solution on p. 620.)
| −12 |
Exercise 10.95 (Solution on p. 620.)
| −5 |
Exercise 10.96 (Solution on p. 620.)
−|8|
Exercise 10.97 (Solution on p. 620.)
−|1|
Exercise 10.98 (Solution on p. 620.)
− | −52 |
Exercise 10.99 (Solution on p. 620.)
− | −31 |

10.4.4 Exercises
Determine each of the values.
Exercise 10.100 (Solution on p. 620.)
|5|
Exercise 10.101
|3|
Exercise 10.102 (Solution on p. 620.)
|6|
Exercise 10.103
| −9 |
589

Exercise 10.104 (Solution on p. 620.)


| −1 |
Exercise 10.105
| −4 |
Exercise 10.106 (Solution on p. 620.)
−|3|
Exercise 10.107
−|7|
Exercise 10.108 (Solution on p. 620.)
− | −14 |
Exercise 10.109
|0|
Exercise 10.110 (Solution on p. 620.)
| −26 |
Exercise 10.111
− | −26 |
Exercise 10.112 (Solution on p. 621.)
− (− | 4 |)
Exercise 10.113
− (− | 2 |)
Exercise 10.114 (Solution on p. 621.)
− (− | −6 |)
Exercise 10.115
− (− | −42 |)
Exercise 10.116 (Solution on p. 621.)
| 5 | − | −2 |
Exercise 10.117
3
| −2 |
Exercise 10.118 (Solution on p. 621.)
| − (2 · 3) |
Exercise 10.119
| −2 | − | −9 |
Exercise 10.120 (Solution on p. 621.)
2
(| −6 | + | 4 |)
Exercise 10.121
3
(| −1 | − | 1 |)
Exercise 10.122 (Solution on p. 621.)
2 3
(| 4 | + | −6 |) − (| −2 |)
Exercise 10.123
2
−[|−10| − 6]
Exercise 10.124 (Solution on p. 621.)
3 2
−{−[− | −4 | + | −3 |] }
Exercise 10.125
A Mission Control Ocer at Cape Canaveral makes the statement lift-o, T minus 50 seconds.
How long is it before lift-o?
590 CHAPTER 10. SIGNED NUMBERS

Exercise 10.126 (Solution on p. 621.)


Due to a slowdown in the industry, a Silicon Valley computer company nds itself in debt
$2,400,000. Use absolute value notation to describe this company's debt.
Exercise 10.127
A particular machine is set correctly if upon action its meter reads 0. One particular machine has
a meter reading of −1.6 upon action. How far is this machine o its correct setting?

10.4.4.1 Exercises for Review

Exercise 10.128 (Solution on p. 621.)


(Section 5.3) Find the sum: 70
9 5
+ 21 8
+ 15 .
Exercise 10.129 3 4
+ 12
(Section 5.6) Find the value of 10
19 .
20

Exercise 10.130 (Solution on p. 621.)


(Section 6.3) Convert 3.2 35 to a fraction.
Exercise 10.131
(Section 7.3) The ratio of acid to water in a solution is 8.
3
How many mL of acid are there in a
solution that contain 112 mL of water?
Exercise 10.132 (Solution on p. 621.)
(Section 10.3) Find the value of −6 − (−8).

10.5 Addition of Signed Numbers5


10.5.1 Section Overview
• Addition of Numbers with Like Signs
• Addition with Zero
• Addition of Numbers with Unlike Signs
• Calculators

10.5.2 Addition of Numbers with Like Signs


The addition of the two positive numbers 2 and 3 is performed on the number line as follows.

Begin at 0, the origin.

Since 2 is positive, move 2 units to the right.

Since 3 is positive, move 3 more units to the right.

We are now located at 5.

Thus, 2 + 3 = 5.

Summarizing, we have
5 This content is available online at <http://cnx.org/content/m35031/1.2/>.
591

(2 positive units) + (3 positive units) = (5 positive units)

The addition of the two negative numbers -2 and -3 is performed on the number line as follows.

Begin at 0, the origin.

Since -2 is negative, move 2 units to the left.

Since -3 is negative, move 3 more units to the left.

We are now located at -5.

Thus, (−2) + (−3) = −5.

Summarizing, we have

(2 negative units) + (3 negative units) = (5 negative units)

Observing these two examples, we can suggest these relationships:

(positive number) + (positive number) = (positive number)

(negative number) + (negative number) = (negative number)

Adding Numbers with the Same Sign


Addition of numbers with like sign:
To add two real numbers that have the same sign, add the absolute values of the numbers and associate
with the sum the common sign.

10.5.2.1 Sample Set A

Find the sums.


Example 10.21
3+7

|3| = 3
} Add these absolute values.
|7| = 7

3 + 7 = 10

The common sign is +.

Thus, 3 + 7 = +10, or 3 + 7 = 10.


Example 10.22
(−4) + (−9)

| − 4| = 4
} Add these absolute values.
| − 9| = 9

4 + 9 = 13

The common sign is  −.


592 CHAPTER 10. SIGNED NUMBERS

Thus, (−4) + (−9) = −13.

10.5.2.2 Practice Set A

Find the sums.


Exercise 10.133 (Solution on p. 621.)
8+6
Exercise 10.134 (Solution on p. 621.)
41 + 11
Exercise 10.135 (Solution on p. 621.)
(−4) + (−8)
Exercise 10.136 (Solution on p. 621.)
(−36) + (−9)
Exercise 10.137 (Solution on p. 621.)
−14 + (−20)
Exercise 10.138 (Solution on p. 621.)
− 23 + − 53


Exercise 10.139 (Solution on p. 621.)


−2.8 + (−4.6)
Exercise 10.140 (Solution on p. 621.)
0 + (−16)

10.5.3 Addition With Zero


Addition with Zero
Notice that
(0) + (a positive number) = (that same positive number).
(0) + (a negative number) = (that same negative number).

The Additive Identity Is Zero


Since adding zero to a real number leaves that number unchanged, zero is called the additive identity.

10.5.4 Addition of Numbers with Unlike Signs


The addition 2 + (−6), two numbers with unlike signs, can also be illustrated using the number line.

Begin at 0, the origin.

Since 2 is positive, move 2 units to the right.

Since -6 is negative, move, from 2, 6 units to the left.

We are now located at -4.

We can suggest a rule for adding two numbers that have unlike signs by noting that if the signs are disre-
garded, 4 can be obtained by subtracting 2 from 6. But 2 and 6 are precisely the absolute values of 2 and
593

-6. Also, notice that the sign of the number with the larger absolute value is negative and that the sign of
the resulting sum is negative.

Adding Numbers with Unlike Signs


Addition of numbers with unlike signs: To add two real numbers that have unlike signs, subtract the smaller
absolute value from the larger absolute value and associate with this dierence the sign of the number with
the larger absolute value.

10.5.4.1 Sample Set B

Find the following sums.


Example 10.23
7 + (−2)

|7| = 7 | − 2| = 2
| {z } | {z }
Larger absolute Smaller absolute
value. Sign is positive. value.

Subtract absolute values: 7 − 2 = 5.

Attach the proper sign: "+."

Thus, 7 + (−2) = +5 or 7 + (−2) = 5.


Example 10.24
3 + (−11)

|3| = 3 | − 11| = 11
| {z } | {z }
Smaller absolute Larger absolute
value. value. Sign is negative.

Subtract absolute values: 11 − 3 = 8.

Attach the proper sign: "−."

Thus, 3 + (−11) = −8.


Example 10.25
The morning temperature on a winter's day in Lake Tahoe was -12 degrees. The afternoon
temperature was 25 degrees warmer. What was the afternoon temperature?

We need to nd −12 + 25.

| − 12| = 12 |25| = 25
| {z } | {z }
Smaller absolute Larger absolute
value. value. Sign is positive.

Subtract absolute values: 25 − 12 = 16.

Attach the proper sign: "+."

Thus, −12 + 25 = 13.


594 CHAPTER 10. SIGNED NUMBERS

10.5.4.2 Practice Set B

Find the sums.


Exercise 10.141 (Solution on p. 621.)
4 + (−3)
Exercise 10.142 (Solution on p. 621.)
−3 + 5
Exercise 10.143 (Solution on p. 621.)
15 + (−18)
Exercise 10.144 (Solution on p. 621.)
0 + (−6)
Exercise 10.145 (Solution on p. 621.)
−26 + 12
Exercise 10.146 (Solution on p. 621.)
35 + (−78)
Exercise 10.147 (Solution on p. 622.)
15 + (−10)
Exercise 10.148 (Solution on p. 622.)
1.5 + (−2)
Exercise 10.149 (Solution on p. 622.)
−8 + 0
Exercise 10.150 (Solution on p. 622.)
0 + (0.57)
Exercise 10.151 (Solution on p. 622.)
−879 + 454

10.5.5 Calculators

Calculators having the key can be used for nding sums of signed numbers.

10.5.5.1 Sample Set C

Use a calculator to nd the sum of -147 and 84.

Display Reads
Type 147 147
Press -147 This key changes the
sign of a number. It is
dierent than −.
continued on next page
595

Press + -147
Type 84 84
Press = -63

Table 10.1

10.5.5.2 Practice Set C

Use a calculator to nd each sum.


Exercise 10.152 (Solution on p. 622.)
673 + (−721)
Exercise 10.153 (Solution on p. 622.)
−8, 261 + 2,206
Exercise 10.154 (Solution on p. 622.)
−1, 345.6 + (−6, 648.1)

10.5.6 Exercises
Find the sums in the following 27 problems. If possible, use a calculator to check each result.
Exercise 10.155 (Solution on p. 622.)
4 + 12
Exercise 10.156
8+6
Exercise 10.157 (Solution on p. 622.)
(−3) + (−12)
Exercise 10.158
(−6) + (−20)
Exercise 10.159 (Solution on p. 622.)
10 + (−2)
Exercise 10.160
8 + (−15)
Exercise 10.161 (Solution on p. 622.)
−16 + (−9)
Exercise 10.162
−22 + (−1)
Exercise 10.163 (Solution on p. 622.)
0 + (−12)
Exercise 10.164
0 + (−4)
Exercise 10.165 (Solution on p. 622.)
0 + (24)
Exercise 10.166
−6 + 1 + (−7)
Exercise 10.167 (Solution on p. 622.)
−5 + (−12) + (−4)
596 CHAPTER 10. SIGNED NUMBERS

Exercise 10.168
−5 + 5
Exercise 10.169 (Solution on p. 622.)
−7 + 7
Exercise 10.170
−14 + 14
Exercise 10.171 (Solution on p. 622.)
4 + (−4)
Exercise 10.172
9 + (−9)
Exercise 10.173 (Solution on p. 622.)
84 + (−61)
Exercise 10.174
13 + (−56)
Exercise 10.175 (Solution on p. 622.)
452 + (−124)
Exercise 10.176
636 + (−989)
Exercise 10.177 (Solution on p. 622.)
1, 811 + (−935)
Exercise 10.178
−373 + (−14)
Exercise 10.179 (Solution on p. 622.)
−1, 211 + (−44)
Exercise 10.180
−47.03 + (−22.71)
Exercise 10.181 (Solution on p. 622.)
−1.998 + (−4.086)
Exercise 10.182
In order for a small business to break even on a project, it must have sales of $21,000. If the
amount of sales was $15,000, by how much money did this company fall short?
Exercise 10.183 (Solution on p. 622.)
Suppose a person has $56 in his checking account. He deposits $100 into his checking account
by using the automatic teller machine. He then writes a check for $84.50. If an error causes the
deposit not to be listed into this person's account, what is this person's checking balance?
Exercise 10.184
A person borrows $7 on Monday and then $12 on Tuesday. How much has this person borrowed?
Exercise 10.185 (Solution on p. 622.)
A person borrows $11 on Monday and then pays back $8 on Tuesday. How much does this person
owe?
597

10.5.6.1 Exercises for Review

Exercise 10.186
(Section 4.6) Find the reciprocal of 8 56 .
Exercise 10.187 (Solution on p. 622.)
(Section 5.3) Find the value of 12
5 7
+ 18 − 31 .
Exercise 10.188
(Section 6.4) Round 0.01628 to the nearest tenth.
Exercise 10.189 (Solution on p. 623.)
(Section 7.5) Convert 62% to a fraction.
Exercise 10.190
(Section 10.4) Find the value of | −12 |.

10.6 Subtraction of Signed Numbers6


10.6.1 Section Overview
• Denition of Subtraction
• The Process of Subtraction
• Calculators

10.6.2 Denition of Subtraction


We know from experience with arithmetic that the subtraction 5 − 2 produces 3, that is 5 − 2 = 3. We can
suggest a rule for subtracting signed numbers by illustrating this process on the number line.

Begin at 0, the origin.

Since 5 is positive, move 5 units to the right.

Then, move 2 units to the left to get to 6. (This reminds us of addition with a negative number.)

From this illustration we can see that 5 − 2 is the same as 5 + (−2). This leads us directly to the denition
of subtraction.

Denition of Subtraction
If a and b are real numbers, a − b is the same as a + (−b), where −b is the opposite of b.

10.6.3 The Process of Subtraction


From this denition, we suggest the following rule for subtracting signed numbers.

Subtraction of Signed Numbers


To perform the subtraction a − b, add the opposite of b to a, that is, change the sign of b and add.
6 This content is available online at <http://cnx.org/content/m35032/1.2/>.
598 CHAPTER 10. SIGNED NUMBERS

10.6.3.1 Sample Set A

Perform the indicated subtractions.


Example 10.26
5 − 3 = 5 + (−3) = 2
Example 10.27
4 − 9 = 4 + (−9) = −5
Example 10.28
−4 − 6 = −4 + (−6) = −10
Example 10.29
−3 − (−12) = −3 + 12 = 9
Example 10.30
0 − (−15) = 0 + 15 = 15
Example 10.31
The high temperature today in Lake Tahoe was 26 ◦ F. The low temperature tonight is expected
to be -7 ◦ F. How many degrees is the temperature expected to drop?

We need to nd the dierence between 26 and -7.

26 − (−7) = 26 + 7 = 33

Thus, the expected temperature drop is 33 ◦ F.


Example 10.32
−6 − (−5) − 10 = −6 + 5 + (−10)
= (−6 + 5) + (−10)
= −1 + (−10)
= −11

10.6.3.2 Practice Set A

Perform the indicated subtractions.


Exercise 10.191 (Solution on p. 623.)
9−6
Exercise 10.192 (Solution on p. 623.)
6−9
Exercise 10.193 (Solution on p. 623.)
0−7
Exercise 10.194 (Solution on p. 623.)
1 − 14
Exercise 10.195 (Solution on p. 623.)
−8 − 12
Exercise 10.196 (Solution on p. 623.)
−21 − 6
Exercise 10.197 (Solution on p. 623.)
−6 − (−4)
599

Exercise 10.198 (Solution on p. 623.)


8 − (−10)
Exercise 10.199 (Solution on p. 623.)
1 − (−12)
Exercise 10.200 (Solution on p. 623.)
86 − (−32)
Exercise 10.201 (Solution on p. 623.)
0 − 16
Exercise 10.202 (Solution on p. 623.)
0 − (−16)
Exercise 10.203 (Solution on p. 623.)
0 − (8)
Exercise 10.204 (Solution on p. 623.)
5 − (−5)
Exercise 10.205 (Solution on p. 623.)
24 − [− (−24)]

10.6.4 Calculators
Calculators can be used for subtraction of signed numbers. The most ecient calculators are those with a
key.

10.6.4.1 Sample Set B

Use a calculator to nd each dierence.


Example 10.33
3, 187 − 8, 719

Display Reads
Type 3187 3187
Press - 3187
Type 8719 8719
Press = -5532

Table 10.2

Thus, 3, 187 − 8, 719 = −5, 532.


Example 10.34
−156 − (−211)

Method A:
600 CHAPTER 10. SIGNED NUMBERS

Display Reads
Type 156 156
Press -156

Type - -156
Press 211 211
Type -211

Press = 55

Table 10.3

Thus, −156 − (−211) = 55.

Method B:
We manually change the subtraction to an addition and change the sign of the number to be
subtracted.

−156 − (−211) becomes −156 + 211

Display Reads
Type 156 156
Press -156

Press + -156
Type 211 211
Press = 55

Table 10.4

10.6.4.2 Practice Set B

Use a calculator to nd each dierence.


Exercise 10.206 (Solution on p. 623.)
44 − 315
Exercise 10.207 (Solution on p. 623.)
12.756 − 15.003
Exercise 10.208 (Solution on p. 623.)
−31.89 − 44.17
Exercise 10.209 (Solution on p. 623.)
−0.797 − (−0.615)
601

10.6.5 Exercises
For the following 18 problems, perform each subtraction. Use a calculator to check each result.
Exercise 10.210 (Solution on p. 623.)
8−3
Exercise 10.211
12 − 7
Exercise 10.212 (Solution on p. 623.)
5−6
Exercise 10.213
14 − 30
Exercise 10.214 (Solution on p. 623.)
−6 − 8
Exercise 10.215
−1 − 12
Exercise 10.216 (Solution on p. 623.)
−5 − (−3)
Exercise 10.217
−11 − (−8)
Exercise 10.218 (Solution on p. 623.)
0−6
Exercise 10.219
0 − 15
Exercise 10.220 (Solution on p. 624.)
0 − (−7)
Exercise 10.221
0 − (−10)
Exercise 10.222 (Solution on p. 624.)
67 − 38
Exercise 10.223
142 − 85
Exercise 10.224 (Solution on p. 624.)
816 − 1140
Exercise 10.225
105 − 421
Exercise 10.226 (Solution on p. 624.)
−550 − (−121)
Exercise 10.227
−15.016 − (4.001)
For the following 4 problems, perform the indicated operations.
Exercise 10.228 (Solution on p. 624.)
−26 + 7 − 52
Exercise 10.229
−15 − 21 − (−2)
Exercise 10.230 (Solution on p. 624.)
−104 − (−216) − (−52)
602 CHAPTER 10. SIGNED NUMBERS

Exercise 10.231
−0.012 − (−0.111) − (0.035)
Exercise 10.232 (Solution on p. 624.)
When a particular machine is operating properly, its meter will read 34. If a broken bearing in the
machine causes the meter reading to drop by 45 units, what is the meter reading?
Exercise 10.233
The low temperature today in Denver was −4 ◦ F and the high was −42 ◦ F. What is the temperature
dierence?

10.6.5.1 Exercises for Review

Exercise 10.234 (Solution on p. 624.)


(Section 6.3) Convert 16.02 15 to a decimal.
Exercise 10.235
(Section 6.6) Find 4.01 of 6.2.
Exercise 10.236 (Solution on p. 624.)
(Section 7.5) Convert 16
5
to a percent.
Exercise 10.237
(Section 8.4) Use the distributive property to compute the product: 15 · 82.
Exercise 10.238 (Solution on p. 624.)
(Section 10.5) Find the sum: 16 + (−21).

10.7 Multiplication and Division of Signed Numbers7


10.7.1 Section Overview
• Multiplication of Signed Numbers
• Division of Signed Numbers
• Calculators

10.7.2 Multiplication of Signed Numbers


Let us consider rst, the product of two positive numbers. Multiply: 3 · 5.

3 · 5 means 5 + 5 + 5 = 15

This suggests8 that

(positive number) · (positive number) = (positive number)

More briey,

(+) (+) = (+)

Now consider the product of a positive number and a negative number. Multiply: (3) (−5).
7 This content is available online at <http://cnx.org/content/m35033/1.2/>.
8 In later mathematics courses, the word "suggests" turns into the word "proof." One example does not prove a claim.
Mathematical proofs are constructed to validate a claim for all possible cases.
603

(3) (−5) means (−5) + (−5) + (−5) = −15

This suggests that

(positive number) · (negative number) = (negative number)

More briey,

(+) (−) = (−)

By the commutative property of multiplication, we get

(negative number) · (positive number) = (negative number)

More briey,

(−) (+) = (−)

The sign of the product of two negative numbers can be suggested after observing the following illustration.

Multiply -2 by, respectively, 4, 3, 2, 1, 0, -1, -2, -3, -4.

We have the following rules for multiplying signed numbers.

Rules for Multiplying Signed Numbers


Multiplying signed numbers:

1. To multiply two real numbers that have the same sign, multiply their absolute values. The product is
positive.
(+) (+) = (+)
(−) (−) = (+)
2. To multiply two real numbers that have opposite signs, multiply their absolute values. The product is
negative.
(+) (−) = (−)
(−) (+) = (−)

10.7.2.1 Sample Set A

Find the following products.


Example 10.35
8·6
604 CHAPTER 10. SIGNED NUMBERS

|8| = 8
} Multiply these absolute values.
|6| = 6

8 · 6 = 48

Since the numbers have the same sign, the product is positive.

Thus, 8 · 6=+48, or 8 · 6 = 48.


Example 10.36
(−8) (−6)

| − 8| = 8
} Multiply these absolute values.
| − 6| = 6

8 · 6 = 48

Since the numbers have the same sign, the product is positive.

Thus, (−8) (−6) =+48, or (−8) (−6) = 48.


Example 10.37
(−4) (7)

| − 4| = 4
} Multiply these absolute values.
|7| = 7

4 · 7 = 28

Since the numbers have opposite signs, the product is negative.

Thus, (−4) (7) = −28.


Example 10.38
6 (−3)

|6| = 6
} Multiply these absolute values.
| − 3| = 3

6 · 3 = 18

Since the numbers have opposite signs, the product is negative.

Thus, 6 (−3) = −18.

10.7.2.2 Practice Set A

Find the following products.


Exercise 10.239 (Solution on p. 624.)
3 (−8)
Exercise 10.240 (Solution on p. 624.)
4 (16)
605

Exercise 10.241 (Solution on p. 624.)


(−6) (−5)
Exercise 10.242 (Solution on p. 624.)
(−7) (−2)
Exercise 10.243 (Solution on p. 624.)
(−1) (4)
Exercise 10.244 (Solution on p. 624.)
(−7) 7

10.7.3 Division of Signed Numbers


To determine the signs in a division problem, recall that
12 = 4 since 12 = 3 · 4
3

This suggests that


(+)
(+) = (+)
(+)
(+) = (+) since (+) = (+) (+)
12 ?
What is −3
12 = −4. That is,
−12 = (−3) (−4) suggests that −3
(+)
(−) = (−)
(+)
(+) = (−) (−) suggests that (−) = (−)
−12
What is 3 ?
−12
−12 = (3) (−4) suggests that 3 = −4. That is,
(−)
(+) = (−)
(−)
(−) = (+) (−) suggests that (+) = (−)
−12
What is −3 ?

−12
−12 = (−3) (4) suggests that −3 = 4. That is,
(−)
(−) = (+)
(−)
(−) = (−) (+) suggests that (−) = (+)

We have the following rules for dividing signed numbers.

Rules for Dividing Signed Numbers


Dividing signed numbers:

1. To divide two real numbers that have the same sign, divide their absolute values. The quotient is
positive.
(+) (−)
(+) = (+) (−) = (+)
606 CHAPTER 10. SIGNED NUMBERS

2. To divide two real numbers that have opposite signs, divide their absolute values. The quotient is
negative.
(−) (+)
(+) = (−) (−) = (−)

10.7.3.1 Sample Set B

Find the following quotients.


Example 10.39
−10
2

| − 10| = 10
} Divide these absolute values.
|2| = 2
10 = 5
2

Since the numbers have opposite signs, the quotient is negative.


−10
Thus 2 = −5.
Example 10.40
−35
−7

| − 35| = 35
} Divide these absolute values.
| − 7| = 7
35 = 5
7

Since the numbers have the same signs, the quotient is positive.
−35
Thus, −7 = 5.
Example 10.41
18
−9

|18| = 18
} Divide these absolute values.
| − 9| = 9
18 = 2
9

Since the numbers have opposite signs, the quotient is negative.


18 = 2.
Thus, −9

10.7.3.2 Practice Set B

Find the following quotients.


Exercise 10.245 (Solution on p. 624.)
−24
−6
Exercise 10.246 (Solution on p. 624.)
30
−5
607

Exercise 10.247 (Solution on p. 624.)


−54
27
Exercise 10.248 (Solution on p. 624.)
51
17

10.7.3.3 Sample Set C

Example 10.42
−6(4−7)−2(8−9)
Find the value of −(4+1)+1 .

Using the order of operations and what we know about signed numbers, we get,
−6(4−7)−2(8−9) −6(−3)−2(−1)
−(4+1)+1 = −(5)+1
18+2
= −5+1
20
= −
= −5

10.7.3.4 Practice Set C

Exercise 10.249 (Solution on p. 624.)


−5(2−6)−4(−8−1)
Find the value of 2(3−10)−9(−2) .

10.7.4 Calculators

Calculators with the key can be used for multiplying and dividing signed numbers.

10.7.4.1 Sample Set D

Use a calculator to nd each quotient or product.


Example 10.43
(−186) · (−43)

Since this product involves a (negative)·(negative), we know the result should be a positive number.
We'll illustrate this on the calculator.

Display Reads
Type 186 186
Press -186

continued on next page


608 CHAPTER 10. SIGNED NUMBERS

Press × -186
Type 43 43
Press -43

Press = 7998

Table 10.5

Thus, (−186) · (−43) = 7, 998.


Example 10.44
158.64 . Round to one decimal place.
−54.3

Display Reads
Type 158.64 158.64
Press ÷ 158.64
Type 54.3 54.3
Press -54.3

Press = -2.921546961

Table 10.6

Rounding to one decimal place we get -2.9.

10.7.4.2 Practice Set D

Use a calculator to nd each value.


Exercise 10.250 (Solution on p. 624.)
(−51.3) · (−21.6)
Exercise 10.251 (Solution on p. 624.)
−2.5746 ÷ −2.1
Exercise 10.252 (Solution on p. 624.)
(0.006) · (−0.241). Round to three decimal places.

10.7.5 Exercises
Find the value of each of the following. Use a calculator to check each result.
Exercise 10.253 (Solution on p. 624.)
(−2) (−8)
Exercise 10.254
(−3) (−9)
Exercise 10.255 (Solution on p. 625.)
(−4) (−8)
609

Exercise 10.256
(−5) (−2)
Exercise 10.257 (Solution on p. 625.)
(3) (−12)
Exercise 10.258
(4) (−18)
Exercise 10.259 (Solution on p. 625.)
(10) (−6)
Exercise 10.260
(−6) (4)
Exercise 10.261 (Solution on p. 625.)
(−2) (6)
Exercise 10.262
(−8) (7)
Exercise 10.263 (Solution on p. 625.)
21
7
Exercise 10.264
42
6
Exercise 10.265 (Solution on p. 625.)
−39
3
Exercise 10.266
−20
10
Exercise 10.267 (Solution on p. 625.)
−45
−5
Exercise 10.268
−16
−8
Exercise 10.269 (Solution on p. 625.)
25
−5
Exercise 10.270
36
−4
Exercise 10.271 (Solution on p. 625.)
8 − (−3)
Exercise 10.272
14 − (−20)
Exercise 10.273 (Solution on p. 625.)
20 − (−8)
Exercise 10.274
−4 − (−1)
Exercise 10.275 (Solution on p. 625.)
0−4
Exercise 10.276
0 − (−1)
Exercise 10.277 (Solution on p. 625.)
−6 + 1 − 7
Exercise 10.278
15 − 12 − 20
610 CHAPTER 10. SIGNED NUMBERS

Exercise 10.279 (Solution on p. 625.)


1−6−7+8
Exercise 10.280
2 + 7 − 10 + 2
Exercise 10.281 (Solution on p. 625.)
3 (4 − 6)
Exercise 10.282
8 (5 − 12)
Exercise 10.283 (Solution on p. 625.)
−3 (1 − 6)
Exercise 10.284
−8 (4 − 12) + 2
Exercise 10.285 (Solution on p. 625.)
−4 (1 − 8) + 3 (10 − 3)
Exercise 10.286
−9 (0 − 2) + 4 (8 − 9) + 0 (−3)
Exercise 10.287 (Solution on p. 625.)
6 (−2 − 9) − 6 (2 + 9) + 4 (−1 − 1)
Exercise 10.288
3(4+1)−2(5)
−2
Exercise 10.289 (Solution on p. 625.)
4(8+1)−3(−2)
−4−2
Exercise 10.290
−1(3+2)+5
−1
Exercise 10.291 (Solution on p. 625.)
−3(4−2)+(−3)(−6)
−4
Exercise 10.292
−1 (4 + 2)
Exercise 10.293 (Solution on p. 625.)
−1 (6 − 1)
Exercise 10.294
− (8 + 21)
Exercise 10.295 (Solution on p. 625.)
− (8 − 21)

10.7.5.1 Exercises for Review

Exercise 10.296
(Section 3.3) Use the order of operations to simplify 52 + 32 + 2 ÷ 22 .


Exercise 10.297 (Solution on p. 625.)


(Section 4.7) Find 3
8 of 32
9 .
Exercise 10.298
(Section 6.2) Write this number in decimal form using digits: fty-two three-thousandths
611

Exercise 10.299 (Solution on p. 625.)


(Section 7.4) The ratio of chlorine to water in a solution is 2 to 7. How many mL of water are in
a solution that contains 15 mL of chlorine?
Exercise 10.300
(Section 10.6) Perform the subtraction −8 − (−20)

10.8 Summary of Key Concepts9


10.8.1 Summary of Key Concepts
Variables and Constants (Section 10.2)
A variable is a letter or symbol that represents any member of a set of two or more numbers. A constant
is a letter or symbol that represents a specic number. For example, the Greek letter π (pi) represents the
constant 3.14159 . . . .

The Real Number Line (Section 10.2)


The real number line allows us to visually display some of the numbers in which we are interested.

Coordinate and Graph (Section 10.2)


The number associated with a point on the number line is called the coordinate of the point. The point
associated with a number is called the graph of the number.

Real Number (Section 10.2)


A real number is any number that is the coordinate of a point on the real number line.

Types of Real Numbers (Section 10.2)


The set of real numbers has many subsets. The ones of most interest to us are:
The natural numbers: {1, 2, 3, 4, . . .}
The whole numbers: {0, 1, 2, 3, 4, . . .}
The integers: {. . . ,-3,-2,-1,0, 1, 2, 3, . . .}
The rational numbers: {All numbers that can be expressed as the quotient of two integers.}

Positive and Negative Numbers (Section 10.3)


A number is denoted as positive if it is directly preceded by a plus sign (+) or no sign at all. A number is
denoted as negative if it is directly preceded by a minus sign ().

Opposites (Section 10.3)


Opposites are numbers that are the same distance from zero on the number line but have opposite signs.
The numbers a and −a are opposites.

Double-Negative Property (Section 10.3)


− (−a) = a

Absolute Value (Geometric) (Section 10.4)


The absolute value of a number a, denoted | a |, is the distance from a to 0 on the number line.

9 This content is available online at <http://cnx.org/content/m35034/1.2/>.


612 CHAPTER 10. SIGNED NUMBERS

Absolute Value (Algebraic) (Section 10.4)


a, if a ≥ 0
|a| = {
−a, if a < 0

Addition of Signed Numbers (Section 10.5)


To add two numbers with

1. like signs, add the absolute values of the numbers and associate with the sum the common sign.
2. unlike signs, subtract the smaller absolute value from the larger absolute value and associate with
the dierence the sign of the larger absolute value.

Addition with Zero (Section 10.5)


0 + (any number) = that particular number.

Additive Identity (Section 10.5)


Since adding 0 to any real number leaves that number unchanged, 0 is called the additive identity.

Denition of Subtraction (Section 10.6)


a − b = a + (−b)

Subtraction of Signed Numbers (Section 10.6)


To perform the subtraction a − b, add the opposite of b to a, that is, change the sign of b and follow the
addition rules (Section 10.5).

Multiplication and Division of Signed Numbers (Section 10.7)


(+) (+) = (+) (+) (+)
(+) = (+) (−) = (−)
(−) (−) = (+)
(+) (−) = (−) (−) (−)
(−) = (+) (+) = (−)
(−) (+) = (−)

10.9 Exercise Supplement 10


10.9.1 Exercise Supplement
10.9.1.1 Variables, Constants, and Real Numbers (Section 10.2)

For problems 1-5, next to each real number, note all subsets of the real numbers to which it belongs by
writing N for natural numbers, W for whole numbers, or Z for integers. Some numbers may belong to more
than one subset.
Exercise 10.301 (Solution on p. 625.)
61
Exercise 10.302
−14
Exercise 10.303 (Solution on p. 625.)
0
Exercise 10.304
1
Exercise 10.305 (Solution on p. 626.)
Write all the integers that are strictly between −4 and 3
10 This content is available online at <http://cnx.org/content/m35035/1.2/>.
613

Exercise 10.306
Write all the integers that are between and including −6 and −1
For each pair of numbers in problems 7-10, write the appropriate symbol (<, >, =) in place of the .
Exercise 10.307 (Solution on p. 626.)
-5  -1
Exercise 10.308
02
Exercise 10.309 (Solution on p. 626.)
-7  0
Exercise 10.310
-1  0
For problems 11-15, what numbers can replace x so that each statement is true?
Exercise 10.311 (Solution on p. 626.)
−5 ≤ x ≤ −1, x is an integer
Exercise 10.312
−10 < x ≤ 0, x is a whole number.
Exercise 10.313 (Solution on p. 626.)
0 ≤ x < 5, x is a natural number.
Exercise 10.314
−3 < x < 3, x is a natural number
Exercise 10.315 (Solution on p. 626.)
−8 < x ≤ −2, x is a whole number.
For problems 16-20, how many units are there between the given pair of numbers?
Exercise 10.316
0 and 4
Exercise 10.317 (Solution on p. 626.)
−1 and 3
Exercise 10.318
−7 and −4
Exercise 10.319 (Solution on p. 626.)
−6 and 0
Exercise 10.320
−1 and 1
Exercise 10.321 (Solution on p. 626.)
A number is positive if it is directly preceded by a sign or no sign at all.
Exercise 10.322
A number is negative if it is directly preceded by a sign.
614 CHAPTER 10. SIGNED NUMBERS

10.9.1.2 Signed Numbers (Section 10.3)

For problems 23-26, how should each number be read?


Exercise 10.323 (Solution on p. 626.)
−8
Exercise 10.324
− (−4)
Exercise 10.325 (Solution on p. 626.)
− (−1)
Exercise 10.326
−2
For problems 27-31, write each expression in words.
Exercise 10.327 (Solution on p. 626.)
1 + (−7)
Exercise 10.328
−2 − (−6)
Exercise 10.329 (Solution on p. 626.)
−1 − (+4)
Exercise 10.330
− (− (−3))
Exercise 10.331 (Solution on p. 626.)
0 − (−11)
For problems 32-36, rewrite each expression in simpler form.
Exercise 10.332
− (−4)
Exercise 10.333 (Solution on p. 626.)
− (−15)
Exercise 10.334
− [− (−7)]
Exercise 10.335 (Solution on p. 626.)
1 − (−18)
Exercise 10.336
0 − (−1)

10.9.1.3 Absolute Value (Section 10.4)

For problems 37-52, determine each value.


Exercise 10.337 (Solution on p. 626.)
|9|
Exercise 10.338
| 16 |
Exercise 10.339 (Solution on p. 626.)
| −5 |
Exercise 10.340
| −8 |
615

Exercise 10.341 (Solution on p. 626.)


− | −2 |
Exercise 10.342
− | −1 |
Exercise 10.343 (Solution on p. 626.)
− (− | 12 |)
Exercise 10.344
− (− | 90 |)
Exercise 10.345 (Solution on p. 626.)
− (− | −16 |)
Exercise 10.346
− (− | 0 |)
Exercise 10.347 (Solution on p. 626.)
2
| −4 |
Exercise 10.348
2
| −5 |
Exercise 10.349 (Solution on p. 626.)
3
| −2 |
Exercise 10.350
| − (3 · 4) |
Exercise 10.351 (Solution on p. 626.)
| −5 | + | −2 |
Exercise 10.352
| −7 | − | −10 |

10.9.1.4 Addition, Subtraction, Multiplication and Division of Signed Numbers (Sec-


tion 10.5,Section 10.6,Section 10.7)

For problems 53-71, perform each operation.


Exercise 10.353 (Solution on p. 626.)
−6 + 4
Exercise 10.354
−10 + 8
Exercise 10.355 (Solution on p. 627.)
−1 − 6
Exercise 10.356
8 − 12
Exercise 10.357 (Solution on p. 627.)
0 − 14
Exercise 10.358
5 · (−2)
Exercise 10.359 (Solution on p. 627.)
−8 · (−6)
Exercise 10.360
(−3) · (−9)
616 CHAPTER 10. SIGNED NUMBERS

Exercise 10.361 (Solution on p. 627.)


14 · (−3)
Exercise 10.362
5 · (−70)
Exercise 10.363 (Solution on p. 627.)
−18 ÷ −6
Exercise 10.364
72 ÷ −12
Exercise 10.365 (Solution on p. 627.)
−16 ÷ −16
Exercise 10.366
0÷ − 8
Exercise 10.367 (Solution on p. 627.)
−5 ÷ 0
Exercise 10.368
−15
−3
Exercise 10.369 (Solution on p. 627.)
−28
7
Exercise 10.370
−120
−|2|
Exercise 10.371 (Solution on p. 627.)
|−66|
−|−3|

10.10 Prociency Exam11


10.10.1 Prociency Exam
Exercise 10.372 (Solution on p. 627.)
(Section 10.2) Write all integers that are strictly between 8 and 3.
Exercise 10.373 (Solution on p. 627.)
(Section 10.2) Write all integers that are between and including 2 and 1.
For problems 3-5, write the appropriate symbol (<, >, =) in place of the  for each pair of numbers.
Exercise 10.374 (Solution on p. 627.)
(Section 10.2) 1  1
Exercise 10.375 (Solution on p. 627.)
(Section 10.2) 0  3
Exercise 10.376 (Solution on p. 627.)
(Section 10.2) 1  2
For problems 6 and 7, what numbers can replace x so that the statement is true?
Exercise 10.377 (Solution on p. 627.)
(Section 10.2) −3 ≤ x < 0, x is an integer.
Exercise 10.378 (Solution on p. 627.)
(Section 10.2) −4 ≤ x ≤ 0, x is a natural number.
11 This content is available online at <http://cnx.org/content/m35036/1.2/>.
617

Exercise 10.379 (Solution on p. 627.)


(Section 10.2) How many units are there between 3 and 2?
For problems 9-20, nd each value.
Exercise 10.380 (Solution on p. 627.)
(Section 10.4) | −16 |
Exercise 10.381 (Solution on p. 627.)
(Section 10.4) − | −2 |
Exercise 10.382  (Solution on p. 627.)
2
(Section 10.4) − −| −4 |
Exercise 10.383 (Solution on p. 627.)
(Section 10.4) | −5 | + | −10 |
Exercise 10.384 (Solution on p. 627.)
(Section 10.5) −8 + 6
Exercise 10.385 (Solution on p. 627.)
(Section 10.5) −3 + (−8)
Exercise 10.386 (Solution on p. 627.)
(Section 10.6) 0 − 16
Exercise 10.387 (Solution on p. 627.)
(Section 10.7) (−14) · (−3)
Exercise 10.388 (Solution on p. 628.)
2
(Section 10.7) (−5 − 6)
Exercise 10.389 (Solution on p. 628.)
(Section 10.7) (−51) ÷ (−7)
Exercise 10.390 (Solution on p. 628.)
−42
(Section 10.7) −7
Exercise 10.391 (Solution on p. 628.)
−32
(Section 10.7) | 8 − −155−5 |
618 CHAPTER 10. SIGNED NUMBERS

Solutions to Exercises in Chapter 10


Solution to Exercise 10.1 (p. 578)
yes
Solution to Exercise 10.2 (p. 578)
yes
Solution to Exercise 10.3 (p. 578)
yes
Solution to Exercise 10.4 (p. 578)
yes
Solution to Exercise 10.5 (p. 578)
yes
Solution to Exercise 10.6 (p. 579)
-5, -4, -3, -2, -1, 0
Solution to Exercise 10.7 (p. 579)

Solution to Exercise 10.8 (p. 579)


N, W, Z
Solution to Exercise 10.10 (p. 580)
W, Z
Solution to Exercise 10.12 (p. 580)
Z
Solution to Exercise 10.14 (p. 580)
Z
Solution to Exercise 10.16 (p. 580)
Neither
Solution to Exercise 10.18 (p. 580)

Solution to Exercise 10.20 (p. 580)


Yes, 10
Solution to Exercise 10.22 (p. 580)
<
Solution to Exercise 10.24 (p. 580)
>
Solution to Exercise 10.26 (p. 580)
{-1, 0, 1, 2, 3, 4, 5}
Solution to Exercise 10.28 (p. 580)
{1}
Solution to Exercise 10.30 (p. 580)
{0, 1, 2, 3, 4}
Solution to Exercise 10.32 (p. 581)
4
Solution to Exercise 10.34 (p. 581)
8
Solution to Exercise 10.36 (p. 581)
yes
Solution to Exercise 10.38 (p. 581)
yes
619

Solution to Exercise 10.40 (p. 581)


yes, -1
Solution to Exercise 10.42 (p. 581)
9
5 or 1 45 or 1.8
Solution to Exercise 10.44 (p. 581)
0.3006 m
Solution to Exercise 10.46 (p. 583)
six plus one
Solution to Exercise 10.47 (p. 583)
two plus negative eight
Solution to Exercise 10.48 (p. 583)
negative seven plus ve
Solution to Exercise 10.49 (p. 583)
negative ten minus three
Solution to Exercise 10.50 (p. 583)
negative one minus negative eight
Solution to Exercise 10.51 (p. 583)
zero plus negative eleven
Solution to Exercise 10.52 (p. 584)
-8
Solution to Exercise 10.53 (p. 584)
-17
Solution to Exercise 10.54 (p. 584)
6
Solution to Exercise 10.55 (p. 584)
15
Solution to Exercise 10.56 (p. 584)
-1
Solution to Exercise 10.57 (p. 584)
7
Solution to Exercise 10.58 (p. 584)
−a is negative
Solution to Exercise 10.59 (p. 584)
−a is positive
Solution to Exercise 10.60 (p. 584)
We must say that we do not know.
Solution to Exercise 10.61 (p. 584)
+ (or no sign)
Solution to Exercise 10.63 (p. 584)
negative seven
Solution to Exercise 10.65 (p. 585)
fteen
Solution to Exercise 10.67 (p. 585)
negative negative one, or opposite negative one
Solution to Exercise 10.69 (p. 585)
ve plus three
Solution to Exercise 10.71 (p. 585)
fteen plus negative three
Solution to Exercise 10.73 (p. 585)
negative seven minus negative two
620 CHAPTER 10. SIGNED NUMBERS

Solution to Exercise 10.75 (p. 585)


2
Solution to Exercise 10.77 (p. 585)
-8
Solution to Exercise 10.79 (p. 585)
7 + 3 = 10
Solution to Exercise 10.81 (p. 586)
0.296
Solution to Exercise 10.83 (p. 586)
6, 000 + 9, 000 = 15, 000 (5, 829 + 8, 767 = 14, 596) or 5,800 + 8, 800 = 14, 600
Solution to Exercise 10.85 (p. 586)
0.0025 hm
Solution to Exercise 10.86 (p. 587)
7
Solution to Exercise 10.87 (p. 587)
3
Solution to Exercise 10.88 (p. 587)
12
Solution to Exercise 10.89 (p. 587)
0
Solution to Exercise 10.90 (p. 587)
-9
Solution to Exercise 10.91 (p. 587)
-6
Solution to Exercise 10.92 (p. 588)
7
Solution to Exercise 10.93 (p. 588)
9
Solution to Exercise 10.94 (p. 588)
12
Solution to Exercise 10.95 (p. 588)
5
Solution to Exercise 10.96 (p. 588)
-8
Solution to Exercise 10.97 (p. 588)
-1
Solution to Exercise 10.98 (p. 588)
-52
Solution to Exercise 10.99 (p. 588)
-31
Solution to Exercise 10.100 (p. 588)
5
Solution to Exercise 10.102 (p. 588)
6
Solution to Exercise 10.104 (p. 589)
1
Solution to Exercise 10.106 (p. 589)
-3
Solution to Exercise 10.108 (p. 589)
-14
621

Solution to Exercise 10.110 (p. 589)


26
Solution to Exercise 10.112 (p. 589)
4
Solution to Exercise 10.114 (p. 589)
6
Solution to Exercise 10.116 (p. 589)
3
Solution to Exercise 10.118 (p. 589)
6
Solution to Exercise 10.120 (p. 589)
100
Solution to Exercise 10.122 (p. 589)
92
Solution to Exercise 10.124 (p. 589)
-1
Solution to Exercise 10.126 (p. 590)
−$ | −2, 400, 000 |
Solution to Exercise 10.128 (p. 590)
9
10
Solution to Exercise 10.130 (p. 590)
3 13 163
50 or 50
Solution to Exercise 10.132 (p. 590)
2
Solution to Exercise 10.133 (p. 592)
14
Solution to Exercise 10.134 (p. 592)
52
Solution to Exercise 10.135 (p. 592)
-12
Solution to Exercise 10.136 (p. 592)
-45
Solution to Exercise 10.137 (p. 592)
-34
Solution to Exercise 10.138 (p. 592)
− 73
Solution to Exercise 10.139 (p. 592)
−7.4
Solution to Exercise 10.140 (p. 592)
−16
Solution to Exercise 10.141 (p. 594)
1
Solution to Exercise 10.142 (p. 594)
2
Solution to Exercise 10.143 (p. 594)
-3
Solution to Exercise 10.144 (p. 594)
-6
Solution to Exercise 10.145 (p. 594)
-14
622 CHAPTER 10. SIGNED NUMBERS

Solution to Exercise 10.146 (p. 594)


-43
Solution to Exercise 10.147 (p. 594)
5
Solution to Exercise 10.148 (p. 594)
-0.5
Solution to Exercise 10.149 (p. 594)
-8
Solution to Exercise 10.150 (p. 594)
0.57
Solution to Exercise 10.151 (p. 594)
-425
Solution to Exercise 10.152 (p. 595)
-48
Solution to Exercise 10.153 (p. 595)
-6,085
Solution to Exercise 10.154 (p. 595)
-7,993.7
Solution to Exercise 10.155 (p. 595)
16
Solution to Exercise 10.157 (p. 595)
-15
Solution to Exercise 10.159 (p. 595)
8
Solution to Exercise 10.161 (p. 595)
-25
Solution to Exercise 10.163 (p. 595)
-12
Solution to Exercise 10.165 (p. 595)
24
Solution to Exercise 10.167 (p. 595)
-21
Solution to Exercise 10.169 (p. 596)
0
Solution to Exercise 10.171 (p. 596)
0
Solution to Exercise 10.173 (p. 596)
23
Solution to Exercise 10.175 (p. 596)
328
Solution to Exercise 10.177 (p. 596)
876
Solution to Exercise 10.179 (p. 596)
-1,255
Solution to Exercise 10.181 (p. 596)
-6.084
Solution to Exercise 10.183 (p. 596)
-$28.50
Solution to Exercise 10.185 (p. 596)
$3.00
623

Solution to Exercise 10.187 (p. 597)


17
36
Solution to Exercise 10.189 (p. 597)
62 31
100 = 50
Solution to Exercise 10.191 (p. 598)
3
Solution to Exercise 10.192 (p. 598)
-3
Solution to Exercise 10.193 (p. 598)
-7
Solution to Exercise 10.194 (p. 598)
-13
Solution to Exercise 10.195 (p. 598)
-20
Solution to Exercise 10.196 (p. 598)
-27
Solution to Exercise 10.197 (p. 598)
-2
Solution to Exercise 10.198 (p. 599)
18
Solution to Exercise 10.199 (p. 599)
13
Solution to Exercise 10.200 (p. 599)
118
Solution to Exercise 10.201 (p. 599)
-16
Solution to Exercise 10.202 (p. 599)
16
Solution to Exercise 10.203 (p. 599)
-8
Solution to Exercise 10.204 (p. 599)
10
Solution to Exercise 10.205 (p. 599)
0
Solution to Exercise 10.206 (p. 600)
-271
Solution to Exercise 10.207 (p. 600)
-2.247
Solution to Exercise 10.208 (p. 600)
-76.06
Solution to Exercise 10.209 (p. 600)
-0.182
Solution to Exercise 10.210 (p. 601)
5
Solution to Exercise 10.212 (p. 601)
-1
Solution to Exercise 10.214 (p. 601)
-14
Solution to Exercise 10.216 (p. 601)
-2
624 CHAPTER 10. SIGNED NUMBERS

Solution to Exercise 10.218 (p. 601)


-6
Solution to Exercise 10.220 (p. 601)
7
Solution to Exercise 10.222 (p. 601)
29
Solution to Exercise 10.224 (p. 601)
-324
Solution to Exercise 10.226 (p. 601)
-429
Solution to Exercise 10.228 (p. 601)
-71
Solution to Exercise 10.230 (p. 601)
164
Solution to Exercise 10.232 (p. 602)
-11
Solution to Exercise 10.234 (p. 602)
16.022
Solution to Exercise 10.236 (p. 602)
31.25%
Solution to Exercise 10.238 (p. 602)
-5
Solution to Exercise 10.239 (p. 604)
-24
Solution to Exercise 10.240 (p. 604)
64
Solution to Exercise 10.241 (p. 605)
30
Solution to Exercise 10.242 (p. 605)
14
Solution to Exercise 10.243 (p. 605)
-4
Solution to Exercise 10.244 (p. 605)
-49
Solution to Exercise 10.245 (p. 606)
4
Solution to Exercise 10.246 (p. 606)
-6
Solution to Exercise 10.247 (p. 607)
-2
Solution to Exercise 10.248 (p. 607)
3
Solution to Exercise 10.249 (p. 607)
14
Solution to Exercise 10.250 (p. 608)
1,108.08
Solution to Exercise 10.251 (p. 608)
1.226
Solution to Exercise 10.252 (p. 608)
-0.001
625

Solution to Exercise 10.253 (p. 608)


16
Solution to Exercise 10.255 (p. 608)
32
Solution to Exercise 10.257 (p. 609)
-36
Solution to Exercise 10.259 (p. 609)
-60
Solution to Exercise 10.261 (p. 609)
-12
Solution to Exercise 10.263 (p. 609)
3
Solution to Exercise 10.265 (p. 609)
-13
Solution to Exercise 10.267 (p. 609)
9
Solution to Exercise 10.269 (p. 609)
-5
Solution to Exercise 10.271 (p. 609)
11
Solution to Exercise 10.273 (p. 609)
28
Solution to Exercise 10.275 (p. 609)
-4
Solution to Exercise 10.277 (p. 609)
-12
Solution to Exercise 10.279 (p. 610)
-4
Solution to Exercise 10.281 (p. 610)
-6
Solution to Exercise 10.283 (p. 610)
15
Solution to Exercise 10.285 (p. 610)
49
Solution to Exercise 10.287 (p. 610)
-140
Solution to Exercise 10.289 (p. 610)
-7
Solution to Exercise 10.291 (p. 610)
-3
Solution to Exercise 10.293 (p. 610)
-5
Solution to Exercise 10.295 (p. 610)
13
Solution to Exercise 10.297 (p. 610)
4
3 = 1 13
Solution to Exercise 10.299 (p. 611)
52 21
Solution to Exercise 10.301 (p. 612)
N, W, Z
626 CHAPTER 10. SIGNED NUMBERS

Solution to Exercise 10.303 (p. 612)


W, Z
Solution to Exercise 10.305 (p. 612)
{−3, −2, −1, 0, 1, 2}
Solution to Exercise 10.307 (p. 613)
<
Solution to Exercise 10.309 (p. 613)
<
Solution to Exercise 10.311 (p. 613)
{−5, −4, −3, −2, −1}
Solution to Exercise 10.313 (p. 613)
{1, 2, 3, 4}
Solution to Exercise 10.315 (p. 613)
none
Solution to Exercise 10.317 (p. 613)
4
Solution to Exercise 10.319 (p. 613)
6
Solution to Exercise 10.321 (p. 613)
+ (plus)
Solution to Exercise 10.323 (p. 614)
negative eight
Solution to Exercise 10.325 (p. 614)
negative negative one or opposite negative one
Solution to Exercise 10.327 (p. 614)
one plus negative seven
Solution to Exercise 10.329 (p. 614)
negative one minus four
Solution to Exercise 10.331 (p. 614)
zero minus negative eleven
Solution to Exercise 10.333 (p. 614)
15
Solution to Exercise 10.335 (p. 614)
19 or 1 + 18
Solution to Exercise 10.337 (p. 614)
9
Solution to Exercise 10.339 (p. 614)
5
Solution to Exercise 10.341 (p. 615)
−2
Solution to Exercise 10.343 (p. 615)
12
Solution to Exercise 10.345 (p. 615)
16
Solution to Exercise 10.347 (p. 615)
16
Solution to Exercise 10.349 (p. 615)
8
Solution to Exercise 10.351 (p. 615)
7
627

Solution to Exercise 10.353 (p. 615)


−2
Solution to Exercise 10.355 (p. 615)
−7
Solution to Exercise 10.357 (p. 615)
−14
Solution to Exercise 10.359 (p. 615)
48
Solution to Exercise 10.361 (p. 616)
−42
Solution to Exercise 10.363 (p. 616)
3
Solution to Exercise 10.365 (p. 616)
1
Solution to Exercise 10.367 (p. 616)
not dened
Solution to Exercise 10.369 (p. 616)
−4
Solution to Exercise 10.371 (p. 616)
−22
Solution to Exercise 10.372 (p. 616)
{−7, −6, −5, −4}
Solution to Exercise 10.373 (p. 616)
{−2, −1, 0, 1}
Solution to Exercise 10.374 (p. 616)
=
Solution to Exercise 10.375 (p. 616)
<
Solution to Exercise 10.376 (p. 616)
>
Solution to Exercise 10.377 (p. 616)
{−3, −2, −1}
Solution to Exercise 10.378 (p. 616)
{1, 2}
Solution to Exercise 10.379 (p. 617)
5
Solution to Exercise 10.380 (p. 617)
16
Solution to Exercise 10.381 (p. 617)
−2
Solution to Exercise 10.382 (p. 617)
16
Solution to Exercise 10.383 (p. 617)
15
Solution to Exercise 10.384 (p. 617)
−2
Solution to Exercise 10.385 (p. 617)
−11
Solution to Exercise 10.386 (p. 617)
−16
628 CHAPTER 10. SIGNED NUMBERS

Solution to Exercise 10.387 (p. 617)


42
Solution to Exercise 10.388 (p. 617)
121
Solution to Exercise 10.389 (p. 617)
51 or 7 2
7 7
Solution to Exercise 10.390 (p. 617)
6
Solution to Exercise 10.391 (p. 617)
0
Chapter 11

Algebraic Expressions and Equations

11.1 Objectives1
After completing this chapter, you should
Algebraic Expressions (Section 11.2)

• be able to recognize an algebraic expression


• be able to distinguish between terms and factors
• understand the meaning and function of coecients
• be able to perform numerical evaluation

Combining Like Terms Using Addition and Subtraction (Section 11.3)

• be able to combine like terms in an algebraic expression

Solving Equations of the Form x + a = b and x − a = b (Section 11.4)


• understand the meaning and function of an equation
• understand what is meant by the solution to an equation
• be able to solve equations of the form x + a = b and x − a = b
x
Solving Equations of the Form ax = b and a = b (Section 11.5)
• be familiar with the multiplication/division property of equality
• be able to solve equations of the form ax = b and xa = b
• be able to use combined techniques to solve equations

Applications I: Translating Words to Mathematical Symbols (Section 11.6)

• be able to translate phrases and statements to mathematical expressions and equations

Applications II: Solving Problems (Section 11.7)

• be more familiar with the ve-step method for solving applied problems
• be able to use the ve-step method to solve number problems and geometry problems

1 This content is available online at <http://cnx.org/content/m18899/1.3/>.

629
630 CHAPTER 11. ALGEBRAIC EXPRESSIONS AND EQUATIONS

11.2 Algebraic Expressions2


11.2.1 Section Overview
• Algebraic Expressions
• Terms and Factors
• Coecients
• Numerical Evaluation

11.2.2 Algebraic Expressions


Numerical Expression
In arithmetic, a numerical expression results when numbers are connected by arithmetic operation signs
(+, -, · , ÷). For example, 8 + 5, 4 − 9, 3 · 8, and 9 ÷ 7 are numerical expressions.

Algebraic Expression
In algebra, letters are used to represent numbers, and an algebraic expression results when an arithmetic
operation sign associates a letter with a number or a letter with a letter. For example, x + 8, 4 − y , 3 · x ,
x ÷ 7 , and x · y are algebraic expressions.

Expressions
Numerical expressions and algebraic expressions are often referred to simply as expressions.

11.2.3 Terms and Factors


In algebra, it is extremely important to be able to distinguish between terms and factors.

Distinction Between Terms and Factors


Terms are parts of sums and are therefore connected by + signs.
Factors are parts of products and are therefore separated by · signs.

note: While making the distinction between sums and products, we must remember that subtrac-
tion and division are functions of these operations.

1. In some expressions it will appear that terms are separated by minus signs. We must keep in mind
that subtraction is addition of the opposite, that is,
x − y = x + (−y)
2. In some expressions it will appear that factors are separated by division signs. We must keep in mind
that
x x 1 1
y = 1 · y =x· y

11.2.3.1 Sample Set A

State the number of terms in each expression and name them.


Example 11.1
x + 4. In this expression, x and 4 are connected by a "+" sign. Therefore, they are terms. This
expression consists of two terms.
Example 11.2
y − 8. The expression y − 8 can be expressed as y + (−8). We can now see that this expression
consists of the two terms y and −8.
2 This content is available online at <http://cnx.org/content/m35038/1.2/>.
631

Rather than rewriting the expression when a subtraction occurs, we can identify terms more quickly
by associating the + or - sign with the individual quantity.
Example 11.3
a + 7 − b − m. Associating the sign with the individual quantities, we see that this expression
consists of the four terms a, 7, −b, −m.
Example 11.4
5m − 8n. This expression consists of the two terms, 5m and −8n. Notice that the term 5m is
composed of the two factors 5 and m. The term −8n is composed of the two factors −8 and n.
Example 11.5
3x. This expression consists of one term. Notice that 3x can be expressed as 3x + 0 or 3x ·
1 (indicating the connecting signs of arithmetic). Note that no operation sign is necessary for
multiplication.

11.2.3.2 Practice Set A

Specify the terms in each expression.


Exercise 11.1 (Solution on p. 680.)
x+7
Exercise 11.2 (Solution on p. 680.)
3m − 6n
Exercise 11.3 (Solution on p. 680.)
5y
Exercise 11.4 (Solution on p. 680.)
a + 2b − c
Exercise 11.5 (Solution on p. 680.)
−3x − 5

11.2.4 Coecients
We know that multiplication is a description of repeated addition. For example,
5 · 7 describes 7 + 7 + 7 + 7 + 7

Suppose some quantity is represented by the letter x. The multiplication 5x describes x + x + x + x + x. It


is now easy to see that 5x species 5 of the quantities represented by x. In the expression 5x, 5 is called the
numerical coecient, or more simply, the coecient of x.

Coecient
The coecient of a quantity records how many of that quantity there are.

Since constants alone do not record the number of some quantity, they are not usually considered as numerical
coecients. For example, in the expression 7x + 2y − 8z + 12, the coecient of

7x is 7. (There are 7 x 's.)


2y is 2. (There are 2 y 's.)
−8z is −8. (There are −8z 's.)

The constant 12 is not considered a numerical coecient.


632 CHAPTER 11. ALGEBRAIC EXPRESSIONS AND EQUATIONS

1x = x
When the numerical coecient of a variable is 1, we write only the variable and not the coecient. For
example, we write x rather than 1x. It is clear just by looking at x that there is only one.

11.2.5 Numerical Evaluation


We know that a variable represents an unknown quantity. Therefore, any expression that contains a variable
represents an unknown quantity. For example, if the value of x is unknown, then the value of 3x + 5 is
unknown. The value of 3x + 5 depends on the value of x.

Numerical Evaluation
Numerical evaluation is the process of determining the numerical value of an algebraic expression by
replacing the variables in the expression with specied numbers.

11.2.5.1 Sample Set B

Find the value of each expression.


Example 11.6
2x + 7y , if x = −4 and y = 2

Replace x with 4 and y with 2.

2x + 7y = 2 (−4) + 7 (2)
= −8 + 14
= 6

Thus, when x = − − 4 and y = 2, 2x + 7y = 6.


Example 11.7
12 , if a = 6 and b = −3.
5a 8b
b +

Replace a with 6 and b with 3.


5a 8b 5(6) 8(−3)
b + 12 = −3 + 12
30 −
= − + 24
= −10 + (−2)
= −12

Thus, when a = 6 and b = 3, 5a


b
8b
+ 12 = −12.
Example 11.8
6 (2a − 15b), if a = −5 and b = −1

Replace a with 5 and b with 1.

6 (2a − 15b) = 6 (2 (−5) − 15 (−1))


= 6 (−10 + 15)
= 6 (5)
= 30

Thus, when a = − − 5 and b = − − 1, 6 (2a − 15b) = 30.


633

Example 11.9
3x2 − 2x + 1, if x = 4

Replace x with 4.
2
3x2 − 2x + 1 = 3(4) − 2 (4) + 1
= 3 · 16 − 2 (4) + 1
= 48 − 8 + 1
= 41

Thus, when x = 4, 3x2 − 2x + 1 = 41.


Example 11.10
−x2 − 4, if x = 3

Replace x with 3.

−x2 − 4 = −3 − 4 Be careful to square only the 3. The exponent 2 is connected only to 3, not -3
= −9 − 4
= −13
Example 11.11
2
(−x) − 4, if x = 3.

Replace x with 3.
2 2
(−x) − 4 = (−3) − 4 The exponent is connected to -3, not 3 as in problem 5 above.
= 9−4
= −5

The exponent is connected to 3, not 3 as in the problem above.

11.2.5.2 Practice Set B

Find the value of each expression.


Exercise 11.6 (Solution on p. 680.)
9m − 2n, if m = −2 and n = 5
Exercise 11.7 (Solution on p. 680.)
−3x − 5y + 2z , if x = −4, y = 3, z = 0
Exercise 11.8 (Solution on p. 680.)
10a +
2 , if a = −6, and b = 2
4b
3b
Exercise 11.9 (Solution on p. 680.)
8 (3m − 5n), if m = −4 and n = −5
Exercise 11.10 (Solution on p. 680.)
3 [−40 − 2 (4a − 3b)], if a = −6 and b = 0
Exercise 11.11 (Solution on p. 680.)
5y 2 + 6y − 11, if y = −1
634 CHAPTER 11. ALGEBRAIC EXPRESSIONS AND EQUATIONS

Exercise 11.12 (Solution on p. 680.)


−x2 + 2x + 7, if x = 4
Exercise 11.13 (Solution on p. 680.)
2
(−x) + 2x + 7, if x = 4

11.2.6 Exercises
Exercise 11.14 (Solution on p. 680.)
In an algebraic expression, terms are separated by signs and factors are separated
by signs.
For the following 8 problems, specify each term.
Exercise 11.15
3m + 7n
Exercise 11.16 (Solution on p. 680.)
5x + 18y
Exercise 11.17
4a − 6b + c
Exercise 11.18 (Solution on p. 680.)
8s + 2r − 7t
Exercise 11.19
m − 3n − 4a + 7b
Exercise 11.20 (Solution on p. 680.)
7a − 2b − 3c − 4d
Exercise 11.21
−6a − 5b
Exercise 11.22 (Solution on p. 680.)
−x − y
Exercise 11.23
What is the function of a numerical coecient?
Exercise 11.24 (Solution on p. 680.)
Write 1m in a simpler way.
Exercise 11.25
Write 1s in a simpler way.
Exercise 11.26 (Solution on p. 680.)
In the expression 5a, how many a's are indicated?
Exercise 11.27
In the expression 7c, how many c 's are indicated?
Find the value of each expression.
Exercise 11.28 (Solution on p. 680.)
2m − 6n, if m = −3 and n = 4
Exercise 11.29
5a + 6b, if a = −6 and b = 5
Exercise 11.30 (Solution on p. 680.)
2x − 3y + 4z , if x = 1, y = −1, and z = −2
635

Exercise 11.31
9a + 6b − 8x + 4y , if a = −2, b = −1, x = −2, and y = 0
Exercise 11.32 (Solution on p. 680.)
8x
3y + 18
2x , if x = 9 and y = −2
y

Exercise 11.33
−3m
2n − −6n
m , if m = −6 and n = 3
Exercise 11.34 (Solution on p. 680.)
4 (3r + 2s), if r = 4 and s = 1
Exercise 11.35
3 (9a − 6b), if a = −1 and b = −2
Exercise 11.36 (Solution on p. 680.)
−8 (5m + 8n), if m = 0 and n = −1
Exercise 11.37
−2 (−6x + y − 2z), if x = 1, y = 1, and z = 2
Exercise 11.38 (Solution on p. 680.)
− (10x − 2y + 5z) if x = 2, y = 8, and z = −1
Exercise 11.39
− (a − 3b + 2c − d), if a = −5, b = 2, c = 0, and d = −1
Exercise 11.40 (Solution on p. 681.)
3 [16 − 3 (a + 3b)], if a = 3 and b = −2
Exercise 11.41
−2 [5a + 2b (b − 6)], if a = −2 and b = 3
Exercise 11.42 (Solution on p. 681.)
−{6x + 3y [−2 (x + 4y)]}, if x = 0 and y = 1
Exercise 11.43
−2{19 − 6 [4 − 2 (a − b − 7)]}, if a = 10 and b = 3
Exercise 11.44 (Solution on p. 681.)
x2 + 3x − 1, if x = 5
Exercise 11.45
m2 − 2m + 6, if m = 3
Exercise 11.46 (Solution on p. 681.)
6a2 + 2a − 15, if a = −2
Exercise 11.47
5s2 + 6s + 10, if x = −1
Exercise 11.48 (Solution on p. 681.)
16x2 + 8x − 7, if x = 0
Exercise 11.49
−8y 2 + 6y + 11, if y = 0
Exercise 11.50 (Solution on p. 681.)
2
(y − 6) + 3 (y − 5) + 4, if y = 5
Exercise 11.51
2
(x + 8) + 4 (x + 9) + 1, if x = −6
636 CHAPTER 11. ALGEBRAIC EXPRESSIONS AND EQUATIONS

11.2.6.1 Exercises for Review

Exercise 11.52 (Solution on p. 681.)


(Section 5.3) Perform the addition: 5 83 + 2 16 .
Exercise 11.53
(Section 5.5) Arrange the numbers in order from smallest to largest: 11 15
32 , 48 , and 16
7

Exercise 11.54 (Solution on p. 681.)


2 2
(Section 5.7) Find the value of 8

3 + 27
Exercise 11.55
(Section 7.3) Write the proportion in fractional form: 9 is to 8 as x is to 7.
Exercise 11.56 (Solution on p. 681.)
(Section 10.7) Find the value of −3 (2 − 6) − 12

11.3 Combining Like Terms Using Addition and Subtraction3


11.3.1 Section Overview
• Combining Like Terms

11.3.2 Combining Like Terms


From our examination of terms in Section 11.2, we know that like terms are terms in which the variable
parts are identical. Like terms is an appropriate name since terms with identical variable parts and dierent
numerical coecients represent dierent amounts of the same quantity. When we are dealing with quantities
of the same type, we may combine them using addition and subtraction.

Simplifying an Algebraic Expression


An algebraic expression may be simplied by combining like terms.

This concept is illustrated in the following examples.

1. 8 records + 5 records = 13 records.


Eight and 5 of the same type give 13 of that type. We have combined quantities of the same type.
2. 8 records + 5 records + 3 tapes = 13 records + 3 tapes. Eight and 5 of the same type give 13 of that
type. Thus, we have 13 of one type and 3 of another type. We have combined only quantities of the
same type.
3. Suppose we let the letter x represent "record." Then, 8x + 5x = 13x. The terms 8x and 5x are like
terms. So, 8 and 5 of the same type give 13 of that type. We have combined like terms.
4. Suppose we let the letter x represent "record" and y represent "tape." Then, 8x + 5x + 3y = 13x + 5y

We have combined only the like terms.

After observing the problems in these examples, we can suggest a method for simplifying an algebraic
expression by combining like terms.

Combining Like Terms


Like terms may be combined by adding or subtracting their coecients and axing the result to the common
variable.
3 This content is available online at <http://cnx.org/content/m35039/1.2/>.
637

11.3.2.1 Sample Set A

Simplify each expression by combining like terms.


Example 11.12
2m + 6m − 4m. All three terms are alike. Combine their coecients and ax this result to m:
2 + 6 − 4 = 4.

Thus, 2m + 6m − 4m = 4m .
Example 11.13
5x + 2y − 9y . The terms 2y and −9y are like terms. Combine their coecients: 2 − 9 = −7.

Thus, 5x + 2y − 9y = 5x − 7y .
Example 11.14
−3a + 2b − 5a + a + 6b. The like terms are

−3a, − 5a, a 2b, 6b


| {z } | {z }
−3−5+1=−7 2+6=8
−7a 8b

Thus, −3a + 2b − 5a + a + 6b= − 7a + 8b.


Example 11.15
r − 2s + 7s + 3r − 4r − 5s . The like terms are

Thus, r − 2s + 7s + 3r − 4r − 5s = 0 .

11.3.2.2 Practice Set A

Simplify each expression by combining like terms.


Exercise 11.57 (Solution on p. 681.)
4x + 3x + 6x
Exercise 11.58 (Solution on p. 681.)
5a + 8b + 6a − 2b
Exercise 11.59 (Solution on p. 681.)
10m − 6n − 2n − m + n
Exercise 11.60 (Solution on p. 681.)
16a + 6m + 2r − 3r − 18a + m − 7m
Exercise 11.61 (Solution on p. 681.)
5h − 8k + 2h − 7h + 3k + 5k
638 CHAPTER 11. ALGEBRAIC EXPRESSIONS AND EQUATIONS

11.3.3 Exercises
Simplify each expression by combining like terms.
Exercise 11.62 (Solution on p. 681.)
4a + 7a
Exercise 11.63
3m + 5m
Exercise 11.64 (Solution on p. 681.)
6h − 2h
Exercise 11.65
11k − 8k
Exercise 11.66 (Solution on p. 681.)
5m + 3n − 2m
Exercise 11.67
7x − 6x + 3y
Exercise 11.68 (Solution on p. 681.)
14s + 3s − 8r + 7r
Exercise 11.69
−5m − 3n + 2m + 6n
Exercise 11.70 (Solution on p. 681.)
7h + 3a − 10k + 6a − 2h − 5k − 3k
Exercise 11.71
4x − 8y − 3z + x − y − z − 3y − 2z
Exercise 11.72 (Solution on p. 681.)
11w + 3x − 6w − 5w + 8x − 11x
Exercise 11.73
15r − 6s + 2r + 8s − 6r − 7s − s − 2r
Exercise 11.74 (Solution on p. 681.)
| −7 | m+ | 6 | m+ | −3 | m
Exercise 11.75
| −2 | x+ | −8 | x+ | 10 | x
Exercise 11.76 (Solution on p. 681.)
(−4 + 1) k + (6 − 3) k + (12 − 4) h + (5 + 2) k
Exercise 11.77
(−5 + 3) a − (2 + 5) b − (3 + 8) b
Exercise 11.78 (Solution on p. 681.)
5 ? +2∆ + 3∆ − 8?
Exercise 11.79
9  +10  −11  −12
Exercise 11.80 (Solution on p. 681.)
16x − 12y + 5x + 7 − 5x − 16 − 3y
Exercise 11.81
−3y + 4z − 11 − 3z − 2y + 5 − 4 (8 − 3)
639

11.3.3.1 Exercises for Review

Exercise 11.82 (Solution on p. 681.)


(Section 4.3) Convert 24
11 to a mixed number
Exercise 11.83
(Section 4.4) Determine the missing numerator: 3
8
?
= 64 .
Exercise 11.84 (Solution on p. 682.)
5 1
6−4
(Section 5.6) Simplify 1 .
12

Exercise 11.85
(Section 7.5) Convert 16
5
to a percent.
Exercise 11.86 (Solution on p. 682.)
(Section 11.2) In the expression 6k , how many k 's are there?

11.4 Solving Equations of the Form x+a=b and x-a=b4


11.4.1 Section Overview
• Equations
• Solutions and Equivalent Equations
• Solving Equations

11.4.2 Equations
Equation
An equation is a statement that two algebraic expressions are equal.

The following are examples of equations:

x+6 = 10 x−4 = −11 3y − 5 = −2 + 2y


[U+FE38] |{z} | {z } |{z}
This
| {z } | {z }
This This This This This
expression = expression expression = expression expression = expression

Notice that x + 6, x − 4, and 3y − 5 are not equations. They are expressions. They are not equations because
there is no statement that each of these expressions is equal to another expression.

11.4.3 Solutions and Equivalent Equations


Conditional Equations
The truth of some equations is conditional upon the value chosen for the variable. Such equations are called
conditional equations. There are two additional types of equations. They are examined in courses in
algebra, so we will not consider them now.

Solutions and Solving an Equation


The set of values that, when substituted for the variables, make the equation true, are called the solutions
of the equation.
An equation has been solved when all its solutions have been found.
4 This content is available online at <http://cnx.org/content/m35044/1.3/>.
640 CHAPTER 11. ALGEBRAIC EXPRESSIONS AND EQUATIONS

11.4.3.1 Sample Set A

Example 11.16
Verify that 3 is a solution to x + 7 = 10.

When x = 3,

x+7 = 10
becomes 3+7 = 10
which is a true statement, verifying that
10 = 10
3 is a solution to x + 7 = 10
Example 11.17
Verify that −6 is a solution to 5y + 8 = −22

When y = −6,

5y + 8 = −22
becomes 5 (−6) + 8 = −22
−30 + 8 = −22
which is a true statement, verifying that
−22 = −22
−6 is a solution to 5y + 8 = −22
Example 11.18
Verify that 5 is not a solution to a − 1 = 2a + 3.

When a = 5,

a−1 = 2a + 3
becomes 5−1 = 2·5+3
5−1 = 10 + 3
a false statement, verifying that 5
4 = 13
is not a solution to a − 1 = 2a + 3
Example 11.19
Verify that -2 is a solution to 3m − 2 = −4m − 16.

When m = −2,

3m − 2 = −4m − 16
becomes 3 (−2) − 2 = −4 (−2) − 16
−6 − 2 = 8 − 16
which is a true statement, verifying that
−8 = −8
−2 is a solution to 3m − 2 = −4m − 16
641

11.4.3.2 Practice Set A

Exercise 11.87 (Solution on p. 682.)


Verify that 5 is a solution to m + 6 = 11.
Exercise 11.88 (Solution on p. 682.)
Verify that −5 is a solution to 2m − 4 = −14.
Exercise 11.89 (Solution on p. 682.)
Verify that 0 is a solution to 5x + 1 = 1.
Exercise 11.90 (Solution on p. 682.)
Verify that 3 is not a solution to −3y + 1 = 4y + 5.
Exercise 11.91 (Solution on p. 682.)
Verify that -1 is a solution to 6m − 5 + 2m = 7m − 6.
Equivalent Equations

Some equations have precisely the same collection of solutions. Such equations are called equivalent equa-
tions. For example, x − 5 = −1, x + 7 = 11, and x = 4 are all equivalent equations since the only solution
to each is x = 4. (Can you verify this?)

11.4.4 Solving Equations


We know that the equal sign of an equation indicates that the number represented by the expression on the
left side is the same as the number represented by the expression on the right side.

This number is the same as this number


↓ ↓ ↓
x = 4
x+7 = 11
x−5 = -1

Table 11.1

Addition/Subtraction Property of Equality


From this, we can suggest the addition/subtraction property of equality.
Given any equation,

1. We can obtain an equivalent equation by adding the same number to both sides of the equation.
2. We can obtain an equivalent equation by subtracting the same number from both sides of the equation.

The Idea Behind Equation Solving


The idea behind equation solving is to isolate the variable on one side of the equation. Signs of operation
(+, -, ·,÷) are used to associate two numbers. For example, in the expression 5 + 3, the numbers 5 and
3 are associated by addition. An association can be undone by performing the opposite operation. The
addition/subtraction property of equality can be used to undo an association that is made by addition or
subtraction.

Subtraction is used to undo an addition.

Addition is used to undo a subtraction.

The procedure is illustrated in the problems of .


642 CHAPTER 11. ALGEBRAIC EXPRESSIONS AND EQUATIONS

11.4.4.1 Sample Set B

Use the addition/subtraction property of equality to solve each equation.


Example 11.20
x + 4 = 6.
4 is associated with x by addition. Undo the association by subtracting 4 from both sides.

x+4−4=6−4
x+0=2
x=2

Check: When x = 2, x + 4 becomes

The solution to x + 4 = 6 is x = 2.
Example 11.21
m − 8 = 5. 8 is associated with m by subtraction. Undo the association by adding 8 to both sides.

m−8+8=5+8
m + 0 = 13
m = 13

Check: When m = 13,


becomes

a true statement.

The solution to m − 8 = 5 is m = 13.


Example 11.22
−3 − 5 = y − 2 + 8. Before we use the addition/subtraction property, we should simplify as much
as possible.

−3 − 5 = y − 2 + 8

−8 = y + 6
6 is associated with y by addition. Undo the association by subtracting 6 from both sides.

−8 − 6 = y + 6 − 6
−14 = y + 0
−14 = y
This is equivalent to y = −14.

Check: When y = −14,


−3 − 5 = y − 2 + 8
643

becomes

,
a true statement.

The solution to −3 − 5 = y − 2 + 8 is y = −14.


Example 11.23
−5a + 1 + 6a = −2. Begin by simplifying the left side of the equation.

−5a + 1 + 6a = −2
| {z }
−5+6=1

a + 1 = −2 1 is associated with a by addition. Undo the association by subtracting 1 from both


sides.

a + 1 − 1 = −2 − 1
a + 0 = −3
a = −3

Check: When a = −3,


−5a + 1 + 6a = −2

becomes

,
a true statement.

The solution to −5a + 1 + 6a = −2 is a = −3.


Example 11.24
7k − 4 = 6k + 1. In this equation, the variable appears on both sides. We need to isolate it on one
side. Although we can choose either side, it will be more convenient to choose the side with the
larger coecient. Since 8 is greater than 6, we'll isolate k on the left side.

7k − 4 = 6k + 1 Since 6k represents +6k , subtract 6k from each side.

7k − 4 − 6k = 6k + 1 − 6k
| {z } | {z }
7−6=1 6−6=0

k − 4 = 1 4 is associated with k by subtraction. Undo the association by adding 4 to both sides.

k−4+4=1+4
k=5

Check: When k = 5,
7k − 4 = 6k + 1

becomes

a true statement.
644 CHAPTER 11. ALGEBRAIC EXPRESSIONS AND EQUATIONS

The solution to 7k − 4 = 6k + 1 is k = 5.
Example 11.25
−8 + x = 5. -8 is associated with x by addition. Undo the by subtracting -8 from both sides.
Subtracting -8 we get − (−8) =+8. We actually add 8 to both sides.

−8 + x + 8 = 5 + 8

x = 13

Check: When x = 13
−8 + x = 5

becomes

,
a true statement.

The solution to −8 + x = 5 is x = 13.

11.4.4.2 Practice Set B

Exercise 11.92 (Solution on p. 682.)


y+9=4
Exercise 11.93 (Solution on p. 682.)
a − 4 = 11
Exercise 11.94 (Solution on p. 682.)
−1 + 7 = x + 3
Exercise 11.95 (Solution on p. 682.)
8m + 4 − 7m = (−2) (−3)
Exercise 11.96 (Solution on p. 682.)
12k − 4 = 9k − 6 + 2k
Exercise 11.97 (Solution on p. 682.)
−3 + a = −4

11.4.5 Exercises
For the following 10 problems, verify that each given value is a solution to the given equation.
Exercise 11.98 (Solution on p. 682.)
x − 11 = 5, x = 16
Exercise 11.99
y − 4 = −6, y = −2
Exercise 11.100 (Solution on p. 682.)
2m − 1 = 1, m = 1
Exercise 11.101
5y + 6 = −14, y = −4
645

Exercise 11.102 (Solution on p. 683.)


3x + 2 − 7x = −5x − 6, x = −8
Exercise 11.103
−6a + 3 + 3a = 4a + 7 − 3a, a = −1
Exercise 11.104 (Solution on p. 683.)
−8 + x = −8, x = 0
Exercise 11.105
8b + 6 = 6 − 5b, b = 0
Exercise 11.106 (Solution on p. 683.)
4x − 5 = 6x − 20, x = 15
2
Exercise 11.107
−3y + 7 = 2y − 15, y = 22
5
Solve each equation. Be sure to check each result.
Exercise 11.108 (Solution on p. 683.)
y−6=5
Exercise 11.109
m+8=4
Exercise 11.110 (Solution on p. 683.)
k−1=4
Exercise 11.111
h−9=1
Exercise 11.112 (Solution on p. 683.)
a + 5 = −4
Exercise 11.113
b − 7 = −1
Exercise 11.114 (Solution on p. 683.)
x+4−9=6
Exercise 11.115
y − 8 + 10 = 2
Exercise 11.116 (Solution on p. 683.)
z+6=6
Exercise 11.117
w − 4 = −4
Exercise 11.118 (Solution on p. 683.)
x+7−9=6
Exercise 11.119
y−2+5=4
Exercise 11.120 (Solution on p. 683.)
m + 3 − 8 = −6 + 2
Exercise 11.121
z + 10 − 8 = −8 + 10
Exercise 11.122 (Solution on p. 683.)
2+9=k−8
Exercise 11.123
−5 + 3 = h − 4
646 CHAPTER 11. ALGEBRAIC EXPRESSIONS AND EQUATIONS

Exercise 11.124 (Solution on p. 683.)


3m − 4 = 2m + 6
Exercise 11.125
5a + 6 = 4a − 8
Exercise 11.126 (Solution on p. 683.)
8b + 6 + 2b = 3b − 7 + 6b − 8
Exercise 11.127
12h − 1 − 3 − 5h = 2h + 5h + 3 (−4)
Exercise 11.128 (Solution on p. 683.)
−4a + 5 − 2a = −3a − 11 − 2a
Exercise 11.129
−9n − 2 − 6 + 5n = 3n − (2) (−5) − 6n
Calculator Exercises

Exercise 11.130 (Solution on p. 683.)


y − 2.161 = 5.063
Exercise 11.131
a − 44.0014 = −21.1625
Exercise 11.132 (Solution on p. 683.)
−0.362 − 0.416 = 5.63m − 4.63m
Exercise 11.133
8.078 − 9.112 = 2.106y − 1.106y
Exercise 11.134 (Solution on p. 683.)
4.23k + 3.18 = 3.23k − 5.83
Exercise 11.135
6.1185x − 4.0031 = 5.1185x − 0.0058
Exercise 11.136 (Solution on p. 683.)
21.63y + 12.40 − 5.09y = 6.11y − 15.66 + 9.43y
Exercise 11.137
0.029a − 0.013 − 0.034 − 0.057 = −0.038 + 0.56 + 1.01a

11.4.5.1 Exercises for Review

Exercise 11.138 (Solution on p. 683.)


(Section 7.2) Is 712calculators
students an example of a ratio or a rate?
Exercise 11.139
(Section 7.5) Convert 38 % to a decimal.
Exercise 11.140 (Solution on p. 683.)
(Section 7.6) 0.4% of what number is 0.014?
Exercise 11.141
(Section 8.3) Use the clustering method to estimate the sum: 89 + 93 + 206 + 198 + 91
Exercise 11.142 (Solution on p. 684.)
(Section 11.3) Combine like terms: 4x + 8y + 12y + 9x − 2y .
647

11.5 Solving Equations of the Form ax=b and x/a=b5


11.5.1 Section Overview
• Multiplication/ Division Property of Equality
• Combining Techniques in Equations Solving

11.5.2 Multiplication/ Division Property of Equality


Recall that the equal sign of an equation indicates that the number represented by the expression on the left
side is the same as the number represented by the expression on the right side. From this, we can suggest
the multiplication/division property of equality.

Multiplication/Division Property of Equality


Given any equation,

1. We can obtain an equivalent equation by multiplying both sides of the equation by the same nonzero
number, that is, if c 6= 0, then a = b is equivalent to
a·c=b·c
2. We can obtain an equivalent equation by dividing both sides of the equation by the same nonzero
number, that is, if c 6= 0, then a = b is equivalent to
a b
c = c

The multiplication/division property of equality can be used to undo an association with a number that
multiplies or divides the variable.

11.5.2.1 Sample Set A

Use the multiplication / division property of equality to solve each equation.


Example 11.26
6y = 54
6 is associated with y by multiplication. Undo the association by dividing both sides by 6
6y
6 = 54
6
9
)6y )54
=
)6 )6
y=9

Check: When y = 9
6y = 54

becomes

,
a true statement.

The solution to 6y = 54 is y = 9.
5 This content is available online at <http://cnx.org/content/m35045/1.3/>.
648 CHAPTER 11. ALGEBRAIC EXPRESSIONS AND EQUATIONS

Example 11.27
x
−2 = 27.
-2 is associated with x by division. Undo the association by multiplying both sides by -2.
x
(−2) −2 = (−2) 27
 
)−2 x
= (−2) 27
)−2

x = −54

Check: When x = −54,


x
−2 = 27

becomes

a true statement.

The solution to x
−2 = 27 is x = −54
Example 11.28
3a
7= 6.
We will examine two methods for solving equations such as this one.

Method 1: Use of dividing out common factors.


3a
7 =6
7 is associated with a by division. Undo the association by multiplying both sides by 7.

7 · 3a
7 =7·6
Divide out the 7's.

)7 · 3a
= 42
)7

3a = 42
3 is associated with a by multiplication. Undo the association by dviding both sides by 3.
3a
3 = 42
3

)3a
= 14
)3

a = 14

Check: When a = 14,


3a
7 =6
649

becomes

,
a true statement.

The solution to 3a
7 = 6 is a = 14.

Method 2: Use of reciprocals


Recall that if the product of two numbers is 1, the numbers are reciprocals. Thus 3
7 and 7
3 are
reciprocals.
3a
7=6
Multiply both sides of the equation by 73 , the reciprocal of 37 .
7 3a 7
3 · 7 = 3 ·6
1 1 2
)7 )3a 7 )6
· = · 1
)3 )7 )3
1 1 1

1 · a = 14
a = 14

Notice that we get the same solution using either method.


Example 11.29
−8x = 24
-8 is associated with x by multiplication. Undo the association by dividing both sides by -8.
−8x 24
= −8
−8

−8x 24
= −8
−8

x = −3

Check: When x = −3,


−8x = 24

becomes

,
a true statement.
Example 11.30
−x = 7.
Since − − x is actually −1 · x and (−1) (−1) = 1, we can isolate x by multiplying both sides of the
650 CHAPTER 11. ALGEBRAIC EXPRESSIONS AND EQUATIONS

equation by −1.

(−1) (−x) = −1 · 7
x = −7

Check: When x = 7,
−x = 7

becomes

The solution to −x = 7 is x = −7.

11.5.2.2 Practice Set A

Use the multiplication/division property of equality to solve each equation. Be sure to check each solution.
Exercise 11.143 (Solution on p. 684.)
7x = 21
Exercise 11.144 (Solution on p. 684.)
−5x = 65
Exercise 11.145 (Solution on p. 684.)
x
4 = −8
Exercise 11.146 (Solution on p. 684.)
3x
8 =6
Exercise 11.147 (Solution on p. 684.)
−y = 3
Exercise 11.148 (Solution on p. 684.)
−k = −2

11.5.3 Combining Techniques in Equation Solving


Having examined solving equations using the addition/subtraction and the multiplication/division principles
of equality, we can combine these techniques to solve more complicated equations.

When beginning to solve an equation such as 6x − 4 = −16, it is helpful to know which property of equality
to use rst, addition/subtraction or multiplication/division. Recalling that in equation solving we are trying
to isolate the variable (disassociate numbers from it), it is helpful to note the following.
To associate numbers and letters, we use the order of operations.

1. Multiply/divide
2. Add/subtract

To undo an association between numbers and letters, we use the order of operations in reverse.

1. Add/subtract
2. Multiply/divide
651

11.5.3.1 Sample Set B

Solve each equation. (In these example problems, we will not show the checks.)
Example 11.31
6x − 4 = −16
-4 is associated with x by subtraction. Undo the association by adding 4 to both sides.

6x − 4 + 4 = −16 + 4

6x = −12
6 is associated with x by multiplication. Undo the association by dividing both sides by 6
6x −12
6 = 6

x = −2
Example 11.32
−8k + 3= − 45.
3 is associated with k by addition. Undo the association by subtracting 3 from both sides.

−8k + 3 − 3= − 45 − 3

−8k = − 48
-8 is associated with k by multiplication. Undo the association by dividing both sides by -8.
−8k −48
−8 = −8

k=6
Example 11.33
5m − 6 − 4m = 4m − 8 + 3m. Begin by solving this equation by combining like terms.

m − 6 = 7m − 8 Choose a side on which to isolate m. Since 7 is greater than 1, we'll isolate m on


the right side.
Subtract m from both sides.

−m − 6 − m = 7m − 8 − m

−6 = 6m − 8
8 is associated with m by subtraction. Undo the association by adding 8 to both sides.

−6 + 8 = 6m − 8 + 8

2 = 6m
6 is associated with m by multiplication. Undo the association by dividing both sides by 6.
2
6 = 6m
6 Reduce.
1
3 =m

Notice that if we had chosen to isolate m on the left side of the equation rather than the right side,
we would have proceeded as follows:

m − 6 = 7m − 8
Subtract 7m from both sides.

m − 6 − 7m = 7m − 8 − 7m
652 CHAPTER 11. ALGEBRAIC EXPRESSIONS AND EQUATIONS

−6m − 6= − 8
Add 6 to both sides,

−6m − 6 + 6= − 8 + 6

−6m= − 2
Divide both sides by -6.
−6m −2
−6 = −6

1
m= 3

This is the same result as with the previous approach.


Example 11.34
7 =
8x
−2
7 is associated with x by division. Undo the association by multiplying both sides by 7.
8x
)7 · = 7 (−2)
)7

7 = − 14
8x

8x= − 14
8 is associated with x by multiplication. Undo the association by dividing both sides by 8.
)8x −7
= 4
)8

−7
x= 4

11.5.3.2 Practice Set B

Solve each equation. Be sure to check each solution.


Exercise 11.149 (Solution on p. 684.)
5m + 7= − 13
Exercise 11.150 (Solution on p. 684.)
−3a − 6 = 9
Exercise 11.151 (Solution on p. 684.)
2a + 10 − 3a = 9
Exercise 11.152 (Solution on p. 684.)
11x − 4 − 13x = 4x + 14
Exercise 11.153 (Solution on p. 684.)
−3m + 8 = −5m + 1
Exercise 11.154 (Solution on p. 684.)
5y + 8y − 11= − 11
653

11.5.4 Exercises
Solve each equation. Be sure to check each result.
Exercise 11.155 (Solution on p. 684.)
7x = 42
Exercise 11.156
8x = 81
Exercise 11.157 (Solution on p. 684.)
10x = 120
Exercise 11.158
11x = 121
Exercise 11.159 (Solution on p. 684.)
−6a = 48
Exercise 11.160
−9y = 54
Exercise 11.161 (Solution on p. 684.)
−3y = − 42
Exercise 11.162
−5a= − 105
Exercise 11.163 (Solution on p. 684.)
2m= − 62
Exercise 11.164
3m= − 54
Exercise 11.165 (Solution on p. 684.)
x
4 =7
Exercise 11.166
y
3 = 11
Exercise 11.167 (Solution on p. 684.)
6 = − 14
−z

Exercise 11.168
−w
5 =1
Exercise 11.169 (Solution on p. 684.)
3m − 1= − 13
Exercise 11.170
4x + 7= − 17
Exercise 11.171 (Solution on p. 684.)
2 + 9x= − 7
Exercise 11.172
5 − 11x = 27
Exercise 11.173 (Solution on p. 684.)
32 = 4y + 6
Exercise 11.174
−5 + 4 = −8m + 1
Exercise 11.175 (Solution on p. 684.)
3k + 6 = 5k + 10
654 CHAPTER 11. ALGEBRAIC EXPRESSIONS AND EQUATIONS

Exercise 11.176
4a + 16 = 6a + 8a + 6
Exercise 11.177 (Solution on p. 684.)
6x + 5 + 2x − 1 = 9x − 3x + 15
Exercise 11.178
−9y − 8 + 3y + 7 = −7y + 8y − 5y + 9
Exercise 11.179 (Solution on p. 685.)
−3a = a + 5
Exercise 11.180
5b= − 2b + 8b + 1
Exercise 11.181 (Solution on p. 685.)
−3m + 2 − 8m − 4 = −14m + m − 4
Exercise 11.182
5a + 3 = 3
Exercise 11.183 (Solution on p. 685.)
7x + 3x = 0
Exercise 11.184
7g + 4 − 11g = −4g + 1 + g
Exercise 11.185 (Solution on p. 685.)
5a
7 = 10
Exercise 11.186
2m
9 =4
Exercise 11.187 (Solution on p. 685.)
3x 9
4 = 2
Exercise 11.188
8k
3 = 32
Exercise 11.189 (Solution on p. 685.)
3a 3
8 − 2 =0
Exercise 11.190
5m
6 − 25
3 =0

11.5.4.1 Exercises for Review

Exercise 11.191 (Solution on p. 685.)


(Section 8.4) Use the distributive property to compute 40 · 28.
Exercise 11.192
(Section 9.5) Approximating π by 3.14, nd the approximate circumference of the circle.

Exercise 11.193 (Solution on p. 685.)


(Section 9.6) Find the area of the parallelogram.
655

Exercise 11.194
−3(4−15)−2
(Section 10.7) Find the value of −5 .
Exercise 11.195 (Solution on p. 685.)
(Section 11.4) Solve the equation x − 14 + 8 = −2.

11.6 Applications I: Translating Words to Mathematical Symbols6


11.6.1 Section Overview
• Translating Words to Symbols

11.6.2 Translating Words to Symbols


Practical problems seldom, if ever, come in equation form. The job of the problem solver is to translate the
problem from phrases and statements into mathematical expressions and equations, and then to solve the
equations.

As problem solvers, our job is made simpler if we are able to translate verbal phrases to mathematical
expressions and if we follow the ve-step method of solving applied problems. To help us translate from
words to symbols, we can use the following Mathematics Dictionary.

MATHEMATICS DICTIONARY
Word or Phrase Mathematical Operation
Sum, sum of, added to, increased by, more than, and, plus +
Dierence, minus, subtracted from, decreased by, less, less than -
Product, the product of, of, multiplied by, times, per ·
Quotient, divided by, ratio, per ÷
Equals, is equal to, is, the result is, becomes =
A number, an unknown quantity, an unknown, a quantity x (or any symbol)

Table 11.2

6 This content is available online at <http://cnx.org/content/m35046/1.2/>.


656 CHAPTER 11. ALGEBRAIC EXPRESSIONS AND EQUATIONS

11.6.2.1 Sample Set A

Translate each phrase or sentence into a mathematical expression or equation.


Example 11.35
| {z }more
Nine | {zthan }some
| number.
{z }
9 + x

Translation: 9 + x.
Example 11.36
Eighteen minus a number.
| {z }| {z }| {z }
18 − x

Translation: 18 − x.
Example 11.37
A quantity less ve.
| {z }| {z }|{z}
y − 5

Translation: y − 5.
Example 11.38
Four | {z }a| number
| {z }times is sixteen.
{z }|{z} | {z }
4 · x = 16

Translation: 4x = 16.
Example 11.39
One
| {zfth }|{z}
of |a number
{z is thirty.
}|{z} | {z }
1 · n = 30
5

Translation: 1
5n = 30, or n
5 = 30.
Example 11.40
{z }|times
|Five {z }|a number
{z }|{z} {z }|more{zthan }|twice
is |two {z }|the number.
{z }
5 · x = 2 + 2· x

Translation: 5x = 2 + 2x.

11.6.2.2 Practice Set A

Translate each phrase or sentence into a mathematical expression or equation.


Exercise 11.196 (Solution on p. 685.)
Twelve more than a number.
Exercise 11.197 (Solution on p. 685.)
Eight minus a number.
Exercise 11.198 (Solution on p. 685.)
An unknown quantity less fourteen.
Exercise 11.199 (Solution on p. 685.)
Six times a number is fty-four.
Exercise 11.200 (Solution on p. 685.)
Two ninths of a number is eleven.
657

Exercise 11.201 (Solution on p. 685.)


Three more than seven times a number is nine more than ve times the number.
Exercise 11.202 (Solution on p. 685.)
Twice a number less eight is equal to one more than three times the number.

11.6.2.3 Sample Set B

Example 11.41
Sometimes the structure of the sentence indicates the use of grouping symbols. We'll be alert for
commas. They set o terms.
A
| number
{z } | {z }six,
minus is twelve
|{z}|{z}| {z }
− = 12
0 1
6
Bxdivided by four, C
B C
@ | {z }| {z }A
÷ 4

Translation: x
4 − 6 = 12.
Example 11.42
Some phrases and sentences do not translate directly. We must be careful to read them properly.
The word from often appears in such phrases and sentences. The word from means a point of
departure for motion. The following translation will illustrate this use.

Translation: x − 20.

The word from indicated the motion (subtraction) is to begin at the point of some number.
Example 11.43
Ten less than some number. Notice that less than can be replaced by from.

Ten from some number.

Translation: x − 10.

11.6.2.4 Practice Set B

Translate each phrase or sentence into a mathematical expression or equation.


Exercise 11.203 (Solution on p. 685.)
A number divided by eight, plus seven, is fty.
Exercise 11.204 (Solution on p. 685.)
A number divided by three, minus the same number multiplied by six, is one more than the number.
Exercise 11.205 (Solution on p. 685.)
Nine from some number is four.
Exercise 11.206 (Solution on p. 685.)
Five less than some quantity is eight.
658 CHAPTER 11. ALGEBRAIC EXPRESSIONS AND EQUATIONS

11.6.3 Exercises
Translate each phrase or sentence to a mathematical expression or equation.
Exercise 11.207 (Solution on p. 685.)
A quantity less twelve.
Exercise 11.208
Six more than an unknown number.
Exercise 11.209 (Solution on p. 685.)
A number minus four.
Exercise 11.210
A number plus seven.
Exercise 11.211 (Solution on p. 685.)
A number increased by one.
Exercise 11.212
A number decreased by ten.
Exercise 11.213 (Solution on p. 685.)
Negative seven added to some number.
Exercise 11.214
Negative nine added to a number.
Exercise 11.215 (Solution on p. 685.)
A number plus the opposite of six.
Exercise 11.216
A number minus the opposite of ve.
Exercise 11.217 (Solution on p. 686.)
A number minus the opposite of negative one.
Exercise 11.218
A number minus the opposite of negative twelve.
Exercise 11.219 (Solution on p. 686.)
Eleven added to three times a number.
Exercise 11.220
Six plus ve times an unknown number.
Exercise 11.221 (Solution on p. 686.)
Twice a number minus seven equals four.
Exercise 11.222
Ten times a quantity increased by two is nine.
Exercise 11.223 (Solution on p. 686.)
When fourteen is added to two times a number the result is six.
Exercise 11.224
Four times a number minus twenty-nine is eleven.
Exercise 11.225 (Solution on p. 686.)
Three fths of a number plus eight is fty.
Exercise 11.226
Two ninths of a number plus one fth is forty-one.
Exercise 11.227 (Solution on p. 686.)
When four thirds of a number is increased by twelve, the result is ve.
659

Exercise 11.228
When seven times a number is decreased by two times the number, the result is negative one.
Exercise 11.229 (Solution on p. 686.)
When eight times a number is increased by ve, the result is equal to the original number plus
twenty-six.
Exercise 11.230
Five more than some number is three more than four times the number.
Exercise 11.231 (Solution on p. 686.)
When a number divided by six is increased by nine, the result is one.
Exercise 11.232
A number is equal to itself minus three times itself.
Exercise 11.233 (Solution on p. 686.)
A number divided by seven, plus two, is seventeen.
Exercise 11.234
A number divided by nine, minus ve times the number, is equal to one more than the number.
Exercise 11.235 (Solution on p. 686.)
When two is subtracted from some number, the result is ten.
Exercise 11.236
When four is subtracted from some number, the result is thirty-one.
Exercise 11.237 (Solution on p. 686.)
Three less than some number is equal to twice the number minus six.
Exercise 11.238
Thirteen less than some number is equal to three times the number added to eight.
Exercise 11.239 (Solution on p. 686.)
When twelve is subtracted from ve times some number, the result is two less than the original
number.
Exercise 11.240
When one is subtracted from three times a number, the result is eight less than six times the
original number.
Exercise 11.241 (Solution on p. 686.)
When a number is subtracted from six, the result is four more than the original number.
Exercise 11.242
When a number is subtracted from twenty-four, the result is six less than twice the number.
Exercise 11.243 (Solution on p. 686.)
A number is subtracted from nine. This result is then increased by one. The result is eight more
than three times the number.
Exercise 11.244
Five times a number is increased by two. This result is then decreased by three times the number.
The result is three more than three times the number.
Exercise 11.245 (Solution on p. 686.)
Twice a number is decreased by seven. This result is decreased by four times the number. The
result is negative the original number, minus six.
Exercise 11.246
Fifteen times a number is decreased by fteen. This result is then increased by two times the
number. The result is negative ve times the original number minus the opposite of ten.
660 CHAPTER 11. ALGEBRAIC EXPRESSIONS AND EQUATIONS

11.6.3.1 Exercises for Review

Exercise 11.247 (Solution on p. 686.)


(Section 4.7) 8
9 of what number is 23 ?
Exercise 11.248
(Section 5.3) Find the value of 21 17
40 + 30 .
Exercise 11.249 (Solution on p. 686.)
(Section 5.4) Find the value of 3 12
1
+ 4 13 + 1 41 .
Exercise 11.250
(Section 6.3) Convert 6.11 15 to a fraction.
Exercise 11.251 (Solution on p. 686.)
(Section 11.5) Solve the equation 3x
4 + 1 = −5.

11.7 Applications II: Solving Problems7


11.7.1 Section Overview
• The Five-Step Method
• Number Problems
• Geometry Problems

11.7.2 The Five Step Method


We are now in a position to solve some applied problems using algebraic methods. The problems we shall
solve are intended as logic developers. Although they may not seem to reect real situations, they do serve
as a basis for solving more complex, real situation, applied problems. To solve problems algebraically, we
will use the ve-step method.

Strategy for Reading Word Problems


When solving mathematical word problems, you may wish to apply the following "reading strategy."
Read the problem quickly to get a feel for the situation. Do not pay close attention to details. At the rst
reading, too much attention to details may be overwhelming and lead to confusion and discouragement.
After the rst, brief reading, read the problem carefully in phrases. Reading phrases introduces information
more slowly and allows us to absorb and put together important information. We can look for the unknown
quantity by reading one phrase at a time.
Five-Step Method for Solving Word Problems

1. Let x (or some other letter) represent the unknown quantity.


2. Translate the words to mathematical symbols and form an equation. Draw a picture if possible.
3. Solve the equation.
4. Check the solution by substituting the result into the original statement, not equation, of the problem.
5. Write a conclusion.

If it has been your experience that word problems are dicult, then follow the ve-step method carefully.
Most people have trouble with word problems for two reasons:

1. They are not able to translate the words to mathematical symbols. (See Section 11.5.)
7 This content is available online at <http://cnx.org/content/m35047/1.2/>.
661

2. They neglect step 1. After working through the problem phrase by phrase, to become familiar with
the situation,
INTRODUCE A VARIABLE

11.7.3 Number Problems


11.7.3.1 Sample Set A

Example 11.44
What number decreased by six is ve?
Step 1: Let n represent the unknown number.
Step 2: Translate the words to mathematical symbols and construct an equation. Read phrases.
What number: n
decreased by: −
six: 6 }n − 6 = 5
is: =
ve: 5
Step 3: Solve this equation.
n − 6 = 5 Add 6 to both sides.
n−6+6=5+6
n = 11
Step 4: Check the result.
When 11 is decreased by 6, the result is 11 − 6, which is equal to 5. The solution checks.
Step 5: The number is 11.

Example 11.45
When three times a number is increased by four, the result is eight more than ve times the number.
Step 1: Let x =the unknown number.
Step 2: Translate the phrases to mathematical symbols and construct an equation.
When three times a number: 3x
is increased by: +
four: 4
the result is: = }3x + 4 = 5x + 8
eight: 8
more than: +
ve times the number: 5x
3x + 4 = 5x + 8. Subtract 3x from both sides.
3x + 4 − 3x = 5x + 8 − 3x
4 = 2x + 8 Subtract 8 from both sides.
Step 3: 4 − 8 = 2x + 8 − 8
−4 = 2x Divide both sides by 2.

−2 = x
662 CHAPTER 11. ALGEBRAIC EXPRESSIONS AND EQUATIONS

Step 4: Check this result.


Three times −2 is −6. Increasing −6 by 4 results in −6 + 4 = −2. Now, ve times −2 is −10.
Increasing −10 by 8 results in −10 + 8 = −2. The results agree, and the solution checks.
Step 5: The number is −2

Example 11.46
Consecutive integers have the property that if

n = the smallest integer, then


n+1 = the next integer, and
n+2 = the next integer, and so on.

Consecutive odd or even integers have the property that if

n = the smallest integer, then


n+2 = the next odd or even integer (since odd or even numbers dier by 2), and
n+4 = the next odd or even integer, and so on.

The sum of three consecutive odd integers is equal to one less than twice the rst odd integer. Find
the three integers.

Let n = the rst odd integer. Then,


Step 1. n+2 = the second odd integer, and
n+4 = the third odd integer.
Step 2. Translate the words to mathematical symbols and construct an equation. Read phrases.
The sum of: add some numbers
three consecutive odd integers: n, n + 2, n + 4
is equal to: = }n + (n + 2) + (n + 4) = 2n − 1
one less than: subtract 1 from
twice the rst odd integer: 2n
n + n + 2 + n + 4 = 2n − 1
3n + 6 = 2n − 1 Subtract 2n from both sides.
3n + 6 − 2n = 2n − 1 − 2n
n + 6 = −1 Subtract 6 from both sides.
Step 3.
n + 6 − 6 = −1 − 6
n = −7 The rst integer is -7.
n + 2 = −7 + 2 = −5 The second integer is -5.
n + 4 = −7 + 4 = −3 The third integer is -3.
Step 4. Check this result.
−7 + (−5) + (−3) = −12 + (−3)
The sum of the three integers is
= −15

One less than twice the rst integer is 2 (−7) − 1 = −14 − 1 = −15. Since these two results
are equal, the solution checks.
663

Step 5. The three odd integers are -7, -5, -3.

11.7.3.2 Practice Set A

Exercise 11.252 (Solution on p. 686.)


When three times a number is decreased by 5, the result is -23. Find the number.

Step 1: Let x =
Step 2:
Step 3:
Step 4: Check:
Step 5: The number is .

Exercise 11.253 (Solution on p. 686.)


When ve times a number is increased by 7, the result is ve less than seven times the number.
Find the number.

Step 1: Let n =
Step 2:
Step 3:
Step 4: Check:
Step 5: The number is .

Exercise 11.254 (Solution on p. 686.)


Two consecutive numbers add to 35. Find the numbers.

Step 1:
Step 2:
Step 3:
Step 4: Check:
Step 5: The numbers are and .

Exercise 11.255 (Solution on p. 686.)


The sum of three consecutive even integers is six more than four times the middle integer. Find
the integers.

Step 1: Let x = smallest integer.


= next integer.
= largest integer.
Step 2:
Step 3:
Step 4: Check:
Step 5: The integers are , , and .
664 CHAPTER 11. ALGEBRAIC EXPRESSIONS AND EQUATIONS

11.7.4 Geometry Problems


11.7.4.1 Sample Set B

Example 11.47
The perimeter (length around) of a rectangle is 20 meters. If the length is 4 meters longer than the
width, nd the length and width of the rectangle.
Step 1: Let x =the width of the rectangle. Then,
x + 4 =the length of the rectangle.
Step 2: We can draw a picture.

The length around the rectangle is


x + (x + 4) + |{z}
|{z} x + (x + 4) = 20
width width
| {z } | {z }
length length
x + x + 4 + x + x + 4 = 20
4x + 8 = 20 Subtract 8 from both sides.
Step 3: 4x = 12 Divide both sides by 4.
x=3 Then,
x+4=3+4=7
Step 4: Check:

Step 5: The length of the rectangle is 7 meters.


The width of the rectangle is 3 meters.

11.7.4.2 Practice Set B

Exercise 11.256 (Solution on p. 686.)


The perimeter of a triangle is 16 inches. The second leg is 2 inches longer than the rst leg, and
the third leg is 5 inches longer than the rst leg. Find the length of each leg.
Step 1: Let x = length of the rst leg.
= length of the second leg.
= length of the third leg.
Step 2: We can draw a picture.
Step 3:
Step 4: Check:
Step 5: The lengths of the legs are , , and .
665

11.7.5 Exercises
For the following 17 problems, nd each solution using the ve-step method.
Exercise 11.257 (Solution on p. 686.)
What number decreased by nine is fteen?

Step 1. Let n =the number.


Step 2.
Step 3.
Step 4. Check:
Step 5. The number is .

Exercise 11.258
What number increased by twelve is twenty?

Step 1. Let n =the number.


Step 2.
Step 3.
Step 4. Check:
Step 5. The number is .

Exercise 11.259 (Solution on p. 686.)


If ve more than three times a number is thirty-two, what is the number?

Step 1. Let x =the number.


Step 2.
Step 3.
Step 4. Check:
Step 5. The number is .

Exercise 11.260
If four times a number is increased by fteen, the result is ve. What is the number?

Step 1. Let x =
Step 2.
Step 3.
Step 4. Check:
Step 5. The number is .

Exercise 11.261 (Solution on p. 687.)


When three times a quantity is decreased by ve times the quantity, the result is negative twenty.
What is the quantity?

Step 1. Let x =
Step 2.
Step 3.
Step 4. Check:
Step 5. The quantity is .

Exercise 11.262
If four times a quantity is decreased by nine times the quantity, the result is ten. What is the
quantity?
666 CHAPTER 11. ALGEBRAIC EXPRESSIONS AND EQUATIONS

Step 1. Let y =
Step 2.
Step 3.
Step 4. Check:
Step 5. The quantity is .

Exercise 11.263 (Solution on p. 687.)


When ve is added to three times some number, the result is equal to ve times the number
decreased by seven. What is the number?

Step 1. Let n =
Step 2.
Step 3.
Step 4. Check:
Step 5. The number is .

Exercise 11.264
When six times a quantity is decreased by two, the result is six more than seven times the quantity.
What is the quantity?

Step 1. Let x =
Step 2.
Step 3.
Step 4. Check:
Step 5. The quantity is .

Exercise 11.265 (Solution on p. 687.)


When four is decreased by three times some number, the result is equal to one less than twice the
number. What is the number?

Step 1.
Step 2.
Step 3.
Step 4. Check:
Step 5.

Exercise 11.266
When twice a number is subtracted from one, the result is equal to twenty-one more than the
number. What is the number?

Step 1.
Step 2.
Step 3.
Step 4.
Step 5.

Exercise 11.267 (Solution on p. 687.)


The perimeter of a rectangle is 36 inches. If the length of the rectangle is 6 inches more than the
width, nd the length and width of the rectangle.

Step 1. Let w =the width.


= the length.
667

Step 2. We can draw a picture.

Step 3.
Step 4. Check:
Step 5. The length of the rectangle is inches, and the width is inches.

Exercise 11.268
The perimeter of a rectangle is 48 feet. Find the length and the width of the rectangle if the length
is 8 feet more than the width.

Step 1. Let w =the width.


= the length.
Step 2. We can draw a picture.

Step 3.
Step 4. Check:
Step 5. The length of the rectangle is feet, and the width is feet.

Exercise 11.269 (Solution on p. 687.)


The sum of three consecutive integers is 48. What are they?

Step 1. Let n =the smallest integer.


= the next integer.
= the next integer.
Step 2.
Step 3.
Step 4. Check:
Step 5. The three integers are , , and .

Exercise 11.270
The sum of three consecutive integers is -27. What are they?

Step 1. Let n = the smallest integer.


= the next integer.
= the next integer.
Step 2.
Step 3.
Step 4. Check:
Step 5. The three integers are , , and .

Exercise 11.271 (Solution on p. 687.)


The sum of ve consecutive integers is zero. What are they?

Step 1. Let n =
Step 2.
668 CHAPTER 11. ALGEBRAIC EXPRESSIONS AND EQUATIONS

Step 3.
Step 4.
Step 5. The ve integers are , , , , and .

Exercise 11.272
The sum of ve consecutive integers is -5. What are they?

Step 1. Let n =
Step 2.
Step 3.
Step 4.
Step 5. The ve integers are , , , , and .

Continue using the ve-step procedure to nd the solutions.


Exercise 11.273 (Solution on p. 687.)
The perimeter of a rectangle is 18 meters. Find the length and width of the rectangle if the length
is 1 meter more than three times the width.
Exercise 11.274
The perimeter of a rectangle is 80 centimeters. Find the length and width of the rectangle if the
length is 2 meters less than ve times the width.
Exercise 11.275 (Solution on p. 687.)
Find the length and width of a rectangle with perimeter 74 inches, if the width of the rectangle is
8 inches less than twice the length.
Exercise 11.276
Find the length and width of a rectangle with perimeter 18 feet, if the width of the rectangle is 7
feet less than three times the length.
Exercise 11.277 (Solution on p. 687.)
A person makes a mistake when copying information regarding a particular rectangle. The copied
information is as follows: The length of a rectangle is 5 inches less than two times the width. The
perimeter of the rectangle is 2 inches. What is the mistake?
Exercise 11.278
A person makes a mistake when copying information regarding a particular triangle. The copied
information is as follows: Two sides of a triangle are the same length. The third side is 10 feet less
than three times the length of one of the other sides. The perimeter of the triangle is 5 feet. What
is the mistake?
Exercise 11.279 (Solution on p. 687.)
The perimeter of a triangle is 75 meters. If each of two legs is exactly twice the length of the
shortest leg, how long is the shortest leg?
Exercise 11.280
If ve is subtracted from four times some number the result is negative twenty-nine. What is the
number?
Exercise 11.281 (Solution on p. 687.)
If two is subtracted from ten times some number, the result is negative two. What is the number?
Exercise 11.282
If three less than six times a number is equal to ve times the number minus three, what is the
number?
Exercise 11.283 (Solution on p. 687.)
If one is added to negative four times a number the result is equal to eight less than ve times the
number. What is the number?
669

Exercise 11.284
Find three consecutive integers that add to -57.
Exercise 11.285 (Solution on p. 687.)
Find four consecutive integers that add to negative two.
Exercise 11.286
Find three consecutive even integers that add to -24.
Exercise 11.287 (Solution on p. 687.)
Find three consecutive odd integers that add to -99.
Exercise 11.288
Suppose someone wants to nd three consecutive odd integers that add to 120. Why will that
person not be able to do it?
Exercise 11.289 (Solution on p. 687.)
Suppose someone wants to nd two consecutive even integers that add to 139. Why will that
person not be able to do it?
Exercise 11.290
Three numbers add to 35. The second number is ve less than twice the smallest. The third
number is exactly twice the smallest. Find the numbers.
Exercise 11.291 (Solution on p. 687.)
Three numbers add to 37. The second number is one less than eight times the smallest. The third
number is two less than eleven times the smallest. Find the numbers.

11.7.5.1 Exercises for Review

Exercise 11.292
(Section 6.7) Find the decimal representation of 0.34992 ÷ 4.32.
Exercise 11.293 (Solution on p. 687.)
(Section 7.4) A 5-foot woman casts a 9-foot shadow at a particular time of the day. How tall is a
person that casts a 10.8-foot shadow at the same time of the day?
Exercise 11.294
(Section 8.5) Use the method of rounding to estimate the sum: 4 12
5
+ 15 25
1
.
Exercise 11.295 (Solution on p. 687.)
(Section 9.3) Convert 463 mg to cg.
Exercise 11.296
(Section 11.6) Twice a number is added to 5. The result is 2 less than three times the number.
What is the number?

11.8 Summary of Key Concepts8


11.8.1 Summary of Key Concepts
Numerical Expression (Section 11.2)
A numerical expression results when numbers are associated by arithmetic operation signs. The expres-
sions 3 + 5, 9 − 2, 5 · 6 and 8 ÷ 5 are numerical expressions.

8 This content is available online at <http://cnx.org/content/m35049/1.2/>.


670 CHAPTER 11. ALGEBRAIC EXPRESSIONS AND EQUATIONS

Algebraic Expressions (Section 11.2)


When an arithmetic operation sign connects a letter with a number or a letter with a letter, an algebraic
expression results. The expressions 4x + 1, x − 5, 7x · 6y , and 4x ÷ 3 are algebraic expressions.

Terms and Factors (Section 11.2)


Terms are parts of sums and are therefore separated by addition (or subtraction) signs. In the expres-
sion, 5x − 2y , 5x and −2y are the terms. Factors are parts of products and are therefore separated by
multiplication signs. In the expression 5a, 5 and a are the factors.

Coecients (Section 11.2)


The coecient of a quantity records how many of that quantity there are. In the expression 7x, the
coecient 7 indicates that there are seven x's.

Numerical Evaluation (Section 11.2)


Numerical evaluation is the process of determining the value of an algebraic expression by replacing the
variables in the expression with specied values.

Combining Like Terms (Section 11.3)


An algebraic expression may be simplied by combining like terms. To combine like terms, we simply
add or subtract their coecients then ax the variable. For example 4x + 9x = (4 + 9) x = 13x.

Equation (Section 11.4)


An equation is a statement that two expressions are equal. The statements 5x + 1 = 3 and 4x
5 +4 = 2
5 are
equations. The expressions represent the same quantities.

Conditional Equation (Section 11.4)


A conditional equation is an equation whose truth depends on the value selected for the variable. The
equation 3x = 9 is a conditional equation since it is only true on the condition that 3 is selected for x.

Solutions and Solving an Equation (Section 11.4)


The values that when substituted for the variables make the equation true are called the solutions of the
equation.
An equation has been solved when all its solutions have been found.

Equivalent Equations (Section 11.4)


Equations that have precisely the same solutions are called equivalent equations. The equations 6y = 18
and y = 3 are equivalent equations.

Addition/Subtraction Property of Equality (Section 11.4)


Given any equation, we can obtain an equivalent equation by

1. adding the same number to both sides, or


2. subtracting the same number from both sides.

Solving x + a = b and x − a = b (Section 11.4)


To solve x + a = b, subtract a from both sides.

x+a=b
x+a−a=b−a
x=b−a
To solve x − a = b, add a to both sides.
671

x−a=b
x−a+a=b+a
x=b+a

Multiplication/Division Property of Equality (Section 11.5)


Given any equation, we can obtain an equivalent equation by
1. multiplying both sides by the same nonzero number, that is, if c 6= 0, a = b and a · c = b · c are
equivalent.
2. dividing both sides by the same nonzero number, that is, if c 6= 0, a = b and ac = cb are equivalent.
Solving ax = b and x a = b (Section 11.5)
To solve ax = b, a 6= 0, divide both sides by a.

ax = b
ax b
a = a
)ax b
= a
)a
b
x = a

To solve x
a = b, a 6= 0, multiply both sides by a.

x
a = b
x
a· a = a·b
)a · x = a·b
)a
x = a·b

Translating Words to Mathematics (Section 11.6)


In solving applied problems, it is important to be able to translate phrases and sentences to mathematical
expressions and equations.

The Five-Step Method for Solving Applied Problems (Section 11.7)


To solve problems algebraically, it is a good idea to use the following ve-step procedure.
After working your way through the problem carefully, phrase by phrase:
1. Let x (or some other letter) represent the unknown quantity.
2. Translate the phrases and sentences to mathematical symbols and form an equation. Draw a picture
if possible.
3. Solve this equation.
4. Check the solution by substituting the result into the original statement of the problem.
5. Write a conclusion.

11.9 Exercise Supplement 9


11.9.1 Exercise Supplement
11.9.1.1 Algebraic Expressions (Section 11.2)

For problems 1-10, specify each term.


9 This content is available online at <http://cnx.org/content/m35050/1.2/>.
672 CHAPTER 11. ALGEBRAIC EXPRESSIONS AND EQUATIONS

Exercise 11.297 (Solution on p. 687.)


6a − 2b + 5c
Exercise 11.298
9x − 6y + 1
Exercise 11.299 (Solution on p. 687.)
7m − 3n
Exercise 11.300
−5h + 2k − 8 + 4m
Exercise 11.301 (Solution on p. 687.)
x + 2n − z
Exercise 11.302
y−5
Exercise 11.303 (Solution on p. 687.)
−y − 3z
Exercise 11.304
−a − b − c − 1
Exercise 11.305 (Solution on p. 687.)
−4
Exercise 11.306
−6
Exercise 11.307 (Solution on p. 687.)
Write 1k in a simpler way.
Exercise 11.308
Write 1x in a simpler way.
Exercise 11.309 (Solution on p. 687.)
In the expression 7r, how many r's are indicated?
Exercise 11.310
In the expression 12m, how many m's are indicated?
Exercise 11.311 (Solution on p. 688.)
In the expression −5n, how many n's are indicated?
Exercise 11.312
In the expression −10y , how many y 's are indicated?
For problems 17-46, nd the value of each expression.
Exercise 11.313 (Solution on p. 688.)
5a − 2s, if a = −5 and s = 1
Exercise 11.314
7n − 3r, if n = −6 and r = 2
Exercise 11.315 (Solution on p. 688.)
9x + 2y − 3s, if x = −2, y = 5, and s = −3
Exercise 11.316
10a − 2b + 5c, if a = 0, b = −6, and c = 8
Exercise 11.317 (Solution on p. 688.)
−5s − 2t + 1, if s = 2 and t = −2
Exercise 11.318
−3m − 4n + 5, if m = −1 and n = −1
673

Exercise 11.319 (Solution on p. 688.)


m − 4, if m = 4
Exercise 11.320
n = 2, if n = 2
Exercise 11.321 (Solution on p. 688.)
−x + 2y , if x = −7 and y = −1
Exercise 11.322
−a + 3b − 6, if a = −3 and b = 0
Exercise 11.323 (Solution on p. 688.)
5x − 4y − 7y + y − 7x, if x = 1 and y = −2
Exercise 11.324
2a − 6b − 3a − a + 2b, if a = 4 and b = −2
Exercise 11.325 (Solution on p. 688.)
a2 − 6a + 4, if a = −2
Exercise 11.326
m2 − 8m − 6, if m = −5
Exercise 11.327 (Solution on p. 688.)
4y 2 + 3y + 1, if y = −2
Exercise 11.328
5a2 − 6a + 11, if a = 0
Exercise 11.329 (Solution on p. 688.)
−k 2 − k − 1, if k = −1
Exercise 11.330
−h2 − 2h − 3, if h = −4
Exercise 11.331 (Solution on p. 688.)
m
6 + 5m, if m = −18
Exercise 11.332
a
8 − 2a + 1, if a = 24
Exercise 11.333 (Solution on p. 688.)
5x
7 + 3x − 7, if x = 14
Exercise 11.334
3k
4 − 5k + 18, if k = 16
Exercise 11.335 (Solution on p. 688.)
−6a
5 + 3a + 10, if a = 25
Exercise 11.336
−7h
9 − 7h − 7, if h = −18
Exercise 11.337 (Solution on p. 688.)
5 (3a + 4b), if a = −2 and b = 2
Exercise 11.338
7 (2y − x), if x = −1 and y = 2
Exercise 11.339 (Solution on p. 688.)
− (a − b), if a = 0 and b = −6
Exercise 11.340
− (x − x − y), if x = 4 and y = −4
Exercise 11.341 (Solution on p. 688.)
2
(y + 2) − 6 (y + 2) − 6, if y = 2
674 CHAPTER 11. ALGEBRAIC EXPRESSIONS AND EQUATIONS

Exercise 11.342
2
(a − 7) − 2 (a − 7) − 2, if a = 7

11.9.1.2 Combining Like Terms Using Addition and Subtraction (Section 11.3)

For problems 47-56, simplify each expression by combining like terms.


Exercise 11.343 (Solution on p. 688.)
4a + 5 − 2a + 1
Exercise 11.344
7x + 3x − 14x
Exercise 11.345 (Solution on p. 688.)
−7b + 4m − 3 + 3n
Exercise 11.346
−9k − 8h − k + 6h
Exercise 11.347 (Solution on p. 688.)
−x + 5y − 8x − 6x + 7y
Exercise 11.348
6n − 2n + 6 − 2 − n
Exercise 11.349 (Solution on p. 688.)
0m + 3k − 5s + 2m − s
Exercise 11.350
| −8 | a+ | 2 | b− | −4 | a
Exercise 11.351 (Solution on p. 688.)
| 6 | h− | −7 | k+ | −12 | h+ | 4 | · | −5 | h
Exercise 11.352
| 0 | a − 0a + 0

11.9.1.3 Equations of the Form ax = b and x


a = b, Translating Words to Mathematical Symbols
, and Solving Problems (Section 11.5,Section 11.6,Section 11.7)

For problems 57-140, solve each equation.


Exercise 11.353 (Solution on p. 688.)
x+1=5
Exercise 11.354
y − 3 = −7
Exercise 11.355 (Solution on p. 688.)
x + 12 = 10
Exercise 11.356
x − 4 = −6
Exercise 11.357 (Solution on p. 688.)
5x = 25
Exercise 11.358
3x = 17
Exercise 11.359 (Solution on p. 688.)
x
2 =6
675

Exercise 11.360
x
−8 =3
Exercise 11.361 (Solution on p. 689.)
x
15 = −1
Exercise 11.362
x
−4 = −3
Exercise 11.363 (Solution on p. 689.)
−3x = 9
Exercise 11.364
−2x = 5
Exercise 11.365 (Solution on p. 689.)
−5x = −5
Exercise 11.366
−3x = −1
Exercise 11.367 (Solution on p. 689.)
x
−3 =9
Exercise 11.368
a
−5 =2
Exercise 11.369 (Solution on p. 689.)
−7 = 3y
Exercise 11.370
x
−7 = 3
Exercise 11.371 (Solution on p. 689.)
m −2
4 = 5
Exercise 11.372
1
4y = 2
Exercise 11.373 (Solution on p. 689.)
−1
3 = −5x
Exercise 11.374
−1 k
9 = 3
Exercise 11.375 (Solution on p. 689.)
−1 s
6 = −6
Exercise 11.376
0
4 = 4s
Exercise 11.377 (Solution on p. 689.)
x + 2 = −1
Exercise 11.378
x − 5 = −6
Exercise 11.379 (Solution on p. 689.)
−3
2 x =6
Exercise 11.380
3x + 2 = 7
Exercise 11.381 (Solution on p. 689.)
−4x − 5 = −3
Exercise 11.382
x
6 +1=4
676 CHAPTER 11. ALGEBRAIC EXPRESSIONS AND EQUATIONS

Exercise 11.383 (Solution on p. 689.)


a
−5 − 3 = −2
Exercise 11.384
4x
3 =7
Exercise 11.385 (Solution on p. 689.)
2x
5 +2=8
Exercise 11.386
3y
2 −4=6
Exercise 11.387 (Solution on p. 689.)
m+3=8
Exercise 11.388
1x
2 =2
Exercise 11.389 (Solution on p. 689.)
2a
3 =5
Exercise 11.390
−3x
7 −4=4
Exercise 11.391 (Solution on p. 689.)
5x
−2 − 6 = −10
Exercise 11.392
−4k − 6 = 7
Exercise 11.393 (Solution on p. 689.)
−3x
−2 +1=4
Exercise 11.394
−6x
4 =2
Exercise 11.395 (Solution on p. 689.)
x + 9 = 14
Exercise 11.396
y + 5 = 21
Exercise 11.397 (Solution on p. 689.)
y + 5 = −7
Exercise 11.398
4x = 24
Exercise 11.399 (Solution on p. 689.)
4w = 37
Exercise 11.400
6y − 11 = 13
Exercise 11.401 (Solution on p. 689.)
−3x + 8 = −7
Exercise 11.402
3z + 9 = −51
Exercise 11.403 (Solution on p. 689.)
x
−3 =8
Exercise 11.404
6y
7 =5
Exercise 11.405 (Solution on p. 689.)
w
2 − 15 = 4
677

Exercise 11.406
x
−2 − 23 = −10
Exercise 11.407 (Solution on p. 689.)
2x
3 −5=8
Exercise 11.408
3z −7
4 = 8
Exercise 11.409 (Solution on p. 689.)
2x
−2 − 7 =3
Exercise 11.410
3−x=4
Exercise 11.411 (Solution on p. 690.)
−5 − y = −2
Exercise 11.412
3 − z = −2
Exercise 11.413 (Solution on p. 690.)
3x + 2x = 6
Exercise 11.414
4x + 1 + 6x = 10
Exercise 11.415 (Solution on p. 690.)
6y − 6 = −4 + 3y
Exercise 11.416
3 = 4a − 2a + a
Exercise 11.417 (Solution on p. 690.)
3m + 4 = 2m + 1
Exercise 11.418
5w − 6 = 4 + 2w
Exercise 11.419 (Solution on p. 690.)
8 − 3a = 32 − 2a
Exercise 11.420
5x − 2x + 6x = 13
Exercise 11.421 (Solution on p. 690.)
x+2=3−x
Exercise 11.422
5y + 2y − 1 = 6y
Exercise 11.423 (Solution on p. 690.)
x = 32
Exercise 11.424
k = −4
Exercise 11.425 (Solution on p. 690.)
3x 5x
2 +4= 2 =6
Exercise 11.426
x
3 + 3x
3 − 2 = 16
Exercise 11.427 (Solution on p. 690.)
x−2=6−x
Exercise 11.428
−5x 2x
7 = 7
678 CHAPTER 11. ALGEBRAIC EXPRESSIONS AND EQUATIONS

Exercise 11.429 (Solution on p. 690.)


2x
3 +1=5
Exercise 11.430
−3x 2x
5 +3= 5 +2
Exercise 11.431 (Solution on p. 690.)
3x
4 +5= −3x
4 − 11
Exercise 11.432
3x
7 = −3x
7 + 12
Exercise 11.433 (Solution on p. 690.)
5y 7y
13 − 4 = 26 + 1
Exercise 11.434
−3m
5 = 6m
10 − 2
Exercise 11.435 (Solution on p. 690.)
−3m
2 + 1 = 5m
Exercise 11.436
2z
−3z = 5

11.10 Prociency Exam10


11.10.1 Prociency Exam
For problems 1 and 2 specify each term.
Exercise 11.437 (Solution on p. 690.)
(Section 11.2) 5x + 6y + 3z
Exercise 11.438 (Solution on p. 690.)
(Section 11.2) 8m − 2n − 4
Exercise 11.439 (Solution on p. 690.)
(Section 11.2) In the expression −9a, how many a's are indicated?
For problems 4-9, nd the value of each expression.
Exercise 11.440 (Solution on p. 690.)
(Section 11.2) 6a − 3b, if a = −2, and b = −1.
Exercise 11.441 (Solution on p. 690.)
(Section 11.2) −5m + 2n − 6, if m = −1 and n = 4.
Exercise 11.442 (Solution on p. 690.)
(Section 11.2) −x2 + 3x − 5, if x = −2.
Exercise 11.443 (Solution on p. 690.)
(Section 11.2) y 2 + 9y + 1, if y = 0.
Exercise 11.444 (Solution on p. 690.)
(Section 11.2) −a2 + 3a + 4, if a = 4.
Exercise 11.445 (Solution on p. 690.)
2
(Section 11.2) − (5 − x) + 7 (m − x) + x − 2m, if x = 5 and m = 5.
For problems 10-12, simplify each expression by combining like terms.
Exercise 11.446 (Solution on p. 690.)
(Section 11.3) 6y + 5 − 2y + 1
10 This content is available online at <http://cnx.org/content/m35052/1.2/>.
679

Exercise 11.447 (Solution on p. 690.)


(Section 11.3) 14a − 3b + 5b − 6a − b
Exercise 11.448 (Solution on p. 690.)
(Section 11.3) 9x + 5y − 7 + 4x − 6y + 3 (−2)
For problems 13-22, solve each equation.
Exercise 11.449 (Solution on p. 691.)
(Section 11.4) x + 7 = 15
Exercise 11.450 (Solution on p. 691.)
(Section 11.4) y − 6 = 2
Exercise 11.451 (Solution on p. 691.)
(Section 11.4) m + 8 = −1
Exercise 11.452 (Solution on p. 691.)
(Section 11.4) −5 + a = −4
Exercise 11.453 (Solution on p. 691.)
(Section 11.5) 4x = 104
Exercise 11.454 (Solution on p. 691.)
(Section 11.5) 6y + 3 = −21
Exercise 11.455 (Solution on p. 691.)
(Section 11.5) 5m
6 = 10
3
Exercise 11.456 (Solution on p. 691.)
−13
(Section 11.5) 7y
8 + 1
4 = 4
Exercise 11.457 (Solution on p. 691.)
(Section 11.5) 6x + 5 = 4x − 11
Exercise 11.458 (Solution on p. 691.)
(Section 11.5) 4y − 8 − 6y = 3y + 1
Exercise 11.459 (Solution on p. 691.)
(Section 11.6 and Section 11.7) Three consecutive even integers add to -36. What are they?
Exercise 11.460 (Solution on p. 691.)
(Section 11.6 and Section 11.7) The perimeter of a rectangle is 38 feet. Find the length and width
of the rectangle if the length is 5 feet less than three times the width.
Exercise 11.461 (Solution on p. 691.)
(Section 11.6 and Section 11.7) Four numbers add to -2. The second number is three more than
twice the negative of the rst number. The third number is six less than the rst number. The
fourth number is eleven less than twice the rst number. Find the numbers.
680 CHAPTER 11. ALGEBRAIC EXPRESSIONS AND EQUATIONS

Solutions to Exercises in Chapter 11


Solution to Exercise 11.1 (p. 631)
x, 7
Solution to Exercise 11.2 (p. 631)
3m − 6n
Solution to Exercise 11.3 (p. 631)
5y
Solution to Exercise 11.4 (p. 631)
a, 2b, −c
Solution to Exercise 11.5 (p. 631)
−3x, −5
Solution to Exercise 11.6 (p. 633)
-28
Solution to Exercise 11.7 (p. 633)
-3
Solution to Exercise 11.8 (p. 633)
-6
Solution to Exercise 11.9 (p. 633)
104
Solution to Exercise 11.10 (p. 633)
24
Solution to Exercise 11.11 (p. 633)
-12
Solution to Exercise 11.12 (p. 634)
-1
Solution to Exercise 11.13 (p. 634)
31
Solution to Exercise 11.14 (p. 634)
Addition; multiplication
Solution to Exercise 11.16 (p. 634)
5x, 18y
Solution to Exercise 11.18 (p. 634)
8s, 2r, −7t
Solution to Exercise 11.20 (p. 634)
7a, −2b, −3c, −4d
Solution to Exercise 11.22 (p. 634)
−x, −y
Solution to Exercise 11.24 (p. 634)
m
Solution to Exercise 11.26 (p. 634)
5
Solution to Exercise 11.28 (p. 634)
-30
Solution to Exercise 11.30 (p. 634)
-3
Solution to Exercise 11.32 (p. 635)
-14
Solution to Exercise 11.34 (p. 635)
56
Solution to Exercise 11.36 (p. 635)
64
681

Solution to Exercise 11.38 (p. 635)


1
Solution to Exercise 11.40 (p. 635)
75
Solution to Exercise 11.42 (p. 635)
24
Solution to Exercise 11.44 (p. 635)
39
Solution to Exercise 11.46 (p. 635)
5
Solution to Exercise 11.48 (p. 635)
-7
Solution to Exercise 11.50 (p. 635)
5
Solution to Exercise 11.52 (p. 636)
181 = 7 13
24 24
Solution to Exercise 11.54 (p. 636)
20
27
Solution to Exercise 11.56 (p. 636)
0
Solution to Exercise 11.57 (p. 637)
13x
Solution to Exercise 11.58 (p. 637)
11a + 6b
Solution to Exercise 11.59 (p. 637)
9m − 7n
Solution to Exercise 11.60 (p. 637)
−2a − r
Solution to Exercise 11.61 (p. 637)
0
Solution to Exercise 11.62 (p. 638)
11a
Solution to Exercise 11.64 (p. 638)
4h
Solution to Exercise 11.66 (p. 638)
3m + 3n
Solution to Exercise 11.68 (p. 638)
17s − r
Solution to Exercise 11.70 (p. 638)
5h + 9a − 18k
Solution to Exercise 11.72 (p. 638)
0
Solution to Exercise 11.74 (p. 638)
16m
Solution to Exercise 11.76 (p. 638)
8h + 7k
Solution to Exercise 11.78 (p. 638)
5∆ − 3?
Solution to Exercise 11.80 (p. 638)
16x − 15y − 9
682 CHAPTER 11. ALGEBRAIC EXPRESSIONS AND EQUATIONS

Solution to Exercise 11.82 (p. 639)


2
2 11
Solution to Exercise 11.84 (p. 639)
7
Solution to Exercise 11.86 (p. 639)
6
Solution to Exercise 11.87 (p. 641)

Substitute 5 into m + 6 = 11. Thus, 5 is a solution.


Solution to Exercise 11.88 (p. 641)

Substitute -5 into 2m − 4 = −14. Thus, -5 is a solution.


Solution to Exercise 11.89 (p. 641)

Substitute 0 into 5x + 1 = 1. Thus, 0 is a solution.


Solution to Exercise 11.90 (p. 641)

Substitute 3 into −3y + 1 = 4y + 5. Thus, 3 is not a solution.


Solution to Exercise 11.91 (p. 641)

Substitute -1 into 6m − 5 + 2m = 7m − 6. Thus, -1 is a


solution.
Solution to Exercise 11.92 (p. 644)
y = −5
Solution to Exercise 11.93 (p. 644)
a = 15
Solution to Exercise 11.94 (p. 644)
x=3
Solution to Exercise 11.95 (p. 644)
m=2
Solution to Exercise 11.96 (p. 644)
k = −2
Solution to Exercise 11.97 (p. 644)
a = −1
Solution to Exercise 11.98 (p. 644)
Substitute x = 4 into the equation 4x − 11 = 5.
16-11=5
5=5
x = 4 is a solution.
Solution to Exercise 11.100 (p. 644)
Substitute m = 1 into the equation 2m − 1 = 1.
683

m = 1 is a solution.
Solution to Exercise 11.102 (p. 645)
Substitute x = −8 into the equation 3x + 2 − 7 = −5x − 6.

x = −8 is a solution.
Solution to Exercise 11.104 (p. 645)
Substitute x = 0 into the equation −8 + x = −8.

x = 0 is a solution.
Solution to Exercise 11.106 (p. 645)
Substitute x = 15
2 into the equation 4x − 5 = 6x − 20.

x = 15
2 is a solution.
Solution to Exercise 11.108 (p. 645)
y = 11
Solution to Exercise 11.110 (p. 645)
k=5
Solution to Exercise 11.112 (p. 645)
a = −9
Solution to Exercise 11.114 (p. 645)
x = 11
Solution to Exercise 11.116 (p. 645)
z=0
Solution to Exercise 11.118 (p. 645)
x=8
Solution to Exercise 11.120 (p. 645)
m=1
Solution to Exercise 11.122 (p. 645)
k = 19
Solution to Exercise 11.124 (p. 646)
m = 10
Solution to Exercise 11.126 (p. 646)
b = −21
Solution to Exercise 11.128 (p. 646)
a = 16
Solution to Exercise 11.130 (p. 646)
y = 7.224
Solution to Exercise 11.132 (p. 646)
m = −0.778
Solution to Exercise 11.134 (p. 646)
k = −9.01
Solution to Exercise 11.136 (p. 646)
y = −28.06
Solution to Exercise 11.138 (p. 646)
rate
684 CHAPTER 11. ALGEBRAIC EXPRESSIONS AND EQUATIONS

Solution to Exercise 11.140 (p. 646)


3.5
Solution to Exercise 11.142 (p. 646)
13x + 18y
Solution to Exercise 11.143 (p. 650)
x=3
Solution to Exercise 11.144 (p. 650)
x = −13
Solution to Exercise 11.145 (p. 650)
x = −32
Solution to Exercise 11.146 (p. 650)
x = 16
Solution to Exercise 11.147 (p. 650)
y = −3
Solution to Exercise 11.148 (p. 650)
k=2
Solution to Exercise 11.149 (p. 652)
m= − 4
Solution to Exercise 11.150 (p. 652)
a= − 5
Solution to Exercise 11.151 (p. 652)
a=1
Solution to Exercise 11.152 (p. 652)
x= − 3
Solution to Exercise 11.153 (p. 652)
m= − 7
2
Solution to Exercise 11.154 (p. 652)
y=0
Solution to Exercise 11.155 (p. 653)
x=6
Solution to Exercise 11.157 (p. 653)
x = 12
Solution to Exercise 11.159 (p. 653)
a= − 8
Solution to Exercise 11.161 (p. 653)
y = 14
Solution to Exercise 11.163 (p. 653)
m= − 31
Solution to Exercise 11.165 (p. 653)
x = 28
Solution to Exercise 11.167 (p. 653)
z = 84
Solution to Exercise 11.169 (p. 653)
m = −4
Solution to Exercise 11.171 (p. 653)
x = −1
Solution to Exercise 11.173 (p. 653)
y = 13
2
Solution to Exercise 11.175 (p. 653)
k= − 2
685

Solution to Exercise 11.177 (p. 654)


x = 11
2 or 5 2
1

Solution to Exercise 11.179 (p. 654)


a = − 54
Solution to Exercise 11.181 (p. 654)
m= − 1
Solution to Exercise 11.183 (p. 654)
x=0
Solution to Exercise 11.185 (p. 654)
a = 14
Solution to Exercise 11.187 (p. 654)
x=6
Solution to Exercise 11.189 (p. 654)
a=4
Solution to Exercise 11.191 (p. 654)
40 (30 − 2) = 1200 − 80 = 1120
Solution to Exercise 11.193 (p. 654)
220 sq cm
Solution to Exercise 11.195 (p. 655)
x=4
Solution to Exercise 11.196 (p. 656)
12 + x
Solution to Exercise 11.197 (p. 656)
8−x
Solution to Exercise 11.198 (p. 656)
x − 14
Solution to Exercise 11.199 (p. 656)
6x = 54
Solution to Exercise 11.200 (p. 656)
2
9x = 11
Solution to Exercise 11.201 (p. 657)
3 + 7x = 9 + 5x
Solution to Exercise 11.202 (p. 657)
2x − 8 = 3x + 1 or 2x − 8 = 1 + 3x
Solution to Exercise 11.203 (p. 657)
x
8 + 7 = 50
Solution to Exercise 11.204 (p. 657)
2
3 − 6x = x + 1
Solution to Exercise 11.205 (p. 657)
x−9=4
Solution to Exercise 11.206 (p. 657)
x−5=8
Solution to Exercise 11.207 (p. 658)
x − 12
Solution to Exercise 11.209 (p. 658)
x−4
Solution to Exercise 11.211 (p. 658)
x+1
Solution to Exercise 11.213 (p. 658)
−7 + x
686 CHAPTER 11. ALGEBRAIC EXPRESSIONS AND EQUATIONS

Solution to Exercise 11.215 (p. 658)


x + (−6)
Solution to Exercise 11.217 (p. 658)
x − [− (−1)]
Solution to Exercise 11.219 (p. 658)
3x + 11
Solution to Exercise 11.221 (p. 658)
2x − 7 = 4
Solution to Exercise 11.223 (p. 658)
14 + 2x = 6
Solution to Exercise 11.225 (p. 658)
3
5x + 8 = 50
Solution to Exercise 11.227 (p. 658)
4
3x + 12 = 5
Solution to Exercise 11.229 (p. 659)
8x + 5 = x + 26
Solution to Exercise 11.231 (p. 659)
x
6 +9=1
Solution to Exercise 11.233 (p. 659)
x
7 + 2 = 17
Solution to Exercise 11.235 (p. 659)
x − 2 = 10
Solution to Exercise 11.237 (p. 659)
x − 3 = 2x − 6
Solution to Exercise 11.239 (p. 659)
5x − 12 = x − 2
Solution to Exercise 11.241 (p. 659)
6−x=x+4
Solution to Exercise 11.243 (p. 659)
9 − x + 1 = 3x + 8
Solution to Exercise 11.245 (p. 659)
2x − 7 − 4x = −x − 6
Solution to Exercise 11.247 (p. 660)
3
4
Solution to Exercise 11.249 (p. 660)
8 23
Solution to Exercise 11.251 (p. 660)
x = −8
Solution to Exercise 11.252 (p. 663)
-6
Solution to Exercise 11.253 (p. 663)
6
Solution to Exercise 11.254 (p. 663)
17 and 18
Solution to Exercise 11.255 (p. 663)
-8, -6, and -4
Solution to Exercise 11.256 (p. 664)
3 inches, 5 inches, and 8 inches
Solution to Exercise 11.257 (p. 665)
24
687

Solution to Exercise 11.259 (p. 665)


9
Solution to Exercise 11.261 (p. 665)
10
Solution to Exercise 11.263 (p. 666)
6
Solution to Exercise 11.265 (p. 666)
1
Solution to Exercise 11.267 (p. 666)
Length=12 inches, Width=6 inches
Solution to Exercise 11.269 (p. 667)
15, 16, 17
Solution to Exercise 11.271 (p. 667)
-2, -1, 0, 1, 2
Solution to Exercise 11.273 (p. 668)
Length is 7, width is 2
Solution to Exercise 11.275 (p. 668)
Length is 15, width is 22
Solution to Exercise 11.277 (p. 668)
The perimeter is 20 inches. Other answers are possible. For example, perimeters such as 26, 32 are possible.
Solution to Exercise 11.279 (p. 668)
15 meters
Solution to Exercise 11.281 (p. 668)
n=0
Solution to Exercise 11.283 (p. 668)
n=1
Solution to Exercise 11.285 (p. 669)
-2, -1, 0, 1
Solution to Exercise 11.287 (p. 669)
-35, -33, -31
Solution to Exercise 11.289 (p. 669)
. . .because the sum of any even number (in this case, 2) o even integers (consecutive or not) is even and,
therefore, cannot be odd (in this case, 139)
Solution to Exercise 11.291 (p. 669)
2, 15, 20
Solution to Exercise 11.293 (p. 669)
6 feet tall
Solution to Exercise 11.295 (p. 669)
46.3 cg
Solution to Exercise 11.297 (p. 672)
6a, −2b, 5c
Solution to Exercise 11.299 (p. 672)
7m, −3n
Solution to Exercise 11.301 (p. 672)
x, 2n, −z
Solution to Exercise 11.303 (p. 672)
−y , −3z
Solution to Exercise 11.305 (p. 672)
−4
Solution to Exercise 11.307 (p. 672)
k
688 CHAPTER 11. ALGEBRAIC EXPRESSIONS AND EQUATIONS

Solution to Exercise 11.309 (p. 672)


7
Solution to Exercise 11.311 (p. 672)
-5
Solution to Exercise 11.313 (p. 672)
-27
Solution to Exercise 11.315 (p. 672)
1
Solution to Exercise 11.317 (p. 672)
-5
Solution to Exercise 11.319 (p. 673)
0
Solution to Exercise 11.321 (p. 673)
5
Solution to Exercise 11.323 (p. 673)
18
Solution to Exercise 11.325 (p. 673)
20
Solution to Exercise 11.327 (p. 673)
11
Solution to Exercise 11.329 (p. 673)
-1
Solution to Exercise 11.331 (p. 673)
-93
Solution to Exercise 11.333 (p. 673)
45
Solution to Exercise 11.335 (p. 673)
55
Solution to Exercise 11.337 (p. 673)
10
Solution to Exercise 11.339 (p. 673)
-6
Solution to Exercise 11.341 (p. 673)
-14
Solution to Exercise 11.343 (p. 674)
2a + 6
Solution to Exercise 11.345 (p. 674)
−4n + 4m − 3
Solution to Exercise 11.347 (p. 674)
−15x + 12y
Solution to Exercise 11.349 (p. 674)
3k + 2m − 6s
Solution to Exercise 11.351 (p. 674)
38h − 7k
Solution to Exercise 11.353 (p. 674)
x=4
Solution to Exercise 11.355 (p. 674)
x = −2
Solution to Exercise 11.357 (p. 674)
x=5
689

Solution to Exercise 11.359 (p. 674)


x = 12
Solution to Exercise 11.361 (p. 675)
x = −15
Solution to Exercise 11.363 (p. 675)
x = −3
Solution to Exercise 11.365 (p. 675)
x=1
Solution to Exercise 11.367 (p. 675)
x = −27
Solution to Exercise 11.369 (p. 675)
y = − 73
Solution to Exercise 11.371 (p. 675)
m = − 85
Solution to Exercise 11.373 (p. 675)
1
x = 15
Solution to Exercise 11.375 (p. 675)
s=1
Solution to Exercise 11.377 (p. 675)
x = −3
Solution to Exercise 11.379 (p. 675)
x = −4
Solution to Exercise 11.381 (p. 675)
x = − 12
Solution to Exercise 11.383 (p. 676)
a = −5
Solution to Exercise 11.385 (p. 676)
x = 15
Solution to Exercise 11.387 (p. 676)
x=5
Solution to Exercise 11.389 (p. 676)
a = 15
2
Solution to Exercise 11.391 (p. 676)
8
x= 5
Solution to Exercise 11.393 (p. 676)
x=2
Solution to Exercise 11.395 (p. 676)
x=5
Solution to Exercise 11.397 (p. 676)
y = −12
Solution to Exercise 11.399 (p. 676)
w = 37
4
Solution to Exercise 11.401 (p. 676)
x=5
Solution to Exercise 11.403 (p. 676)
x = −24
Solution to Exercise 11.405 (p. 676)
w = 38
Solution to Exercise 11.407 (p. 677)
x = 39
2
690 CHAPTER 11. ALGEBRAIC EXPRESSIONS AND EQUATIONS

Solution to Exercise 11.409 (p. 677)


x = − 35
2
Solution to Exercise 11.411 (p. 677)
y = −3
Solution to Exercise 11.413 (p. 677)
6
x= 5
Solution to Exercise 11.415 (p. 677)
2
y= 3
Solution to Exercise 11.417 (p. 677)
m = −3
Solution to Exercise 11.419 (p. 677)
a = −24
Solution to Exercise 11.421 (p. 677)
1
x= 2
Solution to Exercise 11.423 (p. 677)
x = 32
Solution to Exercise 11.425 (p. 677)
x = −2
Solution to Exercise 11.427 (p. 677)
x=4
Solution to Exercise 11.429 (p. 678)
x=6
Solution to Exercise 11.431 (p. 678)
−32
x= 3
Solution to Exercise 11.433 (p. 678)
y = 130
3
Solution to Exercise 11.435 (p. 678)
2
m = 13
Solution to Exercise 11.437 (p. 678)
5x, 6y , 3z
Solution to Exercise 11.438 (p. 678)
8m, −2n, -4
Solution to Exercise 11.439 (p. 678)
-9
Solution to Exercise 11.440 (p. 678)
-9
Solution to Exercise 11.441 (p. 678)
7
Solution to Exercise 11.442 (p. 678)
-15
Solution to Exercise 11.443 (p. 678)
1
Solution to Exercise 11.444 (p. 678)
0
Solution to Exercise 11.445 (p. 678)
-5
Solution to Exercise 11.446 (p. 678)
4y + 6
Solution to Exercise 11.447 (p. 679)
8a + b
691

Solution to Exercise 11.448 (p. 679)


13x − y − 13
Solution to Exercise 11.449 (p. 679)
x=8
Solution to Exercise 11.450 (p. 679)
y=8
Solution to Exercise 11.451 (p. 679)
m = −9
Solution to Exercise 11.452 (p. 679)
a=1
Solution to Exercise 11.453 (p. 679)
x = 26
Solution to Exercise 11.454 (p. 679)
y = −4
Solution to Exercise 11.455 (p. 679)
m=4
Solution to Exercise 11.456 (p. 679)
y = −4
Solution to Exercise 11.457 (p. 679)
x = −8
Solution to Exercise 11.458 (p. 679)
−9
y= 5
Solution to Exercise 11.459 (p. 679)
-14, -12, -10
Solution to Exercise 11.460 (p. 679)
l = 13, w = 6
Solution to Exercise 11.461 (p. 679)
6, -9, 0, 1
692 INDEX

Index of Keywords and Terms


Keywords are listed by the section with that keyword (page numbers are in parentheses). Keywords
do not necessarily appear in the text of the page. They are merely associated with that section. Ex.
apples, Ÿ 1.1 (1) Terms are referenced by the page they appear on. Ex. apples, 1

" "invert the divisor and, 255 Ÿ 3.5(178), Ÿ 3.6(182), Ÿ 3.7(188), Ÿ 3.8(191),
Ÿ 3.9(196), Ÿ 4.1(211), Ÿ 4.2(212), Ÿ 4.3(221),
. . Denominate numbers, 418 Ÿ 4.4(231), Ÿ 4.5(242), Ÿ 4.6(253), Ÿ 4.7(259),
Ÿ 4.8(265), Ÿ 4.9(268), Ÿ 4.10(274), Ÿ 5.1(293),
A absolute value, Ÿ 10.4(586), 586
Ÿ 5.2(294), Ÿ 5.3(298), Ÿ 5.4(304), Ÿ 5.5(308),
Acknowledgments, Ÿ (7)
Ÿ 5.6(312), Ÿ 5.7(316), Ÿ 5.8(320), Ÿ 5.9(321),
add whole numbers, 30
Ÿ 5.10(325), Ÿ 6.1(337), Ÿ 6.2(338), Ÿ 6.3(344),
addends, 30, 69
Ÿ 6.4(348), Ÿ 6.5(352), Ÿ 6.6(358), Ÿ 6.7(369),
Adding, Ÿ 1.1(9), Ÿ 1.7(62), Ÿ 1.9(69),
Ÿ 6.8(381), Ÿ 6.9(386), Ÿ 6.10(392), Ÿ 6.11(395),
Ÿ 1.10(74)
Ÿ 6.12(396), Ÿ 6.13(400), Ÿ 7.1(417), Ÿ 7.2(417),
addition, Ÿ 1.5(29), 30, Ÿ 1.7(62), Ÿ 1.8(67), 69,
Ÿ 7.3(423), Ÿ 7.4(429), Ÿ 7.5(435), Ÿ 7.6(442),
Ÿ 1.9(69), Ÿ 1.10(74), Ÿ 5.1(293), Ÿ 5.2(294),
Ÿ 7.7(446), Ÿ 7.8(456), Ÿ 7.9(458), Ÿ 7.10(462),
Ÿ 5.3(298), Ÿ 5.4(304), Ÿ 5.5(308), Ÿ 5.6(312),
Ÿ 8.1(475), Ÿ 8.2(475), Ÿ 8.3(482), Ÿ 8.4(485),
Ÿ 5.7(316), Ÿ 5.8(320), Ÿ 5.9(321), Ÿ 5.10(325),
Ÿ 8.5(491), Ÿ 8.6(493), Ÿ 8.7(494), Ÿ 8.8(500),
Ÿ 6.1(337), Ÿ 6.2(338), Ÿ 6.3(344), Ÿ 6.4(348),
Ÿ 9.1(511), Ÿ 9.2(512), Ÿ 9.3(517), Ÿ 9.4(522),
Ÿ 6.5(352), Ÿ 6.6(358), Ÿ 6.7(369), Ÿ 6.8(381),
Ÿ 9.5(530), Ÿ 9.6(540), Ÿ 9.7(555), Ÿ 9.8(557),
Ÿ 6.9(386), Ÿ 6.10(392), Ÿ 6.11(395),
Ÿ 9.9(562), Ÿ 10.1(575), Ÿ 10.2(576),
Ÿ 6.12(396), Ÿ 6.13(400), Ÿ 10.5(590),
Ÿ 10.3(582), Ÿ 10.4(586), Ÿ 10.5(590),
Ÿ 11.3(636)
Ÿ 10.6(597), Ÿ 10.7(602), Ÿ 10.8(611),
addition/subtraction property of equality, 641
Ÿ 10.9(612), Ÿ 10.10(616), Ÿ 11.1(629),
additive identity, Ÿ 1.7(62), 64, 69, 592
Ÿ 11.2(630), Ÿ 11.3(636), Ÿ 11.4(639),
algebra, Ÿ 11.1(629), Ÿ 11.2(630), Ÿ 11.3(636),
Ÿ 11.5(647), Ÿ 11.6(655), Ÿ 11.7(660),
Ÿ 11.4(639), Ÿ 11.5(647), Ÿ 11.6(655),
Ÿ 11.8(669), Ÿ 11.9(671), Ÿ 11.10(678)
Ÿ 11.7(660), Ÿ 11.8(669), Ÿ 11.9(671),
Ÿ 11.10(678) C calculator, Ÿ 1.6(44), Ÿ 2.3(105)
algebraic expression, 630 calculators, Ÿ 1.5(29)
Answers to Practice Sets, 2 cancelling common factors, 233
approximation, Ÿ 1.4(21) carrying, 32, 93
area, Ÿ 9.6(540), 540 circumference, Ÿ 9.5(530), 532
associative property, Ÿ 2.6(130) cluster, 482
associative property of addition, Ÿ 1.7(62) clustering, Ÿ 8.3(482)
coecient, 631, 631
B Bar, 160
combining terms, Ÿ 11.3(636)
base, 153, Ÿ 7.7(446), 446
commas, 11, 68
base ten, 11, 68
common factor, 178
Base Ten Positional Number System, Ÿ 1.2(10)
commutative property, Ÿ 2.6(130)
borrow more than once, 49
commutative property of addition, Ÿ 1.7(62)
borrowing, Ÿ 1.6(44), 48
Comparison by division, 418
Braces, 160
Comparison of two numbers by subtraction,
Brackets, 160
418
Burzynski, Ÿ (1), Ÿ 1.1(9), Ÿ 2.1(91),
complex decimals, 345
Ÿ 3.1(151), Ÿ 3.2(152), Ÿ 3.3(160), Ÿ 3.4(170),
complex fraction, 312
INDEX 693

complex fractions, Ÿ 5.6(312) Ellis, Ÿ (1), Ÿ 1.1(9), Ÿ 2.1(91), Ÿ 3.1(151),


composite number, 172 Ÿ 3.2(152), Ÿ 3.3(160), Ÿ 3.4(170), Ÿ 3.5(178),
conditional equations, 639 Ÿ 3.6(182), Ÿ 3.7(188), Ÿ 3.8(191), Ÿ 3.9(196),
constant, 576 Ÿ 4.1(211), Ÿ 4.2(212), Ÿ 4.3(221), Ÿ 4.4(231),
conversion, Ÿ 6.3(344) Ÿ 4.5(242), Ÿ 4.6(253), Ÿ 4.7(259), Ÿ 4.8(265),
coordinate, 576 Ÿ 4.9(268), Ÿ 4.10(274), Ÿ 5.1(293), Ÿ 5.2(294),
counting numbers, 577 Ÿ 5.3(298), Ÿ 5.4(304), Ÿ 5.5(308), Ÿ 5.6(312),
cross products, 231 Ÿ 5.7(316), Ÿ 5.8(320), Ÿ 5.9(321), Ÿ 5.10(325),
cube root, 154, 154 Ÿ 6.1(337), Ÿ 6.2(338), Ÿ 6.3(344), Ÿ 6.4(348),
cubed, 153 Ÿ 6.5(352), Ÿ 6.6(358), Ÿ 6.7(369), Ÿ 6.8(381),
Ÿ 6.9(386), Ÿ 6.10(392), Ÿ 6.11(395),
D decimal fraction, 339 Ÿ 6.12(396), Ÿ 6.13(400), Ÿ 7.1(417), Ÿ 7.2(417),
decimal fractions, 339 Ÿ 7.3(423), Ÿ 7.4(429), Ÿ 7.5(435), Ÿ 7.6(442),
decimal point, 339 Ÿ 7.7(446), Ÿ 7.8(456), Ÿ 7.9(458), Ÿ 7.10(462),
decimals, Ÿ 6.1(337), Ÿ 6.2(338), 339, Ÿ 8.1(475), Ÿ 8.2(475), Ÿ 8.3(482), Ÿ 8.4(485),
Ÿ 6.3(344), Ÿ 6.4(348), Ÿ 6.5(352), Ÿ 6.6(358), Ÿ 8.5(491), Ÿ 8.6(493), Ÿ 8.7(494), Ÿ 8.8(500),
Ÿ 6.7(369), Ÿ 6.8(381), Ÿ 6.9(386), Ÿ 6.10(392), Ÿ 9.1(511), Ÿ 9.2(512), Ÿ 9.3(517), Ÿ 9.4(522),
Ÿ 6.11(395), Ÿ 6.12(396), Ÿ 6.13(400) Ÿ 9.5(530), Ÿ 9.6(540), Ÿ 9.7(555), Ÿ 9.8(557),
denominate numbers, Ÿ 7.2(417), 522 Ÿ 9.9(562), Ÿ 10.1(575), Ÿ 10.2(576),
denominator, 213, 513 Ÿ 10.3(582), Ÿ 10.4(586), Ÿ 10.5(590),
diameter, Ÿ 9.5(530), 532 Ÿ 10.6(597), Ÿ 10.7(602), Ÿ 10.8(611),
dierence, 45 Ÿ 10.9(612), Ÿ 10.10(616), Ÿ 11.1(629),
digits, 11, 68 Ÿ 11.2(630), Ÿ 11.3(636), Ÿ 11.4(639),
distributive property, Ÿ 8.4(485), 486 Ÿ 11.5(647), Ÿ 11.6(655), Ÿ 11.7(660),
divide, Ÿ 2.3(105), Ÿ 2.4(113), Ÿ 2.5(126) Ÿ 11.8(669), Ÿ 11.9(671), Ÿ 11.10(678)
dividend, 105, 135 equal symbol, 30
divisible by 10, 128 equation solving, 641
divisible by 2, 126 equations, Ÿ 11.1(629), Ÿ 11.2(630),
divisible by 3, 126 Ÿ 11.3(636), Ÿ 11.4(639), Ÿ 11.5(647),
divisible by 4, 126 Ÿ 11.6(655), Ÿ 11.7(660), Ÿ 11.8(669),
divisible by 5, 127 Ÿ 11.9(671), Ÿ 11.10(678)
divisible by 6, 127 equivalent fractions, Ÿ 4.4(231), 231
divisible by 8, 127 estimation, Ÿ 8.1(475), Ÿ 8.2(475), 476,
divisible by 9, 128 Ÿ 8.3(482), Ÿ 8.4(485), Ÿ 8.5(491), Ÿ 8.6(493),
division, Ÿ 2.3(105), 105, Ÿ 2.4(113), Ÿ 2.5(126), Ÿ 8.7(494), Ÿ 8.8(500)
Ÿ 2.7(135), 135, Ÿ 2.8(136), Ÿ 2.9(139), exact divisions, 381
Ÿ 4.1(211), Ÿ 4.2(212), Ÿ 4.3(221), Ÿ 4.4(231), exam, Ÿ 1.10(74)
Ÿ 4.5(242), Ÿ 4.6(253), Ÿ 4.7(259), Ÿ 4.8(265), Exercise Supplement, 2
Ÿ 4.9(268), Ÿ 4.10(274), Ÿ 6.1(337), Ÿ 6.2(338), exercises, Ÿ 1.9(69), Ÿ 1.10(74)
Ÿ 6.3(344), Ÿ 6.4(348), Ÿ 6.5(352), Ÿ 6.6(358), Exercises for Review, 2
Ÿ 6.7(369), Ÿ 6.8(381), Ÿ 6.9(386), Ÿ 6.10(392), exponent, 152, 152, 153
Ÿ 6.11(395), Ÿ 6.12(396), Ÿ 6.13(400), exponents, Ÿ 3.1(151), Ÿ 3.2(152), Ÿ 3.3(160),
Ÿ 10.7(602) Ÿ 3.4(170), Ÿ 3.5(178), Ÿ 3.6(182), Ÿ 3.7(188),
divisor, 105, 135 Ÿ 3.8(191), Ÿ 3.9(196)
double-negative, Ÿ 10.3(582) Exponential notation, 152
expressions, Ÿ 11.1(629), Ÿ 11.2(630), 630,
E Elementary, Ÿ 1.2(10), Ÿ 1.3(15), Ÿ 1.4(21)
Ÿ 11.3(636), Ÿ 11.4(639), Ÿ 11.5(647),
elementary algebra, Ÿ (1)
Ÿ 11.6(655), Ÿ 11.7(660), Ÿ 11.8(669),
elementary math, Ÿ 1.5(29), Ÿ 1.6(44),
Ÿ 11.9(671), Ÿ 11.10(678)
Ÿ 1.7(62), Ÿ 1.8(67), Ÿ 1.9(69), Ÿ 2.2(92),
Ÿ 2.3(105), Ÿ 2.5(126), Ÿ 2.6(130), Ÿ 2.7(135), F factor, 170, Ÿ 4.7(259), Ÿ 7.3(423)
Ÿ 2.8(136), Ÿ 2.9(139) factorization, Ÿ 3.1(151), Ÿ 3.2(152), Ÿ 3.3(160),
694 INDEX

Ÿ 3.4(170), Ÿ 3.5(178), Ÿ 3.6(182), Ÿ 3.7(188), graph, 576


Ÿ 3.8(191), Ÿ 3.9(196) Graphing, 68
factors, 93, 135, 170, Ÿ 11.2(630), 630 Graphing Whole Numbers, Ÿ 1.2(10)
facts, Ÿ 2.5(126) greatest common factor, Ÿ 3.5(178), 179
rst partial product, 95 greatest common factor (GCF), 179
ve-step method, Ÿ 7.4(429) grouping symbols, Ÿ 5.5(308)
formula, 533
fourth root, 154 H Hindu-Arabic, 68
fraction, 213 Hindu-Arabic Numeration System, Ÿ 1.2(10),
fraction bar, 213 11
fractions, Ÿ 4.1(211), Ÿ 4.2(212), Ÿ 4.3(221), hyphen, 216
Ÿ 4.4(231), Ÿ 4.5(242), Ÿ 4.6(253), Ÿ 4.7(259),
I improper fraction, Ÿ 4.3(221)
Ÿ 4.8(265), Ÿ 4.9(268), Ÿ 4.10(274), Ÿ 5.1(293),
indeterminant, 108
Ÿ 5.2(294), Ÿ 5.3(298), Ÿ 5.4(304), Ÿ 5.5(308),
index, 154
Ÿ 5.6(312), Ÿ 5.7(316), Ÿ 5.8(320), Ÿ 5.9(321),
integers, 577
Ÿ 5.10(325), Ÿ 8.5(491), 577
fractions of one percent, Ÿ 7.6(442) K k-12, Ÿ (1)
functions, Ÿ 11.2(630) key concepts, Ÿ 1.8(67)
Fundamentals, Ÿ (1), Ÿ (7), Ÿ 1.1(9), Ÿ 2.1(91),
Ÿ 2.2(92), Ÿ 3.1(151), Ÿ 3.2(152), Ÿ 3.3(160), L LCM, 184, 184
Ÿ 3.4(170), Ÿ 3.5(178), Ÿ 3.6(182), Ÿ 3.7(188), least common multiple, Ÿ 3.6(182), 184, 184
Ÿ 3.8(191), Ÿ 3.9(196), Ÿ 4.1(211), Ÿ 4.2(212), like denominate numbers, 418
Ÿ 4.3(221), Ÿ 4.4(231), Ÿ 4.5(242), Ÿ 4.6(253), like denominators, Ÿ 5.2(294)
Ÿ 4.7(259), Ÿ 4.8(265), Ÿ 4.9(268), Ÿ 4.10(274), like terms, Ÿ 11.3(636), 636
Ÿ 5.1(293), Ÿ 5.2(294), Ÿ 5.3(298), Ÿ 5.4(304), line, 13
Ÿ 5.5(308), Ÿ 5.6(312), Ÿ 5.7(316), Ÿ 5.8(320), lowest terms, Ÿ 4.4(231)
Ÿ 5.9(321), Ÿ 5.10(325), Ÿ 6.1(337), Ÿ 6.2(338),
M Mass, 519
Ÿ 6.3(344), Ÿ 6.4(348), Ÿ 6.5(352), Ÿ 6.6(358),
math, Ÿ 1.2(10), Ÿ 1.3(15), Ÿ 1.4(21)
Ÿ 6.7(369), Ÿ 6.8(381), Ÿ 6.9(386), Ÿ 6.10(392),
Mathematics, Ÿ (1), Ÿ (7), Ÿ 1.1(9), Ÿ 1.7(62),
Ÿ 6.11(395), Ÿ 6.12(396), Ÿ 6.13(400),
Ÿ 2.1(91), Ÿ 2.2(92), Ÿ 2.3(105), Ÿ 3.1(151),
Ÿ 7.1(417), Ÿ 7.2(417), Ÿ 7.3(423), Ÿ 7.4(429),
Ÿ 3.2(152), Ÿ 3.3(160), Ÿ 3.4(170), Ÿ 3.5(178),
Ÿ 7.5(435), Ÿ 7.6(442), Ÿ 7.7(446), Ÿ 7.8(456),
Ÿ 3.6(182), Ÿ 3.7(188), Ÿ 3.8(191), Ÿ 3.9(196),
Ÿ 7.9(458), Ÿ 7.10(462), Ÿ 8.1(475), Ÿ 8.2(475),
Ÿ 4.1(211), Ÿ 4.2(212), Ÿ 4.3(221), Ÿ 4.4(231),
Ÿ 8.3(482), Ÿ 8.4(485), Ÿ 8.5(491), Ÿ 8.6(493),
Ÿ 4.5(242), Ÿ 4.6(253), Ÿ 4.7(259), Ÿ 4.8(265),
Ÿ 8.7(494), Ÿ 8.8(500), Ÿ 9.1(511), Ÿ 9.2(512),
Ÿ 4.9(268), Ÿ 4.10(274), Ÿ 5.1(293), Ÿ 5.2(294),
Ÿ 9.3(517), Ÿ 9.4(522), Ÿ 9.5(530), Ÿ 9.6(540),
Ÿ 5.3(298), Ÿ 5.4(304), Ÿ 5.5(308), Ÿ 5.6(312),
Ÿ 9.7(555), Ÿ 9.8(557), Ÿ 9.9(562), Ÿ 10.1(575),
Ÿ 5.7(316), Ÿ 5.8(320), Ÿ 5.9(321), Ÿ 5.10(325),
Ÿ 10.2(576), Ÿ 10.3(582), Ÿ 10.4(586),
Ÿ 6.1(337), Ÿ 6.2(338), Ÿ 6.3(344), Ÿ 6.4(348),
Ÿ 10.5(590), Ÿ 10.6(597), Ÿ 10.7(602),
Ÿ 6.5(352), Ÿ 6.6(358), Ÿ 6.7(369), Ÿ 6.8(381),
Ÿ 10.8(611), Ÿ 10.9(612), Ÿ 10.10(616),
Ÿ 6.9(386), Ÿ 6.10(392), Ÿ 6.11(395),
Ÿ 11.1(629), Ÿ 11.2(630), Ÿ 11.3(636),
Ÿ 6.12(396), Ÿ 6.13(400), Ÿ 7.1(417), Ÿ 7.2(417),
Ÿ 11.4(639), Ÿ 11.5(647), Ÿ 11.6(655),
Ÿ 7.3(423), Ÿ 7.4(429), Ÿ 7.5(435), Ÿ 7.6(442),
Ÿ 11.7(660), Ÿ 11.8(669), Ÿ 11.9(671),
Ÿ 7.7(446), Ÿ 7.8(456), Ÿ 7.9(458), Ÿ 7.10(462),
Ÿ 11.10(678)
Ÿ 8.1(475), Ÿ 8.2(475), Ÿ 8.3(482), Ÿ 8.4(485),
fundamentals of mathematics, Ÿ 2.4(113),
Ÿ 8.5(491), Ÿ 8.6(493), Ÿ 8.7(494), Ÿ 8.8(500),
Ÿ 2.5(126), Ÿ 2.6(130), Ÿ 2.7(135), Ÿ 2.8(136),
Ÿ 9.1(511), Ÿ 9.2(512), Ÿ 9.3(517), Ÿ 9.4(522),
Ÿ 2.9(139)
Ÿ 9.5(530), Ÿ 9.6(540), Ÿ 9.7(555), Ÿ 9.8(557),
G GCF, 179 Ÿ 9.9(562), Ÿ 10.1(575), Ÿ 10.2(576),
geometry, Ÿ 9.1(511), Ÿ 9.2(512), Ÿ 9.3(517), Ÿ 10.3(582), Ÿ 10.4(586), Ÿ 10.5(590),
Ÿ 9.4(522), Ÿ 9.5(530), Ÿ 9.6(540), Ÿ 9.7(555), Ÿ 10.6(597), Ÿ 10.7(602), Ÿ 10.8(611),
Ÿ 9.8(557), Ÿ 9.9(562) Ÿ 10.9(612), Ÿ 10.10(616), Ÿ 11.1(629),
INDEX 695

Ÿ 11.2(630), Ÿ 11.3(636), Ÿ 11.4(639), origin, 13, 577


Ÿ 11.5(647), Ÿ 11.6(655), Ÿ 11.7(660),
Ÿ 11.8(669), Ÿ 11.9(671), Ÿ 11.10(678) P Parentheses, 160
measurement, Ÿ 9.1(511), Ÿ 9.2(512), 512, Parentheses in addition, 69
Ÿ 9.3(517), Ÿ 9.4(522), Ÿ 9.5(530), Ÿ 9.6(540), percent, Ÿ 7.5(435), 436, Ÿ 7.7(446), 446
Ÿ 9.7(555), Ÿ 9.8(557), Ÿ 9.9(562) percentage, Ÿ 7.7(446), 446
metric prexes, 517 percents, 436
metric system, Ÿ 9.3(517) perimeter, Ÿ 9.5(530), 530
minuend, 45, 48 period, 68
minus symbol, 45 periods, 11
mixed numbers, Ÿ 4.3(221), 224, Ÿ 5.4(304) pi, Ÿ 9.5(530)
multiples, 182 plus symbol, 30
multiplicand, 135 polygon, 530
multiplication, Ÿ 2.2(92), 92, Ÿ 2.6(130), positional, 11
Ÿ 2.7(135), 135, Ÿ 2.8(136), Ÿ 2.9(139), positive, 582
Ÿ 4.1(211), Ÿ 4.2(212), Ÿ 4.3(221), Ÿ 4.4(231), positive fraction, 213
Ÿ 4.5(242), Ÿ 4.6(253), Ÿ 4.7(259), Ÿ 4.8(265), positive improper fractions, 222
Ÿ 4.9(268), Ÿ 4.10(274), Ÿ 6.1(337), Ÿ 6.2(338), positive mixed number, 222
Ÿ 6.3(344), Ÿ 6.4(348), Ÿ 6.5(352), Ÿ 6.6(358), positive number, 582
Ÿ 6.7(369), Ÿ 6.8(381), Ÿ 6.9(386), Ÿ 6.10(392), positive numbers, 577, Ÿ 10.3(582)
Ÿ 6.11(395), Ÿ 6.12(396), Ÿ 6.13(400), positive proper fractions, 221
Ÿ 10.7(602) power, 153
multiplication statement, 260 Practice Sets, 1
multiplicative identity, Ÿ 2.6(130), 132, 136 preface, Ÿ (1)
multiplier, 92, 135 prime factorization, 173
multiply.", 255 prime number, 172
multiplicand, 92 product, 93, 135, 170
product statements, Ÿ 4.7(259)
N natural, 577 products, Ÿ 4.7(259)
negative, Ÿ 10.3(582), 582, 582 Prociency Exam, 2
negative numbers, 577 proper fraction, Ÿ 4.3(221)
non-terminating division, 381 properties, Ÿ 1.8(67), Ÿ 2.6(130)
nonterminating divisions, Ÿ 6.1(337), proportion, 423, Ÿ 7.4(429)
Ÿ 6.2(338), Ÿ 6.3(344), Ÿ 6.4(348), Ÿ 6.5(352), proportions, Ÿ 7.3(423)
Ÿ 6.6(358), Ÿ 6.7(369), Ÿ 6.8(381), Ÿ 6.9(386), pure numbers, Ÿ 7.2(417), 418
Ÿ 6.10(392), Ÿ 6.11(395), Ÿ 6.12(396),
Ÿ 6.13(400) Q quotient, 105, 135
number, 10, 10, 67
R radical, 154
number line, 13, 68
radical sign, 154, 154
Numbers, Ÿ 1.2(10), Ÿ 1.4(21)
radicand, 154
numeral, 10, 10, 67
radius, Ÿ 9.5(530), 533
Numerals, Ÿ 1.2(10)
raising a fraction to higher terms, 236
numerator, 213, 513
rate, 419, 423
numerical coecient, 631
rates, Ÿ 7.1(417), Ÿ 7.2(417), Ÿ 7.3(423),
numerical evaluation, Ÿ 11.2(630), 632
Ÿ 7.4(429), Ÿ 7.5(435), Ÿ 7.6(442), Ÿ 7.7(446),
numerical expression, 630
Ÿ 7.8(456), Ÿ 7.9(458), Ÿ 7.10(462)
O Objectives, 2 ratio, 419, 423, 436
Opposites, 583 rational numbers, 577
order of numbers, Ÿ 5.5(308) ratios, Ÿ 7.1(417), Ÿ 7.2(417), Ÿ 7.3(423),
order of operations, Ÿ 5.7(316) Ÿ 7.4(429), Ÿ 7.5(435), Ÿ 7.6(442), Ÿ 7.7(446),
ordered number system, 308 Ÿ 7.8(456), Ÿ 7.9(458), Ÿ 7.10(462)
696 INDEX

reading, Ÿ 1.3(15) Ÿ 5.3(298), Ÿ 5.4(304), Ÿ 5.5(308), Ÿ 5.6(312),


reading strategy, 660 Ÿ 5.7(316), Ÿ 5.8(320), Ÿ 5.9(321), Ÿ 5.10(325),
real number, 577 Ÿ 6.1(337), Ÿ 6.2(338), Ÿ 6.3(344), Ÿ 6.4(348),
real number line, 576 Ÿ 6.5(352), Ÿ 6.6(358), Ÿ 6.7(369), Ÿ 6.8(381),
real numbers, Ÿ 10.3(582) Ÿ 6.9(386), Ÿ 6.10(392), Ÿ 6.11(395),
reciprocal, Ÿ 4.6(253) Ÿ 6.12(396), Ÿ 6.13(400), Ÿ 10.6(597),
reciprocals, 253 Ÿ 11.3(636)
reduced, 233 subtrahend, 45, 48
reduced to lowest terms, 233 sum, 30, 69
reducing a fraction, 233 summary, Ÿ 2.7(135)
relatively prime, 233, 233 Summary of Key Concepts, 2
remainder, Ÿ 2.4(113) symbols, Ÿ 11.6(655)
repeating decimal, 382
roots, Ÿ 3.1(151), Ÿ 3.2(152), Ÿ 3.3(160), T terminating division, 381
Ÿ 3.4(170), Ÿ 3.5(178), Ÿ 3.6(182), Ÿ 3.7(188), terms, 30, Ÿ 11.2(630), 630
Ÿ 3.8(191), Ÿ 3.9(196) test, Ÿ 2.9(139)
round a whole number, 22 total product, 95
rounding, Ÿ 1.4(21), 21, 68, Ÿ 8.2(475),
U unit fraction, 513
Ÿ 8.5(491)
United States System, Ÿ 9.2(512)
S Sample Sets, 1 unlike denominate numbers, 418
second partial product, 95 unlike denominators, Ÿ 5.3(298)
Section Exercises, 2 use parentheses, 63
Section Overview, 1
V value of a digit, 12
sign, 582
variable, 576
signed numbers, Ÿ 10.1(575), Ÿ 10.2(576),
volume, Ÿ 9.6(540), 545
Ÿ 10.3(582), Ÿ 10.4(586), Ÿ 10.5(590),
Ÿ 10.6(597), Ÿ 10.7(602), Ÿ 10.8(611), W weight, 519
Ÿ 10.9(612), Ÿ 10.10(616) whole number, 68
simple fraction, 312 Whole Numbers, Ÿ 1.1(9), Ÿ 1.2(10), 12,
simple fractions, Ÿ 5.6(312) Ÿ 1.3(15), Ÿ 1.4(21), Ÿ 1.5(29), Ÿ 1.6(44),
simplied, 522, 636 Ÿ 1.7(62), Ÿ 1.8(67), Ÿ 1.9(69), Ÿ 1.10(74),
solutions, 639 Ÿ 2.2(92), Ÿ 2.3(105), Ÿ 2.4(113), Ÿ 2.5(126),
solved, 639 Ÿ 2.6(130), Ÿ 2.7(135), Ÿ 2.8(136), Ÿ 2.9(139),
square root, 154, 154 Ÿ 3.1(151), Ÿ 3.2(152), Ÿ 3.3(160), Ÿ 3.4(170),
squared, 153 Ÿ 3.5(178), Ÿ 3.6(182), Ÿ 3.7(188), Ÿ 3.8(191),
standard unit of measure, 512 Ÿ 3.9(196), 577
Subtracting, Ÿ 1.1(9) word problems, Ÿ 11.6(655)
subtraction, Ÿ 1.6(44), 45, Ÿ 1.8(67), 69, writing, Ÿ 1.3(15)
Ÿ 1.9(69), Ÿ 1.10(74), Ÿ 5.1(293), Ÿ 5.2(294),
ATTRIBUTIONS 697

Attributions
Collection: Fundamentals of Mathematics
Edited by: Denny Burzynski, Wade Ellis
URL: http://cnx.org/content/col10615/1.4/
License: http://creativecommons.org/licenses/by/2.0/
Module: "Fundamentals of Mathematics: Preface"
Used here as: "Preface"
By: Denny Burzynski, Wade Ellis
URL: http://cnx.org/content/m18884/1.4/
Pages: 1-5
Copyright: Denny Burzynski, Wade Ellis
License: http://creativecommons.org/licenses/by/2.0/
Module: "Fundamentals of Mathematics: Acknowledgements"
Used here as: "Acknowledgements"
By: Denny Burzynski, Wade Ellis
URL: http://cnx.org/content/m34775/1.2/
Pages: 7-8
Copyright: Denny Burzynski, Wade Ellis
License: http://creativecommons.org/licenses/by/3.0/
Module: "Addition and Subtraction of Whole Numbers: Objectives"
Used here as: "Objectives"
By: Denny Burzynski, Wade Ellis
URL: http://cnx.org/content/m18885/1.4/
Page: 9
Copyright: Denny Burzynski, Wade Ellis
License: http://creativecommons.org/licenses/by/2.0/
Module: "Addition and Subtraction of Whole Numbers: Whole Numbers"
Used here as: "Whole Numbers"
By: Denny Burzynski, Wade Ellis
URL: http://cnx.org/content/m34795/1.4/
Pages: 10-15
Copyright: Denny Burzynski, Wade Ellis
License: http://creativecommons.org/licenses/by/3.0/
Module: "Addition and Subtraction of Whole Numbers: Reading and Writing Whole Numbers"
Used here as: "Reading and Writing Whole Numbers"
By: Denny Burzynski, Wade Ellis
URL: http://cnx.org/content/m34778/1.5/
Pages: 15-21
Copyright: Denny Burzynski, Wade Ellis
License: http://creativecommons.org/licenses/by/3.0/
Module: "Addition and Subtraction of Whole Numbers: Rounding Whole Numbers"
Used here as: "Rounding Whole Numbers"
By: Wade Ellis, Denny Burzynski
URL: http://cnx.org/content/m34780/1.3/
Pages: 21-29
Copyright: Wade Ellis, Denny Burzynski
License: http://creativecommons.org/licenses/by/3.0/
698 ATTRIBUTIONS

Module: "Addition and Subtraction of Whole Numbers: Addition of Whole Numbers"


Used here as: "Addition of Whole Numbers"
By: Denny Burzynski, Wade Ellis
URL: http://cnx.org/content/m34786/1.4/
Pages: 29-44
Copyright: Denny Burzynski, Wade Ellis
License: http://creativecommons.org/licenses/by/3.0/
Module: "Addition and Subtraction of Whole Numbers: Subtraction of Whole Numbers"
Used here as: "Subtraction of Whole Numbers"
By: Wade Ellis, Denny Burzynski
URL: http://cnx.org/content/m34784/1.5/
Pages: 44-62
Copyright: Wade Ellis, Denny Burzynski
License: http://creativecommons.org/licenses/by/3.0/
Module: "Addition and Subtraction of Whole Numbers: Properties of Addition"
Used here as: "Properties of Addition"
By: Wade Ellis, Denny Burzynski
URL: http://cnx.org/content/m34802/1.5/
Pages: 62-67
Copyright: Wade Ellis, Denny Burzynski
License: http://creativecommons.org/licenses/by/3.0/
Module: "Addition and Subtraction of Whole Numbers: Summary of Key Concepts"
Used here as: "Summary of Key Concepts"
By: Wade Ellis, Denny Burzynski
URL: http://cnx.org/content/m34798/1.3/
Pages: 67-69
Copyright: Wade Ellis, Denny Burzynski
License: http://creativecommons.org/licenses/by/3.0/
Module: "Addition and Subtraction of Whole Numbers: Exercise Supplement"
Used here as: "Exercise Supplement"
By: Denny Burzynski, Wade Ellis
URL: http://cnx.org/content/m34800/1.5/
Pages: 69-74
Copyright: Denny Burzynski, Wade Ellis
License: http://creativecommons.org/licenses/by/3.0/
Module: "Addition and Subtraction of Whole Numbers: Prociency Exam"
Used here as: "Prociency Exam"
By: Wade Ellis, Denny Burzynski
URL: http://cnx.org/content/m34805/1.4/
Pages: 74-75
Copyright: Wade Ellis, Denny Burzynski
License: http://creativecommons.org/licenses/by/3.0/
ATTRIBUTIONS 699

Module: "Multiplication and Division of Whole Numbers: Objectives"


Used here as: "Objectives"
By: Denny Burzynski, Wade Ellis
URL: http://cnx.org/content/m18888/1.4/
Page: 91
Copyright: Denny Burzynski, Wade Ellis
License: http://creativecommons.org/licenses/by/2.0/
Module: "Multiplication and Division of Whole Numbers: Multiplication of Whole Numbers"
Used here as: "Multiplication of Whole Numbers"
By: Wade Ellis, Denny Burzynski
URL: http://cnx.org/content/m34863/1.2/
Pages: 92-105
Copyright: Wade Ellis, Denny Burzynski
License: http://creativecommons.org/licenses/by/3.0/
Module: "Multiplication and Division of Whole Numbers: Concepts of Division of Whole Numbers"
Used here as: "Concepts of Division of Whole Numbers"
By: Wade Ellis, Denny Burzynski
URL: http://cnx.org/content/m34864/1.3/
Pages: 105-113
Copyright: Wade Ellis, Denny Burzynski
License: http://creativecommons.org/licenses/by/3.0/
Module: "Multiplication and Division of Whole Numbers: Division of Whole Numbers"
Used here as: "Division of Whole Numbers "
By: Wade Ellis, Denny Burzynski
URL: http://cnx.org/content/m34865/1.2/
Pages: 113-126
Copyright: Wade Ellis, Denny Burzynski
License: http://creativecommons.org/licenses/by/3.0/
Module: "Multiplication and Division of Whole Numbers: Some Interesting Facts about Division"
Used here as: "Some Interesting Facts about Division "
By: Wade Ellis, Denny Burzynski
URL: http://cnx.org/content/m34866/1.2/
Pages: 126-130
Copyright: Wade Ellis, Denny Burzynski
License: http://creativecommons.org/licenses/by/3.0/
Module: "Multiplication and Division of Whole Numbers: Properties of Multiplication"
Used here as: "Properties of Multiplication "
By: Wade Ellis, Denny Burzynski
URL: http://cnx.org/content/m34867/1.2/
Pages: 130-135
Copyright: Wade Ellis, Denny Burzynski
License: http://creativecommons.org/licenses/by/3.0/
700 ATTRIBUTIONS

Module: "Multiplication and Division of Whole Numbers: Summary of Key Concepts"


Used here as: "Summary of Key Concepts "
By: Wade Ellis, Denny Burzynski
URL: http://cnx.org/content/m34868/1.2/
Pages: 135-136
Copyright: Wade Ellis, Denny Burzynski
License: http://creativecommons.org/licenses/by/3.0/
Module: "Multiplication and Division of Whole Numbers: Exercise Supplement"
Used here as: "Exercise Supplement "
By: Wade Ellis, Denny Burzynski
URL: http://cnx.org/content/m34869/1.2/
Pages: 136-139
Copyright: Wade Ellis, Denny Burzynski
License: http://creativecommons.org/licenses/by/3.0/
Module: "Multiplication and Division of Whole Numbers: Prociency Exam"
Used here as: "Prociency Exam "
By: Wade Ellis, Denny Burzynski
URL: http://cnx.org/content/m34870/1.2/
Pages: 139-140
Copyright: Wade Ellis, Denny Burzynski
License: http://creativecommons.org/licenses/by/3.0/
Module: "Exponents, Roots, and Factorizations of Whole Numbers: Objectives"
Used here as: "Objectives"
By: Denny Burzynski, Wade Ellis
URL: http://cnx.org/content/m18890/1.4/
Page: 151
Copyright: Denny Burzynski, Wade Ellis
License: http://creativecommons.org/licenses/by/2.0/
Module: "Exponents, Roots, and Factorization of Whole Numbers: Exponents and Roots"
Used here as: "Exponents and Roots "
By: Wade Ellis, Denny Burzynski
URL: http://cnx.org/content/m34871/1.2/
Pages: 152-160
Copyright: Wade Ellis, Denny Burzynski
License: http://creativecommons.org/licenses/by/3.0/
Module: "Exponents, Roots, Factorization of Whole Numbers: Grouping Symbols and the Order of Opera-
tions"
Used here as: "Grouping Symbols and the Order of Operations "
By: Wade Ellis, Denny Burzynski
URL: http://cnx.org/content/m34872/1.2/
Pages: 160-170
Copyright: Wade Ellis, Denny Burzynski
License: http://creativecommons.org/licenses/by/3.0/
ATTRIBUTIONS 701

Module: "Exponents, Roots, Factorization of Whole Numbers: Prime Factorization of Natural Numbers"
Used here as: "Prime Factorization of Natural Numbers"
By: Wade Ellis, Denny Burzynski
URL: http://cnx.org/content/m34873/1.2/
Pages: 170-178
Copyright: Wade Ellis, Denny Burzynski
License: http://creativecommons.org/licenses/by/3.0/
Module: "Exponents, Roots, and Factorizations of Whole Numbers: The Greatest Common Factor"
Used here as: "The Greatest Common Factor "
By: Wade Ellis, Denny Burzynski
URL: http://cnx.org/content/m34874/1.2/
Pages: 178-182
Copyright: Wade Ellis, Denny Burzynski
License: http://creativecommons.org/licenses/by/3.0/
Module: "Exponents, Roots, Factorization of Whole Numbers: The Least Common Multiple"
Used here as: "The Least Common Multiple"
By: Wade Ellis, Denny Burzynski
URL: http://cnx.org/content/m34876/1.2/
Pages: 182-188
Copyright: Wade Ellis, Denny Burzynski
License: http://creativecommons.org/licenses/by/3.0/
Module: "Exponents, Roots, Factorization of Whole Numbers: Summary of Key Concepts"
Used here as: "Summary of Key Concepts"
By: Wade Ellis, Denny Burzynski
URL: http://cnx.org/content/m34877/1.2/
Pages: 188-191
Copyright: Wade Ellis, Denny Burzynski
License: http://creativecommons.org/licenses/by/3.0/
Module: "Exponents, Roots, Factorization of Whole Numbers: Exercise Supplement"
Used here as: "Exercise Supplement "
By: Wade Ellis, Denny Burzynski
URL: http://cnx.org/content/m34878/1.2/
Pages: 191-196
Copyright: Wade Ellis, Denny Burzynski
License: http://creativecommons.org/licenses/by/3.0/
Module: "Exponents, Roots, Factorization of Whole Numbers: Prociency Exam"
Used here as: "Prociency Exam "
By: Wade Ellis, Denny Burzynski
URL: http://cnx.org/content/m34879/1.2/
Pages: 196-197
Copyright: Wade Ellis, Denny Burzynski
License: http://creativecommons.org/licenses/by/3.0/
702 ATTRIBUTIONS

Module: "Introduction to Fractions and Multiplication and Division of Fractions: Objectives"


Used here as: "Objectives "
By: Denny Burzynski, Wade Ellis
URL: http://cnx.org/content/m18892/1.3/
Page: 211
Copyright: Denny Burzynski, Wade Ellis
License: http://creativecommons.org/licenses/by/2.0/
Module: "Introduction to Fractions and Multiplication and Division of Fractions: Fractions of Whole Num-
bers"
Used here as: "Fractions of Whole Numbers "
By: Wade Ellis, Denny Burzynski
URL: http://cnx.org/content/m34908/1.2/
Pages: 212-221
Copyright: Wade Ellis, Denny Burzynski
License: http://creativecommons.org/licenses/by/3.0/
Module: "Introduction to Fractions and Multiplication and Division of Fractions: Proper Fractions, Improper
Fractions, and Mixed Numbers"
Used here as: "Proper Fractions, Improper Fractions, and Mixed Numbers"
By: Wade Ellis, Denny Burzynski
URL: http://cnx.org/content/m34912/1.2/
Pages: 221-231
Copyright: Wade Ellis, Denny Burzynski
License: http://creativecommons.org/licenses/by/3.0/
Module: "Introduction to Fractions and Multiplication and Division of Fractions: Equivalent Fractions,
Reducing Fractions to Lowest Terms, and Raising Fractions to Higher Terms"
Used here as: "Equivalent Fractions, Reducing Fractions to Lowest Terms, and Raising Fractions to Higher
Terms "
By: Wade Ellis, Denny Burzynski
URL: http://cnx.org/content/m34927/1.3/
Pages: 231-242
Copyright: Wade Ellis, Denny Burzynski
License: http://creativecommons.org/licenses/by/3.0/
Module: "Introduction to Fractions and Multiplication and Division of Fractions: Multiplication of Frac-
tions"
Used here as: "Multiplication of Fractions"
By: Wade Ellis, Denny Burzynski
URL: http://cnx.org/content/m34928/1.2/
Pages: 242-253
Copyright: Wade Ellis, Denny Burzynski
License: http://creativecommons.org/licenses/by/3.0/
Module: "Introduction to Fractions and Multiplication and Division of Fractions: Division of Fractions"
Used here as: "Division of Fractions"
By: Wade Ellis, Denny Burzynski
URL: http://cnx.org/content/m34929/1.2/
Pages: 253-259
Copyright: Wade Ellis, Denny Burzynski
License: http://creativecommons.org/licenses/by/3.0/
ATTRIBUTIONS 703

Module: "Introduction to Fractions and Multiplication and Division of Fractions: Applications Involving
Fractions"
Used here as: "Applications Involving Fractions "
By: Wade Ellis, Denny Burzynski
URL: http://cnx.org/content/m34930/1.2/
Pages: 259-265
Copyright: Wade Ellis, Denny Burzynski
License: http://creativecommons.org/licenses/by/3.0/
Module: "Introduction to Fractions and Multiplication and Division of Fractions: Summary of Key Con-
cepts"
Used here as: "Summary of Key Concepts "
By: Wade Ellis, Denny Burzynski
URL: http://cnx.org/content/m34931/1.2/
Pages: 265-268
Copyright: Wade Ellis, Denny Burzynski
License: http://creativecommons.org/licenses/by/3.0/
Module: "Introduction to Fractions and Multiplication and Division of Fractions: Exercise Supplement"
Used here as: "Exercise Supplement "
By: Wade Ellis, Denny Burzynski
URL: http://cnx.org/content/m34932/1.2/
Pages: 268-274
Copyright: Wade Ellis, Denny Burzynski
License: http://creativecommons.org/licenses/by/3.0/
Module: "Introduction to Fractions and Multiplication and Division of Fractions: Prociency Exam"
Used here as: "Prociency Exam"
By: Wade Ellis, Denny Burzynski
URL: http://cnx.org/content/m34933/1.2/
Pages: 274-275
Copyright: Wade Ellis, Denny Burzynski
License: http://creativecommons.org/licenses/by/3.0/
Module: "Addition and Subtraction of Fractions, Comparing Fractions, and Complex Fractions: Objectives"
Used here as: "Objectives"
By: Denny Burzynski, Wade Ellis
URL: http://cnx.org/content/m18893/1.3/
Page: 293
Copyright: Denny Burzynski, Wade Ellis
License: http://creativecommons.org/licenses/by/2.0/
Module: "Addition and Subtraction of Fractions, Comparing Fractions, and Complex Fractions: Addition
and Subtractions of Fractions with Like Denominators"
Used here as: "Addition and Subtraction of Fractions with Like Denominators"
By: Wade Ellis, Denny Burzynski
URL: http://cnx.org/content/m34934/1.2/
Pages: 294-298
Copyright: Wade Ellis, Denny Burzynski
License: http://creativecommons.org/licenses/by/3.0/
704 ATTRIBUTIONS

Module: "Addition and Subtraction of Fractions, Comparing Fractions, and Complex Fractions: Addition
and Subtraction of Fractions with Unlike Denominators"
Used here as: "Addition and Subtraction of Fractions with Unlike Denominators"
By: Wade Ellis, Denny Burzynski
URL: http://cnx.org/content/m34935/1.2/
Pages: 298-304
Copyright: Wade Ellis, Denny Burzynski
License: http://creativecommons.org/licenses/by/3.0/
Module: "Addition and Subtraction of Fractions, Comparing Fractions, and Complex Fractions: Addition
and Subtraction of Mixed Numbers"
Used here as: "Addition and Subtraction of Mixed Numbers"
By: Wade Ellis, Denny Burzynski
URL: http://cnx.org/content/m34936/1.2/
Pages: 304-308
Copyright: Wade Ellis, Denny Burzynski
License: http://creativecommons.org/licenses/by/3.0/
Module: "Addition and Subtraction of Fractions, Comparing Fractions, and Complex Fractions: Comparing
Fractions"
Used here as: "Comparing Fractions"
By: Wade Ellis, Denny Burzynski
URL: http://cnx.org/content/m34937/1.3/
Pages: 308-312
Copyright: Wade Ellis, Denny Burzynski
License: http://creativecommons.org/licenses/by/3.0/
Module: "Addition and Subtraction of Fractions, Comparing Fractions, and Complex Fractions: Complex
Fractions"
Used here as: "Complex Fractions "
By: Wade Ellis, Denny Burzynski
URL: http://cnx.org/content/m34941/1.2/
Pages: 312-315
Copyright: Wade Ellis, Denny Burzynski
License: http://creativecommons.org/licenses/by/3.0/
Module: "Addition and Subtraction of Fractions, Comparing Fractions, and Complex Fractions: Combina-
tions of Operations with Fractions"
Used here as: "Combinations of Operations with Fractions "
By: Wade Ellis, Denny Burzynski
URL: http://cnx.org/content/m34942/1.3/
Pages: 316-320
Copyright: Wade Ellis, Denny Burzynski
License: http://creativecommons.org/licenses/by/3.0/
Module: "Addition and Subtraction of Fractions, Comparing Fractions, and Complex Fractions: Summary
of Key Concepts"
Used here as: "Summary of Key Concepts "
By: Wade Ellis, Denny Burzynski
URL: http://cnx.org/content/m34943/1.2/
Pages: 320-321
Copyright: Wade Ellis, Denny Burzynski
License: http://creativecommons.org/licenses/by/3.0/
ATTRIBUTIONS 705

Module: "Addition and Subtraction of Fractions, Comparing Fractions, and Complex Fractions: Exercise
Supplement"
Used here as: "Exercise Supplement "
By: Wade Ellis, Denny Burzynski
URL: http://cnx.org/content/m34944/1.2/
Pages: 321-324
Copyright: Wade Ellis, Denny Burzynski
License: http://creativecommons.org/licenses/by/3.0/
Module: "Addition and Subtraction of Fractions, Comparing Fractions, and Complex Fractions: Prociency
Exam"
Used here as: "Prociency Exam "
By: Wade Ellis, Denny Burzynski
URL: http://cnx.org/content/m34948/1.2/
Pages: 325-326
Copyright: Wade Ellis, Denny Burzynski
License: http://creativecommons.org/licenses/by/3.0/
Module: "Decimals: Objectives"
Used here as: "Objectives"
By: Denny Burzynski, Wade Ellis
URL: http://cnx.org/content/m18894/1.3/
Pages: 337-338
Copyright: Denny Burzynski, Wade Ellis
License: http://creativecommons.org/licenses/by/2.0/
Module: "Decimals: Reading and Writing Decimals"
Used here as: "Reading and Writing Decimals"
By: Wade Ellis, Denny Burzynski
URL: http://cnx.org/content/m34957/1.2/
Pages: 338-344
Copyright: Wade Ellis, Denny Burzynski
License: http://creativecommons.org/licenses/by/3.0/
Module: "Decimals: Converting a Decimal to a Fraction"
Used here as: "Converting a Decimal to a Fraction"
By: Wade Ellis, Denny Burzynski
URL: http://cnx.org/content/m34958/1.2/
Pages: 344-348
Copyright: Wade Ellis, Denny Burzynski
License: http://creativecommons.org/licenses/by/3.0/
Module: "Decimals: Rounding Decimals"
Used here as: "Rounding Decimals"
By: Wade Ellis, Denny Burzynski
URL: http://cnx.org/content/m34959/1.2/
Pages: 348-352
Copyright: Wade Ellis, Denny Burzynski
License: http://creativecommons.org/licenses/by/3.0/
706 ATTRIBUTIONS

Module: "Decimals: Addition and Subtraction of Decimals"


Used here as: "Addition and Subtraction of Decimals"
By: Wade Ellis, Denny Burzynski
URL: http://cnx.org/content/m34960/1.5/
Pages: 352-358
Copyright: Wade Ellis, Denny Burzynski
License: http://creativecommons.org/licenses/by/3.0/
Module: "Decimals: Multiplication of Decimals"
Used here as: "Multiplication of Decimals"
By: Wade Ellis, Denny Burzynski
URL: http://cnx.org/content/m34963/1.2/
Pages: 358-368
Copyright: Wade Ellis, Denny Burzynski
License: http://creativecommons.org/licenses/by/3.0/
Module: "Decimals: Division of Decimals"
Used here as: "Division of Decimals"
By: Wade Ellis, Denny Burzynski
URL: http://cnx.org/content/m34968/1.2/
Pages: 369-381
Copyright: Wade Ellis, Denny Burzynski
License: http://creativecommons.org/licenses/by/3.0/
Module: "Decimals: Nonterminating Divisions"
Used here as: "Nonterminating Divisions"
By: Wade Ellis, Denny Burzynski
URL: http://cnx.org/content/m34969/1.2/
Pages: 381-386
Copyright: Wade Ellis, Denny Burzynski
License: http://creativecommons.org/licenses/by/3.0/
Module: "Decimals: Converting a Fraction to a Decimal"
Used here as: "Converting a Fraction to a Decimal"
By: Wade Ellis, Denny Burzynski
URL: http://cnx.org/content/m34970/1.2/
Pages: 386-392
Copyright: Wade Ellis, Denny Burzynski
License: http://creativecommons.org/licenses/by/3.0/
Module: "Decimals: Combinations of Operations with Decimals and Fractions"
Used here as: "Combinations of Operations with Decimals and Fractions"
By: Wade Ellis, Denny Burzynski
URL: http://cnx.org/content/m34971/1.2/
Pages: 392-395
Copyright: Wade Ellis, Denny Burzynski
License: http://creativecommons.org/licenses/by/3.0/
ATTRIBUTIONS 707

Module: "Decimals: Summary of Key Concepts"


Used here as: "Summary of Key Concepts"
By: Wade Ellis, Denny Burzynski
URL: http://cnx.org/content/m34972/1.2/
Pages: 395-396
Copyright: Wade Ellis, Denny Burzynski
License: http://creativecommons.org/licenses/by/3.0/
Module: "Decimals: Exercise Supplement"
Used here as: "Exercise Supplement"
By: Wade Ellis, Denny Burzynski
URL: http://cnx.org/content/m34978/1.2/
Pages: 396-400
Copyright: Wade Ellis, Denny Burzynski
License: http://creativecommons.org/licenses/by/3.0/
Module: "Decimals: Prociency Exam"
Used here as: "Prociency Exam"
By: Wade Ellis, Denny Burzynski
URL: http://cnx.org/content/m34979/1.2/
Pages: 400-401
Copyright: Wade Ellis, Denny Burzynski
License: http://creativecommons.org/licenses/by/3.0/
Module: "Ratios and Rates: Objectives"
Used here as: "Objectives"
By: Denny Burzynski, Wade Ellis
URL: http://cnx.org/content/m18895/1.3/
Page: 417
Copyright: Denny Burzynski, Wade Ellis
License: http://creativecommons.org/licenses/by/2.0/
Module: "Ratios and Rates: Ratios and Rates"
Used here as: "Ratios and Rates"
By: Wade Ellis, Denny Burzynski
URL: http://cnx.org/content/m34980/1.2/
Pages: 417-422
Copyright: Wade Ellis, Denny Burzynski
License: http://creativecommons.org/licenses/by/3.0/
Module: "Ratios and Rates: Proportions"
Used here as: "Proportions"
By: Wade Ellis, Denny Burzynski
URL: http://cnx.org/content/m34981/1.2/
Pages: 423-429
Copyright: Wade Ellis, Denny Burzynski
License: http://creativecommons.org/licenses/by/3.0/
708 ATTRIBUTIONS

Module: "Ratios and Rates: Applications of Proportions"


Used here as: "Applications of Proportions"
By: Wade Ellis, Denny Burzynski
URL: http://cnx.org/content/m34982/1.2/
Pages: 429-435
Copyright: Wade Ellis, Denny Burzynski
License: http://creativecommons.org/licenses/by/3.0/
Module: "Ratios and Rates: Percent"
Used here as: "Percent"
By: Wade Ellis, Denny Burzynski
URL: http://cnx.org/content/m34983/1.2/
Pages: 435-442
Copyright: Wade Ellis, Denny Burzynski
License: http://creativecommons.org/licenses/by/3.0/
Module: "Ratios and Rates: Fractions of One Percent"
Used here as: "Fractions of One Percent"
By: Wade Ellis, Denny Burzynski
URL: http://cnx.org/content/m34997/1.2/
Pages: 442-445
Copyright: Wade Ellis, Denny Burzynski
License: http://creativecommons.org/licenses/by/3.0/
Module: "Ratios and Rates: Applications of Percents"
Used here as: "Applications of Percents"
By: Wade Ellis, Denny Burzynski
URL: http://cnx.org/content/m35007/1.2/
Pages: 446-456
Copyright: Wade Ellis, Denny Burzynski
License: http://creativecommons.org/licenses/by/3.0/
Module: "Ratios and Rates: Summary of Key Concepts"
Used here as: "Summary of Key Concepts"
By: Wade Ellis, Denny Burzynski
URL: http://cnx.org/content/m35008/1.2/
Pages: 456-458
Copyright: Wade Ellis, Denny Burzynski
License: http://creativecommons.org/licenses/by/3.0/
Module: "Ratios and Rates: Exercise Supplement"
Used here as: "Exercise Supplement"
By: Wade Ellis, Denny Burzynski
URL: http://cnx.org/content/m35009/1.2/
Pages: 458-462
Copyright: Wade Ellis, Denny Burzynski
License: http://creativecommons.org/licenses/by/3.0/
ATTRIBUTIONS 709

Module: "Ratios and Rates: Prociency Exam"


Used here as: "Prociency Exam"
By: Wade Ellis, Denny Burzynski
URL: http://cnx.org/content/m35010/1.2/
Pages: 462-463
Copyright: Wade Ellis, Denny Burzynski
License: http://creativecommons.org/licenses/by/3.0/
Module: "Techniques of Estimation: Objectives"
Used here as: "Objectives"
By: Denny Burzynski, Wade Ellis
URL: http://cnx.org/content/m18896/1.3/
Page: 475
Copyright: Denny Burzynski, Wade Ellis
License: http://creativecommons.org/licenses/by/2.0/
Module: "Techniques of Estimation: Estimation by Rounding"
Used here as: "Estimation by Rounding"
By: Wade Ellis, Denny Burzynski
URL: http://cnx.org/content/m35011/1.2/
Pages: 475-482
Copyright: Wade Ellis, Denny Burzynski
License: http://creativecommons.org/licenses/by/3.0/
Module: "Techniques of Estimation: Estimation by Clustering"
Used here as: "Estimation by Clustering"
By: Wade Ellis, Denny Burzynski
URL: http://cnx.org/content/m35012/1.2/
Pages: 482-485
Copyright: Wade Ellis, Denny Burzynski
License: http://creativecommons.org/licenses/by/3.0/
Module: "Techniques of Estimation: Mental Arithmetic- Using the Distibutive Property"
Used here as: "Mental Arithmetic-Using the Distributive Property"
By: Wade Ellis, Denny Burzynski
URL: http://cnx.org/content/m35013/1.2/
Pages: 485-491
Copyright: Wade Ellis, Denny Burzynski
License: http://creativecommons.org/licenses/by/3.0/
Module: "Techniques of Estimation: Estimation by Rounding Fractions"
Used here as: "Estimation by Rounding Fractions"
By: Wade Ellis, Denny Burzynski
URL: http://cnx.org/content/m35014/1.2/
Pages: 491-493
Copyright: Wade Ellis, Denny Burzynski
License: http://creativecommons.org/licenses/by/3.0/
710 ATTRIBUTIONS

Module: "Techniques of Estimation: Summary of Key Concepts"


Used here as: "Summary of Key Concepts"
By: Wade Ellis, Denny Burzynski
URL: http://cnx.org/content/m35015/1.2/
Pages: 493-494
Copyright: Wade Ellis, Denny Burzynski
License: http://creativecommons.org/licenses/by/3.0/
Module: "Techniques of Estimation: Exercise Supplement"
Used here as: "Exercise Supplement"
By: Wade Ellis, Denny Burzynski
URL: http://cnx.org/content/m35016/1.2/
Pages: 494-500
Copyright: Wade Ellis, Denny Burzynski
License: http://creativecommons.org/licenses/by/3.0/
Module: "Techniques of Estimation: Prociency Exam"
Used here as: "Prociency Exam"
By: Wade Ellis, Denny Burzynski
URL: http://cnx.org/content/m35017/1.2/
Pages: 500-501
Copyright: Wade Ellis, Denny Burzynski
License: http://creativecommons.org/licenses/by/3.0/
Module: "Measurement and Geometry: Objectives"
Used here as: "Objectives"
By: Denny Burzynski, Wade Ellis
URL: http://cnx.org/content/m18897/1.3/
Page: 511
Copyright: Denny Burzynski, Wade Ellis
License: http://creativecommons.org/licenses/by/2.0/
Module: "Measurement and Geometry: Measurement and the United States System"
Used here as: "Measurement and the United States System"
By: Wade Ellis, Denny Burzynski
URL: http://cnx.org/content/m35018/1.2/
Pages: 512-516
Copyright: Wade Ellis, Denny Burzynski
License: http://creativecommons.org/licenses/by/3.0/
Module: "Measurement and Geometry: The Metric System of Measurement"
Used here as: "The Metric System of Measurement"
By: Wade Ellis, Denny Burzynski
URL: http://cnx.org/content/m35019/1.2/
Pages: 517-522
Copyright: Wade Ellis, Denny Burzynski
License: http://creativecommons.org/licenses/by/3.0/
ATTRIBUTIONS 711

Module: "Measurement and Geometry: Simplication of Denominate Numbers"


Used here as: "Simplication of Denominate Numbers"
By: Wade Ellis, Denny Burzynski
URL: http://cnx.org/content/m35021/1.2/
Pages: 522-529
Copyright: Wade Ellis, Denny Burzynski
License: http://creativecommons.org/licenses/by/3.0/
Module: "Measurement and Geometry: Perimeter and Circumference of Geometric Figures"
Used here as: "Perimeter and Circumference of Geometric Figures"
By: Wade Ellis, Denny Burzynski
URL: http://cnx.org/content/m35022/1.2/
Pages: 530-540
Copyright: Wade Ellis, Denny Burzynski
License: http://creativecommons.org/licenses/by/3.0/
Module: "Measurement and Geometry: Area and Volume of Geometric Figures and Objects"
Used here as: "Area and Volume of Geometric Figures and Objects"
By: Wade Ellis, Denny Burzynski
URL: http://cnx.org/content/m35023/1.2/
Pages: 540-554
Copyright: Wade Ellis, Denny Burzynski
License: http://creativecommons.org/licenses/by/3.0/
Module: "Measurement and Geometry: Summary of Key Concepts"
Used here as: "Summary of Key Concepts"
By: Denny Burzynski, Wade Ellis
URL: http://cnx.org/content/m35024/1.2/
Pages: 555-556
Copyright: Denny Burzynski, Wade Ellis
License: http://creativecommons.org/licenses/by/3.0/
Module: "Measurement and Geometry: Exercise Supplement"
Used here as: "Exercise Supplement"
By: Wade Ellis, Denny Burzynski
URL: http://cnx.org/content/m35025/1.2/
Pages: 557-562
Copyright: Wade Ellis, Denny Burzynski
License: http://creativecommons.org/licenses/by/3.0/
Module: "Measurement and Geometry: Prociency Exam"
Used here as: "Prociency Exam"
By: Wade Ellis, Denny Burzynski
URL: http://cnx.org/content/m35026/1.2/
Pages: 562-566
Copyright: Wade Ellis, Denny Burzynski
License: http://creativecommons.org/licenses/by/3.0/
712 ATTRIBUTIONS

Module: "Signed Numbers: Objectives"


Used here as: "Objectives"
By: Denny Burzynski, Wade Ellis
URL: http://cnx.org/content/m18898/1.3/
Page: 575
Copyright: Denny Burzynski, Wade Ellis
License: http://creativecommons.org/licenses/by/2.0/
Module: "Signed Numbers: Variables, Constants, and Real Numbers"
Used here as: "Variables, Constants, and Real Numbers "
By: Wade Ellis, Denny Burzynski
URL: http://cnx.org/content/m35027/1.2/
Pages: 576-581
Copyright: Wade Ellis, Denny Burzynski
License: http://creativecommons.org/licenses/by/3.0/
Module: "Signed Numbers: Signed Numbers"
Used here as: "Signed Numbers "
By: Wade Ellis, Denny Burzynski
URL: http://cnx.org/content/m35029/1.3/
Pages: 582-586
Copyright: Wade Ellis, Denny Burzynski
License: http://creativecommons.org/licenses/by/3.0/
Module: "Signed Numbers: Absolute Value"
Used here as: "Absolute Value"
By: Wade Ellis, Denny Burzynski
URL: http://cnx.org/content/m35030/1.3/
Pages: 586-590
Copyright: Wade Ellis, Denny Burzynski
License: http://creativecommons.org/licenses/by/3.0/
Module: "Signed Numbers: Addition of Signed Numbers"
Used here as: "Addition of Signed Numbers"
By: Wade Ellis, Denny Burzynski
URL: http://cnx.org/content/m35031/1.2/
Pages: 590-597
Copyright: Wade Ellis, Denny Burzynski
License: http://creativecommons.org/licenses/by/3.0/
Module: "Signed Numbers: Subtraction of Signed Numbers"
Used here as: "Subtraction of Signed Numbers"
By: Wade Ellis, Denny Burzynski
URL: http://cnx.org/content/m35032/1.2/
Pages: 597-602
Copyright: Wade Ellis, Denny Burzynski
License: http://creativecommons.org/licenses/by/3.0/
ATTRIBUTIONS 713

Module: "Signed Numbers: Multiplication and Division of Signed Numbers"


Used here as: "Multiplication and Division of Signed Numbers"
By: Wade Ellis, Denny Burzynski
URL: http://cnx.org/content/m35033/1.2/
Pages: 602-611
Copyright: Wade Ellis, Denny Burzynski
License: http://creativecommons.org/licenses/by/3.0/
Module: "Signed Numbers: Summary of Key Concepts"
Used here as: "Summary of Key Concepts"
By: Wade Ellis, Denny Burzynski
URL: http://cnx.org/content/m35034/1.2/
Pages: 611-612
Copyright: Wade Ellis, Denny Burzynski
License: http://creativecommons.org/licenses/by/3.0/
Module: "Signed Numbers: Exercise Supplement"
Used here as: "Exercise Supplement "
By: Wade Ellis, Denny Burzynski
URL: http://cnx.org/content/m35035/1.2/
Pages: 612-616
Copyright: Wade Ellis, Denny Burzynski
License: http://creativecommons.org/licenses/by/3.0/
Module: "Signed Numbers: Prociency Exam"
Used here as: "Prociency Exam"
By: Wade Ellis, Denny Burzynski
URL: http://cnx.org/content/m35036/1.2/
Pages: 616-617
Copyright: Wade Ellis, Denny Burzynski
License: http://creativecommons.org/licenses/by/3.0/
Module: "Algebraic Expressions and Equations: Objectives"
Used here as: "Objectives"
By: Denny Burzynski, Wade Ellis
URL: http://cnx.org/content/m18899/1.3/
Page: 629
Copyright: Denny Burzynski, Wade Ellis
License: http://creativecommons.org/licenses/by/2.0/
Module: "Algebraic Expressions and Equations: Algebraic Expressions"
Used here as: "Algebraic Expressions"
By: Wade Ellis, Denny Burzynski
URL: http://cnx.org/content/m35038/1.2/
Pages: 630-636
Copyright: Wade Ellis, Denny Burzynski
License: http://creativecommons.org/licenses/by/3.0/
714 ATTRIBUTIONS

Module: "Algebraic Expressions and Equations: Combining Like Terms Using Addition and Subtraction"
Used here as: "Combining Like Terms Using Addition and Subtraction"
By: Wade Ellis, Denny Burzynski
URL: http://cnx.org/content/m35039/1.2/
Pages: 636-639
Copyright: Wade Ellis, Denny Burzynski
License: http://creativecommons.org/licenses/by/3.0/
Module: "Algebraic Expressions and Equations: Solving Equations of the Form x+a=b and x-a=b"
Used here as: "Solving Equations of the Form x+a=b and x-a=b"
By: Wade Ellis, Denny Burzynski
URL: http://cnx.org/content/m35044/1.3/
Pages: 639-646
Copyright: Wade Ellis, Denny Burzynski
License: http://creativecommons.org/licenses/by/3.0/

Module: "Algebraic Expressions and Equations: Solving Equations of the Form ax=b and x/a=b"
Used here as: "Solving Equations of the Form ax=b and x/a=b"
By: Wade Ellis, Denny Burzynski
URL: http://cnx.org/content/m35045/1.3/
Pages: 647-655
Copyright: Wade Ellis, Denny Burzynski
License: http://creativecommons.org/licenses/by/3.0/
Module: "Algebraic Expressions and Equations: Applications I: Translating Words to Mathematical Sym-
bols"
Used here as: "Applications I: Translating Words to Mathematical Symbols"
By: Wade Ellis, Denny Burzynski
URL: http://cnx.org/content/m35046/1.2/
Pages: 655-660
Copyright: Wade Ellis, Denny Burzynski
License: http://creativecommons.org/licenses/by/3.0/
Module: "Algebraic Expressions and Equations: Applications II: Solving Problems"
Used here as: "Applications II: Solving Problems"
By: Wade Ellis, Denny Burzynski
URL: http://cnx.org/content/m35047/1.2/
Pages: 660-669
Copyright: Wade Ellis, Denny Burzynski
License: http://creativecommons.org/licenses/by/3.0/
Module: "Algebraic Expressions and Equations: Summary of Key Concepts"
Used here as: "Summary of Key Concepts"
By: Wade Ellis, Denny Burzynski
URL: http://cnx.org/content/m35049/1.2/
Pages: 669-671
Copyright: Wade Ellis, Denny Burzynski
License: http://creativecommons.org/licenses/by/3.0/
ATTRIBUTIONS 715

Module: "Algebraic Expressions and Equations: Exercise Supplement"


Used here as: "Exercise Supplement "
By: Wade Ellis, Denny Burzynski
URL: http://cnx.org/content/m35050/1.2/
Pages: 671-678
Copyright: Wade Ellis, Denny Burzynski
License: http://creativecommons.org/licenses/by/3.0/
Module: "Algebraic Expressions and Equations: Prociency Exam"
Used here as: "Prociency Exam"
By: Wade Ellis, Denny Burzynski
URL: http://cnx.org/content/m35052/1.2/
Pages: 678-679
Copyright: Wade Ellis, Denny Burzynski
License: http://creativecommons.org/licenses/by/3.0/
Fundamentals of Mathematics
Fundamentals of Mathematics is a work text that covers the traditional topics studied in a modern prealgebra
course, as well as topics of estimation, elementary analytic geometry, and introductory algebra. It is intended
for students who have had a previous course in prealgebra, wish to meet the prerequisite of a higher level
course such as elementary algebra, and need to review fundamental mathematical concepts and techniques.

About Connexions
Since 1999, Connexions has been pioneering a global system where anyone can create course materials and
make them fully accessible and easily reusable free of charge. We are a Web-based authoring, teaching and
learning environment open to anyone interested in education, including students, teachers, professors and
lifelong learners. We connect ideas and facilitate educational communities.
Connexions's modular, interactive courses are in use worldwide by universities, community colleges, K-12
schools, distance learners, and lifelong learners. Connexions materials are in many languages, including
English, Spanish, Chinese, Japanese, Italian, Vietnamese, French, Portuguese, and Thai. Connexions is part
of an exciting new information distribution system that allows for Print on Demand Books. Connexions
has partnered with innovative on-demand publisher QOOP to accelerate the delivery of printed course
materials and textbooks into classrooms worldwide at lower prices than traditional academic publishers.

You might also like